You are on page 1of 422

ADVANCED LEVEL

PURE MATHEMATICS
PHYSICAL SCIENCE TEXTS
General Editor
SIR GRAHAM SUTTON. C.B.E.. D.Sc., F.R.S.
Director-General, Meieorologkai Office
Fottnerly Dmii of r.c Royal Military College
of Science. Shrivenhani,
and Ba%hfuTth Professor of Mal'ictnolical Physics

ADVANCED LEVEL APPLIED MATHEMATICS


by C. G. Lambe, B.A., Ph.D.
APPLIED MATHEMATICS FOR ENGINEERS AND SCIENTISTS
by C. G. Lambe, B.A.. Ph.D.
ADVANCED LEVEL PURE MATHEMATICS
by C. J. Tranter. O.B.E.. M.A., D.Sc.
TECHNIQUES OF MATHEMATICAL ANALYSIS
by C. J. Tranter, O.B.E., M.A., D.Sc.
GENERAL PHYSICS AND SOUND
(lo -Advanced and Scholarship Level)
by D. H. Fender, B.Sc.. Ph.D.
HEAT
{To -Advanced and Scholarship Level)
by A. J. Woodall, p.B.E., Ph.D., F.Inst.P.
LIGHT
(To Advanced and Scholarship Level)
by C. B. Daish, M.Sc.
ELECTRICITY AND MAGNETISM
(To Advanced and Scholarship Le\cl)
by C. G. Wilson, M.Sc., A. Inst. P.
EXPERIMENTAL PHYSICS
(To Advanced and Scholarship Level)
by C. B. Daish, M.Sc.
D. H. Fender, B.Sc., Ph.D.
A COMPENDIUM OF MATHEMATICS AND PHYSICS
by Dorothy Meyler, M.Sc.
Sir Graham Sutton, C.B.E., D.Sc., F.R.S.

In Preparation
A COURSE IN APPLIED MATHEMATICS
(Covering B.A. and B.Sc. General Degrees)
by D. F. Lawden, M.A.
PHYSICS FOR ELECTRICAL ENGINEERS
by W. P. Jolly, B.Sc.
ELEMENTS OF MATHEMATICAL PHYSICS
b>' 1. N. Sneddon, D.Sc.
ELECTRON PHYSICS AND TECHNOLOGY
by J. Thomson, D.Sc.
E. B. Callick, B.Sc.

PRINCIPLES OF ELECTRONICS
by M. R. Gavin, M.B.E., D.Sc.
J. E. I-IOULDIN, Ph.D.
REACTOR PHYSICS AND TECHNOLOGY
by J. Walker, Ph.D., F.Inst.P., and D. Jakeman, Ph.D., A. Inst. P
ADVANCED LEVEL
PURE MATHEMATICS
By

C. J. TRANTER, O.B.E., M.A., D.Sc.


Bashforth Professor of Mathematical Physics,
Royal Military College of Science, Shhvenham

THE ENGLISH UNIVERSITIES PRESS


LTD
102 NEWGATE STREET
LONDON, E.C.l
First Printed . • « ^953
Reprinted • • L 955
Reprinted • . 1956
Reprinted • • 1957
Reprinted • . 1958

it

ALL RIGHTS RESERVED

S\^- S\A-
4

Printed in Great Britain for The English Universities Press Ltd.


by Butler& Tanner Lid., Frome and London
GENERAL EDITOR'S FOREWORD
General Editor:

SIR GRAHAM SUTTON. D.Sc., F.R.S.


Director-General, Meteorological Office.
Formerly Dean of the Royal Military College of Science, Shrivenham,
and Bashforth Professor of Mathematical Physics

The present volume one of a series on physics and mathematics, for


is
the upper forms at school and the first year at the university. The
books have been written by a team of experienced teachers at the
Royal Military College of Science where, among other things, students
are prepared for London (External) Degrees in Natural Science and
Engineering. The series therefore forms an integrated course of study
based on many years experience in the teaching of physics and
mathematics.
In preparing their manuscripts the writers have been mainly guided
by the examination syllabuses of London University, the Joint Board
of Oxford and Cambridge and the Northern Universities Board, but
they have also taken a broad view of their tasks and have endeavoured
to produce works which aim to give a student that solid foundation
without which it is impossible to proceed to higher studies. The books
are suitable either for class teaching or self study there are many
;

illustrative examples and large collections of problems


for solution
taken, in the main, from recent 'elimination papers.
It isa truism too often forjg'ottfen in teaching that knowledge is
acquired by a student only wheri hft interest is aroused
and maintained.
The student must not only be shown how a class of problems in
mathb-
mati^ is solved but, within limits, why a particular method works
and m physics, why a technique is especially weU adapted for some
particular me^urement. Throughout the series special emphasis
has
been laid on illustrations which may be expected to
appeal to the
experience of the student in matters of daily
life, so that his studies
we reljlpd to what he sees, feels and knows of the world around him.
1reateiTin this way, science ceases to be
an arid abstraction and becomes
vivid and real to the inquiring
mind.
The books have therefore been written, not
only to ensure the
as a preparation for the exciting
world
They incorporate many of the sug-
y®*” teachers and,
It is hoped, will bnng some new points of view into the cl
vi GENERAL EDITOR'S FOREWORD
the study. Last, but by no means they have been written by a
least,
team working together, so that the exchange of ideas has been constant
and vigorous. It is to be hoped that the result is a scries which is
adequate for all examinations at this level and yet broad enough to
satisfy the intellectual needs of teachers and students alike.
O. G. Sutton.
PREFACE
The needs of those taking Pure Mathematics at Advanced Level in
the recently introduced General Certificates of Education have set the
standard for this book. The contents should also prove suitable for
candidates preparing themselves for the Intermediate Examination of
London University, the Qualif5dng Examination for the Mechanical
Science Tripos at the University of Cambridge and for several of the
examinations set by the Civil Service Commission.
The starting point is the Ordinary Level for the General Certificates
and I have included in a single volume the appropriate parts of Algebra,
Trigonometry, Calculus and Geometry. With so wide a field of study,
the order in which the subjects appear is not necessarily the order in
which they should be read. I believe that an early start should be
made with the Calculus and the chapters on this subject may well be
studied concurrently with those on Algebra and Trigonometry. I have
included a large number of worked examples and graded the exercises
in a way which will, I hope, make the book equally suitable for class
or private study.
In preparing this book, I have made great use of the reports of the
Teaching Committees of the Mathematical Association. In particular,
I have found their recent reports on the Teaching of Trigonometry and
Calculus quite invaluable and I wish to acknowledge my debt to them.
Only very occasionally have I differed from their recommendations
and here, of course, I bear full responsibility.
Among nearly 1600 examples and exercises, I have included a large
number taken from recent papers set by the various examining bodies.
My thanks are due to the Senate of the University of London, the
Oxford and Cambridge Schools Examination Board, the Joint
Matricu-
lation Board of the Universities of Manchester,
Liverpool, Leeds, Shef-
field and Bimungham and the Syndics
of the Cambridge University
Press for permission to use their questions. My
thanks are also due to
many friends and colleagues who read the manuscript and offered
instructive criticism. I am particularly grateful
to Dr. E. T. Davies,
Professor of Mathematics, University of
Southampton, Dr. D. r!
DickuiMn, Senior Mathematics Master, Bristol
Grammar School, Mr.
L. Heywood, Senior Mathematics Master,
Manchester Grammar
School Md Mr. H. K. Prout, Head of the Department
of MathemaUcs,
Royal Naval CoUege, Dartmouth,
all of whom made most useful
suggestions when the book was in its first draft.
C. J. Tranter.
Royal Military College of Science,
Shrivenhau.
vti
CONTENTS
CKAF.
rACt

1. The Theory of Quadratic Equations. Miscellaneous


Equations
The roots of a quadratic equation.The sum and product of the roots.
Miscellaneous equations involving one unknown. Simultaneous equa-
tions. The square root of (a + y/ b).
Indices and Logarithms. The Remainder Theorem.

.........
2.
The Principle of Undetermined Coefficients. Partial
Fractions 22
Fundamental laws for positive integral indices. Fractional, zero and
negative indices. Theory of logarithms. Common
logarithms.
Equations in which the unknown occurs as an index. The remainder
theorem. The principle of undetermined coefficients. Partial
fractions.

3. Arithmetical and Geometrical Progressions. Permu-


tations AND Combinations. The Binomial Theorem . 3G
Series. The arithmetical progression. The geometrical progression.
Simple and compound interest. The convergence of the geometric
series. Series involving the natural numbers. Permutations and
combinations. Probability or chance. Binomial theorem for a
positive integral index. Binomial theorem when n is not a positive
integer.

4. Trigonometrical Ratios for any Angle. Graphs of


Trigonometrical Ratios. Trigonometrical Equations 67
Introduction. Trigonometrical ratios for the general angle.
Trigonometrical ratios of some related angles. Graphs of the
trigonometrical ratios for acute angles. Graphs of the trigonometrical
ratios for the general angle. Solution of trigonometrical equations.
6. Addition Theorems. Multiple and Submultiple Angles.

......
Further Trigonometrical Equations. The Inverse
Notation. Small Angles
Addition theorems for the sine and cosine. Addition theorem for the
tangent. Multiple angles. Submultiple angles. Factor formulae.
74

Further trigonometrical equations. The equation a cos ^+6 sin


The inverse notation. Small angles.
6. Relations between the Sides and Angles of a Triangle.
The Solution of Triangles. Heights and Distances 93
^
Notation. The sine formula. The cosine formula. Area of a
triangle.Radius of the inscribed circle. Radii of the escribed
drcles. Formulae for the angles of a triangle in terms of the sides.
The tangent formula. Summary of formulae for the triangle.
Numerical solution of triangles. Examples of the use of the sine
formula. Examples of the use of the cosine formula. Example of
the use of the tangent rule. Alternative methods
of solution of
triangle with three sides given Heights and distances*

7. Introduction TO THE Ideas OF THE Differential Calculus


119
Functions and functional notation. Gradient of a curve.
The
increment notation. DiSerentiation from first principles.
The dif-
ferential coefficient. Differential coefficient of x", n a positive
int^er. Differential coefficients of sin x and cos x. The differential
coefficient as a rate measurer. Approximations.
lx
X CONTENTS
CHAP. PAG!
8. Technical Processes in the Differential Calculus 131
Introduction. Differentiation of a sum. Differentiation of a pro-
duct. Differentiation of a quotient. Differential coefficients of
tan X. cot X, cosec x and sec x. Differentiation of a function of a
function. Differential coefficient of x^ when n is negative or frac-
tional. Differentiation of inverse functions. Differential coefficients
of tan“^ X and sin”* x. Differentiation of implicit functions. List of
standard forms. Higher derivatives.
9. Some Applications of the Differential Calculus . 151
Introduction. Examples of the derivative as a rate measurer.
Some dynamical applications. Maximum and minimum. Applica-
tions to practical problems. Points of inflexion. Curve sketching.
10. Introduction to the Ideas of the Integral Calculus . 1C9
The nature of the fundamental problem. Standard forms. Some
geometrical and dynamical applications. Calculation of an area as a
limit of a sum. The integral as a sum. Examples of definite integrals
and calculation of area. Volumes of figures of revolution.

11. Some Methods of Integration . . . . . 188


Introduction. Generalisation of the list of standard integrals. The
integration of products of sines and cosines. Integration by change
of variable. Definite integrals by change of variable. Integration
by parts. Approximate methods of integration.
12. Some Applications of the Integral Calculus. 211
Introduction. Further examples of the calculation of area. Calcu-
lation of volumes. Mean values. Centres of mass. Moments of
inertia. Length of arc. Areas of surfaces of revolution.
13. The Logarithmic and Exponential Functions • • 236
Introduction. Theareabelowthecurvey=l/jr. Important properties
of the function hyp (/). The logarithmic function. The exponential
function. Differentiation and integration of «. Integrals depending
on/x"*</x. Logarithmic differentiation. Successive approximations
and Maclaurin’s series. Series for «» and log* (1 x). +
14. Coordinates. Lengths of Lines. Areas of Triangles.
Loci. Intersection of Curves . . .
259
Systems of coordinates. Relation between Cartesian and polar co-
ordinates. Distance between two points with given rectangular co-
ordinates. A proof of the addition formulae of trigonometry.
Coordinates of a point dividing a line in a given ratio. Area of a
triangle whose vertices have given coordinates. Equation to a locus.
Intersections of two curves whose equations are known.
274
16. The Coordinate Geometry of the Straight Line . .

An equation of the first degree represents a straight line. Equation


to a line parallel to a coordinate axis. Special forms of the equation to
parallelism
a straight line. Angle between two lines. Conditions for
and perpendicularity. Perpendicular distance of a point from a hn^
The bisectors of the angles between two lines. Line passing through
pair of lines.
the intersection of two given lines. Equation to a
295
10. The Coordinate Geometry op the Circle General equa-
The equation to a circle with given centre and radius.
tion to a circle. Equation to a circle whose diameter is the join of mo
pointe. Ihe
Riven points. Equation to a circle through three given
intersection of a
tangent to a circle at a given point. Points of
straight line and circle. The length of the tangent from
an external
Circle
point. Orthogonal circles. Radical axis of two circles.
through intersections of two given circles.
CONTENTS xl

CRAP. PAGI
17. The Parabola, Ellipse and Hyperbola . . . . 310
Introduction. Tiie equation to a parabola. The tangent and normal
to a parabola. Points of intersection of straight line and parabola.
Parametric equations to a parabola. An important property of the
parabola. The equation to an ellipse. The tangent and normal to
an ellipse. Points of intersection of straight line and ellipse. Para-
metric equations to an ellipse. The eccentric angle. The equation
to a hyperbola. The tangent and normal to a hyperbola. Points of
intersection of straight line and hyperbola. Parametric equations to
a hyperbola. Asymptotes of a hyperbola. The rectangular hyper-
bola. The rectangular hyperbola referred to its asymptotes as axes.
18. Some Theorems in Pure Geometry . . . . . 337
Introduction. Some theorems on the circle. Revision examples.
Theorems on proportion and similar triangles. Revision e.xamplcs.
Similar rectilinear figures. Some ratio and rectangle properties of the
triangle and quadrilateral. Some further properties of a triangle.
The theorems of Ceva and Mcnelaus.
19. Elementary Geometry of the Plane and Sphere . 361
Introductionand definitions. Axioms andfurther definitions.
Some theorems on parallels. Normals. Orthogonal projection and
dihedral angles. Some geometrical properties of the sphere.
20. Mensuration of Simple Solid Figures . . , . 382
Introduction. The volume of a right prism. The volume of an
oblique prism. The volume of a pyramid. The volume and curved
surface of a cylinder. The volume and curved surface of a cone.
The mensuration of the sphere. Summary of formulae.
Answers to the Exercises
Index ... 400
417

The sources from which some of the examples and exercises are taken
are indicated by the following abbreviations ;

L.U. Examinations of Intermediate standard set by the University of

London ;

O.C. Examinations of a similar standard set by the Oxford and Cam-


bridge Schools Examination Board ;

N.U. Examinations of a similar standard set by the Joint Matriculation


Board of the Universities of Manchester, Liverpool, Leeds, Shef-
and Birmingham
field ;

Q.E. The Qualifying Examination for the Mechanical Sciences Tripos


at the University of Cambridge,
CHAPTER 1

THE THEORY OF QUADRATIC EQUATIONS.


MISCELLANEOUS EQUATIONS
1.1. The roots of a quadratic equation
The general quadratic equation can be written
ax^ + c = 0, (1.1)

where a, h and c are numerical coefficients and x is the quantity to


be found. Dividing by a and transposing the term not containing x
to the right hand side

a a

The left hand side can be made into the perfect square
(‘ ^
by adding a term 6y(4a2). If therefore such a term is added to each
side
52 c
1

*'^2aj 4a» a
6® — 4ac
4a*
Taking the square root of each side
^{6* — 4flc)
2a 2a
giving the two roots

^
—5± \/{6* — 4ac)
( 1 . 2)
2a
If 6* > 4ac
the two roots are real and different, if 6* 4ac the =
roots are real and both equal to &/(2a). If 6*— 4ac the expression <
under the square root sign is negative and, since there is no real
quantity whose square is negative, the roots are in this case said to
be imaginary.
The formula (1.2) is quite general and can always be used to obtain
the roots of a quadratic equation. If, however,
factors of the left
hand side of the equation ax* -f~
are more easily obtained by setting each
c + —
0 can be found, the roots
of the factors in turn equal
to zero and solving the resulting simple
equations. This process is
illustrated in the first example below.

Example 1. —
Solve the equations (a) 2x* + 12 -b 0, (&) + 11 . 7x.
13
u PURE MATHEMATICS [1
The left hand side of equation has factors [2x —
the equation can be written
(a) 3)(^ + 4) so that

(2x - 3)(jv + 4) = 0.
Hence cither 2;r — 3 = 0 giving = 3/2. or ;r + 4 = 0 giving ;r = — 4.
Tor equation (6), a = 1, d = — 7, c = 11 and formula (1.2) gives
^ _ 7 ± - 4(l)(il)} 7 ± -v/5 7 ±2-230
2 ~ 2 “ 2
'

giving X = 4-018 or 2-382.

Example 2. Find
the value of k so that the equation — 84- + A = 0 shall
have equal roots.

Here n = 4, 6 = — S, c ~ k. The condition for equal roots (6* = 4ar)


gives
(— 8)* = 4{4){A) or IGft == 64, giving A = 4.

Example 3. Prove that the roots of the equation

(/» — ? — 0-*^* -{-px + q +r— O


are real if p, q and r are real.

The condition for real roots (6* > 4ar) is here that
p^> ^{p - q - r){q + r),
i.e., that /?= — 4p[q + r) + 4{q + r)»> 0,
or Ip — 2{q + r) > 0.
This always true for the left hand side
is is the square of a real quantity
and therefore cannot be negative.

Example If x is real, show that the expression


4. y = (.»* +^
can have «o real value between 3 and 1. — (L.U.)

Rearranging as a quadratic in x,

{X + +X+ 1
l)y = x‘
giving ;r» + (1 — y)x + 1 — y = 0.
For X to be real (1 — y)*> 4(1 — y).

or, (1 — >)(- 3 - y) > 0.

Changing the signs, for x to be real


(y - l){y + 3) > 0.
If y lies between — 3 and I, y + 3 > 0, and y — < 1 0 giving
(y _ i)(^ -(-
3) and the above inequality
0 is not satisfied. Hence
there is no real value between — 3 and 1.

1.2. The sum and product of the roots of a quadratic equation


The general quadratic equation
ax^ + bx +c=0 (
1-1)

can be written as

a
+ fa = 0. (1.3)

If its roots are a and the left hand side of the equation can be written
1] THEORY OF QUADRATIC EQUATIONS 15
as the product of two factors (x — ix){x —
p) and thus the equation
can be written
- a){x -p) = 0,
{X
or, — (a + p)x a/? = 0. -j-
(1.4)
Since equations (1.3) and (1.4) are identical

oi-i-B = ~-= — .Sufficient of in equation (I.l)


a coefficient of x^ in equation (I.l)*
(1.5)
^ coefficient independent of % in equation (1.1).
a coefficient of x^ in equation (1.1)
The formulae enable the values of the sum and product
(1.5)
of the
roots to be written down in terms
of the coefficients in the eiven
equation.

<Mr» -j- L 4. c = 0
ft**
15 three times the other.
^ equation

Let the roots be a and 3a. Then formulae (I.S) give


4a =! — b/a and 3a* = c/a.
Substituting a = - b/{Aa) from the first of these relations
in the second

V 4a/ a*
giving 36* = I6ac as the required relation.

Example 6.
If a, ^ are the roots of the equation x*
equation whose roots are
px
P + q
? = 0 Jorm
.form the
a/fi* and ^/a*

The sum of the roots of the


required equation ^^E^ *
Now, a*;j*

a + = (o + + ^>) = + —
(a ^}{(a + j8)* 3oJ?}
== p[p* — 3y), using (1.6).
Hence the sum of the roots
of the required equation =
smee a*^* = qt ^0^ 9*
second of (1.6).

The product of the roots of the


required equaUon = “
j9* a*
. _^
* 1

From (1.5). when the ^


coefficient of is unity,
= ~ t»o roots.
tte
* =
Hence the
xience tte required T
a equation
is
°‘ the product of the roots,

or.
9*-*^* - /•(#*» - Bq)x + « 0.ff
16 PURE MATHEMATICS [1

EXERCISES 1 (a)
1 Solve the equations, Sx^ ~ 2x — 3 = 0, (u) 5x^ + 10 =
(i) 17;!'.

2 , Show that the equation ^^(1 — = 1 has no real roots if 0 < ;*r) A < 4.

(L.U.)
3, Find the range of values of x for which

^ {x - 1)2 9
0 < .
'

X — ^
2
< -
2
(L.U.)

4. Find the relation between p, q and r if one root of the equation


/>^2 ^ Q jg double the other.
5. If p are the roots of the quadratic equation ax*
a, bx + c = 0,
obtain the equation whose roots are 1/a® and 1//3®.
If, in the above equation a^* = 1, prove that a® =
+ c® + abc 0.

(L.U.)
6 . In the equation ax* hx c = 0, one root is the square of the other.
Without solving the equation, prove that c{a — i>)® = a(c — b)*.
(L.U.)

1.3. Miscellaneous equations involving one unknown


The solution of certain types of equation can sometimes be made
to depend on that of the ordinary quadratic equation. Some of the
artifices employed in such solutions arc illustrated in the examples
which follow.

Example 7. So/ve the equation ^/{Z — x) — + .*) = V(l® + 2.r).

(L.U.)

^Vhcn, as here, one side of an equation contains a single term involving


a square root, this can be removed by squaring both sides. If the terms
in the resulting equation be transposed so that any radical term remain-
ing is again by itself, this can be removed by squaring again.
Applying this process to the equation given here, squaring both sides
gives
Z ~ X - 2^/ {{3 ~ x)il x)} n X = \Q + 2x,

which, on rearrangement and division by 2, gives

3 + ^ - V{(3 -^)(7 + *))•

Squaring again we have


(3 + X)* = (3 - x){l + X),

leading to the ordinary quadratic equation


2x* + IOjt - 12 = 0.

After division by 2, this can be written


{X + 6)(x - 1) = 0,
with roots r = — 6 and = 1.
Only one of these roots (^r = - 6) satisfies the
given equation. The
other value {x = 1) does not satisfy the equation for solution but
s.atisfics

y(3 - x) + V(7 + •’f) = V(I3 + 2-r).


il SIMULTANEOUS EQUATIONS 17

in which the radical terms on the left are separated by a plus instead
of a minus sign. If the same process is applied to this equation it will
be found to lead to the same quadratic 2x^ + 12 = 0 as before. —
Hence, in solving equations of this type it is essential to check the
values found in the actual equation given. In the case of the equation
given here the required solution is = — 6.

Example 8. Solve the equation x* -f — 2 = 8/(x* + 3^)*

Write ^ = X* + 3x and we obtain


y -2=. 8/y.
or, y* — 2^ — 8 = 0.
This gives (y — 4)(y -1-
2) = 0, so that y = 4 or — 2.

Since y ^ X* 2x, y = 4 gives the quadratic x* -f 3x — 4 = 0, or


(jy4- 4)(x — 1) = 0 with roots x = i, — 4. The other value (y = — 2)
gives ;r* + 3x + 2 = 0 or + 2)(;y 4- 1) « 0 with roots x = — 1,
(;y

— 2. Hence the roots of the original equation are — 1, — 2, — 4 and 1.


1.4. Simultaneous equations
It is assumed that the student is familiar with the solution of pairs
of equations such as 3 a; -|- 4^ 7, 2x ^ =
1, in which both equa- — =
tions are of the first degree. Here we shall consider pairs of equations
in which at least one is of a higher degree than the first and where
the solution can be made
depend on that of a quadratic equation.
to
Few fixed rules can be laid down but some of the methods available
are illustrated in the examples below.

Example 9. Solve the pair of equations xy = 10, 3x 4- 2y = 10.


When one of the equations is of the first degree, either unknown is
easily expressed in terms of the other. Substitution in the second
equation then results in a single equation in one unknown.

For the pair given here, the second equation gives


y = 8 — -x.
Substituting in the first equation

- =
1 *)
which, after multiplication by 2 and slight rearrangement,
can be written
3x» 16x 4- 20 0, - =
or. (3x - 10)(x - 2) = 0.
10 3
Thus ^ — y.
.

and since y —
= 8 -x, the corresponding value of
y is 3 ;

or AT =2 and y = 6.
Example 10. Solve the pair of equations x* 4- 4xy 4- y* = 13, 2x* 4- 3;^ m8
When the two equations are same degree in x and y and when the
of the
separate terms involving the unknowns are
all of this degree the soln-

y « mx and proceeding
tion can be obtained by writing
followl m
With y ^ mx, the two equations become
x*(l 4- 4m 4- m“) « 13 and x*(2 4. 3m) » 8. (1.7)
B
18 PURE MATHEMATICS [I

By division.
1 + 4m + m> 13
^ If’
Cross multiplying we have 8(1 + 4m + m*) = 13(2 + 3m). giving the
quadratic in m,
8m* —
- 18 = 0.
7m
This can be witten (m — 2) (8m + 9) = 0,
so that m = 2 or — 9/8.
The value of x can now be obtained by
substitution in one of equations
(1.7), Choosing the second of (1.7) here as it is rather simpler than the
first, wi =2 gives
x^{2 + (3)(2)) = 8.

= so that AT
1 = i 1. Since y = mx and m = 2, the correspond-
ing values of y are ± 2.
The second value — 9/8 for m gives similarly

*'{2 + (3)(- -®)} = 8,

leading to a negative value for xK There are thus no real solutions


corresponding to this value of m.

EXERCISES 1 (6)
1 . Find t from the equation — 1-324 — 2-890 =
t 0.
2 . Solve V(^ + G) - ^/{x + 3) = V(2^ + 5). (L.U.)

3. Solve the equation x^ + 2x


12

-I-
—=
2;r
7.

4. Solve the simultaneous equations, 2x ^y = 5, x* + xy => 2.

(L.U.)
5. Solve the equations x^ y* = 5, xy = 2.

0 . Solve tlie simultaneous equations,


^ + 2 2(y - 4)
+ 3 =
y - 4 X -{- 2
X -y =
1.5. The square root of (a + \/b)
Simultaneous equations of a type considered in the last section
appear in the calculation of the square root of the quantity a \/d
in which <3 is a rational and f\/b an irrational quantity. Before pro-
ceeding to this calculation we consider two important results in
connection with such quantities.
Firstly, the square root of a rational quantity cannot be partly rational
and partly irrational.
To prove this, let c be a rational quantity and suppose it is possible
that
•v/c = /> -f- ^/q,
1

1] SQUARE ROOT OF {a + ^/b) 19

P being rational and irrational. Squaring


c=p‘^ ^p^q,
leading to = o-p^^q
v'?
2p
this requires that an irrational quantity should be equal to a rational
one, and is impossible.
Secondly, if p '^q a ~
-y/t, where p and a are both rational
and -s/q, ^/h are both irrational, then p a and q h. = =
If
p is not equal to a, let ^ a = +
a. Then
a-fa.-\-^fq = a-{^ y/b,
giving =a -f-

This, by our first result, is impossible, so that p = a. It then follows


immediately that q=b.
To calculate the square root of « ^/b we suppose that
^/{a -h ^/b) = ±(^fx -f -v/y).
Squaring both sides,

« + V* == ^ + ^V{^y) + y-
Using the second of the above results, we have
x+y = a,
2V(ay) = ^/b.
The second of these can be written 4*y = 6 and we have therefore
only to solve the simultaneous equations

4xy =b
in order to find x,
y and hence and -s/y.

Example 11. Find ike sqxtare root of 14 + 6\/6. (L.U.)


+ C-/6) = ±
V(i4 + Vy).
Squaring, 14 + 6V6 « x + 2V{xy) + y.
Hence x y = U. 2V{xy) = Q^/5,
The second equation gives Vixy) = 3a/ 6 or irv » 4.R
From the first
equation we have
quadratic equation
y i 14 2 substiSTnly^ = w'X's tte
x(14-x)=45.
This can ^
be written
ar* - + 45 =. 0,
14#
(* - 9)(» - 6) = 0.

i'u.a . i‘ sr-T, rr".£?xn;rj"?r.'" “•


PURE MATHEMATICS fl

EXERCISES 1 (0)

Find the square root of 5 + 2\/6-


Express the square root of 18-12\/2 in the form ^Jx — y/y where
X and y are rational. (L.U.)
Find the square root of a + 6 +
Find rational numbers a and b such that
3 + V2 = {a + by/2){G - V2)*. (O.C.)

EXERCISES 1 (d)

Show that for all real values of y, the expression


3y2 ^ 2y - 1
>^TyT2~
always lies between — 4/7 and 4.

Find the range of values of x for which


x{x - 2)
> 2. (L.U.)
X -{• G
Find the values of X for which the equation
lOx* 4x + 1 = 2}jc{2 - x)
has equal roots. (L.U.)
Find, in its simplest rational form, the equation whose roots are
VV/(V'7 ±
V5). (L.U.)
Show that the roots of the equation 2bx^ + 2[a + b)x + 3a = 26 are
real when a and b are real.
If one root of this equation is double the other, prove that either
a = 26 or 4a = 116. (L.U.)

If the roots of the + bx + c = 0 are a, and the roots


equation x*
of the equation x^ + ?.bx 4- X^c — 0 are y, d show that the equation
whose roots are ay + pd and a6 + is

- ?M^x + 2A*c(6= - 2c) = 0.


Show that the roots of this equation are always real. (L.U.)

The roots of the quadratic equation x^ — px -h (J = 0 are a and p.


Determine the equation having the roots a* 4- and 4-

e.xpressing the coefficients in terms of p and q. Prove further that


if p and q are both real, then this equation can have equal roots

only it p= G or p^ = 4^. (L.U.)

Prove that the roots of the equation


(A 4- 3);r* 4- (6 - 2k)x 4- A - 1 = 0
are real if, and only if, A is not greater than 3/2. Find the values
of A if one root is six times the other. (L.U.)

If the equation a^x^ + 6a6;r + ac 4- 86^ = 0 has equal roots, prove


that the roots of the equation ac[x 4- 1)* = ^b“^x are also equal.
(L.U.)
.

1] EXERCISES
10 . For what values of X has the equation — 3;v + 2 = X{2x — 6) two
equal roots ?

11 . The roots of the equation x* ax 0 are a, p. Find the b =


equation whose roots are pa+ qp, pp+ ga. If the original equation
is — 44r — 5 = 0 find the values of p/q in order that the new
equation shall have one zero root.
12 . Form the equation whose roots are the cubes of the roots of the
equation — 3^ 4 = 0, without solving the equation, giving the
+
numerical values of the coefficients of the new equation. (L.U.)
13. Show that if + bx + c = 0, x* px
the equations x*
-i- q = 0 have
a common root, then {c - q)^ = (6 _ p)[cp - bq). (L.U.)
14. Solve the equation + 2;r)» + 8 = 9(3jr» + 2x),
(3;(r*

16. Solve the equation ^/{x — 6) + 2 = ^/{x 4- 7).

4) - V(;tf - 3) = 3.
16. Solve the equation ^/(Zx -f- (L.U.)
Solve the simultaneous equations,

4 y ^ I
~ 2{y 4 1) 2
^ (L.U.)

Solve the simultaneous equations, jr


^ =
IS.
4 6, x^* 4 2xy — 35 = 0
19. Solve the simultaneous equations,
X y 5
*’ + 2>'’ = 9-

20 . Solve the simultaneous equations,

^+^
Find u and v from the equations.
= 1. * ^y^2a. (L.U.)

« 4 w

^^ + 5:^ =
X
9. 1+1=2.
y X y 4

equations

If a, b. c, d are rational numbers, and if neither b nor

r >“ + « <f

..
is a rirfret

^/b' (L.U.)
Express
1 4 y/Z
(V3 - 1)»'
m the form a 4 where a, 6. c are rational.
25. By (L.U.)
putting solve the equation
24f* - 9^» 4 14;r* - 9;e 4 2 - 0,
CHAPTER 2

INDICESAND LOGARITHMS. THE REMAINDER


THEOREM. THE PRINCIPLE OF UNDETERMINED
COEFFICIENTS. PARTIAL FRACTIONS
2.1. The fundamental laws for positive integral indices
When a quantity a is multiplied by itself any number of times the
product is called a power of a. Thus a x a is the second power of
a and iswritten a^. The number expressing the power is called the
index. Thus the index of is the number 2. Generalising, we have
the definition that if m
a positive integer, a”* denotes the product of
is
m factors each equal to a. We give below three fundamental laws for
the combination of indices. In all cases the indices m and n are
assumed to be positive integers, and in (ii) we assume that m n >
(i) X (2.1)

By definition, a^ = a. a. a . . . to m factors and fl” = a. a. a . . .

to n factors. Hence a”* x a" = a.rt.a . . . to {m n) factors,


_ l3
y definition.

(ii) (2.2)

From the definitions of a”* and

^ ~ ^
= — = a.a.a
a”* . . . io m factors
a^
a'^ a.a.a ... to n factors

= a.a.a ... to (»i — n) factors =


(iii) (a»')” = (2.3)

= a’".a’".a"^ ... to n factors


= (a.a.a ... to m factors) (a. a. a ... to nt factors) . . .

the bracketed terms being repeated n times, so that


(^m)n ^^ ^ a ^ ^ to tnn factors = a"*".
2.2. Fractional, zero and negative indices
It is convenient to have available fractional, zero and negative
indices and for one set of laws to apply in all cases. However, the
definition of a"* as the product of fn factors each equal to a is clearly
meaningless except when is a positive integer. We introduce
fractional and negative indices by determining their meaning
when
the first fundamental law a"* x fl" is true. =
It is then possible

toshow that with the interpretations arrived at on this basis the other

two laws of §2.1 remain valid.


22
2] INDICES 23

The interpretation of p and q being positive integers


Since the first rule of §2.1 is to be true, X
Similarly X a^^^ x X and so on. Hence
flP/« X aP^^ X aP^^ to q factors = = aP,
This implies that {op^^)^ = aP, and taking the ^h root
flP/9 = (2.4)

i.e., that is the ^h root of aP.

The interpretation of a®

Since x = a"»+" is to be true for all values of m and n we


can take w=0 and hence
a® X a” =
giving, rt® = d"/a" = 1, (2.6)

i.e., any quantity with zero index is equivalent to unity.

The interpretation of ar^


The rule a"* x is to hold for all m, n and we can therefore
take m — — «. The rule then gives

fl-" X = A -«+" = A® =
A" 1,

giving, A-" = /a", 1


(2.6)
showing that a“” is the reciprocal of a".
With these interpretations it remains to show that the two laws
A*" -f- A" = A™-" and (a”)" = A">" remain true for all
values of m and n.
To prove the first we have

a" A" = A”* X —


a"
= A"* X A“» since a“" means 1/a",
= A"*-«, by the fundamental law.
To show that {a«)" = a"*" for all w and « we take
to be unrestricted and consider in turn the cases
the value
of m
in which n is a positive
mteger, a positive fraction and any negative
and positive mtegral n,
quantity. For any
^ m
(a")» = . . . to n factors
zss ... to n terms ^
,
- W”''*. ~ i‘- ? are positive integers
{a ) Now
the }th power of (a«]Tlt is {(«")?/«}»
or (a"*)i>’
Tills IS a'v. '
Hence we have, on taking the jth root^
24 PURE MATHEMATICS [2

I'inafly, for unrestricted m and n any negative quantity, we replace


n by - - X. Then

= — = -

Hence, with the interpretation of fractional, zero and negative


indices given in equations (2.4), (2.5) and (2.6) the three fundamental
laws for combination of indices given in (2.1), (2.2) and (2.3) remain
valid.
2b-^x^
Example 1. Express with positive indices, (i) and
7c^’
evaluate (iii)

2b-^x^ ^V(y-q y-r/3 1


^

lc-*y^ Ib^y^’ >-2/3 ^f/3+2/3*

Example 2. Show that {xy)^ = x^y^ for all values of tt.

If na positive integer, {xy)'^


is — xy.xy.xy ... to « factors. This can
be wTitten as the product
{x.x.x ... to « factors) X {y y-y ... to n factors)

which is

If n a positive fraction, say p/q where


is p and q are positive integers,
the ^th power of (^xy)?/^
= {{xy)P/9}^ = {xy)P = xPyP = (xP.^yP/^)^.

Taking tlie ^th root, {xy)P’‘i = xP.'<lyPl9.

If n is any negative quantity, say — A.

= ’‘"y"-

EXERCISES 2 («)
1/2
1. Express with positive integers (i) 4.r, (ii) *Vy^ ^
, ..
/8\-U3
2. Evaluate (i) (64)“3-^ (ii) •

(L.U.)
3. Simplify {x*y::-Y x V{x-^y^ 2 ) {xc)'^^-.

— a~*
4 . Prove that (a — + a “‘^) = - 1/3
(L.U.)
a
x'x/2 xy
Evaluate
5.
xy -y
Simplify -- - .
„ , i ^ 2
LOGARITHMS 25

2.3. The theory of logarithms


The logarithm of a positive quantity to a given base a is defined
as the index of the power to which the base a must be raised to make
it equal the given quantity N. Thus

= N, (2.7)

when X is the logarithm of N to the base a.

X is written X == loga N
and the two formulae (2.7), (2.8) are equivalent statements expressing
the relationship between x^ a and AT.
If we substitute for x from (2.8) in (2.7) we have

fl'og. -V = AT,
a result which is often useful.
If we set jc =0 in (2.7) we have iV = a® = 1. The equivalent
formula (2.8) gives in this case

logal=0, (2.10)
so that the logarithm of unity is zero. If we put =1 in (2.7) N ~ a,
and formula (2.8) then gives
logafl = 1. (2.11)
or, the logarithm of the base itself is unity.
To find the logarithm of the product of two positive numbers
M and N, we have, using (2.9),

MN = ^ .
a'og- ^
Hence, by the definition of a logarithm,

logo MN = logo M -f logo N, (2.12)


showing that the logarithm of a product of two positive numbers is the
sum of the logarithms of the separate numbers. Similarly
logo MNP = logo M + logo N -f logo P,
and so on for products of more factors.
For the logarithm of a quotient of two positive
2v» equation (2.9) gives
quantities M and
M ~~ ^log, M - log,
~N
showing that

log. = log. M N,
^ log. (2.13)

\.e..lhe logarithm of a quotient is


the difference between the logarithm
of ihc ftiiPPCTdioT dnd that of the dcttotniiMiof^
26 PURE MATHEMATICS [2

The logarithm of a positive quantity raised to a power can be


found similarly. Thus, again from (2.9),

Mp = (aiog«-^f)p =
giving loga Mp = p logo M. (2.14)

If, in (2.14) we write p = \/r,

Iog„ M^' = - log. M. (2.15)


r

Thus the logarithm of the pi\\ pon'er of a positive quantity is p times the
logarithm of the quantity, (2.14), and the logarithm of the rth root of
a positive quantity is l/r times the logar ithm of the quantity, (2.15).

Example 3. Prove that hgt, N= r x toga N.

Let X = logft N so that = N. Taking logarithms to base a,

loga(^>^) = logo N,
giving, X log^,6 = logo ^
or, ar = -J—- X logo AT.
togab
The proved here is of importance in that it relates logarithms
result to
different bases. It shows that to transform logarithms from base a to
base b we have to multiply by the quantity l/(loga6).

( 2b fc‘\
Example 4. Prove that 2 togg (a + b) = 2 log^ a + logc (
^ *1*
Ji/
(L.U.)

2 logc (rt + fc) = logc {a by = logf + 2ab + 6*)

/ 26 6 *\
= logc + log,(^l + — + -j
/ 26 6 »\
= 2 logc a + logf(^l +— + --jj-

2.4. Common logarithms


The logarithms used in everyday calculations are those with
base 10. Such logarithms are referred to as common logarithms and
the base is often omitted in written work. Thus log 24 is generally
with
taken to mean logjg 24. The student is assumed to be familiar
very few examples will be
their use in arithmetical work and only
given in this section. Examples involving the use of
common
chapters
logarithms occur throughout the book, particularly in the
on Trigonometry.
2] INDICES AND LOGARITHMS 27

Example 5, Calculate log 5 and 0*125 given that log 2 = 0*3010.

log 5 = log = log 10 — log 2 = 1 — log 2, since log 10 = 1 (by

2.11). = 1 - 0*3010 = 0*6990.


log 0125 = log = log 1 — log 8 = — 3 log 2, since log 1 = 0 {by

2.10), = — 0*9030 = 1*0970, the last form meaning — 1 4- 0*0970.

Example 6. Given (hat log 3 = 0*4771, find the number of digits t» the

integral part of (v'3)**.


89 89
log (*s/3)« = log 3 = X 0*4771 « 21*23
y -^ . . .

Thus 10 has to be raised to rather more than the twenty-first power to


give (\/3)®® and this quantity will therefore contain 22 digits in its
integral part.

2.5. Equations in which the unknown occurs as an index


When the unknown quantity
an equation occurs as an index, in
the laws of combination of indices and the use of logarithms usually
enable the solution to be found. Some of the artificies used are
illustrated in the following examples.

Example 7. Solve the equation 2** 1G*-*1. (L.U.)


Since 16 = 2*, the equation can be written
2** {2‘)®-i =
2-t*—*.
Hence x* = — 4, or *• — 4.t + 4 = 0,
giving, _ 2)« =. 0, so that x 2.

Example 8. Find x from the equation 32* s= 6*-fl,


Taking logarithms.
2x log 3 =* (r 4 1) log 5,
so that (2 log 3 - log 6)* « log 6.
Hence
No. log.
logs 0*6990
X =
2 log 3 — log 5 2 X 0*4771 - 0*6990
0*6990 1*8446
0*2552 1*4068
0*6990
=* 2*74.
0*2552 0*4377

Example 9. Solve the equation 52* _ gx+i 4 4 -- o.


This can be written
(
6*)« - 6 ( 6*) 44 = 0,
which in factor form is
No. log.
(5*- 1)(6*- 4) = 0.
0*6021 1*7797
Hence either 6* =leading to ar s= 0 [by
I
(2.5)], 0*6000
or 6* as 4. 1*8445
Taking logarithms this gives
* log 6 =: log 4, X i*9362
60 that
log 4 0*6021
X
log 6 0*0990

28 PURE MATHEMATICS [2

EXERCISES 2 (6)

1. If a = logfr c. b = \ogc a. c = logo b. prove that ahc = 1 (L.U.)


2. Without using tables, show that
log -v/27 -f log V8 - log VI25 _ 3
log 6 — log 5 ~ 2“ (L.U.)

5.
3. If log^ 10-24 = 2, find (L.U.)
4.
6.
Using logarithms, evaluate the following :

7. 1-405
1
\
8. * (Q-E.)
(
Solve the equations
(a) log (;*'* + 2x) = 0-9031, (6) (2-4)* = 0-59. (L.U.)
Find x from the equation 3* = 0-832. — 3~*
Solve the equation 2^+^ — 32(2')+ 1=0.
Solve the simultaneous equations 2'+^^ = 3' = C{2^).

2.6. The remainder theorem


The polynomial expression
+ CyX^-'^ + -I- , . . -f Cn-iX -h Cn, (2.16)

can be written
Co(.i;" — a") + -t- C2 {x”- 2 _ an-2) Cn-i{x — a)

+ CqU** + + + Cn-ia +
. . (2.17)

Since, as can be verified by actual multiplication,


— a"* = {x ~ a){x”^-^ + ax”*-^ -h a^x^~^ -[-••• + -f

each of the terms in the first line of the expression (2.17) is divisible
by X ~ a. Hence we can write
C(^ + -p . . . -f c^^^x + Cn
= a multiple of (x — a) CqO^ + + ^n* + • • • +
Hence, the remainder when a polynomial expression is divided by (x — a)
is obtained by wrilhig a for x in the given expression. This result is
known as the remainder theorem and it enables the remainder to be
found without having to perform the division.
An alternative proof of the remainder theorem can be given as
follows. Let P(x) denote a pol 3 'nomial expression in x, let Q(x) be
the quotient when P(x) is divided by (x — a) and let R be the remainder.
Then, for all values of x,
P{x) = {x- a)Q{x) + R,
and R is independent of x. Putting x = a we have
P{a) = R,
since the first term on the right hand side vanishes because of the
UNDETERMINED COEFFICIENTS
factor {x — a). Hence the remainder is obtained by writing a for x
in the given expression.
An immediate and important consequence of the remainder
theorem is that if a polynomial expression in x vanishes for a certain
value a of x, then (x —
a) is a factor of the expression.

Example 10, Find the value of k if the remainder when the polynomial
2x^ + ftx^ - lU* + 4jr + 12 is divided by {x 3) is 60. (L.U.) -
By the remainder theorem, the remainder after division by {x — 3) is
obtained by writing ;r =
3 in the expression 2x* hx^ 1 lx* -f 4:r + — + 12.
The remainder is therefore
2{3)« -t- A(3)»- 11(3)» + 4(3) + 12,
or, 162 + 27A - 99 + 12 + 12.
This reduces to 27ft -f 87, and setting it equal to 60 we have
27A + 87 = 60,
^ving A =— 1.

Example 11. Factorise a^b — c) + 6*(c — a) + c*(a — b). (h.U.)


If we a = A the given expression vanishes in other words, there
set
; is
no remainder when the expression is divided by {a b). Hence (a — — fr)
is a factor. Similarly (6 — c) and (c — a) are factors.
The given expression is of the third degree so that, beside (a —
6),
(ft —
c), {c —
a), there can be no further factor involving
a, b or c.
There may however be a numerical factor so we write
a*(ft - c) + ft*(c - a) -f c*(a -
- b)ib - c){c - a), 6) = N{a
where N is the numerical factor. To determine N we can give
a, b and
c any values we find convenient. Choosing a =
0, 6*=1, c aJ 2 the
left hand side becomes — 2 while the right
hand side is 2N. Thus
= — 2 giving N = — 1 and the required factors are
— (a — b)(b — c)(c — a).
2.7. Theprinciple of undetermined coefficients
We start by showing that if a polynomial expression of degree
^
in X vanishes for more than n different
n
values of x, the coefficients of
each power of x must be zero. We write
P(x) CoX” = c^n-i + . + + . . ^
and suppose that = 0 when equals each of the unequal values
a;
«!, oq,. Then (* - ai), {* - a,), {x - are all factors . . .. oq,)
of P(x) and we can write
^ “ *i)(^ — aa) , . . — otj.
p be another value of x which makes P(x) vanish, then
— ai){/5 — aa)
CoiP . . . — 0,
and since none of the factors (5 —
a»Wd — » 1 ta ~ \ • v
.0 U.ust vanish. The expres^on pfe^wt'd^ces
if
P{x) = + c^- + c.
PURE MATHEMATICS
and since this vanishes for more than n values of x we can show
similarly that = 0. In a similar way we can show that each of
the coefficients C2, C3, . . must also vanish.
We can now show
that if two polynomials of degree n in x are
equal for more than n values of x, they are equal for all values of x.

If we suppose that the two expressions

V" + + C^X^-^ -f . . . + Cn-iX + c„.

+ d^x^-'^ + d^x*^-^ + . . . + d^_iX + d^,


are equal for more than n values of x, then the polynomial
(Co — do)x” + (Cx — di)x”-'^ + . . . + (c„_i— dn-i)x +
vanishes for more than n values of x and therefore all the coefficients
must be zero. Hence
^0 ^0 • • • Crt-l ~ ^n-l —
leading to
^0
“ = ^ip • •» ~ ^n-l>
Tlie two expressions are thus identical and therefore equal for all
values of x. Hence we have established the important result that ij
two polynomial expressions in x are identically equal we may equate the
of the like powers of x.
coefficients
The result remains valid if the two expressions are not of the same
degree. For example if one is of degree n and the other of degree
« — 1 we should have
V" + 4- . . . + Cn-iX + Cn
= d^X^-'^ 4- d^x^^^ 4- ... 4- X dn
and hence
Cq = 0, Cx ^ di, C2 — ^^2’ • • •* ^n — 1 ^n — 1*
The result given above is often called the principle of undelermitted
coefficients and it has important applications. Some examples of its
use are given below.

Example 12. Find constants a, b, c such that

2x^ — Oat + 14 = a{x — l)(x — 2) + b{x — 1) + (E.U.)

The sign =
used to denote identity between two expressions.
is
When
two expressions are separated by such a sign we can equate the coefficients
of like powers of the variable. Here we have
— 9^ + 14 = a{x^ - ^x + 2) + b{x — \) + c
s ax^ - (3a - b)x 2a ^ b c.

x and the term independent of x in turn


Equating the coefficients of x*,
gives
a = 2, 3a —6= 9, 2a — 6 +c= 14.

Substituting a =
2 in the second equation we have 6 fc 9 gmng — =
6 = - 3 and substituting a 2. i =
3 in the thLrd equation, = -
4 + 3 + = c 14 leading to c 7. =

2] PARTIAL FRACTIONS 31

Example 13. Find ihe relation between q and r so that + Zpx* + *t-

shall be a perfect cube for all values of x.

Let x^ + Zpx* + + r = (ar 4- a)’


^x^ + 3ax* + Za^x + a*.

Equating the coefficients of x and the term independent of x, = q,


a’ — r. Cubing the first, squaring the second and dividing we have
27a«

giving q* = 27r* as the required relation between q and r.

EXERCISES 2 (c)

1. Find the values of p and q so that [x + 1) and {x — 2) shall be factors


of x^ + px^ + 2jr + What is then the third factor ?
2. Use the remainder theorem to find the factors of
(a - by + (6 - cy + (c - ay,
3. Find the values of X and fi if the expression
Zx^ + Aar® 4- 12;ir* ^x ^
is exactly divisible
(i) by {x — 1) and (ii) leaves remainder 18 when
divided by [x 4- 2).
4. Find the values of a and b in terms of n if

(;*f —» 4- 1)* — (rif — n)* = a^r* 4- 4- 6


for all values of x, (L.UO
6. Find the values of A, B and C if the expression
Ax{x - 2){x 4- 3). 4- Bx{x - 2) 4- Cx{x 4- 3) 4- (x - 2)(x 4- 3)
has a constant value for all values of x. (L.U.)
6. If 4^® 4- kx* -{• px 2 is divisible by x* 4- A*, prove that kp = 8.

2.8. Partial fractions


The student will already be familiar with the process of simplifying
a group of fractions separated by addition
or subtraction signs into
a single fraction. For instance the expression
1 1 2x
X 2 X 2 x^ 4t

can be simplified to give the single fraction 16x/(x^


16) in which -
the denominator is the lowest common denominator
of the separate
fractions. It is often desirable, for instance in expansions
and in the
integral calculus (chapters 3 and 13), to be able to
perform the reverse
proc^. In other words, we require to be able to split up a
single
fraction whose denominator has factors into
two or more partial
fractions,
revere process, the resolution into partial fractions, depends
on the following simple rules :

(i) If the degree of the numerator of the given fraction is equal to


b '

32 PURE MATHEMATICS [2

or greater than that of the denominator, divide the numerator


by the denominator until a remainder is obtained which is of
lower degree than the denominator.
(ii) To ever}' linear factor like {x a) in the denominator there —
corresponds a partial fraction of the form A/{x a). —
(iii) To every repeated linear factor like {x aY in the denominator —
there corresponds two partial fractions of the form A/{x a) —
and B/ix —
a]\ Similarly for factors like {x a)^ we have three —
partial fractions A,— a)- and C/{x — a)^ and so on.
{x — a), B/(x
(iv) To every quadratic factor like x- + ax A- there corresponds a
partial fraction (Cx + D)/{x^- + + Repeated quadratic b).

factors require additional partial fractions as in (iii) above.


Thus
a factor {x- A- ax -^r b)- would require partial fractions
(Cx + D)/{x^ + ax -f- b) and (Ex -[- E)/(x^ + ax + b)K
The application of these rules is illustrated in examples 14 to 1*3

below.

Example 14. Resolve into partial fractioyis 5/[x* + jr - 6).

The factors oi ^ x ~ Q being (x + - 2) we assume


3)(^ that

5 A B A{x - 2) + B{x + 3)
^

_i_ , 6 ^X+ 3 - 2 ^ (X + 3)(4r - 2)

the expressions on the left and right being the


The denominators of
same, the numerators must be the same. Hence
we have
Aix -2) + B{x + 3) = 5. (2.18)

identity by applying the P/i^ciple of


A and B can be found from this
of x and tne
undetermined coefficients. Thus, equating the coefficients
term not containing x, we have
A B= 0 and — 2A + 3B = 5.

— ®
The solution of this pair of simultaneous equations
is 1. ^
we ^have here, ratter
Another and, in the case of linear factors such ^
simpler method of determining A and B from
the identity (- • .

values so that A and B can be found separately.


give r suitable numerical =1
Thus by putting v = in we have (2 + 3)B = 5, giving B
giving rl = - I-
2 (2.18)
and by putting ;r = - 3, we have (- 3 - 2)A = 5.


5 1

Henco X ^ 0 X — 2 x + 3

9
- into partial fractions.
Example 15. Separate
(X - l)(x + 2)

+ the correct assumption is


Here, because of the repeated factor {x 2),

9 A B C
+ + +
(x-l)(x+2)* x-1 (X -)

A{x + 2)* + B{x


= (X - l){x + 2)»

This identity requires that


- + 2' + C(v - 1) = »•
Aix + 2)> + Bix I)(r
2

2] PARTIAL FRACTIONS 33

A can be found immediately by taking x = 1. Thus (1 + 2)M = 9


giving A = 1. To find C, take 2 and we have (— 2 — 1)C = 9 =—
leading to C = —
To find B we can equate the coefficients of, say,

113
3.
ar*. This gives A + B = 0 so that, as ^ 1, B 1. Hence = = —
9
“ x~- “ + 2 “ (i + 2)*'
- l){x + 2)* 1 jr

Example 16. Resolve \^x/[x* — 16) into partial fractions.


The factors of the denominator are {x — 2). {x 2) and (t* + 4). In
view of rule (iv) and the quadratic factor {x* + 4), we assume
IGx A B Cx +D
- IG x-2 x+2 + 4
_A{x+2){x^ + 4.) + B(;r-2)(;r* + 4) + (C;r+ D)(;r-2)(* + 2)
{;r.-2)(,r+2)(*»+4)
This requires that

Mx + 2){x^ + 4) + B{x - 2){x^ + 4) + {Cx -f D)(x - 2)[x + 2) = 10^.


Putting X 2,

+
(2 2)(2* + 4)A = 16 X 2,
giving 32^ = 32 or ^ = 1.
Putting =s 2,

{- 2 - 2){(- 2)* + 4}B = 16 X (- 2).


giving - 32B =- 32 or B= 1.
Equating coefficients of ;r», yf + B
+ C = 0, or, since ^ ^ B = 1,
^ = — 2; and equating coefficients of the term independent of x

8B — 4Z) = 0, which with A *

Hence
x*~lQ
B
^
I, gives D = 0.

x~2^ x-\- *«
^J!_ *
4+
Example 17. Separate xy{x* - 3x + 2) into partial fractions.
In our examples so far, the numerator of the given fraction
all
has been
of lower degree than the denominator. Here the numerator is of the
third while the denominator is of the
second degree. Dividing
X - 3.r + 2 into we find that the quotient is x
remainder is lx — 0. Hence
3 and that the +
x^ - 6
x^ — 3x -^2
-^+3 + ~ 3x + 2*
We now proceed to separate (7x - <!,)/(,• - 3x + 2) into two partial
fractions as in Example 14. Thus we assume that
7x - 6 A B A{x~ 1) 4-B(x-2)
#• — + ~ +
3.^ - I
2 [x — 2)(.r — 1)
X 2 X

Hence a{x - I) + B{x - 2) s 7x - 6.


Setting ^ = 1 we find that B = - 1 and taking
FinaUy wo therefore have
x^2 leads w ^ =
to /i 8
8.

X*
x*-3;r+ 2“^ + ^ +
O

34 PURE MATHEMATICS [2

EXERCISES 2 {d)

Resolve into partial fractions :

5 {x + 1 ) 10 - 11 .*'
5
1 .

(N 1H .

{X -4 )~{x^ + 1 )*

2.
x^
6
3 x^ +
x^ + X — 2
‘ .

- 81
*

x^ 2 ^* - lU + 5
3. 7
2 x - 5)(;r -
.

{X + 1
)^-
{x^ 4-

X^ + 1X 2y + 1
4 . * 8
x^ + 2X + 1
.

(y + I)*!;- - 2 )*

EXERCISES 2 {e)

1 . Simplify the expression 6 x 43” + ^ — 20 x 8-'*.

2 . If a = 2, 6 = 3 show that {a^b^c^)^ = 144v^2c».


3. Simplify Q^n + 2 ^ (35n X 6 X 4«-2)
82/3 + 43/2 \/a^ X -^'b^
=
4. Evaluate (i) (ii)
V--/
when 6 3.
103/4

5 . If logu n = X and log<.n = y, where tt 1, prove that


X -y _ c - a
~ logf, logfc

X y logfr c +
logft a

Verify this result, without using any tables, when a = 4, 6 = 2.

c = 8. » = 4096. (L.U.)

6 . Using logarithm tables evaluate (i) (0-0371)ra, (ii) log, 0-05. (Q-E.)

7 .
+ ^) = logo (I + -jL) = m and logo (1 + ^) =
If loga (1 I,
show
that logo (1 + -gL) = — m — n. I
(L.U.)

8 . Find X from the equation 9* — 12(3^) + 27 = 0, (L.U.)

9 . Solve the equation 4^ + 2 = 3 x 2-*. (L.U.)

10 . If y = a + bx** is satisfied by the values

X = 1 2 4

y = I
7 10 15

show that n = loga (5/3) and deduce the values of a and 6. (L.U.)

11 . If 2 logs N= p, logj 2iV = q, q — p = ‘i, find N. (L.U.)

12 . If = logo {he), y = log* {ca) and = \oge


2 (a6) prove that
X +y 2 = xyz - 2.
13. Find the values of a and 6 if the expression 2x^ - 15.ir* -h ax + 6 is
divisible both by ;r - 4 and by 2x - 1.
(O-C.)

14. A polynomial expression Pix), when divided by [x - 1) leaves a


remainder 3 and, when divided by (x — 2) leaves a renw-inder
1.

SIiow that when divided by {x - l){x - 2) it leaves a


remainder
(O.C.)
- 2.r h 5.
2] EXERCISES 35

15. Express a*(6 — c) + b^{c — a) + c\a — 5) as the product of four


factors. (L.U.)
16. If = a{x + -[)[x + 2){x + 3) + b{x + l){x + 2) + c{x + 1) + tf.

find the ^numerical values of a, b, c and d. (L.U.)


17. Show that x^ Ox ~~ 10 can be expressed in the form
[x — a){x — ^) + 2{x — a )
4- 3;»r

in two different ways and find the values of a and j5 in each case.
(L.U.)
18. Find a and b so that
X* - 7x^ + 17;»r* ~ l7x + 6^ (x - l)^(x* + ax + b).

Hence find all the factors of the quartic expression.


19. Find the value of if the expression
2x^ xy ~ Qy^ + 4x +y +c
can be expressed as the product of two linear factors.
20 . Use the principle of undetermined coefficients to find the square
root of the expression x* + 4;r* + 8^* + 8^ + 4.
5^ + 3
21. Separate mto partial fractions.
{X + l)*(2;r + 1)

X* -- X - I
22. Express in partial fractions.
x^
6;r» 4- 2x* 4- 5x
23. Express
- 1
m
.

partial fractions. (Q-E.)

1 -\-x*
24. Express in partial fractions
(1 + *)(1 + *»)• Q -®-)
(

Use the remainder theorem to


25. find the three factors of ** + 3;r» — 4
and hence resolve
2;r* - - 7;r - 14
xi + 3.r» - 4
into partial fractions.
CHAPTER 3

ARITHMETICAL AND GEOMETRICAL PROGRESSIONS.


PERMUTATIONS AND COMBINATIONS.
THE BINOMIAL THEOREM
3.1. Series
A set of numbers each which can be obtained from some definite
of
law is called a series or progression. Each of the numbers forming
the set is called a term of the series. Thus the sets
(i) 1. 3, 5. 7
(ii) 1. 2, 4, 8
(hi) D, 22 32 42 , ,

are In the first set, each number is obtained by adding 2


all series.

to the preceding one, in the second each term is twice the preceding
one and in the third each number is the square of successive integers.
It is possible to give a formula for the general or «th term of each
of the above series. Thus for (i) the «th term is 2n 1, for (ii) it —
is 2””^ and for (iii) it is n^. If an expression for the «th term of
a series is known it is possible to write down successive terms by
giving successive integral values to n. Thus the series whose «th

term one whose terms are 2, these


is is fy, fff, . . ..

being the values obtained by putting n 1, 2, 3, 4, . in the = . .

formula for the «th term.


in
Series play a very important part in mathematical analysis ;

ones.
this chapter we shall be concerned with a few of the simpler

3.2. The arithmetical progression


A series in obtained from the preceding one
which each term is

by adding (or subtracting) a constant quantity is called an arithmetica


progression (.\.P.). Thus the series
1, 3, 5, 7, , .

a, a d, a 2d, a + Zd, . . .

are arithmetical progressions. The difference between each term and


the preceding one the common difference. When three
is called
in arithmetical progression the middle one is called
quantities are
mean of the other two. Thus a is the arithmetic mean
the arithmetic
between a d and a— d.

In the scries
a, a d, a 2d, a 2d, . - 1 1 •

one less than the number of the


the coefficient of d in any term is
30
; .

ARITHMETICAL PROGRESSION
term in the series. Thus a M the fourth term. If then the
is
series consists of n terms and I denotes the last or «th term
l^a + {n-l)d. (3.2)
To obtain the sum s» of n terms of the series (3.1) we have
Sn^ a + {a + d) + {a + 2d) - 2d) ^ - d)
{I {I
1,

for if I is the last term, the next to last will he — d and the I pre-
ceding one will be I — 2d and so on. If now we write the series in
the reverse order
Sn = + - d)
I {I {I ~ 2d) + {a + 2d) + {a + d) -h a.
Adding and noticing that the sums of terms in corresponding positions
are all a / we have

2sn = (a -h Z) 4- (^^ -f- (a -j. /) -j. , . . to « terms,


= «(a + 1).
Hence
n
(3.3)
or, using (3.2),
n
Sn = -{2(1 4- {» — l)(f}.
(3.4)

Example 1. Insert seven arithmetic means between 2 and


26.
It is ^ways possible to insert any number
of terms between Uvo given
quantities such that the resulting series shall
be an arithmetical pro-
fession. Terms inserted in this way are caUed arithmetic
m^ansran
“«amng of an arithmetic mean between two
given

number
of terms will here be

W^^ch the
^ arithmetical progression of nine terms
of
first is 2 and the last is 26. Le/d he the comnT dSeTnce!
26 = 9th term =2 4- 8d,
so that d =3 The second term is therefore 2 4- 3 or 6 the third
required means are 6. 8,'
20^and°23 11, U, 17,

“umbers in arithmetical progression


be a - d. a and
Thtir prjrt^
o(o* - d*) ^ 604.
S'™™ ;;S 'i,-
;» -±
j;

Here 0 = 26, a + 2d - = -
19, so that 2d 19 a - 19 _ 26 - _ »
38 PURE MATHEMATICS [3

giving i = —
3. With 5„ = 82. formula (3.4) for the sum of n terms of
the series gives

S2 .= ^{50 + fft - 1)(- 3)}

as the equation for the number of terms (u). This reduces to the
quadratic equation
3„2 _ 53 „ _{_ 164 = 0,
or (n — 4}(3« — 41) = 0.

Hence « = 4or« = ^= 13|. The fractional result means that the


sum of 13 terms will be greater than 82 and that of 14 terms will be less
than 82.

3.3. The geometrical progression


A series in obtained from the preceding one
which each term is

by multiplying {or dividing) by a constant quantity is called a geo-


metrical progression (G.P.). Examples are
1 I
*±p O, m •

a, ar, ar^, ar^, .

The ratio between each term and the preceding one is called the
common ratio. When three quantities are in geometrical progression
the middle one is called the geometric mean between the other two.
Ihus a is the geometric mean between ajr and ar.
In the series,
a, ar, ar^, ar^, . .
(3-5)

the index of r in any term is one less than the number of the
term
in the series. Thus ar^ is the fourth term. The last or «th term of
the series is given by
/ = (
3 0) -

To obtain the sum Sn of n terms of the series (3.5) we have


a ar ar^ + . . . +
Multiplying throughout by r
rsn ^ ar -j- ar^ -h ar^ -h . . . + ar^-^ + ar’*.

the terms on the right hand side except a


and af
If we subtract, all

cancel in pairs. Hence Sn ^ — leading to

all - r")
1 -r ’

Example 4. Insert three geometric means between 162 and 1250.


insert any number
As with arithmetical progressions, it is possible to
the resulting series s la
of terms between two given quantities such that
referred to as geomertc
be a geometrical progression. Such terms arc
geometric mean between two given quan i
means, an extension of a
and last terms, the number of terms wiU be
Here, including the first

[3 GEOMETRICAL PROGRESSION 39

five, so we have to find a geometrical progression of five terms o£ whic!»


the first is 162 and the last is 1250. Let r be the common ratio. Then
1250 = 5th term = 162r*.

Hence r* = 1250/162 = 625/81, so that r .5/3. = ± The second term


is therefore {±
5/3) X 162 or ±
270, tlie third is (± 5/3) » X 162 or
450 and the fourth is (± 5/3)® X 162 or 750. ±
Example 5. Fitid three numbers in geometrical progression such that their
sum is 39 and their product w 729.

Let the required numbers bc-a/r, a and ar. Then their product is a*
and hence a® *= 729 giving a = 9. Since the sum is 39, we have
9
- + 9 + 9r = 39,

so that 9r* 30r — + 9 = 0. This can be written 3r» — lOr + 3 = 0


or (3r l)(r 3) — = 0, giving r = 1/3 or 3. The required numbers
are therefore 9/3, 9 and 9 X 3, or 3. 9 and 27.

Example 6. F ind (he sum of ten terms oj the geometrical series 2, — , , ,

Here the first term is 2 and the common ratio is — 2. Hence in the
formula (3.7), a = 2, r =
~ n = 10. Hence the required sum
2{1-(-2)“}
•= ~ “ - 1} * - 1(1024 - 1) = - 682.

EXERCISES 3(a)
1. Write doNvn the first three and the 8th term of the series whose «th
terms are :

(i) 4« - 5, (ii) (iii) (- l)n


2. Show t^t the arithmetic and geometric means between
the two
quantities a, b are respectively ^(a b) and ^/{ab).
3. Find the sum of ten terms of an arithmetical
progression of which
the first term is 60 and the last is — 104.
4. If the first, third and sixth terms of an arithmetical progression are
m geometrical progression, find the common ratio of the geometrical
progression.
6 . The sum of the last three terms of a geometrical
progression having
of tlie progression
If the third term is find the last term.
5,
(L U )
6. Find two numbers whose arithmetic mean
is 39 and geometric
(L.U.)
7. Prove that the series log a, log (ar), log (ar«) is an arithmetical
progression whose sum to n terms is log (aV*-!).
8 .
^ geometrical progression are 24 and
lo/t
12(& +
1 ) respectively. Find 6 if the sum of the first
three terms
of the progression is 70.
40 PL:RE mathematics f3

3.4. Simple and compound interest


If a sum of money P (the principal) isinvested at simple interest
of r per cent, per annum, the amount A (principal plus interest) after
n years is gi\'en by
nr \
^ =P 1 + (3.8)
100 /’
for the interest for one year is Pr/100 and for n 3 ’ears TOO. The
various amounts after one, t’.^o, three, . . . 3 ’cars therefore form an
arithmetical progression.
If, on the other hand, the same principal invested at compound
is

interest of r per cent, per annum, the interest being added annually,

the amount after one j^ear is P^l -f principal for

the second year. Hence after two years the amount is

Pi 1 1 +
or P( 1
100
+
lOO/V 100/ V
and so on. Tlius after n years the amount will be given by

(3.9)

In this case the amounts after one, two, three, years form a . . .

geometrical progression.
If, with compound interest the interest is added
half 3 'oarl 3^ the

interest is half as much as when added j’earl)’ but it is added


twice

as often. Hence in this case

and similarly for cases where the interest is added at other intervals.

Suppose we wish to find the present value (V) of a given


sum 6 ( )

at t le
due n years hence. Then V is the sum put out to interest
present time which in n years will amount to S.
Thus at simple
interest

^c __ v(i ^ ^

100/
s
givmg V = (3.11)
nr
1 + uio
and at compound interest (added yearly),

5 + i^oj"
r (3.12)
giving I + 106/
3] CONVERGENCE OF GEOMETRIC SERIES il

Example Find the amount at the end of \0 years when ;^400 is invested at
7.
4 per cent, compound interest, (i) the interest being added annually and
(«) the interest being added twice a year.

Here P= 400, r = 4, « = 10 and from (3.0), (3.10) we have

(i) interest added annually, amount = = 400(1-04)'®

« p9I-7.

(ii) interest added twice a year, amount = r(^l + = •100{I-0;i)-®

Example Find what sum a man has to invest on his fortieth birthday so
8.
that he may be able to draw out a lump sum
of ;^2000 on his sixtieth birth-
day. the investment being made at 5 per cent, per annum compound interest.

Let V = sum required. Then V has to amount to ^2000 in 20 years.

i.e.. V= 2000(l-05)-»» « = p53-3.


2’G55

3.5. The convergence of the geometric series


Consider the geometrical progression
we stop at the third term the sum is If and this is less than
If + +
2 by the
third term J. Similarly the sum of four terms is
IJ which differs
from 2 by the fourth term Similarly wherever we stop the sum is
J.
less than 2 by precisely the last term added.
Thus the turn of this
senes never exceeds 2, never reaches 2 but may
be made as near to
2 as we please by taking a sufficient number
of terms. The value «>
IS called the limit
of the sum of this series. Series for which such
a
limit exists are said to be convergent.
Consider now the general geometrical progression

a -\- ar + ar^ , . .

By (3.7) the sum to n terms, denoted by is given by


= ~ g
Sn
-r = _ ar»
I 1 - r izry
Suppose r Ues beUveen 0 and 1. Then decreases as » increases and
since It cannot be negative, it must tend to some positive limit
Smee /
and r” are both ultimately equal to I and we
J^n a similar way, if r hes between - 1 and 0, wc can show that the
hmitmg value of r" is also zero. Thus the value of the term ai-V(l -
r)
” increases and the limit of the
.urn of the progression,
denoted by s is, for - 1< r
< 1, given by
a
1 -r (3.13)
42 PURE MATHEMATICS [3

for we may make as near to a/{\ — as we please r) by making n


siifficiently large. \\ e say that the geometric series wJiose first term
is a and common ratio r converges when —1<r< 1 and the limit of
the sum is a/{i — r).

There is no limit of the sum of a geometrical progression whose


common ratio lies outside the range —For instance the sum
1 to 1.

of the progression 2 -f 4 1 + + • gets more and more unmanage-


• •

able as more terms are taken. Each term in fact exceeds the sum
of all the preceding ones. The series is in this case said to be divergent.
Rather loose expressions are sometimes used in connection with
convergent series. Thus the limit of the sum is sometimes called “ the
sum to infinity " and a convergent geometrical progression is some-
times referred to as an “ infinite " geometrical progression. It is pre-
ferable to avoid the words infinity and infinite as far as possible.

Example 9. The limit of the sum


of a convergent geometrical progression is h
and the limit of the sum of the squares of its terms is 1. Find the first term
and the common ratio of the progression.

Let the term be a and the common ratio r. Then by (3.13)


first
k = a/{l — r). For the series comprised of the squares of these terms
the first term is a* and the common ratio f*. Hence / = a*/(l — r*).
Squaring the expression for k and dividing by that for / we have
_ r*1 r 1 -t-

I “ (1 - r)» “ 1 - r'
Solvirj for r, we find r = (A* — /)/(A* + 0-
Substituting this value of r in the expression for k,

r -n 2 kl
a = k{\ -r)= a[i -

Example 10. Evaluate 0-d as a fraction.

0-fl means A+ ifo + and this can be ^vritten

6 r 1 1 1
loL* + io
+ •

-J-

The series inside the square brackets is a convergent geometrical pro-


gression with first term unity and common ratio -fQ. By (3.13) the
limit of its sum is 1/(1 - Hence the value of
iV)
0 (J = X V= i-

3.6. Series involving the natural numbers


The positive integers 1, 2, 3, . are often referred to as the
. .

natural numbers. They form an arithmetical progression with first


n natura
term and common difference unity and the sum of the first

numbers is therefore given by


- = n(n + 1)
(3.14)
5x = |{2 + (» 1 )}
2
'

3] SERIES OF NATURAL NUMBERS 43

The sum of the squares of the first n natural numbers can be found
by starting from the identity
n3 - (» - 1)3 = 3«3 _ 3„ -I- 1.

Changing n successively into (n — 2), 2, — we 1), {n . . 1, have


(« - 1)3 - (« - 2)3 =: 3(n - 1)3 - 3(n - 1) + 1,

(„ _ 2)3 - (n - 3)3 = 3(» - 2)3 - 3(n - 2) + 1,

23-13 = 3.22-3.2 + 1,

13-03 = 3.12-3.1 + 1.

By addition and noticing that apart from n® and 03 all the terms on
the left hand side cancel in pairs,
»3 ^ 3{12 + 22 + . . . + ^
(„ .^ 1)2
— 3{1 + 2 + . . . + (w — 1) + «} +
If we denote the sum of P + 2* + + . . . by Sj, this gives
w3 = 3Sa - 35i + n.
Using the value of given in (3.14) we have
3Sj = »3 + |«(n + 1) - n = |(2»2 + 3» + 1),

or. _»(»+ l){2n + 1)


* (3.15)
6
The sums of the cubes and higher powers of the natural
numbers
^ be found in a similar
tedious.
way but the process gets more and more

Example 11 . Find the sum of n terms of the series 1.2 2, Z + , • •

writing this as (n» +n). the sum of the


senw 13 the sum of the squares of the first n
natural numbers plus the sum
of these numbers. Hence the required sum
«= i«(« + l)(2n + 1) + in{n + 1)

= iP{n + + l} ->
«(» + l){n + 2)
3

EXERCISES 3(6)
1.

si°riXes?®^
interest, (6) If
at 6 per cent, compound interest
(“)
?
2,
purchase a house, valued now at ^^1000. by pavine

(L.U.)
44 PURE MATHEMATICS [3

Prove that the geometrical progression,


2x / 2;»r ^ *
1 -I- T r +
3 + ^ 3 + A-

is convergent for all values of x and find the limit of its sum. (L.U.)
Show that there are two geometrical progressions in which the second
term is —
4/3 and the sum of the first three terms is 28/9. Show
also that one of these progressions is convergent and, in this case,
find the limit of its sum. (L.U.)
Find the term and the common ratio of a convergent geometrical
first
progression in which (i) the limit of the sum is 4 and (ii) the limit
of the sum of the series formed by the cubes of the terms of the
geometrical progression is 192.
a a .

When docs the series a H 1 + . . . converge and


1 + a (1 + a)*
what is then the limit of its sum ?

Starting from the identity


(2n + 1)* - (2n - 1)‘ = 64«s + IGn,
show that the sum n natural numbers is
of the cubes of the first

equal to the square of the sum of these numbers.


The first, second, third and wth terms of a series are 4, — 3, — IC
and (fl«* + bw + c) respectively. Find a, b, c and the sum of n
terms of the series.

3.7. Permutations and combinations


Suppose we have four objects denoted by A, B, C and D and we
select groups of two. Possible selections are AB, AC, AD, BC, BD
and CD, Each selection is called a combination and it is possible to
make six different combinations from four objects taken two at a
time. If, however, we are concerned with the
arrangements of the
different
four objects taken two at a time we can do this in twelve
ways, viz.,
AB, AC, AD, BC, BD, CD,
and BA, CA, DA, CB, DB, DC,
to make
Each arrangement is called a permutation and it is possible
different permutations from four objects taken two at a time.
twelve
combinations we are concerned only with the number
Thus in forming
contains whereas in forming permutations we
of things each selection
with the order of the component objects as weU.
are concerned
number permutations which can be made
A formula giving the of
things taken r at a time can be obtamed as ^o\\o\vs.
from n unlike
from n things. The first place can be fiUe
We have to fill up r places
things at our disposal. When it
has een e
in n ways for we have n
the second place can be filled in (« - 1) ways
tn fill it. Each way Y! firrnlS
of filling the first place
3] PERMUTATIONS AND COMBINATIONS 45

may be associated with each way of filling the second, so that the
first two places may be filled in n(n 1) different ways. —
Proceeding
in this way the first three places can be filled in «(« 1)(» 2) ways — —
and all the r places can therefore be filled in
«(n — 1)(» _ 2) , , , (n — r 4- 1)

ways. A
convenient notation for the number of permutations of
« things taken r at a time is and we therefore have
«Pr = n(w — \)(n — 2) . . . (« —r -I- 1), (3.16)
the number number in the suffix of the symbol
of factors being r (the
”Pf). Putting f = n we have for the number of permutations of
n things taken all at a time (or the number of ways of arranging
n things among themselves)
"Pn = n{n — 1)(« — 2) . . , 3.2.1,
there now being n factors. The product «(n — l)(n — 2) , , . 3.2.1
is called factorial n * and written (w)!, or sometimes, [k.
To find the number of combinations wliich can be made from n
unlike things taken r at a time, let (a notation similar to that for "G
the number of permutations) be the required number.
Then each of
these »Cr combinations consists of a group of r
things. These can
be arranged among themselves in (r)\ ways. Hence the
product of
^Cr and (r)! is the number of arrangements of n things taken r at a
time, so that

"C, X {r)\ =
= n{n - l)(7j - 2) ... (w - r -f 1 ).

Hence «r. = - 2) r + 1)
(3.17)
«!
An alternative form of (3.17) can
be obtained by multiplying numerator
and denommator by (n r)I. Since (n —
r)\ (n r)(« r l)
— = — — —
3.2.1 the numerator wiU now contain
(» down
aU the numbers n. (n i) * -
2), to unity and will therefore be (m)1.

Hence {n)\

{r)\ (n - r)\'
(3.18)

Example 12. Find how many different numbers can he made h'y using four out
of the nine digits 1. 2. 3 9,
The required number is the number of
permutations of nine things taken
lour at a time and is therefore

•P4*dx8x7x8* 3024.
Exainple 13. In how many ways can an escort of four soldiers be chosen

The required number is the number ot selections which can be m^e


46 PURE MATHEMATICS [3

ixom nine things taken four at a time. This is which by (3.17) is


9x8x7x6 = , 12C.
4 X 3 X 2 X 1

WTien a particular soldier is always to be included, we have to find the


number of wavs in which selections for the other three places in the
escort can be made from the remaining eight men. This is or

3x2x1 = 56.
The method of the last part of example 13 can be employed to

obtain a formula which we shaU use later in this chapter {page 49).
Suppose we have {n -f 1) objects : the number of combinations of
these objects taken r at a time such that a particular object is always
excluded is "G for we have to select from only n objects. The number
of combinations of the objects taken r at a time such that a
particular

object is always included is "Q_i for we have to select from only


n objects for the remaining (r — 1) places in a selection. Since
the

object must be either included or excluded, the sum is the total


number
of combinations of (n + 1) objects taken r at a time or ”+^Cr.
Hence
«+iC, = + (3-19)

objects
So far we have based our work on the assumption that the
of which arrangements have been made or from
which selections have
of permuta-
been taken are all dissimilar. Formulae for the number
tions or combinations when the objects are not all unlike are rather
cases are best treated on their ments and
we
complicated. Such
consider the following as an example.
time
To find the number of arrangements of n things taken all at a
another
when p are exactly alike of one kind and q are exactly aUke of
kind, let x be the required number of permutations.
Then if the p Uke
unlike objects different from any of the
objects were replaced by p
the x arrangements we could form {p)\ new per-
rest, from any one of
of the remaining
mutations without altering the position of any
of the x arrangement,
objects. If then this change were made in each
permutations. Similarly if the ? like
we should obtain x X (p)\
by unlike ones, the number of ^mutations
objects were replaced q
would be X X (>)! X (?)!. But the objects are now^
different and

among themselves in (w)! ways. Hence


can be arranged
^ X (/>)! X (?)! = («)!,

givmg. ^ _ W (3.20)

arrangements of « things taken aU at a tinae


Similarly the number of
one kind, alike of a second kmd and r alike of
when ^ are alike of q
a third kind and so on is
(«)! (3.21)
,

(r)! . .
Ipy. (9)1
3] PROBABILITY OR CHANCE 47
Example 14. How many different arrangements of letters can be made by
using all the letters of the word contact? In how many of these arrange-
ments are the vowels separated? (L.U.)
Here we have seven letters including two c’s and two Vs. The required
number of arrangements by (3.20) is

(7)! 7x6x5x4x3x2xl “
(2)! (2)! 2 X 1 X 2 X 1

If we treat the vowels o, a as one letter, the number of arrangements


with the vowels together is

2(6) i 2X6X6X4X3X2X1 ”
(2)! (2)! 2 X 1 X 2 X 1

the multiplier 2 being introduced in the numerator to allow for the two
possible arrangements ao, oa of the vowels among themselves. The
number of arrangements with the vowels separated is the difference,
1260 -360 or 900.
Sometimes the number of permutations of n things taken r at a time
is required when each thing may be repeated
any number of times (up
to r) in any arrangement. Here the first place may be filled in n ways
and, when it has been filled, the second place may also be filled
in
n ways for we are able, if we wish, to use the same thing again. Thus
the first two places can be filled in » x « or ways. Similarly the
first three places can be filled in n
x n x n or ways and so on.
The total number of arrangements is therefore n'.
Example 15. How many entries must be made in a football pool consisting
of twelve matches to ensure a correct forecast ?
The result of each match may be win, draw so that the forecast
lose or
of the first match can be made in three ways.
The result of the second
game can similarly be entered in three ways so that a correct
forecast
of the first two matches will require 3* entries.
For the first three
matches 3« entries will be required and so on.
Hence for all twelve
matches the required number of entries wUl be 3“
or 631441.

3.8. Probability or chance


Suppose any one oim + n events is just as likely
to happen as
any other and that one event is certain to happen.
Then of the if >»
events are considered favourable and n
unfavourable the probability
or c^nce of a favourable event is said to
be m/[m
For example
m tossing a com, heads or tails are equaUy likely and either a head or + n).
m must occur. The chance of throwing a head
bmulMly the chance of throwing a two with a
is therefore 1/2
six-sided die is 1 /6 for
omy the two IS favourable, any one of the numbers
one to six is equallv
^ely and one number must occur. The phrase "
just as likely to
toppen m
the aWe definition is open to criticism
but the general
Thus the belief that a coin will be equally
f likelv to

proof to the contrary. ^


48 PURE MATHEMATICS [3

the probability of an event happening is^ and of


If its not happen-
ing is q, we liave from the above

m n
P=m
^

n -\~
^
^ m ^

n
Thus p q
= \ and q = 1 — p. Probabilities can therefore range
between 0 and 1, 0 indicating impossibility and 1 certainty. Some-
times percentages are used, a one per cent, chance means a chance of
one in a hundred. If the chances for and against an event are /> and
q, the odds against an event happening are q to p. Thus odds of
5 to 2 against an event implies that the chance of the event happening
is 2/7 and of it not happening is 5/7.

Example 16. What


chance that a hand of thirteen cards dealt from
is the
a pack shall contain only red cards ?
The total number of possible hands is and the totalnumber of
favourable hands is *®Ci 3 for in this case selections of 13 have to be made
from the 26 red cards. The required chance is therefore
= 1/61055
approximately.
the chance of one event happening is p and that of anotlicr in-
If
dependent event happening is p* the chance of both happening \spp'.
The chance of the first happening and not the second is p{\ p'), —
that of the second happening and not the first is p'{l p) and the —
chance of neither happening is (1 p)(\ P')-

These results follow —
directly from the definition of probability. For example the chance
of throwing a six with one die is 1/6, the chance of two sixes when
two
dice are thrown is 1/36. The chance of one six only from two dice is
JH
i X l + f X i
'
3 6*

and the chance of no sixes is 25/36.

EXERCISES 3(0
fifteen possible
1. In how many ways can a team of eleven be picked from
players ?

2. How many different arrangements can be made by taking (i) five,

(ii) all the letters of the word special ?

2000 and 3000 can be made from the


3. How many numbers between
digits 7, 3. 2, 5 ?

how many ways books be


candistributed to four readers when
five
4. In
each reader can have all the books ?
four digits can be formed from the digits
6. How many numbers each of
each digit can be repeated four times ? Calculate
1. 2, 3, 4 wlien
the sum of all these numbers.
2 of which are alike and the rest all different
0. Tl.cre are 10 articles.
articles be made } (L.U.l
In how many ways can a selection of 6
THE BINOMIAL THEOREM 49

7. A signaller has six flags, of which one


two arc white and is blue,
three are red. He sends messages by hoisting flags on a flagpole,
the message being conve 3’’ed by the order in which the colours are
arranged. Find how many different messages he can send (i) by
using exactly sL\ flags, (ii) by using exactly five flags. (L.U.)
8. \\Tiat are the odds against drawing three black balls from a bag
containing four white and five black balls ?
9. Find the chance of throwing head and tail alternately with three
successive tosses of a coin.

3.9. The binomial theorem for a positive integral index


By actual multiplication, we can show that
(1 + x)2 = 1 + 2x -f
(1 + = 1 -f + 3%2 -I-

(1 -f x)« = 1 -f 4- 6x24;i;

In the expressions for these powers of (1 -j- x) we observe that


(i) the indices of x increase by unity as we go term by term from
left to right, the index of the last term being the same as the
power to which (1 x) is raised,
(ii) the first term and the coefiheient of x in the
last term are both
unity and those of the other terms are in the expression for
(1 + x)2, and 2Cj in that for (1 x)® and in + *
that for (1 + ^)*.

This suggests that the result for any positive


integral power n of
(1 4- x) will be

(1 4- x)« =1 4- "CiX 4- ”CjX2 4- . . , 4. ^ ^ (3 £2)


A^ummg that this result is valid, multiplication by
collection of the terms in like powers
(1 + and
of x gives
(1 + *)”+! = 1 + (-C, + 1)* + ("C, + + . . .

+ ("C. + + . . . + ;v«+l.
Since "Cj 1 « + = + =
1 '•+’Ci and "+>C, = "C, 4- "C
' , a result
already established in (3.19), this can
be written
(1 +
*)»+! 1 = ^
n+ic,x . . . +
”+'CrX^ + +
+ . . .
+ (3.23)
gumption made in (3.22) is true for a positive integral
md
mdex «, shows that it is also true when n
(3.23)
increased to n 4- 1. is
But we know the assumption to be true
we
for n 2, 3 and 4, so that =
infer that it is also true forn =
6 and therefore for « 6 and so =
H^^ce the result is true for any positive
integer ».
® 'coefficients in (3.22) are abbreviated
« . -fu XU by omitting® the
» , with this notation we should have

(l+*)«=l-t-Q* + C,*» + + C,*>' +


. . . . . , + *« (
3 24)
.

60 PURE MATHEMATICS [3

where the coefficient of is given by

C, = ’C, = ^. .(«->-+ 1)
_ 25)
(?')!

The first and


terms are sometimes written as
last and Cn, so that Q
Cq and C„ are both unity and Cr is then always the coefficient of
in the expansion. The result established here is known as the binomial
theorem as it gives the expansion (a series of 1 terms) for the «th +
power of the binomial expression (1 x). The method used here in +
establishing the theorem is known as a proof by induction and is a
very powerful method in many branches of pure mathematics.
If we require the expansion of (a we have

(a -[- xY = ^ ,

and, writing xja in place of x in (3.24),

(a + x)^ = a"^I + Cj? -b + * * • + + • +


= a" + -f .

-b Cra^-’‘x^ + . . . + a:". (3.26)

The numerical coefficients in the binomial expansion are given by


the following table (Pascal's arithmetical triangle) :

Power. Coefficients.

1 1 1

2
3
4
13
1

1
2

4
1
3
6
1

4 1

6 1 5 10 10 6 1
C 1 6 15 20 15 6 1

• * •

Apart from the and last coefficients which are unity, any entry
first
adding together the one immediately above it
in the table is given by
entry on the Thus the entry 16 in the sixth Ime
and the next left.

and The coefficients in the expansion


is the sum of 10 and 6 so on.
the last line of the
of (I are therefore immediately obtained from
7 and 1 and once these are available, those
table as 1, 7, 21, 35, 35, 21,
for (1 + x)^ can be obtained similarly.

Example 17. Expand [x + by the binomial theorem and


decimals. (L.U.)
expansion to evaluate (l-03)‘ correct to five places of

Writing for a, 3y for x and taking « = 6. (3.26) gives

(, + 3>,. = ,. + 6.H3y) + 15.‘(3yy +


.

3] THE BINOMIAL THEOREM


the coefficients being taken from Pascal’s triangle [or calculated trom
(3.25)]. This gives
(x + Zyy = X* + ISx^y + 135;r‘>'* + 540x^y* +
+ + 729>'*.

Taking x ^ 1, y = 10“*, x Zy = 1-03 and hence


(103)« = 1 + 18 X 10“» + 135 X 10-* -f 540 X 10-* -f 1215 X lO"'
+
1458 X 10-*® 729 X 10-** +
= 1 0000000 + 0-1800000 + 0-0135000 + 00005400
+ 0-0000122 + 0-0000001 + . . .

= M940523 = M9405 (to five places).

Example 18, Expand (I — — ;r*)® in ascenditig powers of x as far as the


term in x*.

Writing —
f.r(l + fj»r) in place of x in (3.24) and taking the coefficients
from Pascal’s triangle,
(1- I* - *’)» = {1 - + I*) )•
= 1 + 6(- |.r)(l + f,) + 10(- + fr)« + 10(- + |;r)'

+ 5(- i*)«(l + I*)* + . . .

The term in the expansion need not be included as it involves x* and


last
higher powers only. Simplifying and retaining only terms which
involve x^ and lower powers we have
(1 - - **)‘ = 1 - ¥*(i + I*) + ¥**(1 + !* + 1^*)
- + 2* + + .
+ ...) +
. .) .

= 1 -¥*+(- 6 + ¥)*• + (30 - H*)**


+ (10 - ¥» + W)*‘ + .

Example 19. Find Urm


independent of x in the expansion
the
of
(2x + 1/*')*’ in descending powers
of x and find the greatest term tii
the expansion when = |. (L U )

We can write (2x + l/x*)» as 2»x«> (l + so the term inde-


pendent of ^ is 2**4f*» times the term in *-*> in the expansion of

. This is 0,( -) or
2
12 X 11 X 10 X 9 1

1 X 2 X 3 X 4 ’2^
495
‘®* multiplying by 2**x>*, the required term is 496 X 2*
or 126720.

Let Tf be the rth term in the expansion


of -|
Then
T, = and
^ , Hence
^
_ “C, 1 12 —r+ 1 I
'f **C,^l-2x* '
r *^/3)»
27 /13 - r\

62 PURE MATHEMATICS [3

when X the factors in the expressions for


=5 2/3, all and
cancelling except the last in numerator and denominator.
Thus the (r + l)th term is greater than the rth so long as

27/13 - r

lev T > 1.
)
so long as 351
i.e., 43»'. >
The largest value of r consistent with this
inequality is eight so the greatest term is the ninth.

Example 20. If Cr denotes the coefficient of in the expansion of (I + x)^,


n being a positive integer, prove that
Cq — C| + Cj — . . . + — 1)”C„ =
( 0,

Cq + Cj 4* C, + . . . + Cn = 2'*,

and deduce that, if n is even

2Co + C, + 2C* + Ca + 2C* + Cg + . . . + 2C„ = 3.2n-l.

We have
Co + C,x + + . . . + Cnxn = (1 + x)n, (3.27)

Putting x= — 1,

Co - Cj + C, + . . . + {- 1)"C„ = (1 - 1)« - 0, (3.28)

the coefficients of the various C’s being ± 1 according as their suffices


are even or odd.
Putting = 1 in (3.27),

Co + C. + C. + . . . + C„ = (1 + 1)» = 2". (3.29)

Multiplying (3.29) by 3/2, (3.28) by 1/2 and adding

2Co + Cl + 2Cj + + 2C„ -l.2n = 3.2«-l. . . .


=
the coefficient of C„ in (3.28) being + 1. since n is even.

3.10. The binomial theorem when n is not a positive integer


When n is fractional or negative, it can be shown (but the proof
is outside the range of the present book) that the series
n{n — 1 «(« — 1 )(" — 2)
I + nx-\- -X 2)
H ,
X^ + . (3.30)
(2)! (3)!

is convergent if 1 — <
^ <C 1 and that the limit of its sum is (1 .^) + .

known as the binomial theorem for a fractional or


This result is
between it and the theorem
negative index. The points of difference
for a positive integral index are :

«(« 1)
2 _L- .

the series I nx x- 4 r .

(i) for positive integral n,

terminates at the term in and its sum is (1 + *)’’• for all

values of X, * •

negative n, the series does not terminate, it is


(ii) for fractional or
its sum only when
convergent and has (1 x)» as the limit of +
—1<X< 1.
,

3] THE BINOMIAL THEOREM 63

The following particular cases are worth noting. Putting n =— 1


in (3.30) we deduce that the series
—X I . .

converges for — I < x <1 and the limit of its sum is — ^ , Changing
I +X
X to — X wc see that the series
I + x-{- X^ + X^ . .

converges for — 1 < < 1 and that the limit of


:5c
its sum is ——
» X I
These two series ar^geometrical progressions with first terms unity
and common ratios -j- x respectively and the limits of their sums as
given here agree with those obtained from (3.13). Putting « 2 =—
and changing x to —xm
(3.30) we find that the series

1 + 2jc + 3^2 ^ + . . .

converges for - 1< < 1 and the Umit jc of its sum is L_


n — x)^'
Finally writing » = J in (3.30). the series

converges for —1 <^< I and the limit of its sum is V(1 + x).

Example 21. Expand ascending powers of x as far as


the term in x*.

Resolving the given expression into partial


fractions, we find
7 -f X
~ 3 4 - 3x
4-
(1 + X)(l -{- X*) r4- X 1 + ;r*

« 3{1 4- x)-i + {4 - 3x)(l + x*)-i


«= 3(1 - ar + . . .)

+ — 3x)(l ~ x^
(4 X* — X* , . .)
= 7 - Ox - 4- 7x* + . . ..

the series being convergent if — 1 <x< 1 .

/»«’• of

V(l-06) = (1 4- 0-05)1/3 = 1 + j X O-OS — i X (0*06)>


4- iV X - (0-06)» . . .
= 1 4- 0 025 — 0-000313 + 0*000008 — , . ,
= 1*0247 (to four places).

EXERCISES 3(d)
1.

^ IS so small that


and higher powers can be neglected,
show
(1 fx)»(2 4- 3x)» s. 64 4
- ga^. _ 720x>. (L.U.)
54 PURE MATHEMATICS
2 . Find, by the binomial theorem, the coefficient of in the expansion
of (3 — 5x*)* in ascending powers of x.
{Q*E.)
3. Write down and simplify the term independent of x in the expansion

of ( . Which is the numerically greatest term in this

expansion when x = I? (L.U.)


4. In the binomial expansion of (I + n being an integer greater
than t^vo, the coefficient of x* is six times the coefficient of x* in the
expansion of (1 + Determine the value of n. (L.U.)
6 If denotes the coefficient of x^ in the expansion of
.
(1 + x)", prove
that "C, + 2(«C,+i) + «C ,+2 “ "+2c^+2- (L.U.)
6 . Find the value of « for which the coefficients of x, x* and x* in the
expansion of (1 + x)" are in arithmetical progression. (L.U.)
7. Express 2x^/(l + x®)(l — x)* as a sum of three partial fractions and ;

obtain an expansion, in ascending powers of x, of this expression as


far as the term involving x’. (Q-E-)
8 . Use the binomial theorem to evaluate 0-90* * correct to four significant
figures. Check the result by using logarithms. (Q-E.)

EXERCISES 3 [e)
6.
1. Find the sum of the terms from the (n 4* l)th to the mth term
inclusive of an arithmetical progression whose first term is a and
whose second term is b. If m = 13, « = 3 and the sum is 12a, find
the ratio b : a. (L.U.)
2. If a~', b~^, d-^ are in arithmetical progression, prove that
b = 2ac/{a + c) and find b/d in terms of a and c. (L.U.)
3. Three unequal numbers a, b,\/c are in
c are such that 1/a, 1/6,
arithmetical progression and a, c, b are in geometrical progression.
Prove that 6, a, c are in arithmetical progression. (L.U.)

4. S the sum of n terms of a geometrical progression,


is is the product P
of the n terms and R is the sum of the reciprocals of the terms.
Prove that (S/i?)” = P*.
The amplitude of the first oscillation of a pendulum is 15®. If the
amplitude of each succeeding oscillation is 0-89 of the amplitude of
the preceding oscillation, find after how many oscillations toe ampli-
tude will first be less than 1®. (L.U.)

6. £1000 is borrowed at 5 per cent, compound interest. Two-thirds


of the amount then owing is paid back at the end of each year.
How much will have been paid back at the end of five years ?
7. The sum of borrowed from a building society at 4 per cent,
£2000 is
per annum compound interest, and the capital and interest is repaid
by 20 annual payments of £A. Find A and the sum which would
(L.U.)
pay off the balance after 10 such payments.
8. If a geometrical progression with common ratio (1 4- c)/(l
— c) is
otherwise
convergent, prove that c must be negative, but that it is
3] EXERCISES 55
unrestricted. such progressions, one with c =
Two and the other
with c = c„ each have their first terms unity. If and S, are
the corresponding limits to their sums, show that
9.
5^1 ~ {Ci
Hence deduce that when Cj is less than numerically.
(L.U.)
The term of a geometrical progression is 7 and its common ratio
first
is Find how many terms of the progression must be taken in
order that the sum may differ from the limit of its sum
bv
^ less than
001 .

10. Show by induction that the sum of the cubes of the first « positive
integers is J«*(« + 1)* and deduce that the sum of the cubes of the
(n + 1) odd integers from 1 to (2n + 1) inclusive is
(« + 1}*{2h2 + in + 1). (L.U.)
11. If « consecutive terms are taken from an arithmetical progression
of common difierence 2, show that
3(n S„ - 5«») = n*(n* - 1),
where is the sum of the n terms and is the sum of their squares.
(L.U.)
12. A typical car registration number contains three letters of the
alphabet and three of the digits 0, 1. .2
9. How many such
numbers can be formed ? (L U )
13. In how many ways can a party of five people
be selected from six
men and four women so that there are always more men than women
m the party ?
14. Using all the digits 1. 2. 3, 4, 6, 6 how many arrangements can be
made (i) begmning \vith an even digit, (ii) beginning and ending
an even ® with
digit ?

15. Two straight lines intersect at O. Points .4 ...4, .4„ are taken
on one Ime and points B,. B
B, on the other. Prove that
the number of triangles that can be
drawn with three of the points
for vertices is
1
( )
«!(» _ 1 )^ a the pQj-jjt o is not to be used.
(ii) «*, if the point O may be used. (LU
)
16. In a hand of twelve cards, hve are red
and seven black. If two
“dom. find the odds against them both being
buck
17. The odds gainst a student solving a

18.
«
Prove
attempt
in the binomial expansion
t
of (1 + 0 03)n the rth term
B less than one-tenth of the (r l)th term -
certain problem are 4 to 3

>
and

19 . If tte coefficients of *r-i,


if r 4. (L ) U
expansion of
(1 +
») are in arithmetic^ progression,
prove that
»• - n(4f -f 1) -t- 4r* - 2 = 0.
PURE MATHEMATICS
Find three consecutive coefficients of the expansion of {1 +
which form an arithmetical progression. (L.U.)
In the expansion of (1 ax powers of x, the coefficients
in
of are 27 and — 192 respectively. Find a and the coefficients
of x^ and x^^. (L.U.)
If Cf the coefficient of x^ in the expansion of
is (1 + x)^, n being
a positive integer, prove that
Co + Cl + C2 + . . . "T C„ — 2**,

and that
Co + 2Ci + SCj + . . . + {n 1)C„ = (n 4- 2)2«-L (L.U.)

By using the identity (1 — (1 — a-)"( 1 + or otherwise, =


prove that, if n is a positive integer and Cf is the coefficient of x' in
tlic expansion of (I + x)^. then
1 - Ci=^ + Cj^ _ + . . . l)«C„® = 0
equal to (—
-(n + 2)(» + 4) . . . 2n ..
«
if « is odd but is 1)" if is

even. (L.U.)

V{1 -\-3x - 4a-*) .

Expand in ascending powers of x as far as the


(1 - 2 a-)2
term in x^, assuming that the value of x is such that the expansion
converges.
Express
6.V x*
(1 - ,r)(l - x~^

in theform of partial fractions.


Prove that, when the function is expanded as a series in descending
powers of X, the coefficient of x~^ is 3» — 1 + — 1)’*. (Q-E.)
(

Show that, if a: is so small that x* and liigher powers of x can be


neglected, then
ISat*
(N.U.)
'T*
CHAPTER 4

TRIGONOMETRICAL RATIOS FOR ANY ANGLE.


GRAPHS OF TRIGONOMETRICAL RATIOS.
TRIGONOMETRICAL EQUATIONS
4.1. Introduction
The student is assumed to be familiar with the idea of circular
measure and the definitions of the trigonometrical ratios for acute
angles. Thus we take for granted that
X degrees = .xr/180 radians. (4.1)
Abo if X, y and t are respectively the base, height and hypotenuse
of the right-angled triangle ABC in which the angle BCA is a right

angle and the angle ABC is denoted by 6 (Fig. 1), then


sin 0 = y/r, cos 6 = x/r^ tan 0 = y/x^ (4.2)
and
1
COSCC Q
/I
= -r—
1
7r» SCC 0
n = cot 0 = (4.3)
sm 6 cos Q tan 0
Immediate and important consequences of these definitions are

tan 0 =^ = (4.4)
X x/r cos 0
and, since by Pythagoras* theorem, ~ r®,
^
sin-0 4- cos-Q = 1. (4.5)

Abo,

1 + tan= 0 = 1+^* = + = !!’


= _L. - sec*0, (4.C)
x^ cos* 0
and

i + cot*o = i + ^!=J:l±i!==L‘_
'
-
1 _
y% 2

67
68 PURE MATHEMATICS [4

These relations enable all the trigonometrical ratios to be found


vviicn one is given and are often very useful in casting expressions
involving the trigonometrical ratios into alternative forms.

Example 1 . If sin 9 = 1/3 find the values of all the other trigonometrical
ratios.

The relation (4.5) gives cos® 0 = 1 — (1/3)® = 8/9, so that


cos 9 = 2v'2/3.
1/3 1
From (4.4), tan 9 =
2^2/3 2-v/2'

Formulae (4.3) then give cosec 0=3, sec 9 = 3/2 v/2, cot 9 = 2\/2.

Example 2. Show that sin^ 9 — cos® 9 = (sin 9 — cos 0)(l + sin 9 cos 9).
sin® 9 — cos® 9 = (sin 9 — cos 0){sin* 0 sin 0 cos 0 + cos® 0)
-}-

== (sin 0 — cos 0)(1 + sin 0 cos 0),

using relation (4.5) in the second factor on the right.

4.2. The trigonometrical ratios for the general angle


Suppose a radius OP, starting from a standard initial position OX,
is rotated in an anti-clockwise direction. It sweeps out angles which
are conventionally termed positive angles and these may be of any
magnitude. Thus the angle shown in Fig. 2 is 240° (2407r/180 or
4;T/li radians). The same position will be reached by OP after rota-
tions 240° + 3G0°, 240° + 720° or 240° + any integral multiple of
3G0°. We should speak of these angles as being of magnitudes 600°
(or 10,-r/3 radians), 000° (or IGrr/S radians) and 240° w X 360 [or
(6;j 4- 4);r/3 radians] respectively.

Angles generated when OProtates in a clockwise direction are


called negative angles: that shown in Fig. 3 is an angle of
Taking 0 as origin, two Y
perpendicular lines X'OX. as axes OY
and ordinate of the point P witn
and OP of length r, let the abscissa
and The axes divide the diagram into
respect to these axes be x y.
X'OY' and Y'OX these are refeired
four quadrants XOY. YOX'. :

quadrants respectively. I he
in :,S the first, second, third and fourth
4] TRIGONOMETRICAL RATIOS 69

usual sign conventions used in elementary graphical work * are taken


to apply to the coordinates x and y of the point P. Thus when P is in

Fig. 4

the quadrant x and y are both positive, when P is in the second


first

quadrant x is negative and y positive, when P is in the third quadrant


X and y are both negative and when P is in the fourth quadrant x is
positive and y negative, r is taken to be positive for all positions of
the line OP.
The trigonometrical ratios for angles XOP of any magnitude are
defined in precisely the same way as for acute angles : thus

and

cosec 6
sin 0

11
= y/r,

sm 6
,
cos B

sec 6 =
= x/r,


cos B
but the appropriate signs are attached to x andy according
tan 6

cot 0
=y/x,

=
to the position
1

tan B
(
4 9)
.

of the point P. Hence for angles in which OP


hes in the first quadrant,
since all of x,
y and r are positive, the sine, cosine be and tangent will
positive. For angles in which OP lies in the second quadrant, since x
IS negative,
y and r positive, the sine is positive, cosine and tangent
negative. For angles in which OP is in the third
quadrant we can
similarly deduce that the sine and cosine are both
negative but the
tangent is positive, while for the fourth quadrant the
sine and tangent
will be negative and the cosine
positive. The diagram below shows
which of the ratios are positive in each quadrant
and may be useful
as an aid to memory.

Sine All

Tangent Cosine

• See
§ U.l.
60 PURE MATHEMATICS [4

4.3. The trigonometrical ratios of some related angles


Some useful relations connecting the trigonometrical ratios of
certain related angles can be obtained as follows.

In Fig. 5 the radii OP, OP' correspond respectively to angles 0


and —
0. It is clear that the abscissae of P and P' are the same and
that their ordinates are the same in magnitude but opposite in sign.
In other words, changing from 0 to — +
0 is equivalent to reflecting
OP' in the axis X'OX and this changes the sign of y but leaves x
unaltered. Hence, using (4.8),

sin (— 0) = — sin 0, cos (— 0) = cos 0, 14.10)

and, by division,
tan (— 0) = — tan 0. (4.11)

In Fig. 6, a diameter of the circle centre 0. The angles


POP' is

XOP, XOP' are respectively 0 and 180^ + 0* The addition of 180°


to 0 is equivalent to reflecting OP through 0. This changes the signs
of both X and y. Hence
sin (180° + 0) = - sin 0, cos (180° + 0) = - cos 0, (4.12)

and, by division,
tan (180° + 0) = tan 0. (4.13)

Changing the sign of 0 in (4.12), (4.13) and using the results of (4.10),

(4.11) we have

sin (180° = - sin (- 0) = sin 0,


-0) I

cos (180° — 0) = — cos (— 0) = — cos 0,


(4.14)
^
tan (180° - 0) = tan (- 0) = - tan 0. J

In Fig. 7, the angle XOP 0 and XOP'


is 90° + is 0. The ad^'tion

of 90° to the angle is equivalent to measuring 0 from the axis Y OY


4] TRIGONOMETRICAL RATIOS 61

instead of from X'OX. The ordinate of P' is therefore the same as


the abscissa of P and the abscissa of P' is minus the ordinate of P.
Hence ^ x,
y are the coordinates of P,

sin (90° 4- 0) = x/r = cos 6, cos (90" + 0) = - y/r = _ sin 6, (4,15)


‘ \ )
and, by division,
tan (90° + 0) = - cot 0. (4.16)
Changing the sign of 0 in and using the
(4.15). (4.16) results of (4.10),

- 0) = cos (- 0) = cos 0,
sin (90°
cos (90° — 0) = — sin (— 0) = sin '>
0, (4.17)
tan (90° - 0) = cot 0.
The tripnometrical ratios for angles 270°
addition of 180° in those for 90°
± 0 are obtained by the
± 0. Thus, using (4.15) and (4.12)
sin (270®
-f 0) ^ sin+ (90® + 0)}
{180®
= — sin (90® H- 0) = — cos 0,
cos (270® + 0) = cos {180®
4- (90® + 0) }
^

= - cos (90® 4- 0) = sin 0,


and, by division,
tan (270° -f- 0) = — cot 0. (^-10)
Changing the sign of 0 and using
(4.10). (4.11)
sin (270° - 0) = - cos (- 0) = - cos 0.'!

cos (270° - 0) = sin (-


0) = _ sin 0. I (4.20)
tan (270° — 0) = _ cot (—
0) = cot 0. J
Finally, the addition of 360°
(or any integral multiple thereof)
does
62 PURE MATHEMATICS [4

not alter the position of P, Hence we can drop the 360° and
+ 0) = sin 0, cos (360° + 0) = cos d.
sin (360° (4.21)
sin (360° -0)= sin (- 0) = - sin 0,
cos (360° — 0) = cos (— 0) = cos 0,
(4.22)

and the results for tan (360° i 0) can be obtained by division.


It should be noted that for angles — 0, 180° 0, 360° ± sines 0,
remain sines and cosines remain cosines. For 90° i 0, 270° ± sines 0,
become cosines and cosines become sines.

Example 3. Express sin 13o®. (an 140°, sin 1220'’ and cos (- 840°) m
terms of the trigonometrical ratios of positive acute angles.

=
sin 135° - 45°) = sin 45°.
sin (180°
tan 140° = tan (180° - 40°) = - tan 40°.
sin 1220° = sin (3 X 360° + 140°) = sin 140°
= sin (180° - 40°) = sin 40°.
cos (- 840°) = cos 840° = cos (2 x 360° + 120°) = cos 120°
= cos (180° - 60°) = - cos 60°.

EXERCISES 4 (a)

1. If 0 is an acute angle and sin 0 = 1/4, find the values of the other
trigonometrical ratios.
2. If tan 0 = 3/4, find possible values for sin 0 and cos 0.

3. If cos 0 = — 3/5, find the values of sin 0 and tan 0 when 0 is in (i) the
second and (ii) the third quadrant.
4. Show that tan 0 -f cot 0 = sec 0 cosec 0 and that
(sin 0 + cos 0)(cot 0 -f tan 0) = sec 0 + cosec 0.

5. Prove that tan*.,4 — sin*.,*! = sin* .<4 sec*

6. Prove that
sin 0 1 + cos 0 2
1 + cos 0 ^ sin 0 sin 0

7. Prove that sin 330° cos 390° - cos 570° sin 510° = 0.

8. Show that sin (270° - 0) - sin (270° + 0) = cos 0 + cos (180° + 0).

4.4. The graphs of the trigonometrical ratios for acute angles


The graph of sin 0 can be constructed as follows, lake points
Pi, Pz, P3. ... on a circle of unit radius and
draw a line marked off
in degrees (or radians) in prolongation of the initial
line {Fig. 8). CX
is equal to the
If we then plot a point Qi such that its abscissa ONi
radians) in the angle XCP^ and its
ordinate
number of degrees (or
equal to the height of P^ above CX. will lie on the graph
Q,N, is
can be plotted similarly. The
of sin0. Other points (?3. • • •

reaches JU
graph commences at zero and rises to unity when 0
.

table of values of sin 0 could be made by measuring the heights


rough
4] GRAPHS OF TRIGONOMETRICAL RATIOS 63

of P„ p2 « • above CX and recording these heights against


the corresponding angles KCP^, XCP^, XCP^, . . .

Since, from (4.17), cosO = sin (90*^ — 6),


the values taken by the
cosine as the angle increases from 0° to 90° will be the same as those

taken by the sine as the angle decreases from 90° to 0°.


The graph
of cos 0 for acute angles is shown
in Fig. 9.
To construct the graph of tan 0 we draw a base line CX of unit
le^h and mark off a line in degrees (or radians) in prolongation of
CX, Points P;, ^ ^3 are taken on the line XY
2' * • - • (perpendicular

aW.^ nw ^
* IS equal to the number of degrees (or radians) in
^ its
the
ordinate is equal to P^X. the point 0, ivill
nn lie

simiurly. The graph commences at zero and rises faster and faster
e ajyroaches 90°.Again, a rough table of values of tan
6
and recording these
^labies of the trigonometrical
e accurate than could
.
agamst the corresponding angles XCP^, XCP^,
XCP., . .
ratios for angles between 0° and 90°
.

be obtained by measurement as mdicated


64 PURE MATHEMATICS [4

above are available. Details of their construction are beyond the


scope of the present book but the student will be expected to be able
to use the information contained in such tables.

Fig. 10

Accurate values of the ratios for certain angles such as 45® and 60®
can be obtained from the isosceles right-angled triangle and the
equilateral triangle as follows. In Fig. 11, ABC is an isosceles triangle
right-angled at C ; the angle at B (and at A) is 45®, If CB, CA are

Fig. 11

each taken to be of unit length, the hypotenuse AB will, by Pythagoras


theorem, be of length \/2. Hence
sin 45® = AC/AB = 1/V2, cos 45® =BC/AB = 23)
tan45® = ^C/5C = l. J

In Fig, 12, ABC is an equilateral triangle each of whose


sid^ is
taken to be of length 2. The perpendicular from A to BC
wUI bisect
it at D. Hence ABD is a right-angled triangle of
hypotenuse 2, b^e 1
and height, by Pythagoras' theorem, ^/^2- — 1-) or Therefore,

since the angle ABC is 60°,


— J

4] GRAPHS OF TRIGONOMETRICAL RATIOS 66

sin 60^* = AD/AB = V3A cos 60^ = BD/AB = 1/2,1


(4.24)
tan 60° = AD/BD = VS-J
The trigonometrical ratios for 30° can also be obtained from the
triangle ABD
of Fig. 12, using AD
as its base and BD as its height,
for the angle DAB
is 30°. In this way we find
sin 30° = BD/AB = 1/2, cos 30° = AD/AB == ^3/2,1
(4.25)
tan 30° = BD/AD = 1/^3.
The following table may be a useful aid to memory for these
results :

e sin* 6 cos* 6 tan* 0

0 1 0
30* 1/4 3/4 1/3
45* 1/2 1/2 1
60* 3/4 1/4 i

3
90* 1 0 00

The cosecant, secant and cotangent of these angles follow directly


from (4.3). For example,

° 1
sec 60
cos 60°

4.5. The graphs of the trigonometrical ratios for the general


angle
Theresults of §§ 4.3, 4.4 enable us to calculate the trigonometrical
ratios of any angle from a table of
the ratios for acute angles. Thus
cos 170° = cos (180° -
= - cos 10° = - 0-9848,
10°)
sin 1220° = sin
(3 X 360° + 140°) = sin 140° = sin (180° - 40°)
= sin 40° = 0-6428,
sin (- 663°) = - sin 663° = -
sin (2 x 360° — 67°)
= sin 67° = 0-8387.
. graph of any angle we have (4.15),
sin 0 for
sm (90° -f 0) = cos 0 so that the graph for sin for
0 values of 0 between
0 and 180° is the same
as that of cos 0 for 0 between 0° and 90°.
(180° -{- 0) = — sin 0 so the graph for 0 between
ifino
A8U and 360° is the same as that for 0 between 0°
and 180° but is
on tte other side of
the 0-axis. Since sin (360° -|- 0)
graph repeat? iteelf after 360°
sin 0 the =
and again after 720° and so on. Finally,
j
(— 0) =—
sin0, the graph for negative 0 can be
med from that for positive 0 by reflection in the origin. The graph
IS shown
by the full curve of Fig. 13,
a
66 PURE MATHEMATICS [4

Since, from (4.17), cos0 = sin (90* — B), the graph of cos0 can
be obtained b}^ displacing the graph of sin B to the left by 90*. This
is shown dotted in Fig. 13.

For the graph of tanO, we have by (4.11), tan (— 0) — tan 0, =


so the graph for — 90* < 0 < 0 is obtained from Fig. 10 by reflection
in the origin. Also from (4.13), tan (180° -f- (3) = tan B, so the graph
for B between 90* and 270* is the same as that for B between — 90*
and 90° and so on. The full graph is shown in Fig. 14.

Fig. 14. — Graph of tan d

The graphs of the trigonometrical ratios show their periodic nature.


Each ratio repeats itself after a certain interval (called the period).
The trigonometrical ratios are examples of periodic functions the :

of
periods of sinO and cosO are both 360* (or 27t radians) while that
tanO is 180* (or n radians). The magnitude of sin0 and cos0 is
half of this range of variation (i.e., unity)
is
always between 1± :

called the amplitude.


The fundamental relation, sin*0 -f- cos^B = 1, proved in §4.1 for
values of For, as we have seen
acute angles remains true for all B,
4] GRAPHS OF TRIGONOMETRICAL RATIOS 67

in § 4.3, the addition or subtraction of any multiple of 90° can at


worst change a sine into a cosine or vice versa and lead to an alteration
in sign. But sin*0 +
cos^O only involves a sine and a cosine and
variations in sign are unimportant as both terms are squared. Hence
the relation remains valid for all values of 0, Similar arguments can
be applied to show that the relations 1 tan^fi sec^ 0 and + =
1 + cot^ 6 = cosec^ 0 remain true for all values of 0,

Example 4. Draw the graph of y = sin {2x + 2it/Z) from x = 0 to x = 2it.


Use your graph to find the positive values of x which satisfy the equation
AT = 5 sin {2x
+ 2tt/Z). (L.U.)

Working in radians, using tables and the relations of we can plot


§ 4.3,
the graph shown in Fig. 15.

Plotting the graph of


y =
x/5 on the same diagram, we find that the
p-apbs intersect at three points A,B,C given by ;r =»
0-48, 2*34 and 3*30
(approx.). These are the required positive
values of x which satisfy
the equation x = 6 sin {2x + 27r/ 3 ).

EXERCISES 4(6)
1 . Use tables to find the values of
(i) sin212», (u) cos(- 110“), (ui) tan 1145°, (iv) sec 1327”.
2 ^
.

sm
^ a cos
® values of
2^4 and 2-4.
(O C )
3.
between 0° and 180° which satisfy the
fquatio^
sm (4r
+ y) = 0-6, sin (X - y) * ^ 0*6. (L.U.)
4.
diagram the graphs of sin
^dd r
I cos ^ for values of x between 0* and 180®.
acute angle which satisfies
Hence find an
the equation sin 2;r = l - cos (O C
5. )

STo.'s.r'Sr.i;”’

(O.C.)
68 PURE MATHEMATICS [4

6. Draw the graph of ^ = sin a: + i sin for values of x between 0®


and 180®. Hence find positive values of x less than 180® which satisfy
the equation sin at + ^ sin Sat — § = 0. (O.C.)

4.6. The solution of trigonometrical equations


Trigonometrical equations differ from algebraic equations in that
they often have an unlimited number of solutions. Some equations
can only be solved by graphical methods (see, for instance, Example 4,
page 67). In such cases it is usually best to rearrange the equation
so that the simplest trigonometrical graph is drawn. Thus in solving
the equation a; 5 sin (2x= +
2.t/ 3) it is preferable to plot graphs of

y =
sin (2x -f- 27i/3) and y xj^ rather than y = 5 sin {2x 2ji/ 3) = +
and y = X,

be
When a theoretical solution of a trigonometrical equation can
obtained, the equation can often be reduced to one of the
foms
sin (? =
t, cos 0 =
c or tan 0 =
c, where c is a numerical
quantity.

We now consider the solution of these three equations.


The simplest to deal with is tan 0 c and we commence with this =
abscissae
equation. Angles satisfying this equation are given by the
of the points of intersection of the graph of y tan 0 with a line =
parallel to the 0-axis and at distance c from it. Suppose that (Fig. 16)
ABC, . . .,
are the points of intersection of the graph
B', C', , , .

'= tan'0 Vith this line. abscissae of all such points satisfy the
The
y
equation tan 0 c. = Suppose one of these abscissae (for convenience
the smallest numerical one is usuaUy selected) is a°. T^is then is the
abscissa of A, The abscissa of B is a°. that of ^ is 300 a + + ,

the right, whUe the abscissae of B t .


, , , .

and so on for points to


360° — and on. All these results can be
are -- 180° a°, + -f a° so
where « is any positive or
included in the formula (« X 180° 4- a°)
If a were quoted m. radians mstead of
negative integer or zero.
the formula (tm 4- a). Hence the gener
degrees wc should write
4] TRIGONOMETRICAL EQUATIONS 69

solution of the equation tan 0 = c is


(« X 180 + a) degrees or {im + a) radians,
where a any solution of the equation, but generally taken for con-
is

venience to be the smallest numerical solution. As an example the


general solution of the equation tan 0 = 1 is, since tan 45° = 1,
(« X 180 + 45) degrees or (n -|- radians.
If we deal with the solution of cos 0 = c in
the same way. Fig, 17
applies. It should first be noted that since cos & always lies between

± 1, there will be no solutions to an equation of this type for which

c ISnumerically greater than unity. Taking the


smallest solution a**
as the abscissa of the point A, other
solutions given by the abscissae
of points B, C, ... and B', C',
. , . are 360° a°, 360° a° — +
~ + a°, . . . These are included in the formula
all
{« X 360 ±a ). Thus the general solution of the equation
cos 61 = c when -'l<c<lis
{n X 360 ± a) degrees
where n
or {2im ± a) radians,
isany positive or negative integer or zero and
a is any solution

example the general solution of the equation
cose = 1/2, smce cos 60° = 1/2, is
^
{» X 360 ± GO) degrees or {2«
± i)n radians.
y

and Taking
70 PURE MATHEMATICS [4

of tlic point A, other solutions, given by the abscissae of points B,


C, D. . , and B'. C, , , are 180^
. a°, 360^^ -j- a°. 540° a°. . -
. . . and — 180° — a°, — 360°+ a°, The . . . general formula
including all these is n x 180°+ (— l)"a°. Hence the general solu-
tion of the equation sin 0 = c when — 1 < c < 1 is

{n X
180 -f (— l)"a) degrees or (uti (— l)"a) radians,
where again n is any positive or negative integer or zero and a is any
solution of the equation. For example, the general solution of the
equation sin 0 is, since sin 45° =\/y/2, =
[n X 180 -J- (— 1)" X 45) degrees or (n -f ^ radians.

Taking n = 0, 2, 3, 1, . , . solutions are 45°, 135°, 405°, 495°, . . .

while = — — 2, — 1, 3, . . . gives the solutions — 225°, — 315°,


- 585°. . . .

To sum up, if a radians is one solution of the equation


(i) sin 0 ~ c,(— I <c<l), the general solution is nn -f (— l)"a,

(ii) cos 0 = c, { — l<c<l), the general solution is 2n7t i a,

(iii) tan 0 = c, the general solution is nn -F a,

where n is any positive or negative integer or zero, and corresponding


expressions for the solutions in degrees.
Many trigonometrical equations can be reduced to one of these
forms and the general solution can then be written down. Examples
are given below and further examples will be found in Chapter 5,

page 82,

Example 5. Fiud all the angles less than four right angles which satisfy the
equation 2 cos* ^ = I sin 0. (L.U.)

Since cos^ 0=1— sin* 6. the equation can be \vritten


2—2 sin* = + 1 sin or 2 sin* (?+ sin 0 — 1 = 0.

This equivalent to (2 sin 6


is I)(sin 0 Hence either — + 1) = 0-
sin 0=1/2 giving 0 = « X 180^ (- 1)« X 30^ angles between + ;

O'* and 300* included in this arc those for m = 0, n


= 1. i.e. 30* and
150* or sin 0 1 giving 0 = —
n X 180* -I- (— 1)"(— 00*} ^the = ;
;

only angle between 0* and 300* in this is that for n 1, i.e. 270 = .

Hence the required angles are 30®, 150* and 2i0®.

Example 6. Find all the angles which satisfy the equation


4 sec* 0 = 3 /a» 0 + 6. (L-U.)

Since sec* 0=1-}- tan* the equation can be written


0,

0=3 tan 0+5. or 4 tan* 0 3 tan 0 — — 1 = 0.


4 + 4 tan*
This is equivalent to (tan 9 - 1)(4 tan fl + 1)
= 0. One set of solutions
corresponds to tan 0 = 1 and is 9 = « X 180- +
45". The other corre-
whose tangent is
sponds to tan 9 = - 1/4. From tables, the angle
14* 2', so that another set of solutions is given
by
— 0*25 is
0 = » X 180* - 14*2'.
4] TRIGONOMETRICAL EQUATIONS 71

Example 7. Find all the angles between 0® and 360® which satisfy the
equation sin 2d = cos 36. (L.U.)

^ince cos 36 sin {90® =


3d), we have sin 26 — sin (90® 3d) so = —
that the general solution is 2d n x 180® (— 1)" X (90® =30). + —
Putting « =
0, 2, 4, 6 and 8 respectively gives

(i) 2d = 90® —
3d or 6 = 18®.
(ii) 2d * 360® + 90® -
3d leading to d == 90®.
(iii) 2d = 720® +
90® —
3d leading to d = 162®.
(iv) 2d = 1080® + 90® -
3d giving d « 234®.
and (v) 2d = 1440® + 90® -
3d giving 6 = 306®.
Higher even values of n lead to values of 6 greater than 3G0® while odd
positive values and negative even values of n all lead to negative values
of d. The value m = —
1, gives 2d = — 180® - 90® + 3d leading to
d = 270® and values like n = — 3, —
5, etc., give angles in excess of
360®, Hence the required angles are 18®, 90®, 162®, 234®. 270® and 306®.

Example 8. Find the general value of 6. in degrees, which satisfies simul-


taneously the equations tan 6
V3. sec 6 2.
— (L.U.) = —
If tand= y/3. the general solution is d = n x 180® + 60®; if
sec d = - 2. cos d = - 1/2 and 6 => 2n x 180®
± 120®. Solutions
of the first equation are therefore — 480®, — 300®, — 120®, 60®, 240®,
420®, GOO®, etc., while those of the second are — 480®,
240®, 120®,
120 240®, 480®, COO®, 840®, etc.
, Values simultaneously satisfying the
two equations arc therefore 480®, -
120®. 240®, 600®, etc., all of —
which are included in the formula n x 360®
-f 240®.

EXERCISES 4(c)
1. Find all the values of 0 which satisfy the equation
2 tan 0 3 sec 0 = 4 cos 0.+ (L.U.)
2 . Find the values of * between 0“ and 380° satisfying
the equation
10 sin* X + 10 sin ar cos X — cos* x =
2. (L U )
3. Find the values of A and B between 0® and 180®
which satisfy the
equations A - B = 12^ 18', cos (^
+ B) = 0-4467. (L.U.)
4. Give the general solution (in radians) of the
equation
cos (0 - 71/4) = sin 20.
5. Find the general solution of the equation
10 sec* 0 - 3 = 17 tan 0.
C. What IS the most general value of 0 which satisfies
both the equations
quauons
tan 0 = 1/V3 and sin 0 = - 1/2 ?

EXERCISES 4(d)

1. T# • e a* - 0*
^ “ aa 6*’ values of cos 0 and tan
+ 0.

2 . Find the value of

sin* A cosec cos A, (I».U.)


+

72 PURE MATHEMATICS [4

3. If 9 is an angle quadrant and tan 6


in the first = t, express all the
other trigonometrical ratios in terms of t.
4. Kind the values of cos 3360^ cosec (— 840°). (L.U.)
Prove the identity
(1 +sec .* + tan ;r)(l +cosec at + cot x)
= 2(1 +tan AT-f-cot sec x +cosec at a-),

and verify this result when x = rr/4. (L.U.)


If tan* a — 2 tan* ^ = 1, find the possible ratios of cos a to cos fi.

(L.U.)
If sec 9 — cos 6 = a and cosec $ — sin 0 = b, prove that
a*6*(a* + + 3) = 1.

Prove that
,, .
cot a + tan 8
=
8 . r cot a tan 8.
cot ^ + tan a
9. Show that
cos 0 — 1 cos 0 + 1
+ = 2(1 + tan 0).
sec 0 + tan 0 sec 0 — tan 0
10 . Prove that
1 + sin 0 cos 0
=
— sec 0 + tan 0.
cos 0 1 sin 0
11. If X cos 6 + y sin = a and «;in 0 — y cos 0=5, prove
0 a: that
bx + ay
tan 0 = and AT* + y* = a* + b^. (L.U.)
ax — by
12 . If tan 0 + sin 6 ~ x and tan 0 — sin 0 = y, prove that
-y2)2 = IGA-y.
(;p
2 (L.U.)

13. Plot on the same diagram the graphs of cos 20 and tan (40° — 0)
between 0 = — 20° and 0 = 60°. Hence find two approximate
solutions of the equation cos 20 cot (40° — 0) = 1.

14. Sketch the graph of y = {rr/2) sin* at and use your graph to solve
the equation 2Ar = ,-isin*Ar. (O.C.)

15. Find graphically the values of x between 0° and 180° which satisfy
the equation sin at = 3 cos* x.
(O.C.)

IG. Draw on the same diagram the graphs of 4 sin {x + 30°) and 2 + tan x
for values of a- from 0° to 360°. From your graphs obtain the solu-
tions, within this range, of the equation 4 sin (at + 30°)
— tan x = 2.
t • j

Draw the graph of y = 2 sin (at + 7t/4) between x — rr and x n


solve
By drawing another graph, using the same scales and axes,
approximately the equation 4Ar“ + 16 sin* (at + 7i/4) = 7t*. (
• •)

satisfy the equation


Find allthe angles between 0° and 360° which
6 sin* AT + 6 cos AT = 7.
Find the general solution of the equation tan 30
= cot 20.
equation
Find the general values of x satisfying the
4 cos AT + 6 = Csin*Af.
4] EXERCISES 73

21. Assuming r is positive, find r and a value of B between — 180® and


180® to satisfy the equations r cos 0 = — 4, r sin 0 — 2*5.

22. Find all the angles between 0® and 360® wliich satisfy the equation
3 tan® 6 — 3 tan* 6 = tan 6 — 1.

23. Find the general solution of the equation


(2 tan ;ir — 1)® = 3(sec* — 2).

24. Find the general solution of the equation tan ;r tan 4r = 1.

25. Find the values of x, in radians, between 0 and 2;r, which satisfy
the equation 6 tan* — 4sin*;»r — 1. v^.U.)
CHAPTER 6

ADDITION THEOREMS. MULTIPLE AND SUB-


MULTIPLE ANGLES. FURTHER TRIGONOMETRICAL
EQUATIONS. THE INVERSE NOTATION.
SMALL ANGLES
5.1. The addition theorems for the sine and cosine
We now consider formulae expressing the trigonometrical ratios of
the sum of two angles in terms of the trigonometrical ratios of the
separate angles. Such formulae arc known as addition theorems and
we start by deri\-ing them
a restricted range of angles. Generalisa-
for
tion of the results to cover all angles can be made but the process is
rather troublesome a compact general method of derivation of the
:

formulae making use of a result in coordinate geometry is available


and this is given in Chapter 14, page 262.
Formulae for sin (A -f 5) and cos (H -|- B) for cases in which the
angle {A +
B) is acute can be obtained from Fig. 10. Here the angle
AOB is A, the angle BOC is B, P is any pomt on OC and PM, PN are
perpendicular to the lines OA, OB respectively. and NH NK
are

perpendiculars from the point on to OA, N PM


respectively. Since

PM, PN are perpendicular to the arms OA, OB respectively of the

angle AOB, tlic angle is A.MPNFrom the right-angled triangle


MPO,
OP sm {A -f B) = MP = MIC KP
= //iV + /vP. (5-1)

The right-angled triangle


since MHNIC is a rectangle by construction.
= ON right-angled triangle ONP gives
OHN gives IIN sin A, and the
ON OP^ cos B. Hence HN = OP sm A cos B. Also, from the
74
;

5] ADDITION THEOREMS 75

right-angled triangle PKN, KP = PN cos A, while the triangle ONP


gives PN = OP sin B. Thus KP == OP cos A sin B. Substituting
for HN, KP in (5.1) and dividing both sides by OP, we have
sin (A -\- P) = sin A cos B -f- cos A sin B. (5.2)

The corresponding formula for cos (^ B) is obtained similarly


thus, from Fig. 19,

OP cos OM = OH - HM
(A -\-B)=^
= OH - NIC
Also OH = ON cos A and ON = OP cos B so that OH = OP cos A cos B.
NK ^PN sin A and PN = OP sin B, giving NK = OP sin A sin B.
Substitution and division by OP leads to
cos (^ -f" ^) = cos A cos B — sin A sin B, (5.3)

The formulae for sin {A -{- B), cos {A -f B) given in (5.2), (5.3) are

the fundamental addition theorems. They have been derived only for
the case in which the angle
(4 S) +
acute and Fig. 19 applies. For
is
the case in which A and B are both acute but
in which their sum
(A -f B) is obtuse, we should work from Fig.
20. The lettering has
the
We now
same significance as in Fig. 19 but now M lies on AO produced.
have, from the triangle MPO
MP = OP sin MOP = OP sin (180® ^ A — B),
Since sin (180® — 6) = sin this can be written
0, still

OP sin (AA-B)==MP = MK-\-KP=.HN KP,


A- (5.4)
and we obtain (5.2) in exactly the same way as
before. The formula
tor cos (A + B) can be similarly
extended to cases in which (A 4- B\
' ^
IS obtuse. '

The theorems can be extended to cases in


which one of the ancles
say X , hes between 90® and 180® as foUows.
Let A' 90'*+/ so =
+ ^) = cos ^ ind
76 PURE MATHEMATICS [5

sin (A' A- B) = sin (90° + A + B)


= cos {A + B)
= cos A cos B — sin ^ sin B,
since A and B are both acute,

= sin A' cos B + cos A' sin


Similarly,
cos {A' A- B) = cos (90° A- A A- B)
= — sm (A + B)
= — sin ^ cos B — cos A sin B
= cos A' cos B — sm A' sin B.
Thus both the addition theorems are true when A' lies between one
and two right angles. A similar argument holds if B is increased by
90°. Hence the theorems are true for any angles between 0° and
180°. The argument can be extended to show that the theorems are
valid for angles of any magnitude but the full proof becomes rather
long. As stated previously, a much shorter proof for general angles
can be given when an elementaiy^ result in coordinate geometry is
available (see Chapter 14, page 262) for the present we shall take
:

the theorems as applying to angles of any size.


By writing — B for B in the theorems, we have, using (4.10), the
two results,
sin (A — B) = sin A cos (— B) -f cos A sin (— B)
= sin A cos B — cos A sin B, (5.5)
and
cos (^ — B) = cos A cos (— B) — sin A sin (— B)
= cos A cos B sin sin B. -j- /I (5.6)

= a/3 + 1
Example 1. Show that cos 15® *
2 a/2
cos 15® = cos (45° -
30°)
= cos 45® cos 30® + sin 45® sin 30®

1 a/3
•v/3 II I \/3 + 1

*"
V2’“r V2'2 2v/2

Example 2. Use Hie addition jornnila to show that cos (90® + A) = -- sin A
cos (90® A) = cos 90® cos A — sin 90® sin A
= — sin A,
since cos 90® = 0, sin 90® = 1.

5.2. The addition theorem for the tangent


cosine give
By division, the addition theorems for the sine and
tan (A A- B) = sin (A + B)
cos {A + B)
A
5] MULTIPLE ANGLES 77

^ sin A cos B + cos ^ sin B


cos A cos B — sin i4 sin B
tan A tan B
1 — tan A tan B‘
(5.7)

dividing numerator and denominator by cos A cos B.


Writing —B in place of B,

tan (^ - B) = (-^)
1
«
— tan A tan (— B)
tan A — tan B
(5.8)
1 tan A tan B*

^ ^
Example 3. Show that tan (45* + ^) =
'
I --tan A

= tan 45® + tan^ 1 + tan A


tan (45* + W)
1 — tan 45“ tan A
~~
1 — taiT^'
since tan 45* = 1.

EXERCISES 5(fl)

I. Show that cos (a + cos (a - /J) = cos* a — sin*

Prove that cot {A « cot A cot B — I


2.
+ B) '
cot A + cot B
3. Show that
sin (^ +B + C) = cos ^ cos B cos C(tan A + tan B
+ tan C — tan A tan B tan C)
and deduce that, if B, C are the angles of a triangle, then
cot A cot B + cot B cot C + cot C cot A = h (L U )
4. Show that a sin + 6 cos = V(a* + 6») sin (* + a) where
tan a = b/a.
5. If A cos 0 = cos (0 - a), show that tan 0 = A cosec a - cot a
G. Prove that
sin (^ -B ) shi_(B_- C) sin{C-A)
** ""
cos ^ cos B cos B cos C *"
cos“C cos /I

5.3. Multiple angles

m B=^ m
the three addition formulae
(5.2), (5.3) and
( ) we obtaui expressions for the sine,
.
cosine and tangent of
terms of the trigonometrical 2 in
ratios of A, Thus, from^(6.2)
sin (.4 + .4) = sin .4 cos .4 + cos ^4 sin A,
sin 24 = 2 sin .4 cos 4,
(6.9)
A

78 PURE MATHEMATICS [5

The addition formula for the cosine, (5.3), gives similarly


cos 2A = cos- A — sin^ A. (5.10)
By writing sin^ A = — cos^ A,
1 this can be written in the alternative
form
cos 2/1 = 2 cos^ A — 1, (5.11)

and by writing cos® A = 1 — sin® A in (5.10), yet another equivalent


form is

cos 2.'! =1- 2sin2^. (5.12)

Sometimes, particularly in the integral calculus, it is necessary to


express sin® A and cos® A in terms of cos 2/1. This can be done by
rearranging the last two formulae to give
cos® /I = J(1 + cos 2A) and sin® A = \(\ — cos 2A). (5.13)

By wTiting B—A in the addition formula for the tangent (5.7)


we have
2 tan A
tan 2.-1 = (5.14)
1 — tan® A
Expressions for the trigonometrical ratios of 3.4 can be obtained
as follows. By WTiting B ~ 2A in (5.2) we have
= sin (A 2 A)
sin 3/1
= sin A cos 2 + cos A sin 2/4.
Substituting cos 2/1 = — 2 sin® A, sin 2/4 = 2 sin A cos A gives
1

sin 3/1 = sin .4(1 — 2 sin® /4) + 2 sin A cos® A,

and, writing cos® = — sin®.4 we have, after slight reduction,


/I 1

sin 3.4 = 3 sin /4 — 4 sin® A. (5.15)

A similar process applied to the addition formula for the cosine, (5.3),

gives
cos 3/4 = cos 4-2/4)
(.4
= cos A cos 2/4 — sin A sin 2A
= cos (2 cos® A — 1} —2 sin® A cos A
/I

= cos A (2 cos® /4 - 1) — 2 (1 - cos® .1) cos A


— 4 cos® A — 3 cos A. (5.16)

Proceeding similarly from (5.7), we have


tan 3/4 = tan {A 4-2/4)
tan A 4- tan 2A
tan A tan 2.4

2 tan A
tan A 4-
i — tan® A

1 — tan A
SUBMULTIPLE ANGLES 79
using (5.14). After reduction this gives

„ 3 tan A — tan® A
,
tan ZA
-
= (5.17)
1 - 3tanM
5.4. Submultiple angles
By writing A = xj% in the formulae of the last section we have,
from (5.9),
sin a; = 2 sin Jjc cos Jj;. (5.18)
from (5.10), (5.11) and (5.12),

cos X = cos® — sin® Ja; la:'|

= 2cos®i:t-l I (5.19)
= 1—2 sin® \x, 1
and from (5.14),
, 2 tan \x
(5.20)
ts-20)

These formulae enable us to express the sine, cosine and tangent


of an angle in terms of the tangent of the half angle.
If we write

t = tan ^x, (5.21)


(5.20) gives immediately
tan :¥ = 2//(l - ^2). (5.22)
Formula (5.18) can be written

sin = 2 tan ^x cos®


__ 2 tan ix
sec® ^x

_
~ 2 tan ix
~ 2(
(5.23)
1 + tan^ ix r+T*’
Also, from (5.19),
cos « = cos® 4x(l — tan® Jx)
_ 1 — tan® ix
sec® ix

_ 1 — tan® Jx 1 — /®
“ 1 + tan> ix ~ r+T>‘ (6.24)

The three formulae

= = = 6 26 )
(6.26)
-
(

where I = tan i* are useful in the solution of a


certain tvoe of
80 PURE MATHEMATICS
_ ^ 30 cos 60
Example . ^
4. Prove that = 2 {cos 20 - cos 40). (L.U.)
^
cos 6 cos 26 ‘

cos 30 cos 60 4 cos® 0—3 cos 0 4 cos® 20 — 3 cos 20


cos 0 cos 20 ~ cos 0 cos 20
= 4 cos® 0 — 3 — (4 cos® 20 — 3)
= 4 (cos® 0 — cos® 20)
= 2{(1 cos 20) — (1 + cos 40) =
-f- } 2 (cos 20 — cos 40).

Example // tan 0
5. 4/3 and t/ 0® < 0 = < 360®, without tables, the
possible values of tan ^0 and sin \d.

Let / = tan ^0, then 4/3 = tan 0 = 2t/{\ — /®), giving 4 — AP = 6/


or 2/® + 3/ — 2 = 0. This gives (2/ — 1)(/ 2) = 0 leading to t = 1/2
or —
2, and these are the required values of tan J0.
To find sin ^0, we have
/ = tan ^0 = sin |0 sec jO — sin J0(1 + tan® ^0)*^*,

so that sin jO = With t = 1/2 this gives


sin id = (l/2)/V(l+ 1/4) = 1/V5 ;

and with / = — 2, sin J0 = — 2/v'(l + 4) = — 2/(±\/~>) = 2/v'5 if

0 is to be less than 360® and therefore 10 less than 180®.

EXERCISES 5 (t)

1. If tan® a — 2tan*^ = 1, prove that cos 2a + sin®/J = 0. (L.U.)

2. If f = tan iO, express the square root of


(1 + sin 0)(3 sin 0 + 4 cos 0 + 5)
in terms of /.
(L.U.)

3. If sin 30 = p and sin® 0 = J


- <?. prove that />® + = 12?®.

4. Prove that 2 cot iA + tan ^ = tan A cot® iA. (L.U.)

5. If 2cos0 = ;r + 1/x, show that 2 cos 30 = jr® + \/x^, (L.U.)

(L.U.)
6. If sec A - tan ^ = ;r, prove that tan ^A = ^
»

5.5. The factor formulae


to express
I he addition formulae for the sine and cosine can-be used
sums and and cosines as products. Starting from
differences of sines
the addition formulae
sin (+ + Z?) = sin ^ cos 5 + cos A sin B,
sin (A — B) = sin ^ cos ^ — cos A sin B,
addition leads to nx
~ sin / .
.
(5.26)
2 sin AcosB (^4 +
and subtraction gives
2 cos .+ sin B = sin (A A~
(5.27)

the cosine
Similarly the addition formulae for
6] FURTHER TRIGONOMETRICAL EQUATIONS 8l
cos (A B) = cos A cos B — sin ^ sin B,
cos — B) = cos A cos B + sin ^ sin B,
(i4

give 2 cos ^ cos B = cos + B) + cos {A — B), (5.28) (^4

and — 2 sin ^ sin B = cos (A B) — cos (A — B). (5.29)


These four formulae express products as sums.
By writing v4 + B = C, ^ — B = B, so that A = \[C -D) and
B ~\(C — D) these formulae become
sin C + sin B = 2 sin \{C + B) cos i(C - D), ]
sin C — sin B = 2 cos J{C + B) sin ^(C — B), I

cos C cos B = 2 cos f (C + B) cos J{C — B),


-j-
'
(5.30)

cos C - cos B = - 2 sin \(C 4- B) sin i(C - B)..


These formulae, which express sums as products, are of great use and
are often called the factor formulae. It is useful to remember the
results in words e.g., the sum of the sines of two angles is equal
;
to
twice the sine of half the sum of the angles multiplied
by the cosine
of half the difference of the angles and so on. The minus sign in the
last formula should he noted.

Example 6. ^ 3A ^
tan 6A,
sxn cos QA + 5in A cos 2A
Using {S.29) and (5.26),
sin, 3A sin, 6A+ sin A sin 2^4
sin 3^ cos 6i4 + sin A cos 2^4

_ " i cos 9.4 + t cos (- 3.4) - J cos 3.4 + ^ cos (- ^4)


sin 9.4 + 4 sin (- 3.4)
+ * sin 3.4 + i sin (- A)
cos A — cos 9.4
__
sin .4 + sin 9A

-
_ 2 sin i(A + 9.4) sin i(A - 9.4)
i(9.4 — A)
2 cos i(A + 9.4) sin *

sin 6.4 si n 4.4


**
^s 5.4 sin 4.4
^
Example 7. m/kout using tables, prove that cos 165® + si» 166® « cos 136®.
By the last of*the factor formulae
(6.30),
cos 166® -
cos 136® = -
2 sin i(165® 136®) sin *(186® +
136®) -

2 sin 160® sin 16®
«—
2 sin (180® 160®) sin (180® 16®)— —
=—
2 sin 30® sin 165®
= - sin 165®.
since sin 30® *= 1/2. Hence cos 166® + sin 166® = cos 136®.

5,6. Further trigonometrical equations


The formulae of the last section enable
certain trigonomelrical
82 PURE MATHEMATICS [5

equations to be reduced to equations of the form sin 0 c, cos 0 = =c


or tan 0 =
c, the general solution of which is known
(Chapter 4,
page 70). The following examples illustrate the methods.

Example 8. Find all the values of $ which satisfy the equation


cos pd -f cos (p + 2)e = cos d.

Using the third o£ the factor formulae, the equation can be written
2 cos [p -f 1 )
cos 6 = cos d,

or, cos ^{2 cos {/> + 1)0 — 1 ;


= 0.

Hence either cos 0 = 0, giving 0 = ^2h ± or cos (p + 1)0 = i

giving + 1)0 = ^2n ± i.c.. 0 = ± ^)Tr.


Example 9. Find the general solution of the equation 2 sin 2x sin r = 1.

By formula (5.29), the equation can be written



+ cos 2x = 1.
cos -Lr

and using the formula 2 cos* 2;r — 1 = cos 4x. we have


“ 2 cos* 2.r q- cos 2x = 0 or cos 2x{2 cos 2x — \) = 0,
so that either cos 2.r = 0 giving 2.r = (2n ± \)it. i.e., x = {n ±\)-n:
or cos 2x = ^ giving 2;r = (2« ± J)ff. i.e., x = {n ± ^)7r.

5.7. The equation a cos 6 + 6 sin 0 = c


The equation a cos 6 + i sin 0 = c in which a, b and c arc supposed
known numerical quantities, often occurs in practical applications.
There are various methods of solution here we : shall consider two.
In the first method, we divide by \/(^* obtain

®
""
vWTb^)'

a
Fig. 21

If we introduce an angle y whose tangent is h/a, a glance at Fig. 21


shows that a/\/{a^ + b^) is cos y and b/'\/(a^ b^) is sin y. Hence +
the equation can be written
c
cos 0 cos y + sin 0 sin y =
.

6] further trigonometrical equations 83

or, cos [0 ^ y) =
The equation has now been reduced to one of the standard forms
whose general solution is known. Hence a general value of 0 — y
can be written down and, since y is a known angle, 0 can be found.
For real solutions to exist it is necessary for c to be numerically less
than ^/(a^ +
b^). More precise details of the method of solution can
be obtained from Example 10 below.
The second method makes use of the formulae (5.25), i.e., if
t= tan then

sin d = 2/
COS0 =1-
l+t- l +

Substitution in the given equation a cos 0 + 6 sin 0 = c and multiplica-


tion throughout by 1 -|- gives
- + 2bt = c(l
a{l /
2
) -i-

or, {a + c)t^ — 2bi — (a — = 0. c)

This quadratic equation gives two values of i {or tan


J0) from which
general values of 6 can be derived. Again precise details will be found
in Example 10 where the equation is solved by both methods.

Example 10. Find (He general solution of the equation 2 cos 0 — sin 0=1,
Method Dividing by ^/{2* -f (—
{*) !)•} or V5 we have


V6
2
cos $
V5
1
— sin 0 s —1

V6
Taking tan y =—
1/2 so that, from tables, y 26® 34'. 2/\/5. = —
— Z/\/5 are respectively the cosine and sine of this angle and we have
cos 0 cos (— 26® + sin 0 sin {— 26® 34') =
34')
I/V6.
cos (0 + 26® 34') = 1/V6 => 0*4472.
Now the angle whose cosine is 0*4472 is 03® 26', so using the general solu-
tion given in (4.26),
0 + 26® 34' «n X 300® ± 63® 26'.
The positive sign on the right leads to the solutions

0 =n X 360® + 36® 62',

while the negative sign gives 0 =«x 360® -- 00®.

Method (it) Writing sin 0 = 2//(l -f (»), cos 0 = (I -<»)/(! + f*)


where / = tan 10, and multiplying throughout by I
+ (>, we have
- - 2( = + i\
2(1 1

3l* + 2/ - 1 = 0.
This can be written + l)(3t - 1) = 0. The root « - 1 gives
(t
/
tan 10 = - so that 10 = n x 180® - 46® and
1,
0 « n x 360® - 90®
pie root = 1/3 leads to tan 10 =.
1, 10 = n x 180®
<
18® 26' and
8 = « X 360® -H 30® 62' as before.
-f-
84 PURE MATHEMATICS [5

EXERCISES 5(c)
1. Prove that sin A sin (60° - A) sin (60° + ^) = J sin 3A. (L.U.)
2. If ^ + R -|- C = 180°, prove that
sin A 4- R +
C = 4 cos ^A
sin sin cos ^R cos JC. (Q.E.)
If cos A — cos B ~
3.
p and sin A — sin B = q, express cos {A — R) and
sin {A 4- R) in terms of
p and q. (L.U.)
4. Find all the angles between 0° and 360° which satisfy the equation
cos 0 4- sin 30 = cos 20.
5. Find the general solution of the equation
sin 0 — sin 20 = sin 40 — sin 30.
6. Use the appropriate factor theorem to find the general value of x
satisfying the equation cospx 4- cosqx = 0.
7. Find all the angles between 0° and 360° which satisfy the equation
cos ;r 4- 7 sin ;r = 5.

8. Find the value of 0 less than 360° which satisfies the equation
3 cos 0 — 4 sin 0 = 5.

5.8. The inverse notation


If sinO =X where ^ a given quantity numerically less than
is
unity we know that 0 can be any one of a whole series of angles. Thus
if sin 0 = = un (— l)"(7r/r))
1/2, 0 and 0 " many-valued
is The
inverse notation 0 = sin"^;r is used to denote the angle whose sine

is X and the numcncally smallest angle satisfying the relation x = sin 0


is chosen as the principal value. Here and in what follows we shall
deal only with principal values and understand the statement
0 = sin'-'^x to mean that 0 is the angle lying between —n/%
and n/2
radians whose sine is x. The statement 0 = sin~^ x is read as 0 equals
5] THE INVERSE NOTATION 85

the inverse sine of x (or sine minus one x) and an alternative notation,
more commonly used on the Continent, is (? = arc sin x.
The graph of 0 = sin“^ x is, on this understanding, easily seen to
be that part of the graph x = sin 0 given by — ji/2 < & <:7c/2 with
the if-axis horizontal and the 0-axis verticah This is shown in
Fig. 22.

In a similar way, 0 =
cos'^^; wiU be taken to denote the
smallest
angle whose cosine is a:. Since the cosine takes the
same values for
negative as for the corresponding positive angles
and we require a
notation which gives an unique value to 0 when ^ is
given, we conven-
tionaUy take B to be the angle lying between 0 and
n radians whose cosine
For example, =-
is X. and cos**^
(- s) - ?
graph of 0 = cos ^ ^ is easily derived from that of 4? = cos 0 and is
shown in Fig. 23.

similarly deHned but since, unlike the sin

v^ue. e _ tan 1 *
^ ^ unrestricted i
taken to mean that 0 is the smallest
is
tangent ts
angle who>
x and 9 bes between - n/Z and
n/2 radians. The
y

86 PURE MATHEMATICS [5

tan’ ^ (1) = ,-7/4 and tan — 1) = — :t/4. The graph of 0 = tan”* x


^ (

is given in Fig. 24.


The
inverse cosecant, secant and cotangent are similarly defined.
In order that they ma}' be single-valued, we choose cosec”* x to denote
the angle between —
:t/ 2 and 7i/2 whose cosecant is x while sec”* x
and cot”* x are taken to mean the angles lying between 0 and n radians
whose secant and cotangent respectively are x.
It follows from these definitions that

sin (sin ^ x) = x, cos (cos ^ x) — x, tan (tan”* x) = x, etc., (5.31)

and these relations be found useful in some of the examples and


will
exercises given below. Care should be taken to avoid confusion
between the inverse sine, cosine, etc. of x and the reciprocals of sin x,

cos X, etc. The latter should always be written or cosec x, —?—


'
sin X cos X
or sec X, etc.
The general solutions of the three equations smd c, cos 0 c, = =
and tan 0 =
c given in (4.26) can be compactly expressed in this inverse
notation. Thus if
(i) sin 0 = c, (— 1 <c< 1), 0 = 1)" sin”* c,'j

(ii) cos 0 = c, (— 1 <c< 1), 0 = 2njt ± cos”* c, > (5.32)

(iii) tan 0 = c. 0 = tan”*


7171 c. J

Example 11. Show that


(a) {— x) = IT — cos~^ X. (6) 5im“' (—*) = — sin-^ x.

(a) Suppose that x is positive and


(i) let ^ = cos”*^. Then 6 lies
between 0 and it/2 and x = cos $. Hence
= — cos 6 — cos {n — 6),
— X
giving cos-* {— x) = n — 6=7t — cos~^ x.

(ii) If ;r is negative, let x = y so that y is positive. Then


COS'* {— y) = n — cos-^ by above. Hence (i)

COS'* X — n — COS'* (— x)
and slight rearrangement gives
COS'* ( — X) = 7T — COS'* X.

(iii) If X is neither positive nor negative, it must be zero. Hence


COS'* (— x) =
COS'* (0) iTf2 and cos'* x = = cos'* (0) = n/2, so

that
IT — COS'* X = n — ^TT = iiT = COS'* (— x).
Hence cos'* (- ;r) = tt — cos'* for all values oi x (provided, of

course, that x is not numerically greater than unity).

(fc) The identity sin-* (- ^r) sin-* = -


can be established in a similar
way and is left as an exercise for the student.

Example 12. Show that cos^^ x + sin~^ x = ‘n/2.

B = cos'* Then B lies between 0 and


(i) Let X be positive and let jr.
^

6] THE INVERSE NOTATION 87

jr/2, and x — cos B. Hence


X — cos B = sin (i^ir— B),
giving sin~‘ x = ^ir — B = — cos-' x,
and the required result follows.

(ii)Let x be negative and let ar = —^ so that y is positive. By (i)

above, cos-* y + sin-'y = it/ 2 giving cos“' {— ^) + sin-'(— x) ^ -nf'Z

But, by Example 11,


cos“' (— ;r) » TT — cos“' X and sin"' (— t) = — sin"' x,
so that
IT — COS"' x — sin"' X =» ff/2,

giving COS"' x 4- sin"' x = Tr/2.

(iii) If X is neither positive nor negative, it must be zero. Hence


= COS"' (0) = 7r/2 and sin"' x = sin"* (0) = 0,
COS"' X
giving COS"' X + sin"' x = 7r/2.

Hence the identity cos"';r + sin"' = »r/2 is valid for all values jr of
1 < ^ < 1 ).
4

Example 13. Show that (a«-' (1/3) + si«-' (l/VS) = ir/4.

Let o = tan-i (1/3), = sin-' (l/VS). so that tan a 1/3 and =


sin /J = 1/ V6. Hence ^ is the angle shown in the right-angled triangle

(Fig. 25) in which the height is 1 and the hypotenuse is The base
is \/(6 — 1) or 2 and we deduce that tan = 1/2. Hence
tan-' (1/3) + sin-' (1/V5) = « -1-
= tan-' (tan (a + ^)), by (5.31),
tan
tan a + tan
Hi? tan a tan }
tan
U - i X iJ
tan-' (1) = ff/4.

EXERCISES 5(<f)

1. Evaluate sin-' {1/V2), cos-' (- ^3/2), sec-' 2 and cot-'


( V3).
2. Show that tan-' [— x) = — tan-' x*
3. Prove that tan-' x + cot-' x = n/2.

4. Prove that 2 sin-' = sin-'


6. Show that cos-' (^) + 2 tan-' = sin-' (|).
(|)
88 PURE MATHEMATICS [5

6- If all tlie angles are acute, show that


cos-1 + cos-1 jy ^ V{(1 — ^*)(1 —
cos-i [xy —
7. Show that there is a positive value of x which satisfies the equation
tan“i (2x + 1) + tan“i (2x — 1) ^ tan-i 2, and find this value.
8. By drawing the graphs of y = tan x and y = 2x show that the smallest
positive root of the equation tan”i 2x — x is the circular measure of
an angle of about 67®,

5.9. Small angles


If we plot on the same diagram the graphs of sin 0, 0 and tan d

Fig. 26

for values of 0 in radians a figure like Fig. 26 results. It is apparent


that for 0 < 0 < jt/2,
sinO <0 < tan 0, (5.33)

and that these three quantities are approximately equal to one another
for small values of the angle 0.

Fig. 27

and approximations can also be inferred from


These inequalities
chord PQ subtends an acute angle 0 at the centre 0
Fig. 27 in which the
of a circle of radius r. The tangent to the circle at P "'cfs O(?
pro-

the bisector of the angle POQ


it wUl bisect the
duced at r. If ox is
6] SMALL ANGLES 89

base of the isosceles triangle POQ at right angles at X. Hence


OX = r cos PQ = 2PX — 2r sin and the area of the triangle
POQ = \PQ.OX = l{2r sin W.r cos i0) = sin 0. The area of the
sector POQ is (where 0 is in radians) and since OPT is a right
angle, PT = r tan 0 and the area of the triangle OTP

= \OP.PT — tan 0.
From the figure,
area of triangle POQ < area of sector POQ < area of triangle POT,
or, sin 0 < Ir-O < tan 0 .
which, on division by give
sin 0 < 0 < tan 0 .

It is also clear
that the areas of the three figures considered approach
equality as the angle 0 diminishes.
Dividing the inequalities (5.33) by sin 0 we find 1<
and this can be written 1
0/sin 0 1 /cos 0. <
>
(sin 0)/0 >
cos 0. This in turn can be
written

0 <1— < 1 _ cos 0.


Since 1 — cos 0 = 2 sin^ {0 and since sin 0 < 0, sin i0 will be less than
J0 so we have 1 - cos0 < 2(J0)2 or 1 - cos 0 <\o\ Hence

0 <1- < ie\


and 1 - (sm 0 )/O can therefore be made as small
as we please by
making 0 sufficiently small. Another way of expressing
this is to
wnte
sm 0
=1— e,

where e a quantity which we may make as small as


is
we please by
taking 0 to be sufficiently smaU. StiU
another way of expressing the
same thing is to say that the limit of (sin
0)/0 as 0 tends to zero is
unity or, symbolically
/sin 0 \
("Tj (5.34)

We also infer from the foregoing that when 0 smaU, sin 6 is is


approximately eqml to 0 (in radians). Also,
since cos 0 I 2 sin> iO = -
approximately equal to JO, an approximate
o?tn=T ^
r ^
of COS0 when 0 is small will
be 1 2(J0)* or 1
- i0»
value
A coar«.r —
® As examples, consider
^
the values given by these
approximations when the is 40 Thf
radian me^ure of 4'’ is 0-0698 so
gives sin 4 —
that the approximation sin' 0 =0
0-0698, while the approximation cos
6 1 = — giyg^

90 PURE MATHEMATICS [6

cos 4® = — 1(0 00487) = 0*9976. Both of these


1 values are correct
to four decimal places.

Example 14. The elevations of the top Q of a flagstaff PQ from three distant
points A. B.C which are in a horizontal line with P ate 6, 26 and 36 respec-
tively. Prove that AB =
3BC approximately. (L.U.)

The three right-angled triangles CPQ, BPQ. APQ give


CP cos 30
cot 36 =
UP sin 30’

BC -f CP cos 20
cot 20 =
sin 20'

AB -h BC A- CP cos 0
cot 0
. «
= -T
QP sm 0

Since the points A, B, C are all far from P, the angles 6, 26, 36 will all
be small. The cosines can therefore be replaced approximately by
unity and the sines by the angles in radian measure. Hence we can
write approximately
CP \ BC CP 1 AB BC CP 1

gp
^

^ ^

gP “ 0’

Subtracting the first of these relations from the second and the second
from the third we shall have approximately

PC 1 1 1 ^P 1 1 1

gp ^ ^~^ ^ gP ” 0
” 20 “ 20*

By division we then have AB/BC = 3, approximately.

Example 15. Find an approximate value of the acute angle which satisfies
the equation sin 6 0*52. =
Since sin 0 is nearly equal to 0*5, 0 must be nearly »r/6 radians. Let

0 = —be where
€i
e is therefore small. Then
/n \ tr ft .

0*52 = sin (
“ + ej = sm
.
- cos e +
,
cos - sin e.

Since sin (tt/G) = 1/2, cos (tt/G) =


^3/2. and, because e is smaU,
cos < =
1, sin € = e approximately, we have
1 -v/S
0-52 = 2
+ -F*'

giving c= 4- X 0 02 = 0-0231.
V3
Hence « = 0-0231 radians or 1* 19' approximately and 0
= 31* lO'.
6] EXERCISES 91

EXERCISES 5 (e)

1. Find an acute angle which approximately satisfies the equation


sin 6 = 0-48.

2 . The diameter of the bull’s eye of a certain target is one inch. At


what distance will it subtend an angle of 30 minutes ?
3. In a right-angled triangle ABC, C is the right angle, the side BC — a
and the side AC
= b. Show that the angle A BC hes between
b/\/(a* -f b*) and b/a radians.
4, Assuming that sin 0 = Q — kB'^, where A is a numerical constant, is
a sufficient approximation to the value of sin 0 when 0 is a small
angle, use the formula sin 30 = 3 sin 0 - 4 sin® 0 to show that A = 1/6.
(This result gives, of course, a better approximation than sin 0 = 0.)
Prove that the perimeter of a regular polygon of n sides inscribed
in a circle of radius R is 2nR sin [n/n] and use the
approximation
of Exercise 4 above to find the diSerence between this perimeter an
d
the circumference of the circle when i? = 30 ft., n = 200.
Prove that if 0 < 0 < 7i/2. sin 0 > 0 - J0*.

EXERCISES 5 if)

1. Prove that tan (45° -h 0) - tan (45° - 0) = 2 tan 20.


2. Iftan a = a/{a -f 1) and tan p = l/(2a + find the smallest value
1)
of the angle a -f
3. Show that sin (a + sin {a ~ p) = sin* a - sin*
Express tan -f B
4. (.<4 -f- C} in terms of A, tan B and tan C.
Prove that

sin 2 tan ~ tan -


tt sin 2 2
cos a + cos (Q.E.)
1 — tan* - tan* -
2 2
Prove the identity
(sin 2a - sin 2P) tan (a + = 2(sin* a - sin* P). (L.U.) ,
If tan = cosec x - sin x, prove that tan* = — 2 ± ^6.
^ ^
If sin 0 -f sin 20 = a and cos 0 + cos 20 = b. prove that
(a* -t- 6*) (a* + 6* _ 3) = 26. X.U.l
Prove that
tan ot + tan (eo' + a) + tan (120° + a) = 3 tan 3a. (L.U.)^
Prove that
sinM + sin" (120° + A) + sin" (240° + = -
.4) fsin (L.U.)
Establish the identity
sin 0 -h sin (0 + a) + sin (0 -f 2a) + sin (0 -f 3a)

= 4 sin
^9 + cos a cos (L.U.)
|.
92 PURE MATHEMATICS [5

12 . Prove that
sin 3x -h 2 sin 5x sin* x + sin 7x — cos ;r{sin Qx + sin ix). (L.U.)
13. Prove that
= cos A 1 -f cos A
2 cot 2A cot (L.U.)
1 — cos A cos A
14. Find all the angles between 0® and 180® (inclusive) which satisfy
the equation cos x — cos 7x = sin (L.U.)
15. Solve the equation 10 sin* 0 — 5 sin 20 = 4, giving the values of 0
between 0® and 360®. (L.U.)
16. Solve the equation 2 sin 0 + 3 cos 0 = — 1, stating all solutions for
the range 0 < 0 < 27i. (Q E )

17. Solve completely the equation sin 30 cos 30 — cos* 20 + ^ = 0.


(L.U.)

IS. Find a pair of angles lying between 0® and 180® and satisfying the
equations sin .(4 + sin S = 0-95, A —B = 120®. (L.U.)

19. Find all the angles between 0® and 360® satisfying the equation
sin 20 — 2 cos 20 = sin 0 — 2 cos 0 + 2. (L.U.)

20 . If tan 0 = A tan {A — 0), show that


(A — 1) sin + = (A + I) sin (20 — +).
Hence, or otherwise, find the values of 0 between 0® and 360® which
satisfy the equation tan 0 = 2 tan (60® — 0). (L.U.)

Find two values of 0 less than 7t/2 satisfying the equation


8 sec 0 — 4 tan 0 = 7.

Write down the general solution of the equation. (L.U.)

Prove that
4 tan-* il)
- tan-i (^) =
23. Prove that
cot-* =
+ cos-* (f).
(i) cot-* (3)

24. Find’jir from the equation tan“* 2x + tan“* 3x = fr/4.

25. If 0 is an acute angle such that cos 0 = 1


— where .* is so small
that x^ is negligible compared with unity, prove that cos 20
= 1 — 4x
and cos 30 = 1 — 9;r approximately. (L.U.)

I
CHAPTER 6

RELATIONS BETWEEN THE SIDES AND ANGLES


OF A TRIANGLE. THE SOLUTION OF TRIANGLES.
HEIGHTS AND DISTANCES
6.1. Notation
A triangle has six parts or elements —three sides and three angles.
\i A, B and C are used to denote the angles of the triangle, it is con-
ventional to denote the sides opposite these angles by the corresponding
small letters a, b and c respectively (Fig. 29).

The sides of a triangle are independent of


one another except for
the fact that the sum of any two of them must
be greater than the
third. The angles, however, are not independent. Since the sum of
the angles of any triangle is 180°, the third angle is
known if two angles
are given. There are thus five independent
elements in a triangle.
three sides and two angles. Later in this chapter we shaU see that if
three elements of a triangle, one at least of which
is a side, are known
the other three can be found. The process of
calculating the unknown
elements of a triangle when three of its elements
are given is termed
the solution of the triangle.
The early part of this chapter is devoted to
the derivation of
relations between the sides and angles
of a triangle. In the later
sections, the arrangement of the computaUons
involved in the numerical
solution of trmgles is discussed and examples
are given of some prac-
tical apphcations to problems in heights and distances.

6.2. The sine formula


be the centre of the circle circumscribing
T the triangle ABC
jom BO and produce it to meet the circle again at D.
Toin DC
pgs. 30, 31 apply respectively when the
angle A of the triangle ABC
acute or obtuse. In both diagrams,
the angle BCD, being the angl
m a sejm-circle, is a nght angle. In Fig.
30. the angle BDC is equ«
93
94 PURE MATHEMATICS [6

to the angle BAC the same segment, while in Fig. 31, the angle BDC
in
is equal to the supplement of BAC,
since the points B, A. C and D
are concyclic. If 7? is the radius of the circumcircle so that BD 2R, =
A

the right-angled triangle BCD gives,


in Fig. 30, = 2Rsm A,
BC
in Fig. 31, = 2R sin (180° ~ A)^2R sin A.
BC
Hence in both cases, since BC = a, = 2Rsin
By joining AD
instead of DC
we could prove similarly that
c = 2R sin C. By starting our construction from C, instead of B,
we could show in the same way that b =
2R sin B, These three
results can be displayed in the formula

= 2R. (6.1)
sin A sin B sin C
a result usually kno^vn as the sine formula. In certain cases (see for
instance, page 104) this formula enables the solution of a triangle to
be carried out and it also enables the radius R of the circumcircle of
a given triangle to be found.

6.3. The cosine formula


In Figs. 32 and 33, BD is the perpendicular from B on to the base

diagram applies
CA, or CA produced, of the triangle ABC, The first

when the angle A is acute, the second when A is obtuse. In each


COSINE FORMULA 95

figure the right-angled triangle DAB gives

BD = c sin
In Fig. 32,
DA = c cos A and CD = CA — DA —h — c cos A.
In Fig. 33,
^
AD = c cos DAB = c cos (180° — — c cos /I
and CD = CA AD = b — c cos A. -j-

Applying Pythagoras' theorem to the right-angled triangle DCB in


either figure,
CR2 = CZ>2 -f- BD\
or, a® = — c cos
(6 -f- c® sin® A
= 6® — 2bc cos ^ -f- c® (cos® A -f sin® A),
which, using the identity sin® + cos® =
.4 .<4 1, gives

^ ^2 _ 25c cos .(4. (6.2)


The two similar formulae
52 = c® -f fl®— 2ca cos B,
c® = fl® 5® — 2ab cos C,

can be similarly derived. These are the cosine Jormtdae and are useful
in the solution of triangles when at least two sides are given (see
page 108).

Example 1, In a triangle ABC, prove that a* = - c)*


(6 -f- 46c sin* and
hence that a = (b ~~ c) sec
^ where tan ^ (L.U.)

Since cos^ = 1 _ 2 sin* ^A, the cosine formula


a* 3= 6* 4- c* — 2bc cos A
gives
a* = 6* + c* - 26c(l - 2 sin* 4^)
= (6 — c)* 4- 46c sin* }A.
Using the given expression for Un this can be written

a**(6-c)*4.(6-c)»taa*^
— c)*(l 4- tan*^)
*3 (6
= (6 — c) * sec*
leading to « = (6 — c) sec A.

*
Example 2. Prove that in any triangle ABC, - B)
c* sin {A 4- B)'

From ^ ^
the sine formula, a 2J2 sin A. b = = 21? «iin n ^ on
that after dividing numerator
and denominator by 4R\
^ sin* ^ sin* B _ sin* .4 - sin* B
sin* C “ ^n* (^1 4. B) *
since C« ISO-* - ^ ^ B and hence
sin C « sin (IgO*^ - - B) - sin (.< + B).
90 PURE MATHEMATICS
This can be witten
a~ — b- (sin A -f sin £')(sin A — sin jB)
c* sin= (.-I -}- B)

_ 2 sin ^{A + B) cos ^(.1 - B).2 cos \{A + B) sin i(A - B)


2 sin \(A 4- B) cos ^{A *f B) sin {A -f B)
using (5.30) and (5.9).
This simplifies to
2 sin j(A — B) cos \{A — £) sin (A — B)
~
c* sin {A A- B) sin {A + B)’

EXERCISES 6 (a)

1. With the usual notation for a triangle ABC, prove that


a cos A A- b cos B = c cos {A — B). (L.U.)

2 . With the usual notation for a triangle ABC, show that c* can be
expressed in the form
[a + 6)®{1 — A* cos* ^C),
and obtain the value of A*.

Prove that in a triangle ABC with circumcircle of radius R,


a cos A A- b cos B A- c cos C = 4R sin A sin B sin C.

If, in a triangle ABC, ab = c*, prove that


cos {A — B) A- cos C + cos 2C = 1.

With the usual notation for a triangle ABC, prove that


(b + c) tan ^A — (6 — c) cot ^A = 2b cot B.

Prove, in the usual notation for a triangle, that if

b A- c c A- a a A- b
^
~Tr ~i2 13 '

tlicn.
sin A sin B sin C cos A cos B cos C
and
6 19 25
(L.U.)

6.4. The area of a triangle


Let A denote the area of the triangle ABC and, in Idg. 34, let BD
3

B on to AC. Then since BD = csin A.


be the perpendicular from
A = iCA.BD = ibc sin A. (6-3)
6] RADIUS OF INSCRIBED CIRCLE 97

In the same way we could show that A= ^ca sin B or sin C.

By writing (6.3) in the form we could rewrite tlie


sin A
sine formula (6.1) as

a
= 2R = abc (6.4)
sin A sin B sin C 2A
To an expression for the area of a triangle in terms of the
find
sides alone, we have from (6,3),

2bc sin A = 4A,


and from the cosine formula (6.2),

_ ^2
2bc cos A = ^2 ^2
Squaring and adding, and using sin^ A + cos® ^ = 1, these give
46®c2 = 16A2 -f (6® + c® — a 2 2
) ,

so that A® = — (6® -f c® — a®)®}


= i(26c H- 6® + c® - a®)(2/)c - _ ^2 ft2 ^ 2^
= *{(& + c)® - a®}{fl® - (6 - c)®}
“ + c -f a)[b -f c — a)(a — 6 c)(a -j- -j- ^ — c)*
If we write
2s =a+ 6 -i- c,
(
6 5)
,

so that s is half the perimeter of the triangle, this can be written

A® = 2s. (2s - 2a) (2s - 26) (2s - 2c),


or. A= V{s(s -a){s-~ 6)(s - c) }. (6.6)

Example 3 . The
sides of a triangle are in arithmetical progression
and its
area ts Z/5lhs that of an equilateral triangle
of the same peritneter. Prove
that its sides are in the ratio 3:6:7.

the sides of the triangle he x



d, x, x d. + If 2s is its perimeter,
2s Zx. From (6.6) the square of its area

“ - - = - <2*).
T (I *)(t -*-'*)
For the equilateral triangle of the same perimeter
each side will be x and
the square of its area

3**
-
l(¥ ')(¥ - )(¥ - .) ’
16
Hence i**(}*« _ d») = ^ X ***,
i** -
giving
Thus '*• = (i - tSts)*’ =
“ **
T *''o* /«
3*/«- *•
**'®
which are in the ratio
triangle are
1 7
6.5. The radiusof the inscribed circle
Let / be the centre of the inscribed circle
and A A F the points
93 PURE ^IATHE^^ATICS [6
of contact of the circle with the sides BC, CA, AB of the triangle ABC,
1 hen
area triangle BIC -f area triangle CIA -f area triangle AIB
— area triangle ABC = A.
4

Fig. 35

If r isthe radius of the inscribed circle, the heights of the triangles


BIC, CI.A, AIB are each r and their bases are respectively a, b and c.
Hence
Ira + \rb -f {rc = A,
or since i + ft -f c) '= s,

rs = A,
giving r = A/s. (6.7)

This formula, together with (0.6) enables the radius of the inscribed
circle to be found in terms of the three sides of the triangle.
Alternative expressions for r can be found as follows. Since AI
bisects the angle CAB, the angles AFI, lEA are right angles and
AI is common, the triangles AFI, lEA are congruent and AE AF, =
Hence
2AE ^ AE A- AF
and similarly 2BD = BD -f BF,
2CD ==CD + CE,
By addition
2AE + 2{BD CD) ^ AE + AF A- BD A- BF A- CD CE
-f-

or 2AE 4- 2BC = perimeter of the triangle ABC.


Hence 2AE A-2a = 2s.
giving AE = s — a.
In the right-angled triangle IE A, the angle EAI is ^A and since

IE = r.

-I— = tan i/I,


AE
gi\'ing r = (s — o) tan l.-l.
6] RADII OF ESCRIBED CIRCLES 99

Similarly we can show that y = (s — tan \B, y = {s — c) tan JC,


and, combining these with (6.7), we have
y = (5 — a) tan = (s — fi) tan iR = (s — c) tan IC = A/s. (6.8)

Yet another expression for y can be obtained by starting from the


relation (see Fig. 35), BD + DC = BC = a. Now the angle DBI is

and the angle DCI is ^C, so that the right-angled triangles DBI,
DCI give
DB = r cot IB, DC = r cot iC.
Hence y{cot -h cot JC} = a.
This can be written
cos \B cos JC
= «.
sin JB sin JC }
giving
y{sin JC cos JB + cos JC sin JB} — a sin JB sin JC,
or, y sin J(B -}- C) = rt sin JB sin JC."
Since ^4 +B -|- C = 180^
J(B-f-C)=90^-U.
sin J(B 4- C) = sin (90" - J4) ^ cos lA,
Also, from the sine formula (6.1),

= 2B sin .(4 = 4B sin J^ cos J^.


Hence
r cos J^ = 4B sin J.4 cos J^ sin JB sin JC,
giving, r = 4B sin sin JB sin JC. (6,9)

6.6. The radii of the escribed circles


Let be the centre of the escribed circle touching BC internally

at F,. E, respectively. Let the radius


f be
of this circle
r,. Then, Fig. 36,
area triangle BI,A + area triangle CI,A - area triangle BI,C
= area triangle ABC = A
^00 PURE MATHEMATICS [6
The heights of the triangles BI,A, CI,A. BIfi are each r. and their
bases are respectively c. b and a. Hence
4- IrJ) — \T^a = A,
+ b ~ a) lr^{c
Since 2s = a b c b ~ a = 2s ~ 2a, and c,
therefore
r^is — = A. fl)

Hence = A
s — a
''i (0.10)

and we can derive similar formulae = A /(5 — b), = A/(s — c) for


the radii of the escribed circles opposite B and C respectively.
Since, from (0.7), A = rs, this can be written
rs
=
s —a ,

and use of (
6 8 ) for r then gives
.

ri = stan^^. (G.ll)
Similarly we can show that r^=:s tan IB and ^3 = 5 tan IC,
Example 4. Jn a Mangle ABC. r^. r^. r, res,pectivfly denote the radii of
the three escribed circles. Prove that r^r^ + r^^r^ + =
5 *, where
2s =
+ 6 + c.a (L.U.)
Since r^ = A/(s - b), r, = A/(s - c). r,r, = A^is - b){s - c). But
A = s{s - a){s — b){s — c) so that r^r^ = 5(5 — a). Similarly
2

fjrj = s{s — b) and = s{s - c).


Hence
*'z^s + ^ 3^1 + = s(s — a) + 5 (^ — 6 + 5{s —c)
— s(3s — a — b — c) )

= 5 3 s - 2s) =
(

EXERCISES 6 (6)

If E is the middle point of the side CA of the triangle ABC and if

A is the area of the triangle, prov’e that


BC* - BA^
cot A EB = (L.U.)
4A
2. IfA is the area and R the radius of the circumcircle of the triangle
ABC, prove that cos + cos {B — C) = 2A/rti?.
3. If Athe area and r, r^, r^.
is are respectively the radii of the inscribed
and the three escribed circles of a triangle, prove that A = (rri^gr,)
If r, are respectively the radii of the inscribed and the three
r^, r^.

escribed circles of a triangle and if R is the radius of the circumcircle,


piove that ri A- f 2 + ~^ (L.U.)

If r^, r^, ^3 are respectively the radii of the three escribed circles of
a triangle ABC and if R the radius of the circumcircle, prove that
is

rj = AR sin lA cos \B cos ^C, rg = 47? cos lA sin JB cos jC,

y, = 47? cos ^A cos iB sin JC.


6] ANGLES IN TERMS OF SIDES 101

6. If r, fi, r^, arc respectively the radii of the inscribed and the three
escribed circles of a triangle ABC, prove that
cot* cot* cot* JC. (L.U.)

6.7. Formulae for the angles of a triangle in terms of the sides


By rearranging the cosine formula
= 6^ + c* — 26c cos A
in the form
^ = (62 -I- c* _ a^)/2bc,
cos
we can express the angle A in terms of the sides
This of a triangle.
formula is not, however, well suited to computation by logarithms
and we develop alternative formulae as follows.
From the formula (6.8) we have
tan iA — r/{s — a).
Using (6.7), r = A/s, so that

and using the expression (6.6) giving the area in terms of the sides,
we have
tan iA = V^- «)(s -b){s-c)}
s(s — a)
or, tan
{s — b)(s-c )
lA
s(s “ a)
(6.13)

The corresponding formulae

^
taniB- / r (s-c)(s-an
/f
VI s(s-6) /
= ~
tan iC
s(s c)
}
can be similarly derived.
If in (6.12) we use the formula A = Ibc sin A instead of (6.6) we
have

tan iA = = 6c sin jA cos jA


2s(s - a) s(s — a)

=
« •

givmg cos* iA
6o

or, cos iA
s(s — a)
(6.14)
be
Multiplication of (6.13) and (6.14) gives

-M - (6.15)
c

102 PURE MATHEMATICS


and, of course, there arc formulae corresponding to (6.14), (6.15) for
the angles B and C,

Example 5. Prove that in a triangle ABC,


-
1 1 1 1 1 .1
=
(a + 6 + c)»

a
COS^ -A
2-
+ - COS^ Tfi
b 2 c
cos* -zC
2
J-T
4abc

From (6.14) and the two similar formulae,


1 1 1 s{s — a)/ + — 6) + s{s - - c)
- cos2 + x cos= i/? 4- - r.os*
+ cos» iC = v

b
3s* — s(a + + c)

abc
3s‘ - 2s*
abc
(g + 6 + c)«

abc 4abc

6.8. The tangent formula


We now derive another set of formulae which are useful in the
numerical solution of triangles. Starting from the sine formula (6.1),
b = 2R sin'B, c = 2R sin C, and after division of numerator and
denominator by 2R, we have
— c _ sin R — sin C
5
b c sin Z? + sin C

_ 2 cos J(S + C) sin 1{B — C)


" 2 sin J(“BTcy^^l(^ - Q
= cot i(B + C) tan i{B - C).
Hence tan l(B — C) =
-|-
)
and since \(B C) = 90® — \A, this can be written

tan \{B-C)=
The two corresponding formulae

tan J(C -A) = “t IB. tan i{A - B) =


can be similarly derived.

Example 6. Prove that in any triangle ABC,

sin i{B-C) = cos lA

Working as in § 0.8, we have


ft — c sin B— sin C
a sin A
sin B— sin C
^ sin (i3 +Q
*
6] SUMMARY OF FORMULAE 103
since A *» 180® —B~ C. This can be written
b c 2 cos J(B + C) sin |{B — C)
a 2 sia + C) cos + Cj
_ sin {{B - C) _ sin - C)
'
sin i(B + C) cos
since \{B
C) +
90® =
{A. — The result follows immediately after
multiplication by cos ^A,

EXERCISES 6(c)

1 . With the usual notation for a triangle ABC, show that

sin.^ = b){s-c)).
oc
2 . Prove that in any triangle ABC,
(a + 6 -f- c)(tan + tan JB) = 2c cot JC.
3. Show that in a triangle ABC,
b A- c = a cos J(B — C) cosec \A,
4. Show that in a triangle A BC, if 2s = a + 6 + c,
1 — tan \A tan |B = (L.U.)

6 . Assuming the cosine formula for a triangle ABC, prove that


/rt^ -*)(*-")
smi^= (L.U.)
he
6. If in a triangle ABC,

prove that a = (6 - c) cos \A sec


(L.U,)

6.9. Summary of formulae for the triangle


The more important relations between the sides,
angles and area
of a triangle and the radii of its associated circles
are here collected
for reference.
If R
the radius of the circumcircle, r that of the
is
inscribed circle
f„ fa the radii of the three escribed circles,
the area of the A
triangle and 2s a A = + +
c, then

^ b
^ C ahn
(6.47
sinil sinB sin C 2A'
— 2Ac cos A,
-j- c®
(6.2)
A = -^{s{s - a){s — b){s- e ) }, 6 6)
.
(

tan IA=
~ - i I
c)
l
(6.13)
s(s -a) I*
tan - C) = cot iA,
(6.16)
104 PURE MATHEMATICS [0

r = A/s = (s — i?) tan = (s — b) tan JS = — (s c) tan JC, (6.8)

= — = s tan lA,
r. 6 10 6 11 )
— s a (
.
, .

and similar relations for rg, r^.

6.10. The numerical solution of triangles


When three elements of a triangle, one at least a side, of which is
are given the other three can be found. The sine and cosine formulae
(6.4), (6.2) are all that are required to effect the calculation but some
improvement in the numerical work involved can be made in certain
cases by using the half angle formula (6.13) or the tangent formula
(6.16). However, we shall start by giving examples in which only
the sine and cosine formulae are employed.
It is advisable to set out the computational work in a systematic
manner and to employ checks whenever possible. Some suggested
lay-outs are shown in the examples given in the following paragraphs.

6.11. Examples of the use of the sine formula


(i) One side and two angles given
In this case the remaining angle can be found immediately from
the fact that the sum of the angles of a triangle is 180®. Suppose the
given side is c, then since the angle C is known, the diameter 2R of
the circumcircle can be found from

sm C
The remaining sides a and b can then be calculated from

a = 2R sin A, b ~2R sin B,


the angles A and B being known. The adaptation of the method to
the case where either of the sides a or i is given instead of c should
cause no difficulty.

Example 7. Solve the triangle in which


c = 26-83 in., A = 80* 30', B= 40* 12'.

C = 180* - {A B) No. log.


= 180 — (80 30 4- 40 IJ )
26-83 1-4286
= 180* — 120*42' = 59* 18'.
sin 59* 18' 1-9344

27? = c/sin C = 2C-83/sin 59* 18'. 27? 1-4942


a = 27? sin ^ = 27? sin 80* 30'. sin 80* 30' 1-9940

5 = 27? sin B= 27? sin 40* 12'.


a = 30-77 1-4832
The required solution is therelore 1-4942
27?
C == 59* 18', sin 40* 12' 1-8099
a = 30-77 in.,
"fc = 20-14 1-3041
b = 20-14 in.
6J THE SOLUTION OF TRIANGLES 105

Fig. 37

(ii) Two Sides and the non-included angle given {the ambiguous case)
To fix ideas, we shall take b, c as the given sides and B as the given
angle. The angle C can be found from the sine formula in the form

^ c sin R
.

sin C = (6.17)

Various possibilities may arise.

{a) The side b may be sufficiently small for 6 < c sin B. This would
require sin C to be greater than unity and no triangle will exist
with the given values of b, c and B. This is illustrated in Fig. 38.

38 Fig. 39

(6) The given values and B may be such that 5 = c sin B. In


b, c
this case, sin C = 1 and C — 90°, Here the triangle is right-
angled at C (Fig. 39). Since, when one angle of a
triangle is a
right angle the other two are necessarily acute, this
case can only^
occur when B < 90°.
(c) When fi, c and B are such that 6 > c sin B, sin C < 1 and there
^ be two values of C (less than 180°) which can satisfy
(6.17). One of these values of C, say
equation
will be acute and the
other, Cj, will be obtuse. We now have to enquire if both these
values give possible solutions.

If B is obtuse, then
the only obtuse angle of the triangle and the
it is
3Jiglo Cj must be excluded as a
possible solution.
If B is acute, values of C greater than 90° are not immediately
excluded.
2

106 PURE MATHEMATICS [6

If,however, h >
c, such values are excluded on the grounds that the

angle C would then be greater than the angle B and the greater angle
would be opposite the lesser side.
4 For the case B 90°, b c, both < <
values Cl, of C are possible
(Fig. 40). This case gives rise to two
possible triangles ABC^, ABC 2 and is
often called the ambiguous case.
To sum up, when the sides b, c and
the angle R of a triangle are given, we
have to consider the following cases.
(a) b < c sin B. There is no solution.
{b) b = c sin B. There is one solution and the triangle is right-
angled at C.
(c) b c sin B. If R > 90°, there is one solution and C is the acute
angle derived from equation (6.17). If B 90° and b c, there < >
is one solution and C is the acute angle derived from (6.17). If

< <
B 90° and 6 c, there are two possible triangles ABC^, 0- ABC 2
1-
,

the angles Cj, Cg being respectively the acute and obtuse angles
satisfying equation (6.17).
Once C has been found, the remaining angle and side are then
found as in Example 7. In the ambiguous case there will be two
values a-i, two values
of the side a to be found corresponding to the 0-
A^, A deduced for the angle A. Some numerical examples 1- follow.

Example 8. Js there a triangle in which b = 5, c = 7 and B = 48® 35'/


Jf so, solve the triangle.
0-
Tlio value of C is given by No. 1- log.

sin C = ——=
csinB
r
b
7 sin 48® 35'
5
7
sin 48® 35'
8451
8751

0-7202
This leads to a value of log sin C of 0-0212
5 0-6990
and hence sin C = 1-05. Since this is greater
than unity, there is no possible triangle with sin C 0-0212
the given sides and angle.

Example 9. Solve the triangle in which b = 5*6, c =s 7*0 and B= 53® 8'.

Here No. log.


7 sin 53® 8'
sin C = . 7 8451
S'O sin 53® 8' 9031
This gives log sin C = 0, sinC = and 1
0-7482
C= 90®. The remaining angle A is given by 5-C 0-7482
A = 180® - (/i + C)
= 180® — (53® 8' + 90®) sin C 0-0000

= 3G® 52'. 7 8451


we have sin 36® 52' 7781
For the side a,

30® 52' a = 4-200 0-6232


a = —.----.sin 4 = ..a®
sin 90®
.sin
s«n C
= 7 sin 30® 52' = 4-2.
6] THE SOLUTION OF TRIANGLES 107

The solution, is therefore A = 36® 52', C= 90®, a «= 4*2.

Example 10. Solve the triangle in which b = 24*93 cm., c =« 12*10 cm.,
B= 122® 51'.

Here No. log.


12*1 sin 122® 51' 12*1 sin 57® 9'
sin C = 12*10 1*0828
24*93 24*93
sin 57® 9' 1*9244
since sin 122® 51' = sin (180® - 122® 51').
1*0072
Hence log sin C= 1*6105, giving C= 24® 4',
24-93 1*3967
an obtuse value for C being impossible as B is
greater than 90®. sin C 1*6105
A = 180® - (B + C) 1210 1*0828
= 180® - (122® 51' -f 24® 4')
sin 33® 5' 1*7371
= 33® 6'.
0-8199
a = .sin A sin 24® 4' 1*6105
sin C
12*10 a ^ 16-20 1*2094

sin 24® 4‘
.sin 33® 6' = 16*20.

The solution is therefore


A * 33® 5', C= 24® 4', a = 16*20 cm.

Example 11, Solve completely the triangle in which b » 2*718, e = 3*142,


B = 64® 18'. (L.U.)

No. log.

3*142 0*4972
sin 64® 18' 1*9096

0*4068
2-718 0*4343

sin C 1*9726

3*142 0*4972
sin 65® 61' 1*9178

0*4160
sin 69® 61' 1*9726

Oj = 2*770 0*4425

3*142 0*4972
3*142 sin 64*18' sin 16® 33' 1*4283
sin C
2^7l8 1*9266
Hence log sin C = 1*9725 giving Cj = 69® 61' sin no® 9' 1*9725
and C, « 180* - 69® 61' = 110® 9'. since
this is the ambiguous case in which B
a, = 0*8974 1*9630
<
and 6 < c. If we denote the angles BA Cj,
00®

BAC^ respectively by A^ and A^ we have


(i) Aj *= 180® - (64* 18' + 69® 61') « 180* — 124* 9' = 65® 61'
By the sine rule, if BCi = a,.

3*142 sin 66® 51'


sin A, 2*770,
sin C| ^69® 61'
!

108 PURE MATHEMATICS [6


and
(ii) = ISO° - (54“ IS' + 110“ 9') = 180° — 164° 27' = 15° 33',
so tliat if BCj = flj.

3142 sin 15° 33'


a, =
sm
sin A,* = = 0-897.
C, sin 110° 9'

The required solution is therefore,


^=55° 51', C=69°5r. a = 2-770.
or. W = 15°33'. C=110°9', a = 0-897.

6.12. Examples of the use of the cosine formula


(i) Tzc’o sides and the included angle given

Suppose for example that the two sides b, c and the included angle A
are given. The side a can be calculated from the cosine formula

^2 „ ^2 _j_ ^2 _ 2bc cos A,


tables of squares and square roots being useful in the arithmetical
work. The remaining angles B and C can then be found from the
sine rule arranged as

sm B
. ^ = 6 sin /I
and
, .^
sm C = c sin ^ •
a a
Since the sides are known, any question of the values of the angles B
all

and C is settled by taking angles which are in the same order of magni-
tude as the sides opposite them. Alternatively, once one of the
angles B ox C has been found from the sine formula, since A is given,
the other can be found from the fact that the sum of the angles is 180®.
0-

Example 12, Solve the triangle in which h = 10-67 »«., c — 21*7 1-


in..

A = 44° 40'.
+ — 26c cos A
c*
= {10-G7)» + (21-7)* - 2 (I0-67){21*7) cos 44° 46
= 584-8 — 328-8 = 25G-0.
a = lC-0 in. No. I
log.

bsinA 10-67 sin 44° 46' 6» = (10-67)* 113-9


c* = (21-7)* 470-9

Hence, log sin B = T0718. Since the 6* + c* 584-8


angle B, being opposite the smallest
necessarily acute, this gives
26 = 2 X 10-67 21-34 1.3292
side, is 21-7 1-3365
c
B = 28° 1'.
cos A cos 44° 46' 1-8512
C = 180° - {A A- B)
26c cos A 328-8 2-5169
= 180° — (44° 46' + 28° 10
= 180° — 72° 47' = 107° 13'. b 10-67 1-0282
sin A sin 44° 46' 1-8477
The required solution therefore,
is

B= 28° r. C = 107° 13', a 16*0 in. = 8759


a 16-0 2041

sin B ^6718
. —
6] THE SOLUTION OF TRIANGLES 109

(ii) Three sides given


When all three sides are given, the angles can be found from the
cosine formula arranged in the form

=
^2 -f- c2 _ a®
cos A
2bc
with corresponding formulae for cos B and cos C. Alternatively, one
of the angles can be found in this way and the other two found from
the sine formula.

Example 13. Solve the triangle in which a = IG in,, b = 10-67 in,,


c = 21-7 in.

cos A = No. log.


2bc
(10C7)>+(2l-7)*-{lC-0)»
6* = (10-67)* 113-9
c* = (21-7)> 470-9
2 (10-07) (21-7)
44® 46'.
6* + c* 5S4-8
a* = (16-0)* 250-0
B and C then follow as in
Example 12,
6* + c* — a* 328-8

2* « 2 X 10 C7 21-34 1-32U2
c 21-7 1-3365

2bc 2-6657

6= + - a* 328-8 2-5169
2bc 2-0657

cos A 1-8512
(

EXERCISES C(rf)

Solve the triangles in which-:


1. c = 15-6 in., B = 34® 20', C = 62® 9',

2. a = 60 cm.. 6 = 70 cm.. A = 02®.


3. fl = 17-00 in., b = 21-42 in., D = 61® 34'.
4. 6 = 107-2 ft., c = 76-69 ft.. B = 102® 25'.
6. 6 « 16-0 ft., c = 12-3 ft.. C = 34® 20'.
6. a = 7-00 in., b = 3-69 in.. C = 47®.
7. a = 17 cm.. 6 = 11 cm.. c = 10 cm.
8. Find the third side and the radius of the circumcircle for the triangle
in which a « 6-324 in., b = 8-222 in.. C = 64® 32'.

6.13. Example of the use of the tangent rule


As show when two sides and the included
in the last paragraph,
angle are given a triangle can be solved by using
the cosine formula
to find the remaining side and then the sine
formula for the remaining
angles, 'fhe cosine formula is not, however, well suited to work with
110 PURE MATHEMATICS [6

logarithms. Unless the given sides are two-figure numbers and there-
fore easy to square, it is best to use the tangent formula to deal with
this case.
Suppose the sides h, c and the angle A are given. Then the tangent
formula (G.16)

tan - C) = cot \A,

enables \[B —
C) to be found. Since 4- C) = 90® — \A, we can
then find B and C by addition and subtraction. The remaining side a
is then found from the sine formula.

Example 14. Solve the triavgle in which b = 10*67 in., c = 21*7 in.,

A = 44:“ 4G'.
Here b < c and it is best to rewrite the tangent formula as
1 /c -b\ 1
tan (C - B) = cot -A.
2 +b
1 21-7 - 10*67
Hence tan -{C - 13) = cot 22“ 23'
21.7 + 10 07
11*03
cot 22“ 23'.
32*37
Hence No. log.

i(C - B) = 39“ 37'.


11 03 1*0426
Also, cot 22“ 23' 0*3853
i(C B) = 90“ - iA 1*4279
= 90“ - 22“ 23' = 67“ 37'.
32-37 1*5101
By addition and subtraction,
B=
tan KC — B) 1*9178
C = 107“ 14', 28“.
21*7 1*3365
c 21*7 sin 44“ 46'
= —— sin A sin 44“ 40' 1*8477
a - ‘
' •
oil'
I
sin 107 14
sin 6
1*1842
21*7 sin 44“ 46'
= 10 0 *
. sin 72“ 46' 1*9801
sin 72“ 46'
a 1*2041
Hence the required solution is

B = 28“. C = 107“ 14', a = 16*0 in.

of solution of triangle with three


6.14. Alternative methods
sides given
formula, rearranged so that the angles ^e e.vpressed
The cosine
the sides, again not well suited to work with logarithms,
in terms of is
probably best to use formula
l" auThree angles am to be
found it is

(6.13),
/ r(s _ h)(s - c)
tan \A = s(s — a)
Vi
for B and C. To save repetition in the
and tlie two similar formulae
written
logarithmic work, this can be
6] THE SOLUTION OF TRIANGLES 111

tan M - (6.18)

with corresponding expressions for B and C. The logarithm of the


expression under the square root sign has only to be worked out once
and the log tangents of the half angles then follow by subtracting
log (s — a), log (s — b) and log (s — c).

Alternatively, the angles could be calculated from (6.14) or (6.15),

or from
sin = 2A/6c
= ^V(sls - a){s - b){s - c)},
a formula which is easily deduced from (6.4) and (6.6).

Example 15. Solve the triangle in which a = 16, 6 = 10*67, c = 21*7.

We first find s from the


bg.
formula
2s == a b + c. « 16
Then form {s — a), (s — b), ^ 10-67
{s - c).
^ 21-7

A check is provided from 2s = sura 48-37'


w — s.
f^ /%\ i
o; -r t* “* /A m 0 -- - -
...
I*
We then find ^ 24-10
log {(s - a)(s — 5)(s ~c)/s) s - a 8-10 0-9133
by adding the logarithms of s — b 13-52 M310
(s — a), (« — b), (s — c) and s ^ c 2-49 0-39G2
subtracting that of s. The ^w.. .x, v
2*4405
logarithm of the square root ^
J 24-19 1*3830
of this ouantitv follows bv
division by two, and the — &)(« — c)/s 1-05G9
angles aie found from (6 18)
" 0*5286
s ^ a
and two similar formulae. *
0-8133
From the workincf on th€> riaVif —
we find tan ^^4 1*6152
ia4=22-24'. 44M8', V {{s - a){s ~ b)is - c)/s) 0*6286
iB = 14* 1', 28® 2^ s-b 1*1310
IC sa B3® 36' r 107® 10'
There is a
check that the
final 1*3976
sum of the angles should be V fU — a) — 6 — c)/s\
(s ) Is 0*6285
180®. As there is a possible s —c 0*3962
error ox ar least nau a zzunute
in each angle due to the use of 0*1323
four-figure tables,
the slight
difierences between the angles
found here and those in
Examples 12 and 13 and the
slight difference of .<4 B+C
from 180^ is not surprising.
112 PURE MATHEMATICS [6

EXERCISES 6 [e)

1. Calculate the remaining angles of a triangle in which two sides are


13-45 cm., 54-31 cm. and the included angle is 67® 24'.
2. Find the angles of a triangle whose sides are 10-4, 12-8 and 17-6 ft.

3. In a triangle a ~ 4-832 in., b = 2186 in., A -- B = 34° 16'. Solve


the triangle.
4. In a triangle ABC. h = 6-27, c = 4-32, A = 51°. Calculate the angles
B and C and the length of the internal bisector BD of the angle ABC.
(L.U.)

5. The sides a, 6, c of a triangle are respectively (A* +A + 1). (2A + 1)

and (A^ — 1) where A > 1. Show that the angle A is 120°.

6. Find the area of a triangle having sides of length 322-2 ft., 644-7 ft.

and 432-1 ft. (Q.E.)

6.15. Heights and distances


In the work of the surveyor and in navigation, elementary
trigonometry finds an important application. It is often possible by
measuring certain distances and angles to calculate other distances
and angles which cannot be measured directly. Such calculations are
usually simply practical applications of the formulae relating to the
sides and angles of a triangle. We give below a few typical examples
explaining the few technical terms used in this type of work as they
occur.

Example 16. The angle of elevation of the top of a vertical tower from a point
A is a. From a point B, in a direct line between A and the foot of the tower
and at distance d from A the angle of elevation to the top of the tower is p.
,

Find the height of the tower.

angle of elevation of the top T of the tower from A is the angle


the
The
line A T makes with the horizontal through A and the foot of
the tower.
this angle
If the level of T had been below that of A we should speak of
as the angle of depression.

the tower, T being the top


In the diagram, TT' is
T AT f
ABT' is horizontal, TT' vertical, the angle 'YrfJlt,
oi T from A [ntB
distance AH
is d and the angles of elevation a,

arc as shown.
'

HEIGHTS AND DISTANCES 113


Applying the sine formula to the triangle ABT, since the external angle
T'BT is equal to the sum of the angles TAB, BTA and therefore the
angle BTA is — <x, we have
TB d
sin a sin — «)*

. .

giving
^
TB =
d. sin a
sm - a)
The right-angled triangle TBT now
gives for the height of the tower
TT = TB sin
d sin a sin
sin ip — a)*
Thus, by me^uring two angles and one distance, this formula
will
enable the height of an inaccessible object to be found.

Example 17. The angle of elevation of the top T of a vertical tower


from a
point A is a. B is a second point on the same level as A and the foot T*
of the tower. B
is not in the straight line joining A and
the foot of the
tower and the distance is c. AB
The angles TAB, TBA are measured
and found to be y and S respectively. Derive a
formula giving the heiehf
of the tower in terms of a, y, 5 and c.

From the triangle AST,


AT
sin^rs’
Since the angle ATB is 180» -y- S, this gives

sm (y + 8)
The right-angled triangle AT'T, g-ives
TT' = AT ain «,
c sin a sin 8

when the
sin (y + 8)’
expression for is subsUtuted.

.V. c,7. V -
14. .ri'j
m‘«" •W'lr- ~
B
114 PURE MATHEMATICS (6

I'lG. 44

The angle ^CB = 180® — 43® 20' - 65® 52' No. 1


log.
= 180® — 109® 12' = 70® 48'.
no 20414
ABC
1

From the triangle sin 65® 52' 1-9003


sin 43® 20' 1-8365
no sin 65® 52'
"" 48'"'* 1-8382
sin 70®
sin 70® 48' 1-9751
If CD is perpendicular to AB, the right-
angled triangle ADC gives CD = 72-96 1-8631

CD = .^Csin 43® 20'


no sin 65® 52' sin 43® 20'
^^0® 48'
= 73 ft.

EXERCISES 6(/)

1. An aeroplane is ohserv'ed at the same inslant from three stations


through
A. B.Cin a. horizontal straight line but not in a vertical plane
the aeroplane. If = BC = c and the angles of elevation from
A. B,C are respectively a, prove that the height of the aeroplane is
y
,
(L.U.)
(cot* a A- cot* y - 2 cot*

2. An aeroplane is observed simultaneously from two points A and B,


north of B. From
at the same level, A being at a distance c due
the bearing of the aeroplane is 0 east of south at an elevation a and
that the aeroplane is
from B the bearing is 4> east of north. Show
from B. (L.U.)
at a height and find its elevation
sin (0 -I-

and 10 is he
3. A vertical rectangular hoarding 8 ft. long ft.

attached to^^*:**
in position by four equal stay ropes, two
the bottom end of each rope being fixed
to a point ® ST m
vertices of a square of ^
side 12 ft. calculate
If these four points are the
inclination to the honzontol. (UU.
the length of each stay and its

tower AB. of height H. and the base of X


4. The base ^ of a vertical and > A. H
H. are on level
a vertical flagpole Xy. of height respective y.
of Y from A and B are a and
The angles of elevation
Prove that H = h sin a cos /? cosec (a — ^ i

TaUng A = 40 ft,, a = 30“ and p = 10“. find the

from X, giving your answer to the nearest tenth of a de^ee. f


of B
*

6] EXERCISES 115
5. A straight path rises at an angle 6 to the horizontal ; O, P and Q
arc three points on the path, being higher than O, P and Q higher
than Pthe distance OP is x ft. At Q there is
: a vertical pole QP,
the height of P
above Q being h ft. Prove that, if QR subtends
angles a and ^ at O and respectively, then P
X sin a
h
sin
— a) cos d
—— /3

sm
Prove also that the height of R above the level of O is

~
X sin 8 sin (a -f
n (/?-„)
6)

(O C-)

6. A tripod consists of three rods AB, and AD which are 100 ft., AC
100 ft. and 125 ft. long respectively. The ends B. C and D of the
three legs stand on a horizontal plane D is equidistant from B and ;

C and 50 ft. from the line BC. Find the height of the apex A above
the ground, given that the angle is 25®. BAC
(Q.E.)
7. A, B, C are three towns B is 10 miles from
:
in a direction 47® E.
of N. C is 17 miles from B in a direction 20® N. of W. Calculate
;

the distance and direction of A from C.


(O.C
8 . A horizontal tunnel AB
is bored through a ridge in
a direction per-
)

pendicular to the line of the ridge, and


over ndge. Show that if / is the length
a path goes from .d to B
of the tunnel, and a.
are the inclination of the two portions
of the path to the horizontal
^
the height of the ridge above the
tunnel is
/ sin a sin ^
sin (a + ^)*
What is the length of the path = 1000 yd.,
if I a = 10® and ^ ^ 7^® ?
(O.C.)

EXERCISES 6(g)
1. Assuming that, in any triangle ABC,
sin A sin B sin C

prove that
ct + b — c
'
tan lA tan IB.
a + b c
Calculate the value of c for the
triangle in which
a 6 + =
18-6 in., A 72® = 14^ B =- 46® 42 '.
2. (O.C.)
If in any triangle ABC,
= h^cosiA.
sin 9

prove that (6 + c) cos 6 = o.


For the case b * 123, c = 41*2
sin 9 and hence
^ « 4ft®
A ’
Kft' ^“<1 au
«>e vai ue of
the value of a.
3.
,

116 PURE MATHEMATICS


terms of those of the triangle ABC and prove that the perimeter
of the triangle DEF is
a cos A + b cos D c cos C. (L.U.)
Prove that in a triangle ADC,
a + 6* + c* = 2{hc cos A F c(i cos B F ab cos C).
In a triangle ABC, X is the mid-point of the side BC, Prov’e that
26 sin C
sm AXD
A Or, =
V(262 -1- 2c* - a*)'

a sin C
and that sin XAC = ^2'6* -
8. -H 2c* a*)'

6 . The area of a triangle is 336 sq. in., the sum of the three sides is

84 in. and one side is 28 in. Calculate the lengths of the other two
sides. (L.U.)
9.

7. The angle A of a triangle ABC


60® and the area of the triangle
is

is equal to that of an equilateral triangle with sides of length x.


Show that 6* + c* — a* = .*•*.
In the triangle ADC the incentre and circumcentrc are at the same
distance from the side BC. Prove that
4 sin ^A sin \B sin \C = cos .<4,

and deduce that cos B + cos C = 1. (L.U.)

The radii of the incircle and circumcircle of the triangle ABC are
r, R respectively. Prove that the area of the triangle ADC is
r* cot ^A cot \B cot ^C.
If thetangents at B, C to the circumcircle meet in D. E, F, prove
that the area of the triangle DEF is
B* tan A tan B tan C. (L.U.)

10. Prove that the area of a triangle ABC is 2/?* sin A sin B sin C where
R the radius of the circumcircle.
is
If r is the radius of the circle inscribed in the triangle
ABC, and
if DEF
is the triangle formed by joining the
points of contact of
the inscribed circle with the sides, prove that
(i) the area of the triangle DEF is 2r* cos ^A cos \B cos ^C
(ii) the radius of the circle inscribed in the
triangle DEF is

2r cos lA cos JB cos fL.U.l


cos p + cos JB + cos iC
11. O is the centre and R the radius of the circle circumscribing the
in L. M,
triangle ABC. AO, BO and CO meet tlic opposite sides
N respectively. Show that
6 sin C
(‘J " cos (B - C)
'

2
1 1 1
_ (L.U.)
EXERCISES 117
12 . In any triangle ABC prove that
6 — c sin 4(B — C)
a cos \A
Theinscribed circle of the triangle touches at ABC
and one BC E
of the escribed circles touches BC at F, If 5 c, prove that >
BE = 5—6
and => b EF
c, where 2s = a —
b 4- c.
If -4 =
36® 42', a = 4*32 and EF
= 1-67, calculate the lengtlis
of 6 and c.
(L y j
13. In the triangle ABC. prove that c cos ^{A — B) = (a -|- &) sin §C.
the sum of the lengths of two sides of a triangle is 21
If
in., the
length of the third side is IS in. and the angle
opposite the third
side is 52°, solve the triangle completely.
(L.U )
14. If 2s = a + b + c, show that the area
of the triangle A BC is given
by s* tan ^A tan |B tan }C. ^
Find the angles and area of a triangle whose
sides are 413 328 *
and 167 in.
15. In a triangle ABC, prove that

tan B cot C =
a« _ + <;»•

16. Find the difiference between the areas


o£ the two possible triangles
ABC in which A = 29» 13'. a = 81-47 It., b = 102-3 ft.
17. In a triangle .4 BC, a = 18-9 in,,
(Q E )

b = 12-2 in., ^ - B = 37». Find


the values of A and c.
18.
^ j

IneleTse* *Z° ^ ‘"c'^ded


’ remaining angles when Uie third
side is 16 3 cm
19.

i^loOS^^ft^anV"'’
ISiud 8 sq. ft. and in which = 65 ft,, 6 4 ^ = 97
a .
20 . The Sidra AB. AC Ota. triangle ABC are equal to one another
The
^ ^ “tt the area of the triangle.
!
21 .
Bis 4 mara'‘/3Py7r;m "J^rc'Ts
S ToMsTf
22 .

a w4‘«4of^0 t^dtnTLlf
straight line 100 »

w ^nYrh^o^t^fiir^ wirrLr
plane, find the height
of the tower.
r
ft

s*"** horizontal
23.

lo dmuto. » IS-.
118 PURE MATHEMATICS [G

24. Two towers A and R on a level plain subtend an angle of 90® at


an observer's eye. The observer walks directly towards B a distance
of 630 m. and then finds the angle subtended to be 143® 24'. Find
the distance of the tower A from each position of the observer.
25. At a point in the horizontal plane through the base of a circular
tower, the eIe\’ation of the top of the tower is
^ and the elevation
of the highest point of a flagstaff which stands on the top of the
tower in the centre is y. At a point a ft. nearer the tower, the
highest point of the flagstaff is just visible above the edge of the
tower at an elevation a. Prove that the height of the flagstaff is
a sin^ a sin (y — fi)
cosec (a — coscc (a — y).
- ;

CHAPTER 7

INTRODUCTION TO THE IDEAS OF THE


DIFFERENTIAL CALCULUS
7.1. Functions and functional notation
When two variable quantities x and y are so related that the value
of one quantity y depends on the value of the other x, then y is said
to be ^function of x. The relation containing the two variable quan-
tities may be a simple formula such as y + a; 2 or y = sin x, = +
or the relation may be expressed by means of a graph. Sometimes
the graph relating two variable quantities is available but it may be
impossible (or very difficult) to express the relationship by a formula.
For example, a recording barometer gives a record showing the atmos-
pheric pressure plotted against the time t the record displays p as
p —
a function of i but it is not generally possible to express p in terms
of / by a formula.
A means of expressing that y is a function of x when the formula
connecting the two variable quantities is not known or when we are
dealing with a general rather than a particular function of x is to write
y =5 J(x), y = <l)(x) or y =
This notation does not mean that
F(x).
y is / multiplied by x but is simply an abbreviation for the words
*'
function of x ”. The different letters . are used to denote .

different functions.
If y = f(x) and we wish to specify y for a given value of x, say
= 3, we write the result as /(3). For example, if f(x) = -f- x -f 2,
/(3) = 4x3^4-3-1-2 = 41 and this is the value of y when x == 3.
Similarly, = sin x,
if <l>(x) 1/^2. = sin^r/^ =
Functions such as we have considered above are termed explicit
functions. The variable quantity x is called the independent variable.
The second variable y, whose value depends on that given to x, is
refer.ed to as the dependent variable.
Sometimes the relation between two variable quantities x and
y
is given in a form such as
x^-{-y^ = 2x, or a; f- y 4- cosy =4
these are called implicit functions. In the first example given above
we could solve for y and obtain
y = ^/[2x - x%
^d in this form, y is an explicit function of x. In the second example
not possible to find y
it is explicitly as a function of x. We shall be
concerned in this book chiefly Tvith explicit ftmctions.
=
A function like y x® is a single-valued function of x — since, to
119
120 PURE MATHEMATICS [7

a given value of x, say x = there corresponds one and only one value
4:,

of y (^ — 42 — On the other hand, a function like y = y/x a is


two-valued function — for a given value of x, say x = 4, there are two
values y = i 2 which satisfy the given relation. In general, to if
a given value of x there corresponds more than one value of y,
y is
said to be a multi- or many-valued function of x.

7.2. The gradient of a curve


It is often helpful to depict the variation of a function by means
of its graph. Suppose, to fix ideas, we consider the simple explicit
function y = x'^. The graph is easily drawn and is shown in Fig. 45.

We notice that as increases from zero through positive values,

y increases. The reverse is true when x increases to zero from negative


values. We could describe the changes in y as x increases by saying
that y decreases so long as x <. 0 and that y increases when x 0. >
If Xj, yj are the abscissa and ordinate of a point P and X 2
those ,

to x^
of a point Q, the change in the value of y as x changes from Xj
is yg — The average rate of change of y as x changes from Xi to
yi-
Xj is defined as
y^ -y\
Xo — Xi
seen to be the tangent of the angle QPR.
and from the figure this is
This quantity is referred to as the
PR being parallel to the *-axis.
clearer to take
slope of the chord PQ. It may help to make matters
the distance J^oved y
variables s and t instead of x and y, s bemg
boS in time f. Instead of the graph y we =
should have the
=
GRADIENT OF A CURVE 121
“ space-time graph s = The average rate of change of distance
as changes from
t to would be {s^ — Sj)/(t — /J and this is simply 2
the average speed of the body in the time interval — ^i).
Let us now consider a numerical example. Take the simple func-
tion y =
whose graph is sho\\m in Fig. 45 and take the abscissa of
the point P to be ~ 2. The ordinate of P is 2^ 4. If we = =
take the abscissa of the point Q to be 3, its ordinate =
3^ 9. =
The tangent of the angle QPR, or the slope of the chord PQ, is

If we take another point Q^, nearer to P than Q, say one whose abscissa
is 2-4, its ordinate will be 2-42 or 6-76 and the slope of the chord PO,
VI
will be

Taking other points Q^, Q^, , each one nearer to P than the
. ,

preceding one and taking, for example, their abscissae


to be 2*2 2*1
2 05 ... the slopes of the chords PQ^.
PQ„ PQ^ calculated
m the same way, are 4-2, 4-1, 4-05, . . ,

When we join the point P


to points Q, Q„
latter approach nearer and nearer
Q„ Q„ as the
to P, the slopes of the chords PQ,
PQi, PQ^> - . decrease and the
.
calculated values of these
slopes suggest that they are approaching
a limiting value which might
well be of magnitude The same
4. is suggested by Fig. 46 which is
a magnified version of part of the
curve of Fig. 45.

To decidethe slopes of the various chords


if
do in fact approach
approaches P
and to find the value of tWs limit
let the abscissa of
+
0 be 2 A. The ordinate of Q is (2- A)« e +
,

122 PURE MATHEMATICS [7


slope of the chord PQ is

(2+ /i)2_22_A(4 + ;i)


^
(2 + A) _ 2 h
^
,

As Q approaches P
the value of h becomes smaller and smaller and it
is apparent that the limiting value,
as h decreases to zero, of the slope
of the chord PQ is 4. In other words, we can find the abscissa (2
of a point Q so that the slope of the chord PQ differs from
h) +
4 by as
little as we please. For example, if we wish to find a point Q such
that the slope of the chord PQ is 4-0001, we have to take the abscissa
of Q to be 2-0001. through P with slope 4 is the limiting
The line
position of the chord PQ as
Q approaches P and is the tangent to the
curve at P. The slope of this tangent line is defined as the gradient
of the curve at the point P.

7.3. The increment notation


A
convenient notation for a small increase, or increment, in the
value of a variable x is the symbol called “ delta x This notation
does not mean <5 multiplied by x but dx x^ = —
x where Xi differs
from X by a small quantit 3^ When is a function of x, the symbol 6y
y
is used to denote the change in the value of
y corresponding to a
change 6x in the value of the independent variable x. In Fig. 47,

Fig. 47

P a point on the graph of a function y


is =
f(x) whose abscissa and
ordinate arc x and y respectively and ? is a neighbouring point on
the graph with coordinates x dx and y dy. and are the M
projections of the points and Q on the ^-axis, and PR is drawm
P
parallel to NM. Then ON = x,PN= y. OM = x dx. QM=y^ 6y.

It follows that QR = QM - RM ^ QM - PN = y +
dydy -- y ^
and that PR = NM = OM - ON x dx x dx. The average - =
rate of change oi y zls x changes io x -\r dx is measured
by the tangent
ratio dy/dx. The
of the angle QPR and this is clearly equal to the
7] THE DIFFERENTIAL COEFFICIENT 123

gradient of the curve at the point P, or the slope of the tangent to the
curve at the point P is the limiting value of the ratio by/dx as bx
approaches zero.
As an example of the use of this notation, let us find the gradient
of the curve ^ at the point P whose coordinates are {x, y). At
the point P we have ,

y ==
and at the neighbouring point Q whose coordinates are {x bx,y -\- (5y),

y + by ^{x-^ bx)\
By subtraction,
<5y == (^ + bxY — (2x -}- bx)bx,

and ^ = 2;r 4- bx.


bx
As bx approaches a convenient notation for which is
zero, as
bx >*0 ”, it is clear that this ratio approaches the limiting value 2x.
The gradient of the curve y' —
x^ at the point {x, y), or the slope of the
tangent to the curve at this point, is therefore 2x. At the point
where x 2, = this result gives the gradient of the curve to be 4 as
found in the last section.

EXERCISES 7 (a)

If f{x) = 2x* — I, find the values of /(2), /(O) and /(— 1).
X
If F(/) = 3/ — 2, find the values of F(i) and F(—
+ What
J).
values of make F{t) « 0 ?
t

I _ sin e, find the values of ^(0) and 4>{n/2).


3. If ^(0)
What values
of 0 make ^(0) — 0 ?
4. Express y explicitly in terms of x when
(i) + 4y* = x, (ii) xy y* ^ x*.
6 . The distance s ft. travelled by a body
time / sec. is given by the in
formula s = + 2t. Find the average speeds of the body in time
intervals of
^ and sec. from the commencement of its motion.
6. P and 0 are neighbouring points on the curve = 2(x — x*), P
the point {x, y) and
y is
Q the point [x -f
of the ratio by/bx and determine
y + <5y). Find the value
the gradient of the curve at the
pomt P.
Find the gradient of the curve = at the point whose abscissa
y
IS Xm

7.4. Differentiation from first principles. The differential


coefficient
The
process of calculating the ratio of the
incremental change in
a fimcUon of * to the incremental change in
*, that is of deter-
muung an expression for dy/6x, and then
finding the limiting value
124 PURE MATHEMATICS [7
of this ratio as 6x approaches zero, is known as
differentiation from
first principles. The limit found in this way is generally denoted by
^V
the symbol — and is called the differential coefficient of
y with respect
to X.

Alternative notations for the differential coefficient of a function


f(x) of X are

and alternativ^e terms for this quantity are the derivative or derived
function. The process of differentiation
that of finding the gradient
is
of the curve representing the function under consideration and the
differential coefficient or derivative is the slope of the tangent to the
curve at a given point.
Consider as an example the function y = 5x\ To differentiate this
function from first principles we have
= bx\
y
y + dy — b(x 6x)\
By subtraction,

dy = ^ [(x — x^} = 5{2x dx)dx.


dx)^ -f-

Hence = 5{2x + dx) = \0x + 5dx,


^
OX
and, since the limiting value as (5-c—>-0 of the expression on the right
is lOx,

^
dx
= lim.
^-*oydx/
= 10;,.

Thus, if
y = the differential coefficient of y with respect to x
S.r-, is

10;t. Alternatively we could express the result as

^{5x^) = lOx.

or, if f{x) — f’(x) = \0x.

For a general function = f(x), we should


y have
/(-^ + 6x) - f(x)
^^ lim. f 1

dx 6x y
7.5. The differential coefficient of n a positive integer
If y = ;r«, then
y + 4v =
and, by subtraction,
= (x <5^:)” — x^.
7J DIFFERENTIATION OF SIN ^ AND COS ;e 125

Since nassumed to be a positive integer, {x -f


is can be expanded
in a terminating series by the binomial theorem and we have
”^”
dy = x’' + tixo-^dx) + + . + (•5*)’“ - x’^
2? • •

= nx’'-^dx) + ~ ^^
x-^idxy + . . . + ( 5 a:)".

Division by dx gives

fx
= + -' "a? + • . • + (5.v)"->.

Since the terms on the right except the first contain Sx raised to
all
a positive index, the limit of this expression as 6 x approaches zero is
and we have the result that if
dy
y = X” -I- = nx^~\ (7.2)
dx
The same result holds when n is negative or fractional but we shall
delay a proof of this until
pp. 140, 141.

7.6. The differential coefficients of sin* and cos*


If we take y = sin *, then

y + dy = sin {* + dx),
and, by subtraction,
6y = sin (* d*) — sin *
-|-

= 2 cos (* + J dx) sin ^ dx,


by formula (5.30). Division by dx and a slight rearrangement gives

N«, tam (6.34) „ &->0. (.In fa) tend, to unity,


cos (*
nnd
-f- i dx) tends to cos *, so that if

= cos *.
^ (7,3)

Similarly if
y = cos *, y + dy = cos (* + dx) and
dy = cos (* 4- dx) — cos *.
This can be written in the
form

3- = -sin(z + jfa).(«ii.;
and the limit of dy/Sx as 5x tends to zero is now - sin *. Hence if

dy
y=:cos*, / = -sin*. (7.4)
dx
126 PURE MATHEMATICS [V

Example 1. Differentiate from first principles, -f- x^.


Let y = x^ then y + 8y = {x 8^)3 {x + 8a-)*.
By subtraction,
8y = {x + Sat)* — jc* + (jr -{- Sat)* — x*
= 3x^hx) + 3Ar(8Ar)* + (5 a:)* + 2A'(8Ar) -f (5 a)».
8y
Hence — = Sat* + 3Ar(5A:) + (8a-)* + 2a: + 8^.

and the limit of this as 8x tends to zero is 3^* + 2;r. Hence the
derivative of a* -f a* is 3a* 2a. +
should be noted that the differential coefficient of this function is
It
the sum of the separate differential coefficients of a* and a*. This
is a special case of a general result that the differential cocfiicient of
the sum (or difference) of two functions is the sum (or difference) of
their separate differential coefficients (sec
§ 8.2).

Example 2. Differentiate \/x^ from first principles.

Here we let y = 1/a*, so that y 8y — l/(x + Sa)*.


1 1
Hence
(x + 8a)* a*
A* — (a + 8a)*
^
~^*(a + SaI*
- 2a(8a) - (5a)*
'

a'*(a +
Sy - 2a - (8a)
Thus
dx a»(a+8a)*'
and the limit as 8a —> 0 is —
2x/x*. Thus the derivative of I/a* is
“ 2/a*. It should be noted that this can be expressed by saying that
the derivative of a-* is 2a-*. Hence the result that if y = a",

dy
— = nx»-^, proved for positive integral n, holds when n = — 2.

Example 3. Differentiate 3a* -h cos 2a from first principles.

Let y = 3a* + cos 2a, so that y + Sy = 3(a + 5a)* + cos 2(a + 8a).

Then, by subtraction,
5y = 3{(a + Sa)* -
A*} 4- cos 2{a + 8a) - cos 2a
= 3 (2a + 8a) 8a —
2 sin (2a + 5-*^) sin 8a.

Sv /sin 8 a\
8a) —
Thus
^= 3(2a 4- 2 sin (2a +

and, taking the limit as 8 a tends to zero, we have

= 6a — 2 sin 2 a.
dx
coefficients of 3a*
Notice that this re.siiU is the sum of the differential
and that the differential coefficient of 3a> is 3 times tnc
and cos 2a
differential coefficient of a*.

EXERCISES 7(6)

Differentiate from first principles :

3. 1/(2 a*). 6. cos 3 a.


1. fiA^.
sin 2a. 6. A 4- sin A.
2. A* -A*. 4.
APPROXIMATIONS 127

7.7. The differential coefficient as a rate measurer


Suppose that a body moves a distance s in time t and that s is a
function of t given by s =/(/). We have already seen that if s,
are the distances moved in times t, respectively, the average speed
of the body over the time interval {t^ — t) is (sj — s)/(/j — t), or in
the incremental notation, Ss/dt. The limit of this quantity
as St
approaches zero is ds/dt. This is the rate of change of distance
with
respect to time and is the instantaneous speed of the
body at time t.
Similarly a body is moving in a straight line with velocity v,
if

the rate of change of velocity with respect


to time is dv/dt and this
ISthe acceleration of the body at the instant considered.
In general,
if a variable quantity
y is a function of another variable x, the differ-
ential coefficient dy/dx can be regarded as
giving not only the gradient
of the graph of the function but also the
rate of increase of with
y
respect to x
The sign convention adopted is that the function
is
increasmg where iU differential coefficient
is positive and decreasing
where its differential coefficient is negative.

Example 4. A
body moves in a straight line a distance
s ft. in t see. and
bind the velocity and acceleration
of the body after 3 sec.
ds
Velocity, v = -=3,..

Acceleration 0/.
dt
Putting / =s 3. the velocity and
acceleration after 3 sec. are respectively
27 ft./sec. and 18 ft./sec.*

Example 5. The diam‘Ur of an expanding moke


ring at ti,ne I is proportional
lot . If the diameter w 6 cm. afUr 0 see., at what rate is it then hanging >
UD is the diameter D=
kf »hcre k is a constant.
- 6 = Since D= fi whoa
/ 6, 36A so that A =3 1/6 and i»/6. D=
The rate of change of Z) =
When =
= ^ i cm./scc.

t 0, the rate of change of D is therefore 2 cm./scc.

7.8. Approximations

SfL tie" Hence

/'(x) -|- (T,


6x

This
iy = {/'(*) + cr}dx.
12 $ PURE MATHEMATICS [7

and as dx approaches zero, the second term on the right becomes more
and more insignificant compared with the first. Hence we may use
the equation
dy f'{x) dx (7.5)

as an approximate formula to find the effect on the value of a function


of a small change in the value of the independent variable.

Example 6. If y =
find the approximate percentage increase in y due to
an increase of 0-1 per cent, in x.
Here /{x) = x\ f'(x) = Zx^
and = 3^* Bx.

By 3x^ Bx Bx
Hence — =
x^
— = 3—,
r
x
y
dividing by the equal quantities y and x^. Now Bx/x is the ratio of the
change in ;r to x and the percentage change in x is therefore 100 Bx/x.
Similarly the percentage change in y is 100 By/y and we can write
percentage change in y '= 3 X percentage change in x.

The percentage change in x being OT, the approximate change in y is

therefore 0*3 per cent.

Example 7. Find approximate error made in calculating the area of a


the
triangle in which two of the sides are accurately measured as 18//. and
25 ft., while the included angle is measured as 60°, But is wrong.

Taking the given sides as 6. c and the included angle as A, the area A is
given by A = ibc sin A. If is in error by 8^. the area will be in error

by 8A where
8A '= \hc^ (sin/f) BA = ^feccos.^ 8^.

Now 6 = 18. c = 26, A = 60° = n/Z rad., 8.4 = = 7r/3G0 rad..

so that
18 X 25 X cosff/3
X
2

EXERCISES 7 (c)

that the distance moved s ft. is


1. A body moves in a straight line so
<’ - hmd an
given in terms of the time ( sec. by 5 =
body at time < and find the times at which
for the velocity of the
the body is at rest. t * . •

moidng straight
2 The velocity v ft. /sec. at time « sec. of a body
of the body when
.

Find the acceleration


line is proportional to (*.

t = 2 sec. if its velocity


is then 16 ft./sec.
^
given by f + < •
ink stain
After 1 sec., the area A sq. cm. of an
is
3.
is increasing after 2
sec.
Find the rate at which the area
can be measured accurately to 0 01
4 If the side of a square
side is measured
.

area of a square whose


is the possible error in the
to be 20 in. ?
— —
7] EXERCISES 129
5. Find the approximate percentage change in the square of a quantity
when the quantity itself changes by 01 per cent. Hence calculate
an approximate value for (10 01 )*.
6 . Find the increase in the area of a circle when its radius changes from
6 in. to 51 in.

EXERCISES 7 {d)

1. If f{x) = -f S’n X, find the values of /(O), /{-’r/2) and /(— ti).
2 . If 4x^y —
2x* + 3xy = 0, express y explicitly in terms of x and find
the value of y when x = 2.

3. If/(:r) = ax^ bx c where a, b, c are constants, find an expression


for f{x -f- !)•

4. f{x) denotes an expression of the third degree in x. If /( — 1) = 6,


/(O) t= 9, /(2) = 19, /(3) = 11, find the expression for f{x).
6 . If f{x) = log X, show that :

(i) f{ab) = f{a) + f{h),


(ii) /(a/6) =/(a) -/(6).
6 . P is the point [x, y) and Q the point (x + dx, y + dy) on the graph
of y = ^x. Show that
^ _ 1

dx V(* + <5,*:) + V-**


and hence find the gradient of the curve at the point P.
Find the slope of the curve
y = ax* + bx
7.
c, where a, b. c are
constants, at a point whose abscissa is x. At what point is the
tangent to the curve parallel to the ;r-axis ?

y = — x*
8 Calculate the gradient of the curve
.
4jr* — 3x at each
of the points where it crosses the axis of
x. (L U )
Show that
y =-- I + 3x ~ {x*/4) when
9. the gradient of the curve
^ = 2 is double that when x = 4. Find also the
abscissa of the
point on the curve at which the gradient is —
1 . (O.C.)
10 . An expression of the second degree is denoted by /{;*•).
If /(i) =7
/(2) = 23, /(3) =
17, find the gradient of the graph of
f{x) 3it x = 2.
(O.C.)
11 . Find the value of the constant c so that the
tangent at the origin
of coordinates to the curve
y ^ x{c x*) makes an angle of 45 ®
with the 4r-axis.
12 . For what values of x is the tangent to the curve

y = 1^* +
equally inclined to the two coordinate axes ? (O.C.)
13. Differentiate from first principles ;

(i) (ii) l/x.


14 Differentiate
.
from first principles ;

(i) sina^, where a is a constant.


(ii) icos2jr.
1
130 PURE MATHEMATICS [7

15. Obtain from first principles the differential coefficient of l/{x + 2)


with respect to x. (L.U.)
16. If f{x) = + sin;ir, find f'{x). ^^^lat is the value of /'(O) ?

17. An excavator removes V cu. ft. of soil in i min., where

At what rate is the soil being removed after 20 min. ? (L.U.)


18. Find the gradients of the curve y = x^ + \92x at the points
where (i) x = 2, (ii) x = ^.
19. If the distance s travelled by a particle in time t is given by
s = tit \al~,

where u and a are constants, show that the velocity at time t is


u -f at and that the acceleration is constant.
20 . Tiie distance s ft. which a body’ has travelled in t sec. is given by
s = y- — /. Find when the body’ is at rest and the acceleration
at that time.
21 . Assuming the path traced out by a mortar bomb is given by’
tliat
the equation y = 4x — (a'®/3-2), the x- and y-axes being horizontal
and vertical lines through the point of projection, find tlie angle of
projection of the bomb.
A particle moves along a straight line in such a manner that its
distance x from a fi.xed point in the line at time t is given by x = cos t.
Prove that its velocity’ is equal to — v/(l — x*) and that its accelera-
tion is — X.

A point moves along a straight line and. at the end of t sec., its

distance (s ft.) from a fixed point in the line is given by


5 = - 9/2 + 24/ - 18.

Sliow the velocity vanishes for two values of / and the accelera-
tliat
tion for one value of /. Find also the value of the velocity’ when
the acceleration vanishes and the values of the acceleration when
the velocity’ vanishes. (O.C.)

The distance moved in a straight line by’ a particle in / sec. is 5/® ft.

Show that V, the velocity, and /, the acceleration, are connected by


= (O.C.)
the relation /* 60i'.
5 ft.
A body moves a straight line so that the distance travelled
in
in time t sec. is given by 5 = 8/® + 3/. Find the approximate space
the average
travelled during iV sec. after 5 sec. of motion and deduce
velocity in that interval of time.
CHAPTER 8

TECHNICAL PROCESSES IN THE DIFFERENTIAL


CALCULUS
8.1. Introduction
In the last chapter, where an attempt was made to introduce the
basic ideas of the differential calculus, examples were chosen which
involved only very simple functions. The differential coefi&cients of
more complicated functions can be found from first principles in the
same way but the labour involved in some cases is considerable.
Fortunately certain general theorems can be set up and these, together
with the differential coefficients of quite a small number of standard
functions, enable the differential coefficients of more complicated
functions to be found readily.
It is the object of this chapter to deal with the technical processes
involved in finding the differential coefficients of functions which can
be considered as sums or differences, products or quotients, etc. of
simpler functions. The student should work through a large number
of examples on differentiation until he is thoroughly familiar with
these processes.

8.2. The differentiation of a sum


(a) Let y = w -f C, where « is a given function of x and C is a con-
stant. Then, if x increases to x dx, u increases to « m and y
-f <5
increases to y + so that

y + dy == « + -t- C.
By subtraction, dy = du, and division by 6x gives
6y _ du
dx
In the limit as dx tends to zero, we have
dy du
dx dx'
Thus an additive constant disappears on differentiation,

(^) Let y = M -f V,
where u and v are given functions of x.
If x
increases to -f dx, « and v increase to «
and y increases to
dw and v -f- du respectively
^ +
y dy. Hence +
y dy —u ~\- du V dv.
and by subtraction
dy ^ du dv.
131
+
132 PURE MATHEMATICS [8

Dividing by 6x and proceeding to the limit as dx tends to zero we


have
dy _ du ^ dv
dx dx dx*

showing that the differential coefficient of a


of tivo functions is the sum
sum of the differential coefficients of the separate functions. It is clear
that the plus sign can be replaced throughout by a minus sign and
that the differential coefficient of the difference of two functions is the
difference of the differential coefficients of the separate functions.

{c) liy u V w, where a third function of x, we can write


ze? is

this as y {u v) w, and application of the preceding result gives


dw
" + >
% U
- dx
du
dx
In this way we can show, step by step, that the differential coefficient
of the sum of any finite number of functions is the sum of the differential
coefficients of the separate functions.

Example 1. Find the differential coefficient of the function


-X Z.

Since the differential coefficient of ;r« is nx^-^. application of the above


rules gives for the differential coefficient, 5x* -f* 2^
— “

8.3. The differentiation of a product


(rt) Let y = Cu, where u is a function of x and C is a constant. Then

y dy C{u + Su),
where ^y, du are the increments in y and u respectively corresponding
to an increment dx in By subtraction we find dy = C du, and
;i;.

division by dx gives
dv
-- = C--.
dx dx
In the limit as (3;t: tends to zero, we have
'(y
= C-, (8-3)
dx dx

and the differential coefficient of a constant multiplied by a function ff


coefficient of the
X is equal to the constant multiplied by the differential
function.
If dy. du, dv
(b) Let y = where » and r are given functions of
u and v respectively correspon mg o an
are the increments in y.
=
8] DIFFERENTIATION OF PRODUCT 133

increment 6x in x,

y dy = {u + 6tt){v + 6v)
= uv V du u dv dit dv.
By sirbtraction,

6y = V (5» + u dv -f- dit dv,

A slight rearrangement and division by dx gives


dy 5 m .dv
r- V—
<

{u du)--,
dx dx dx
As dx tends to zero, du tends to zero and du/dx, dv/dx, dy/dx tend

respectively to and Hence


dx dx dx
dy du ,
dv
dx dx dx
showing that the differential coefficient of the product
of two functions
of X is equal to the second function multiplied by the differential coefficient
of the first plus the first function multiplied by the differential
cocfficieni
of the second.

{c) The differential coefficient of uvxa where xo is a third function


y
of X can be found as follows. If we divide the left-hand side of
(8.4)
by y and the right-hand side by «v(=y) we can express
the result
m the form
l^_ldu^ldv
y dx it dx v dx'
If now we apply this result to = (uv)w we have
y
I
“-J-
y
dy
dx
= — 1

uv dx
d ..
7-(«v) H
I

w
dw
dx
,

But, by (8.4),

d ,
. du

so that

Multiplication by y, or uvw, then gives

dy du dv . dw

For a more general function = uvu>...,


y we can obtain similarly
,

134 PURE MATHEMATICS [8

dy dw
dx
= vw • •
dii

dx
“h • • •
dv

dx
— ,
f- uv . • ,

dx
— (8.5)

so that the derivative can he obtained by differentiating each function


separately multiplying by the remaining functions and adding the results.

Example 2, Find the differential coefficients of (i) and (ti)

(i) The differential coefficient of 6jf* is 6 times the differential co-


efficient of and so is 6 x or 24jr».

(ii) The differential coefficient of is one-quarter of the differential

COS X
coefficient of sin x and is therefore .

4
dy
Example 3. Find ~ if (i)
y s= x^sinx, (m) y = x sin x cos x.

(i) By (8.4).
dy d d
di =
*= 2x sin X -y X* cos x.
(ii) By (8.5).

dy d d d
— — s\n X cos ^

+ X cos ;r— (sin x) + x sin x—(cos x)

= sin X cos X X cos ;*r.cos ;r -j- ;r sin jr(— sin x)


« sin X cos X 4- ;r(cos* x — sin* x).

EXERCISES 8(a)

Differentiate the following functions of x :

1. -- sin 4- 2. 6. x^ cos X.

2. 10 sin X cos X. 7. {Zx + 2)».

3. sin X — X^ cos X. 8. X* sin X cos X.


4. X{1 -- X). 9. cos* X.
5. (1 + x^){l - 2x*). 10. 10;r* — sin* x + x cos x.

8.4. The differentiation of a quotient


Suppose that y — u/v where u and v are given functions of x.

Then « = yv and application of the rule for differentiating the product


yv gives

du
dx
= v-fi
dx
dy
+,
dv
y-r»
dx

Solving for ^
dx
we have
dy_ldH __y^
dx~ V dx vdx
X
( —
8J
DIFFERENTIATION OF TAN ETC 135

Since y : m/v, y/v = u/v^, and liencc,

dy 1 dn u dv
dx V dx dx

du
— — dv

or.
dy
dx
— V
dx
«
dx
(
8 6)
.

Thus the differential coefficient of the quotient of tivo functions of x is


equal denominator times the differential coefficient of the numerator
to the

minus the numerator times the differential coefficient of the denominator


all divided by the square of the denominator^

_
Example , .

4. Ftnd — when
. ...
(») y = -1— —X ;

(«) y = —
sin X
1 +
,

(i) Using (8.6).

<* + -*)-(!- *)^(1 + *)


_

dx
(1 H. x){- ;,)(!) — 2
(I +xy (1
(ii) Again using (8.6).

d d
(sin ;r) - sin (ar)
dy dx dx
_ ~
dx ~ X*
X cos X — sin AT

8.5. The differential coefficients of tan x, cot x, cosec * and sec x


The differential coefficients of tan x, cot x, cosec x and sec x can
be derived from those of sin aj and cos x and the rule for differentiating
a quotient. Thus ^

l{tan X) =
dx dx\cos

cos a;— sin a;) — sin x^ (cos x)


cos^a;

^ a;(cos x) — sin x{— sin x)


cos^x
_ cos^ X 4- sm ^ X
cos^x
1
= sec* X. (8.7)
C0S*AJ
136 PURE MATHEMATICS rs

Similarly
d /cos X
dx
(cot x) = if;c\sin
x

sin (cos x) — cos (sin x)


dx dx
sin^ X
— sin® X — cos® X
sin^ X

= — cosec X, (
8 8)
.

sin® X
Also

— df 1
(cosec x)
dx i/;r\sin x

sin ^-^(1) — l.-^(sin^)


dx dx
sin®;t

sin AT. (0) — (1) cos a;

sin® a;

cos X
= — COSCC X cot X, (8.9)
sin® X
It is left as an exercise for the student to show in the same way that

d
(sec x) = sec x tan at.

dx

EXERCISES 8 (6)

Differentiate the following functions with respect to x :

sin X cos X —
1. 6 .

x’^+ 1 sin ;r cos x +


2 - X X
2 7.
.

1 + 2x tan X
1 - 3^®
3. 8 . sec* X,
2 +
sin X
4. (3 - 2Ar®)-®, 9.
1 + tan X
1 + sin X
10 . cot* X,
6. T
1 — sin X

of a function
8.6. Differentiation of a function , ,
.

A function like^' = + 3)Ms a


U a function of a: and (* + 3) Ms a function
of {* +%^rotWexLtles
3) Other examp
.
8] FUNCTION OF A FUNCTION 137

are sin a*, tan (x^), etc. The object of this section is to establish a
very simple rule for the rapid determination of the differential
coefficients of such functions.
The differential coefficient of a function like our first example can,
of course, be found by first squaring out the right-hand side and
differentiating term b}^ term. Thus
y=^(x + 3)2
= _j_ 9^

^ = 2a: +6= 2(:i: 3).

It should be observed that the result exactly the same as if we had


is
treated (:v 3) as if it were x and used the standard result for the
differential coefficient of Similarly if y (a: -f where a is a =
constant,

y = {x-^ a)3
= + 3flx2 -f- ^a^-x d- a\
dv
/
dx
= + Grt.tr + 3rt2.

= 3(ar2 + 2ax + a^) = Z{x -{- a)^,


and again the result is the same as if we had treated {x
-f- a) as if it
were and used the standard result for the differential coefficient
of x\
Now consider y = + S)*. Working as before
{2x

y= + I2x -h 9,
4.t:2

= + 12 = 4(2a: + 3).
^ Sac

SO that the result not now 2(2x


is
3) but twice this. +A rough
explanation is that whereas {x
+
3) changes at the same rate as x,
(2x -h 3) changes twice as fast. Similarly if y (cx -|- d)\ where c —
and d are constants,

y = c^x^ 2cdx -f d\
= 2c^x H- 2cd = 2c{cx + d)
^
and we observe that the result is the same as if we had
treated (cx + d)
as if It were x, used the standard result
for the differential coefficient
multiplied by c, the differential coefficient of cat
This suggests that if is a function of u where
+ d.
y « is a function of x,
the formula giving might well be
^
dx du (
8 10 )
,
dx*
138 PURE MATHEMATICS [8

Applied to some of the examples already given this would give


[a) y = (x -f 3)2^ or y = u- where u =x -{- Z,

(^) ^ = (^ + or y = 7(3 wliere ti =x a.

‘^y

du
t
=
dx
= zu^ i.

dx du

^ ^ X du
dx
= 37(2 X 1 = Z(x + a)\
(c)
y = (cx i/)2, or y = u~ where u = cx -I- d.

~
dy
du
du
= 2«, ^ = c,
dx

^=
du
— = 27(Xc = 2c(c.t: + d).
dx dx
A proof of the important formula (8.10) is rather beyond the
strict
scope of the present book. The following, although it assumes a
result not already proved, must suffice. If y is a function of u and
It is a function of x, let 6u be the increment in « corresponding
to an
increment dx in x, and let 6y be the increment in y corresponding to
the increment du in tt. Then, provided du 0, ^
^^ X —
dx du dx
and, assuming that the limit of a product is the product of the limits,
this gives

^ X —
dx du dx

since — ,
— are respectively the limiting values of ^ and
^
dx du dx
as dx tends to zero. ,
.
shou wor
This formula is a most important one and the student
through many examples of its use. At first it is ?
below u wi
introduce the auxiliary variable « as in the examples
unnecessary and the results can be written
practice tliis soon becomes
down at once.

Example 5. Find when (*) y = (1 — 3^*)*,


dx
(i) Let M = 1 - 3x*. so that y = u‘. Then
.

81 FUNCTION OF A FUNCTION 139


du dy
T- = — 6jr and -- =; 5«‘.
dx du
dy dy du
Hence ^ (-
-di^Tu^Tx^
= - SOxtt* = - 30^(1 — 3-r2)*.

= 1 + 2x
(ii) Let u
I
, so that y— rt*.

dy du
Here —= 2m, but to find — wc have to apply the rule (8.6) for

differentiating a quotient. This gives

rfM
~
dx (1 +^)‘
(1 +;r)(2) - (1 + 2;r)(l) 1

(1 +^)*‘
dy dy du
Hence ~
dx du ^ dx

2m X
(1 +Ar)»
1 2(1 + 2;r)

(1 -hx)* (!+;.)»•

Example 6. Find (i) - ^)} ~ {„,• (28 -


| {«„ (41 ,
(ii)
^)}

(i) Let « ;= 4/ - tt/S, so that ^= 4. Then


at
^ ^ d ^\L
-{sin (4/ - 7r/5) = -(sin h) = —(sin m) x
}
^
= cos tl X 4 *= 4 cos (4/ — ir/5).

(u) Lety = cos* {26 - w/5) and let u 20 7t/5. Then y cos* = — = ii.
This a function of a function and we now let cos u
is still
— v.

= v*
Then y and
^ = 4y>. By (8.10),

dy dy dv


since w s* cos tt and so —
dv
du
= — sin m.

dy dy du
Also — 4i;* sin u x 2,
du^Te
= du
since « - n/5, so that
:^ = 2.
UQ
Replacing the values « = 20 — vr/5, v =* cos it s= cos (20 — v/5)
we have ' * **

d$
— 8 cos* 2 $ - j) sin (26 -
{ ^).
— —
140 PURE MATHEMATICS IS

EXERCISES 8 (c)
dy
Find —
dx
when :

1. y = {4x - 5)\ 0. y = tan* (3^ -h 1).


2. y = (^2 ^ 2x)^. 7. y = siii^ (2 — x).
3. y = sin 2[x — a), a constant. 8. y (1 - A-)*.

4. y = tan 2x. 9. y = sin* .y sin 3x.


1 + sin* X
5. y = sec Zx. 10. y = —
1 sin* X
du
11. If M = sin”* 9 cos" 0, find the value of — if m and n are positive
integers.
12. Find the differential coefficient of s with respect to f if 5 = sin* (a/t)
and a is a constant.
13. If m is a positive integer, find the differential coefficients with respect
to X of :

(i) sin™;ir, (ii) sin (hi) sin (cos ^).

14. If y = (tan x + sec x)”*, where m is a positive integer, show that


dy

dx
= my sec x.

15. Show that the differential coefficient with respect to x of


tan x{l + 2 sec* x) — 3x sec* x
is C sec* tan ;*r(tan — x). Hence show that, if ^ is a positive
acute angle, this differential coefficient is positive.

8.7. The differential coefficient of a:” when n is negative or


fractional
It was shown chapter {§ 7.5) that, if n is a positive integer,
in the last
the differential coefficient with respect to a; of x" is nx^-K
The rules
established in §§ 8.4, 8.6 for differentiatint; a quotient and
a function
function enable us to show that the same result is true when
of a
n is negative or fractional.

(a) Let n be a negative integer and let


n = — w so that m is a positive

integer. Then if y we have =

Bv the rule for differentiating a quotient, since


m isa positive integer
of a:-" is mx’^-\ we have
and therefore the differential coefficient
dy a:" X 0 - 1 X mx"^
dx~

since « =— wj.
81 FUNCTION OF A FUNCTION 141

(^) Let n=p/q where p and q are integers. There is no loss of


generality in assuming that q is positive. Then, it
y = we have
y =
Put = u, then y = uP and x = m?, so that
dy dx
dii
= puP-^ * '
dxi

by the result already established.


The rule for differentiating a function of a function can be written
in the form

^
du
=^ V
dx du
so that puP-'^ =^ X

giving ^
dx a *

Since u = a;'/?, this can be written

dx q

9
= 7tX^~\
since n ^p/q.

^®^ce ^{xn) = nx^-^ for all values of w.

V(
(i) Let « =^^ 3/^1 _ 3,-,^ Then y = u* and
dy
- = 2«, ^ du
=i_3(-2)^-. = 1 + Q/xK

Hence ^
dx
^ X —
du dx
= 2tt(l + Q/x*)
= 2{x - 3A*) (1 + 6A").
(u) Let tt = (1 + x)/{l - x). Then y « V« = and
dy
du
^ 1

and. by the rule for the


differential coefficient of a
quotient.
_ 1 —X -t- (I H- ^r) 2
dx (I - X)*
* X
'
»

142 PURE MATPIEMATICS [8
dy _ dy du
Hence
dx du dx
I 2
2v/« (1 - x)^

(1 + x)^^l - x)^'^'

EXERCISES 8 {d)

Differentiate with respect to x :

1. ('• - A)- 4. 1/V(1+^).


2 . (2 - 5a:3)-*. 6. ~ x%
3. V(1 + X), 6. sin {^/x).

7. li y — \/{l 4- sin x), show that —


dx
= h^/{\ — sin x).

8 .

A
8.8. Differentiation of inverse functions
sin X

If in Fig. 48, FT is the tangent at the point F to the curve repre

senting the function y = /W* if FT makes an angle ^ with the


^-axis,

tan y> = dy
dx
If the equation y f{x) = is written in the form x = g{y),
the curve of

Fig. 48 also represents this function. If FT makes an angle f with


8] INVERSE FUNCTIONS 143
the ^-axis, we have
tan ^ .

dy
But. from the figure,
^ -f yj 90 ^, so that <l>
= 90° - y, and

tan ^ = tan (90° — V’)


= cot = tp
1

tan
so that —= 1 /^, (
8 11 )
.

dy / ifA?

An analytical proof of formula (8.11) would run as


follows. If
y ~/W ^rid X is the inverse function given hy
x we have — iiy),
y=f[g{y)]’
Differentiating this with respect to
y as a function of a function

or, since x = g{y),


Hence g'{y) == l//'{x), and this can be written

^-1 /^y
dy / dx

8.9. The differential coefficients of tan"^^ and sin“^*


Hy= tan~^ x, we have x = tany and
dx

dy
= 7-
sec* y
,

= -f tan*y1
= 1 -f
Hence, usmg 8 11 ),
.
(

dy /dx 1

dx “I+ 3: 2
’ (^*12)

Similarly, if
y sin-i x, x = siny and
dx
Ty =
= ^{1 — sin^y)
= V(1 -jf*),
giving —
dx
'
=t 1

K~
/ dy V(1
of the other in^
- (8.13)

trigonometrical
as exercises for
tlw stude^
144 PURE MATHEMATICS 18

dv
shota that ~ =
1
Example 8. If y = sec-^ x.
dx xy/{x^-\)'
If y = sec-' X, X = secy, so that
dx
— = secy tany = secy\/(sec2y — 1) = x\/(x^ — 1).

Hence ^
dx
= 1
dy
=
x\/{x'
^

/ 1 )

— ;r*\ dy
/I 2
Example 9. Jf y = co5"*( zl, show that -r — ,

\1 + ;rV
i
dx x^ \

1 - ;r>
I«t u = so that y = cos-' m, or m = cosy.
1 + ;tr

dit
= — siny =a — V^(l — cos*y) = — \/(l — «*)
dy

+ x\
dit rf /I — Jr*\
Also, ~
dx rfAl + xV
(1 + ;*•*){_ 2x) - (1 - x^)[2x)
(1 +

(1

dy dy du / d\t
But ~
dx du ^ dx V / dy) ^ dx
(1 +x*) - ix
2x (1 -hxy

1 +x a*

EXERCISES 8 (e)

1
1. Show that —(cot-^ x) = — z.
dx 1 + X*

2. If a is a constant, show that —d Icos-' a)


- = “ ,
;r\
) - X*)
dx\
d
3. Show that —
dx
-1
(coscc-^ x) —
xV{x^ - 1)

dy 2
=
i. If y = tan
(^) show that --

5. Differentiate x sin”' x with respect to x.

- show that _
- x*)^ —
- xy = 1.
0. If y = (sin-' x)/Vi^ ^*)» (1

functions
8.10. Differentiation of implicit
rules for finding the differential coefficients
So farwehavecstablished
When the dependent variable y is not given
of explicit functions only.
IMPLICIT FUNCTIONS U5
explicitly in terms of the independent variable ^
not necessary it is
nor indeed is it usually possible, to
solve for y in terms of x.
Suppose, for example, that
y is given implicitly in terms of ^ by
uy
the equation

x‘+y^ = 2x. (8.14)

L"tZ^gV^^l"Z°n oi a Son
|(y) - ifr.) X I . i/y.

2x + 2y-l = 2.
ax
giving X
dx
UA y
In this example we can
first solve equation
(8.14) for y to give
,,
^ x^), v{^x
and then find dy/dx from
,

Ihis procedure would


the equations
give
y = v«
^ V« where u
wnere « -
— zx
2^ - x^

_ 1 du
= 2 - 2x.
du 2'\/u dx
and then, ^ du 1
~
dx du dx~ 2
1 ^ I ~x
y/w y
as before, but
this method is more laborious

dx *“>
y^x’ differentiation of
(8. 16) gives
1 _idy . dy
"^rx-^'-y/x-o.
leading to dy i

dx — I'
Example 10. Find iyn^
(i)
V* + Vy =
(i)
' ^ ,.•>

2, so that

_1 1 dy
0.
146 PURE MATHEMATICS [8

giving = - ^0.
I
(ii) = const., and the rule for the differentiation of a product gives
dy
dx
ivx^ — ^yf^ my
SO that
dx mt/n— 1
iix^yn

8.11. List of standard forms


The standard functions and the
differential coefficients of certain
rules for differentiation established in this chapter are most important.
They are here collected for easy reference. In all cases C denotes a
constant and u, v are functions of x,
d,
-{» + .

.)
,
= du dv

d du

d du dv
-(uv)
,
.

= ,

^ (sin x) = cos X.
dx

(a;") = ^(cos .) = — sin X,


dx

— (tan x) = sec* x, ^(cot x) = — coscc*


dx dx

(coscc x) = — coscc X cot X, ^(sec x) = sec X tan x.


dx dx

d
(sin“^ x) = ^(tan- .) =
dx V(1 - x^)

8.12. Higher derivatives


wUl itself be a
If V be a function of a:, the differential coefficient
differentiating dy/dx with respect
function of it. The result of

HIGHER DERIVATIVES 147
is called the seconi differential coefficient
of y with respect to x or the
second derivative. Proceeding further, the differential coefficient of the
second derivative is called the third differential coefficient or third
derivative and so on.
A
conventional notation for the 6rst, second, third, ... and nth
differential coefficients of
y with respect to x is
dy d^ d^y d’*y

dx dx^’ Ib^'
If is a function of ^ given by
y =/(r), the notation
/'M. r{A> r(x) /('i)W.
IS also sometimes used for the first, second, third «th derivatives.
which the general expression for the
«th derivative of a function can be found.
Here we shall be concerned
only with the first few differential
coefficients and shall not attempt
to discuss the general derivative.

Example II. Find


d^y
Ix^
and ~
d*y
d^
when (i)
y == *10, (n) y == cos 2x.

d*y
= =
(i)
dx
10 ,.. 90..
2 = 720.T.

dx
~ — 2 sin 2 ^, ^
dx^
— 4 cos 2x, ~=
dffy
dx^
g sin 2x.

Example 12. Find


0 when (i)
y = x^l - x)\ (,i) y = x sin x.
(i)
y = ^»(l _ xy = _ +
2x^
dy
~ « 2t - 6.r* + Ax\
rf*y

dx^
= 2 - 12;r + 12.»>,

(ii)
y e= sin x,

dy
s= sin ^ -f cos x.

rf*y

dx*
- cos * + cos jv - ^ sin jt = 2 cos ;r - ;r sin x.

EXERCISES 8(/)
Find dy/dx when :

1. x*y*'~x^0.
2. y» — sin = 4 .

3. x*^xy-i^y*=. a\ {a constant).
4. Find dr /do when r* cos
6 « const. -

5. If y =, tan X, show
that
d*y
77t =» 2taa^ + 2tan*y.
— — X

U8 PURE MATHEMATICS [8

6. Show that
d^(VV) d^U^^ d^UdV dU d^V d^V
dx~ ~dx^ ^ dx^ dx d^ ^d^*
where U and V are functions of x» (QE.)
d^
7. Evaluate
dx*
{(1 ^ -f x^i sin t}. (QE.)

8. If = (cos^)/a’, prove that


d^y 2dy
-j—k ^
dx^
—X dxr y — 0 » (L.U.)

EXERCISES 8{^)

1. Find (i) ^{Ix^ - + 5). (ii) 4(3/"^* - + 6/),


dx dt

(iii) ^g[0-’‘ - 40->^=).

Find dy/dx when ~ cosx + ;vsin;*r, y = (3^ — 1)(-*' — 8).


2 . (i)
y (ii)
(O.C.)

3. Find dOfdt when (i) 0 = sin f sin 3f, (ii) 6 = /*sia”^f. (L.U.)

4 . Differentiate with respect to x :

(i) (2 — x^) zos X + 2xs\nx, (ii) (1 — lA) tan

5. Find (i) - 3^)’}. (") ^{0“cos2 20}.

6 . Find dyjdx when


x^ X „ 1 “
(i) 3^ = (“) =
.....
>' =
X V ' ' :v
^ ^-r-:a72'
\X + 1 + y/

7. Differentiate with respect to x :

... x-\ ....


(;r-2)(;r-4)
« (“0
cos ;r

1
AT* - 4
(‘0
;r - 3
*
1 -hx

8. Find dyjdx when :

X tan X sin x
(ii) y = (iii) y =
{') y =z^.
^in X '
X 2 + 3 cos

following functions with respect to x and simplify


9. Differentiate the
the resulting functions as far as possible.
cos {x^
- + 1) (QE.)
(i) x^ tan «.T, (ii)

sin {x* 1)

differential coefficients with respect


If a function of x. find the
10 . y is

x/y, (iv) y/x.


(i) xy\ (ii) {3y + 2)*, (iii)

11 .
Differentiate with respect to ^ :

—- 2rr.
rr
(L- n 1

(i) {x' - *)•, (ii) cos (2 3rf), (iii)


—— —
8] EXERCISES 149

12. Find

dl{yr(\ V/')}’

(iii,
'
dx\ V(x- - 1)

(i) li y = tan 4r + J tan® x, show that = sec* x.


r/ V

^ii) If >• = sin-^ fcos;r), show that — 1.


dx
14. Find the value of :

(i) ^tan (ii) ^sin

15. Find dO/dt wlicn :

(i) 0 cos-1 (1 - 2/*), (ii) 0 = sin-1 - 1).


IG. Differentiate with respect to x :

(i) cos (l/x). (ii) tan (x>), (iii) sin-*


(Q E.)
17. Show that X = (^sin 07r/)/4{6^ — 36,^®) satisfies the relation

~
d^x
Ggx
g
^ -sin Qnt. (Q.E.)
(Q-E.)
18. If a and ^ are constants, show that the derivative
with respect to
X of each of the functions

and 2 tan-
y(:
IS {(« -x)(x -/3)}-i/2
19. It y ~ sin-1 {Zx — 4x^), show that \/(l — x^)~ = 3
dx
20, If U =
« 0* + (sin -1 fl)> _ 20 V(1 - 0®) sin-1 0, show that
~ = 40® sin-10.
aO
21 Find dy/dx when —
.
Zyx^ + 2.v® = 0.
22 . Ii y' - 2y^{l
+ *«) + ,!! = 0, show tliat
dy X
dx~ va +
23. Find dy/dx when
y is given by
(i)
J-* + *> = 4* + 1, (ii) iy, 3,, ^ 0
24. Find dy/dx when
y is given by :

(i) ar'siny -ycos* = 0, (ii) ^cosy -;v«sin* = 0.


26 . Find the slope of the tangent
to the curve
xy^ - 2x^y* x* ^ I 0
at the point where ^ » 1, y = 2.

150 PURE MATHEMATICS [8

26. Find d^y/dx^ when:

(i)
y = sin X, (ii) y = x tan-' x, (iii)
y = -~-
1 + X
27. If ^ = tan* X, prove that
d^y
= 2(1 +>0'{1 + 3>'). (O.C.)
dx
dy
28 If y =
sin X
x^
^
find
^

dx
and ^ d~y
-r^.
dx^
,
and prove that
, . .

+i/^
dx
+ + 2)y = 0. (I.U.)
dx^
29. If y = (tan-';r)*, prove that

30. Show that if « = tan-' 0, then


d^u A
(1 + -i-
20-- = 0.
CHAPTER 9

SOME APPLICATIONS OF THE DIFFERENTIAL


CALCULUS
9.1. Introduction
We have seen Chapter 7 that the derivative is a measure of the
in
slope of the tangent to the curve representing a function. The process
of differentiation has therefore the geometrical application of finding
the slope of a tangent to a curve but we shall delay giving examples
of this application until we discuss the methods of coordinate geometry
(Chapters 16, 17).
Other important applications already discussed are the use of the
derivative as a rate measurer and in finding velocities and accelerations
in dynamical problems. More elaborate examples can be given now
that the technical processes of differentiation have been studied.
Further uses of the differential calculus occur in finding maximum
and minimum values, and in curve tracing. These applications are
discussed in the paragraphs which follow.

9.2. Some examples a rate measurer


of the derivative as
We give below two examples in which the methods of the last
chapter can be used in solving problems on rates of change.

Example 1. The volume of a solid cube increases uniformly at cubic inches


per second. Find an expression for the rate of increase
of its surfau-area
when the area of a face w 6* square inches.
IfX is the length of an edge of the cube at time the volume V is given
by K s X*. Differentiating with respect to /, we have

But dV/dt is the rate of increase of volume and this is A*, so that

giving ^
dt
A*

The surface-area S of the cube is 6*‘, so that the


rate of increase of
surface-area at time t is

dt
Substituting for dx/dt we find

A* 4A*

161
152 PURE MATHEMATICS [9
\ATien the area of a face is b^, x —
b and the required rate of increase of
surface-area at this instant is obtained by writing x =:
b in the expres-
sion for dS/dt, giving ^h^jb.

Example 2. A hollow circular cone with vertical angle 90® and height
2 ft. is
inverted and filled with water. This water begins to leak away through a
small hole in the vertex. If the level of the water begins to sink at the rate of
1 in. in 2 min., and the water continues to leak away at the
same rate, at what
rate is the level sinking when the water is
2 ft. from the top? {The volmne
of a cone is one-third the area of the base times the height.)

Let {Fig. 49) the height of the


water at time / min. be h in., and let
the volume of the water then be V cu. in. Then, V = l-nr^h, where r in.

w r **

isthe radius of the water surface. Since the semi-vertical angle of the
cone is 45®, it is clear that r =
h, so that

V =
By the rule for differentiating a function of a function,
dV _ dV dh
It ^ dh^Tt

Since the level of the water is decreasing at the rate of J in. /rain, when
the cone is full, i.c. when A =
3 ft. 36 in..=
—=
dV
X (36)* X (— i) cu. in./min.,

(a negative sign meaning a negative rate of increase or a rate of decrease).


dV/dt remains constant at this value. When the water is 2 ft. from the
top. A =
3 “
2 s= 1 ft. = 12 in., and substitution in (9.1) gives
dh
v X (36)* X (- i)
= X (12)*^»

so that ^ * ^ “ '
( 12/

showing that the water is sinking at 4-5 in./min. at this instant

9.3. Some dynamical applications


that a body, moving in a straight
We have already seen (§ 7.7) if
velocity tun m
a distance s and acquired a i/ ,

line, has traveUed


0] DYNAMICAL APPLICATIONS 153

then
ds
(9.2)

and its acceleration (a) at time t is given by


dv
(9.3)

Alternative expressions for the acceleration can be found as follows


Firstly, combining (9.2) and (9.3),

dt-

Secondly, since r is a function of s and s is a function of /,

dv dv ds dv dv
(9.5)
at ds dt ds ds
Thus the acceleration may be expressed in any one of the three
equivalent forms
dv dH dv
dC di^* ds
In mechanical applications, differential coefficients with respect to
the time are often denoted by dots placed above the dependent variable.
Thus ds/dt, dH/df^, dv/dt are denoted by and v respectively.
s, s In
this notation, equations (9.2), (9.3) and (9.4) would be written
V

Example 3. The distance s moved in a straight line hy a particle in time t i$


given by s ^ at* + btwhere a, b and c are constants.
c,
Jf v is the
velocity of the particle at time t, show that 4<i(s
c) v« — = — b*.
(L.U.)
Id the notation just given,
V =4 «= 2a^ 4- 5,
so that
4a(5 - c) = 4a(a/» ^ bt c ^ c) = 4a{at* -f bt)
=s 4a*t* + 4abt = {2at
+ 6) — 6* »

«= V* - b*.

Example 4. If the of a body varies inversely as the square root of the


velocity
distance, prove that the acceleration varies
as the fourth power of
velocity.
' r j the

Denoting the distance traveUed by the velocity and acceleration by v


ana a we have, '

k
^ ** where A is a constant

ds A/«A2s3/2y
154 PURE MATHEMATICS
_ _ ^^ ~
*3
¥'i^
j,4
^ “ 2^'
since \/5 = A/i’. and we have established the result required.

EXERCISES 9 (a)

1. A conical vessel lias a vertical angle of 60“. If liquid is poured in


at a rate of 1000 cu. in. /min., find the rate at which the level is rising
when the depth of liquid in the vessel is 5 in.
2. 1 he radius of a sphere is r in., the area of its surface is 4;rr* sq. in.,
and its volume is cu. in. ; if, when the radius of the sphere is

21 increasing at the rate of 0-01 in. /sec., find the rates


in., it is
at which the surface and volume are increasing at the same time.
(Take n = 22/7.) (O.C.)
3. The inner and outer radii of a cylindrical
tube of constant length
change in such a way that the volume of the material forming the
tube remains constant. Find the rate of increase of the outer radius
at the instant when the radii are 3 cm. and 5 cm. and the rate of
increase of the inner radius is 3J cm. /min.
4. The displacement x at time f of a moving particle is given by
X = a sin 2f + 6 cos 2/.

wliere a and b are constants. If v is the speed at time t, prove that


V = 2^/{a^ + - X*). {L.U.)

6. Ifthe velocity of a body varies as the square of the distance travelled,


show that the acceleration of the body varies as the cube of the
distance.
6. 5 isthe distance moved and v the velocity acquired by a body moving
in a straight line at time t. If (i) t; = « + ft, (ii) v* = + 2fs, where
u and / are constants, show that in each case the acceleration of the
body is /.

9.4. Maximum and minimum


Suppose the grapli of y = f{x) is as sho\vn in Fig. 50. Points such
as A, B, C are called turning points. As x increases, tlie values taken
0] MAXIMUM AND MINIMUM 155

by y increase until the point A is reached, decrease from A to


increase again from B to C and then decrease. At A, B and C, y is
neither increasing nor decreasing.
Sometimes the points A, B and C are referred to as points of
maximum or minimum values, maxima at A and C and minimum at
B. It should be noted that a maximum or minimum value is the
greatest or least value in the neighbourhood, but it need not be the
absolutely greatest or least value. For example, there are points on
the left of the curve of Fig. 50 for which the values of y are less than
the minimum value ” at B and there are points on the right where
the values are greater than the maximum value at A. Again in

Fig. 51

Fig. 51,which shows the graph of y 2x* = —


x^, there are maximum
values where x =^
\ and these are also greatest values, but the
minimum value at a; =
0 is not a least value, for there are points on
the curve with smaller values of
y than that at 0.
y

X
Fig. 52

Wehave already seen that where the value of the derivative is


positive, the function itself is increasing and
where it is negative, it
is decreasing.
This is illustrated geometrically in Figs.
62. 63 which
show the graphs of functions which increase and decrease
respectively
156 PURE MATHEMATICS [9

as the independent variableIn the first diagram, the


increases.
tangent at a representative point P
makes an acute angle rp with the
a:-axis since the tangent of an acute angle is positive and since, by
;

definition, dy /dx =
ta.ny), the derivative will be positive. In the
second diagram, the angle ip is obtuse, and since such angles have nega-
tive tangents, the derivative will be negative. At points like A, B
or C 50 or 51, the tangents to the curves will be parallel to
in 1-igs.
the .t-axis. At such points, the tangent will make a zero angle with
the x-axis and we shall have
d_y

dx
Returning to Fig. 50, shown again in Fig. 54, the signs of the deriva-
tive have now been marked in. Immediately to the left of the point A,
the function is increasing and its derivative is positive. At point A,

the function neither increasing nor decreasing and the derivative


is

is zero. Immediately to the right of point A, the function is decreasing


and so has a negative derivative, and so on. can formulate the We
following rules for determining the position of turning points
and
distinguishing between maximum and minimum values :

{a) At a turning point, dy/dx = 0.


At a point giving a ntaximum value, dy/dx changes from
positive
(6)
greater respectively
to negative as x takes values just less and just
than the value at the turning point.
a minimum value, dy/dx changes from nega ix'6
(c) At a point giving
just greater respective y
to positive as x takes values just less and
than the value at the turning point.

Example 5. Find the turning point on the curve y = - 2x and determine


whether it is a point of maximum or minimum y*

Hero dx
MAXIMUM AND MINIMUM 157
and dyfdx vanishes when 2^ — 2 « 0, i.e., when x = 1. Hence the
point X ^ I, y ^ (I)* — 2(1) — 1 is a turning point on the curve.
For X s=s 0-9 (a value a little less than the value ;r 1 at the turning=
point),
dy
j^
dx
= 2x 0-9-2= — 0-2.

For X = 1*1 (a value a little greater than x — 1),

^ = 2x M-2=+0-2.
The derivative therefore changes from negative to positive, so the point
*— ^>y= —1 gives a point of minimum^. The graph of^ 2^ = —
is shown in Fig. 55.

An alternative method of discrimination between


maximum and
minimum values can be obtained as follows. If we
plot on the same
diagram the graphs of
y =/(*) and y =J'{x) (the derived function).

» pMtiv., for belmn A md


IS decreasmg and “|^/M
/ (x) is negative, and for points to the
righfo/s
158 PURE MATHEMATICS
f(x) again increasing and f'(x) is positive. The derived function
is

f'(x) is zero for values of x corresponding to the points A and B.


Considering the graph y =
f'(x), we see that for a value of x corres-
ponding to the point A,f’(x) is decreasing and therefore has a negative
derivative. Thus at point A ,f”(x) (or d'^y/dx’^) is negative. Similarly
at point B, f'{x) is increasing and derivative f”(x)
its is positive.
Hence at turning points giving maximum values

and at turning points giving minimum values

>0 .

Applied to Example 5, y = — 2x, dy/dx — 2x —2 and there


is a turning point where 2x — 2 = 0, i.e., where x = I, For this
curve

dx^
and, this being positive, the turning point is one giving a minimum
value toy. This method is further illustrated in the following example.

Example 6. Find the maximum and minimum ordinates o/ the curve

y = x^{x -f 1).
y = x^{x + I) = + ;r*.

d^y
dx
3;r» + 2x,
57*
6x + 2.

dy/dx vanishes when 3jr* 2x — 0, i.e., when x = 0 and when


X = — 2/3. When x = 0, d^y/dx* = 2 ;
this being positive, x = 0
gives a minimum value to y, the minimum ordinate being y = 0.

When ;r=-2/3, d^y/dx^ = G{-


2/Z) 2 2; + this being
negative, x = — 2/3 gives a maximum ordinate of
(- 2/3)*(- 2/3 + 1). or 4/27.

EXERCISES 9 (5)

which the expression {x - 2){x


for
- 3)* has
1. Find the values of x-

maximum and minimum values and discriminate between them.


iiy = — 2){x + 1)*. find the maximum and minimum values of y,
2
stating which is which.
Find the maximum and minimum values of the expression
3.
3x
- l){x - 4)*

maximum and minimum values of the function


4. Find the
{x-l){x-2)/x
the function between
and illustrate your result by drawing the graph of
X ^ ^ 3 and x =» 3.
9] APPLICATIONS TO PRACTICAL PROBLEMS 159

6. Find the maximum and minimum values of the function 2 sin t + cos 2/
and discriminate between them.
6. Show that the function — 6;»r® ISx + 5 increases with x for all
+
values of x. Find the value of the function when the rate of increase
is least.

9.5. Applications to practical problems


Many problems can be solved by the method of the last
practical
section. It sometimes happens that the quantity whose maximum
or minimum value is sought appears at first to be a function of more
than one variable. In such cases it is often possible, by means of
geometrical or other relations between the variables, to eliminate all
but one of these variables. Once the quantity has been expressed in
terms of a single variable, the procedure is identical to that given in
the previous section. We differentiate with respect to the single
variable remaining and the values which make the derivative vanish
include those giving maximum and minimum values to the quantity
under discussion. In many cases it is unnecessary to examine the
changes in sign of the derivative (or the sign of the second derivative,
if that method is used) to discriminate between
maximum and mini-
mum values, for it is often possible to see at once on physical grounds
whether the solution leads to a maximum or minimum. Some
illustrative examples follow.

Example 7. Find the height


of the tight circular cylinder of greatest volunte
which can be cut from a sphere of radius a.
(L.U.)
In Fig. 67, 0 is the centre of the sphere and ABCD
is a plane section of the
cylinder through its axis. If Xis the mid'point of the generator BC of

the cyUndcr and if the radius and height


of the cylinder are respectivelv
r and h. the right-angled triangle BOX
gives
r» + iA* = o*. (9-6)
The volume V of the cylinder* is given by
V = ?rf*A.
160 PURE MATHEMATICS 19

Since, from (9-0),

V = 7Th{a^ - l/r-) = n(aVi ~ JA*).

the volume is now expressed in terms of the single variable h. T is a


maximum or minimum when dl’/dh = 0. i.c., when
7r(d2 - JA=) = 0
or when h = 2a/\/3.
This value of A does in fact give a Maximum value for the volume of the
cylinder since

~ = - y 37Ta,
which is a negative quantity.

Example 8. Adespatch rider is in open country at a distance of 6 miles from


the nearest point P of a straight road. He wishes to proceed as quickly as
possible to a point Q on the road 20 miles from P. If his maximum speed,
across country, is 40 miles per hour and, along the road. 50 miles per hour,
find at what distance from P he should strike the road, (L.U.)

In Fig. 58, the rider starts from a point A and strikes the road at a point
13, X miles along the road from P. Then = d, PB = x, PQ = 20, AP
BQ = PQ - PB = 20 X miles.

Since the angle APB is a right angle,


AB = ^(39 +
speed is 40 m.p.h. and along BQ it is 50
m.p.h.,
Along AB. the rider’s
so that the times taken to traverse AB
and BQ are respectively

+ x^) 20 - X
V(36 and hr.
40 50

journey therefore given by


The total time T for the is

- X
r=
accomplished as

a/OO +
io
quickly as possible, this
,
20

must
and for the journey to be
be a minimum, or dTfdx 0. Now
dT 1
2x J_
^ “ 40 ^ 2%/(30 4- X*) 50’

= 5-/^- = 38 +
This vanishes when ^(36 + *>)
i -

3tr/4 = 0 = 8 mdes.
or when OV/IO = 30. giving
or tr
9] APPLICATIONS TO PRACTICAL PROBLEMS 16]

Example 9. A straight line AB


ends on two fixed perpendicular lines
has its

OX, OY
and passes through a fixed point C whose distances from the fixed
lines are a, b. Find the position of AB
which makes the triangle AOB
of minimum area and calculate that minimum area. (L.U.)
Draw CD, CE perpendicular respectively to OX and OY. Let AB make
an angle Q with OX. Then
AD = a cot B, BE = b tan 6.
Thearea of the triangle AOB is the sum of the areas of the rectangle
ODCE and the two triangles DAC, ECB.

Hence if A is the area of the triangle AOB,


E ^ ab \a cot Q.a 4* \b.h tan 0
= ab + la* cot 0 + 16* tan 0.
( 9 7i
.

This is a minimum when dl^jdB = 0, wheni.e.,

— la* cosec* 0 + 16* sec* 0 *= 0,


orwhen tan 0 = a/b.
The value of the minimum area is obtained by substituting the value
of 0 given above in (9.7), giving
A min = 0* 4- \a*{b/a) 4- ib*{a/b)
= 2a6.

EXERCISES 9 (c)

1 An open cylindrical vessel is to be constructed from a given amount


ofuniform thin material. Show that it contains the greatest possible
volume when its height is equal to the radius of its base.
(O.C.)
A piece of wire, which forms the circumference of a circle of
12 in.
radius. Is cut and bent so as to form two new circles.
Find the radius
of each circle in order that the sum of the areas
of the two circles
shallbe as small as possible.
(O.C.)
Square pieces are cut out of a square sheet of metal as shown
in the

FtG. 60
162 PURE MATHEMATICS [9

figure,and the remainder is folded about the dotted lines so as to


form an open box. If the length of the edge of the square sheet is
IS in. find the maximum volume of the box.
r*
(O.C.)
4. A cylindrical tin canister without a lid is made of sheet metal. If
S is the area of the sheet used, without waste, V the volume of the
canister and r the radius of the cross-section, prove that
2V = Sr - nr\
If 5 given, prove that the volume of the canister
is is greatest when
the ratio of the height to the diameter is 1:2. (L.U.)

5. A BCD a square ploughed field of side 132 yards, \vith a path along
is
its perimeter. A man can walk at 5 m.p.h. along the path, but only
at 3 m.p.h. across the field. He starts from A along AB, leaves AB
at a point P. and walks straight from to C. Find the distance ofP
P from A. if the time taken is the least possible. (L.U.)

6. An isosceles triangle of vertical angle 2$ is inscribed in a circle of


radius a. Show
that the area of the triangle is 4a* sin 0 cos* 6 and
hence that the area is a maximum when the triangle is equilateral.
(L.U.)

7. The sum of the perimeters of rivo rectangles is 198 in. The ratio of
length to breadth is 3 2 for one rectangle : and 4 3 for the other.
:

Find the minimum value for the sum of their areas.


8. A piece of wire 10 in. long is cut into two parts one of which is
bent into a circle, and the other into a square. If the sum of the
areas of the circle and of the square is to be a minimum, find the
radius of the circle. (Q-^ )

9.6. Points of inflexion


Consider the function v {x = — l)®(l2x“ — 9x — 43). By the rule

for differentiating a product,

^ = 3(x- -9x- 43) + - 1)’(24 - 9)


l)=(12Ar= (a: a:

dx
= - {12a:2 _ _ 43 + _ l)(8x - 3)}
3(jr 1) = (jt

.= ^x - 1)2{20a:2 - 20x — 40}


= 60(;c - + 1)(^ - 2).
l)2(;r

Hence dy/dx vanishes when x = — 1, x = \ and x = 2.


x<-l, dy/dx is positive, when a; Ues between - 1
^d
when x and 2, dy/dx is negative
between 1
w en
is negative, lies
changes sign from positive to
x> 2. dy/dx is positive. Since dy/dx
negative as x increases through x — x = I gives a

dy/dx changes from negative to positive as x p^s


value and since iy/rf
= 2 gives Although
through = 2, ;t: a minimum value to y.
vanisLs when x = dy/dx does not
\.
change sign as
tangent to the curve is parallel to tn
tins ^'alue and. although the sucn
= neither a maximum nor a mmimum.
X-axis at X 1, this point is
9] POINTS OF INFLEXION 163

a point is called a point of inflexion a : rough graph of the function


is shown in Fig. 61 and the tangents at and adjacent to the critical
points A, B and C are shown in the subsidiary diagrams.

As we pass through thepoint of inflexion B, the derivative changes


from negative, through zero, to negative again. A
graph of the deriva-
tive would therefore show a maximum
at B
and another method of
finding points of inflexion would be
to seek maximum (or minimum)
values of the derivative. Thus, at a point
of inflexion at wlxich the
tangent to the curve is parallel to the ;r-axis
both

^
dx
=0 and ^=
dx*
0,’

and the second derivative changes sign


as we pass through the critical
pomt.
At a point of inflexion, the curve " crosses its
tangent **
and such
pomts can, of course, occur when the tangent
not parallel to the
is
^ second derivative can be shown to vanish
r^4.^u n
but the first derivative is, of course,
not itself zero. It is beyond the
scope of the present book to
consider these points in further detaU
^d we simply state the rule that the second
derivative changes sign as
we pass through and vanishes at
a point of inflexion.
To sum up the results of the
last three sections we have

d^
positive ; minimum value for y.

~ d^
^ negative ; maximum value for y.

A
^ dJT*
changing sign, point of inflexion.
164 PURE MATHEMATICS [9

Example 10, Find the turning points and point of inflexion on the curve
y ~ — 5x* -f 5x^ — 1.

Here,

5x* -
+ 15;v> = 5x\x* - 4;r + 3)
20,*-»

= 5x^(x - l)(x - 3),


d^y
= 20;»r8 - G0;r» + 30^ = 10jr(2Ar* - 6^ + 3).
Hence the first derivative vanishes when = 0, 1 and
;r 3. Wlien
X = 0, the second derivative vanishes, when x =s
the second deriv-
1,

ative is —
10. and when x 3 it is 90. = Hence x =
1 gives a maximum,

X =3 gives a minimum. Since the second derivative is negative for


small negative values of x, positive for small positive values of x
and zero at ;r =
0, there is a point of inflexion for this value of x. Since
the first derivative also vanishes when ;r =
0. the tangent to the curve
is parallel to the ;r-axis at the point of inflexion.

9.7. Curve sketching


It isoften useful to be able to make a rough sketch of a curve
without going to the labour of actually plotting a large number of
points on it. The following procedure, either wholly or in part, should
enable a good idea of the shape of a curve to be obtained.

(i) Determinethe curve is symmetrical about either or both axes


if

of coordinates. Symmetry about the ^-axis occurs if the equation


contains only even powers ofy and about they-axis if the equation
contains only even powers of x.
(ii) Determine if there is symmetry about the
origin. Such sym-
metry occurs when a change in the sign of x causes a change in
the sign of y without altering its numerical value.
(iii) Seek values of x which make y® negative
and therefore y imagin-
ary. No real points occur on the curve for such values of x.
(iv) Find where the curve crosses the axes of
coordinates. The curve

cuts the ^-axis at points for which y =


0 and it cuts the y-axis
where ^ 0. =
It passes through the origin if y 0 when ^ 0. = -
which make very large and values ot
(v) Find values of (if any) y
y (if any) which make x very
large.
m
- . •
>, •

its behaviour this neign-


(vi) If the curve passes through the
origin,
be decided by studying the value of the
bourliood can sometimes
smaU the curve keeps close to the
ratio y/x. If this ratio is
unity the direction of the
^-axis near the origin, if is nearly yA js large the
curve bisects the angle between the axes, while if/A
and better me
curve keeps near the y-axis. An alternative ^r g
of the derivative dyjdx near .

is to study the value


quantity measures the slope of the tangent to
Since this ^
the curve lies near the
a small value means that
unity means
while a value near
value that it lies near the y-axis,
9] CURVE SKETCHING 165

that the tangent at the origin approximately bisects the angle


between the axes.
(vii) Find turning points and points of inflexion (if any) by the methods
of this chapter.

Some illustrative examples follow.

Example 11. Sketch the cxtrve y = ;r* — 6^* + 8;r + 10.


Since odd values of x and y occur, there is no S 5'mmetry about the
coordinate axes. A
change in sign of x alters the value of
y so that
there is no symmetry about the origin. Points exist on the curve for
all values of x. The curve crosses the y-axis where y 10. =
It is not
convenient to find quickly the points at which the curve crosses the
4r-axis for this requires the solution of a quartic equation. The curve
does not pass through the origin, men
x is large, the dominant term
is X* and this is positive whatever the sign of x
hence y is large and
:

positive when x is large and positive or large and negative.

dy
- 12;r + 8
= 4(x - l)»(x + 2).
d*y
« 12x» — 12.
17*

Hence turning points or points of inflexion occur when x = 1 and


ar «- 2.

Fig. 62

When X = the second derivative vanishes and changes


1,
sign so
x^l gives a point of inflexion where the tangent
is paraUel to the
x-axis. When x =
1, the value of
y is easily found to be 13. When
^ ^— 2, the second derivative is positive, so
that x = —
2 eives a
ZterS
tbe value of y at this point being
Sketch of the curve is shown in
-
U. A rough
Fig. 62.

Example 12. Sketch the curve y* = *•. ,

y « symmetrical about the


It 18 not symmetrical about the
y-axis for an odd power of *
occur, m the equation. When x is negative,
y. is negativeTd toert
'

166 PURE MATHEMATICS [9

Fig. 63

are no real points of the curv’c for such values of x. The curve passes
through the origin and since its equation can be written in the form
(y/x)^ = X, the ratio y/x is small when x is small. The curve therefore
lies close to the ;«r-axis near the origin. As x becomes large so does y.
A sketch is given in Fig. 63.

EXERCISES 9 [d)

1. Find the values of x at the points of inflexion (if any) of the curve
y = — 4jc^ + 1.

2. For what values of x are there points of inflexion on the curve


y = AT + sin ;r at which the tangent is parallel to the ^-axis ?
3. Discuss the nature of the points on the curve
y = Zx^ ~ 8x^ - 2-ix^ + 9G;tr

at which the tangent to the curve is parallel to the ;r-axis.

4. Give a rough sketch of the curve a^y = 4.r*(3a — 4x), where a is

constant. (L.U.)

5. Sketch the graph of the curve y* ^ x{5 — x)*, (L.U.)

0. Find (i) the slopes of the tangents to the curve whose equation is
y^ = x^{\ — x^) at the points where ^ = 0 and x = \, and (ii) the
values of x at the turning points of the curve. Sketch the curve.
(O.C.)

EXERCISES 9 (e)

it is
1. A vessel is constructed so that the volume of water contained in

192'
(x^ 4- 24;r» + 192^r) cu. in.,

when the depth is in. What is the rate of increase of volume


per unit increase of x when (i) ;r = 2, (ii) x — ^
How many times faster does the surface rise when x —
when = 4, if water is poured in at a constant rate ? '

a cone remains constant while the radiiis of its


If the volume of
rate of per cent, per second find the
base is increasing at the 1
is dirainis img.
percentage rate per second at which its height
9] EXERCISES 167

3. A trough 10 ft. long has its cross-section in the form of an isosceles


triangle. The depth of the trough is 8 in. and it is 10 in. wide at
the top. If water runs into it at the steady rate of 40 cu. in. /sec.
at what rate is the surface rising when the depth of water is 4 in. ?
4. A body moves in a straight line so that its distance s ft. from a
fixed point O at time t sec. is given by
s ^ {t - 2)^(2t - 7).

Find when the body passes through O and the velocity and accelera-
tion each time it passes. Find also the minimum value of the
velocity.
6. The velocity v ft./sec. of a particle w'hich has travelled a distance
5 ft. from a fixed point is given by v* 16s. Find tlie acceleration =
of the particle.
6. Find the values of x and y at the turning point of the curve
ay = bx* -i- cx,
where a, h and c are positive constants. Is the turning point a
maximum or minimum ?

Find the values of x at the turning points of the curve


— X \
^ ~ x' +
X + 1'
State which maximum is a and which a minimum.
8 . Show that maximum and minimum values of cos* x sin x occur when
sin* ar = 1/4.
9 . Show that the minimum value of a sec 0 — 6 tan Q is ^/{a* — 6*).
10 . Determine the value of x for -which

is a maximum ; « is a constant. (Q-E-)


A prism of square section contains 64 cu. in. of clay, the side
of
the square being x in. Express the length of the prism
in terms
of X and find the total area of its faces.
Show that the total area is a minimum when the prism is a
(O.C.)
A piece of wire of length cut into two portions of lengths x
/ is
and Each portion is then cut into t\velve equal parts which
are soldered together so as to form
the edges of a cube. Find an
expression for the sum of the volumes of the two
cubes so formed.
What is the least value of the sum of the volumes (O.C.)
?
Find the height of the right circular cone of
maximum volume, the
sum of the height and radius of the base being 12
in. (O.C.)
Post Office regulations restrict parcels to a
maximum length of
3 ft. 6 in. and a maximum girth
of 6 ft. Find the maximum per-
nussible volume of a rectangular parcel.
Find also the length of the longest thin rod
which can be packed
mside a parcel of maximum permissible
volume, giving your answer
in feet to three significant figures.
/n c \
168 PURE MATHEMATICS [9

15. The point is 7 miles due north of a point B. One man starts
from A and walks due east at the rate of 3 m.p.h. Simultaneously
a second man starts from B and walks due north at the rate of
4 m.p.h. Find the rate at which the distance between them is
increasing when they are 15 miles apart. Find also the minimum
distance between them.
10. A water tank with an open top and square horizontal cross-section
is to contain 32 cu. ft. Find the cost of lining the tank with sheet
lead at gd. per sq. ft. when that cost is the least possible.
17. In a triangle ABC, the angles B and C are equal. Prove that the
maximum value of cos cos B is 9/8.
18. Find the dimensions of the rectangle of greatest area which can be
inscribed in a circle of radius r.
19. Given that the stiffness of a beam of rectangular cross-section varies
directly as its breadth and as the cube of its depth, find the breadth
of the stiflfest beam of rectangular cross-section which can be cut
from a cylindrical log of diameter 2 ft. (Q-E*)

20. Find the quantity which, when added to the square of its reciprocal,
gives a minimum sum.
21. Find the maximum and minimum value of y when
y = ~ + 1.

Find also the value of x at the point of inflexion. (Q-^*)

22. Find the abscissa of the point of inflexion on the curve


y = ax^ + bx^ A- cx A- A,

where a, b, c and d are constants.


23. Sketch the curve {a - x)y^ = {o -h x)x^. where a is constant.

24. If a a constant, trace the curve


is x^ = 4a*(2a - y).
25. Sketch the graph oi y = x/{x‘ + 1). finding the maximum and
minimum values of ;y. , j j v
entirely within the region bounded by.

Prove that the graph lies

the lines y = ± J.
I «
CHAPTER 10

INTRODUCTION TO THE IDEAS OF THE INTEGRAL


CALCULUS
10. 1. The nature of the fundamental problem of the integral
calculus
The preceding chapters on the differential calculus have been con-
cerned with the rate of variation of various known functions. The
integral calculus is concerned with the inverse problem—*/ the rate oj
variation of a function is known, what is the function itself? In symbols,
we have to find a function oi x when the derivative dy/dx
y is known!
i.e., we have to find
y from the equation

2 = (
10 . 1)

where <l>(x) is a known function of x.


As an example, suppose we know that the velocity at
time / of
a particle moving in a straight line is
(« -f at) where u and a are con-
stants, and we ^h
to find an expression for the distance s travelled
by the particle in this time. Since
the velocity is expressed by ds /dt
we have to find s from the equation
ds
(
10 2 )
.

This entails finding a function of


t whose derivative with
respect to
* IS (M
-f at). An inspired guess wiU lead to the result s =
for the denyative of this
«< +
expression is «
only solution to our problem.
at. +
This, however, is “not

Where C is any constant whatever,


The function s td -f- lat^ 4- C =
has the same derivative- and the
general solution of equation
(10.2) is

s = «/ 4- lai^ _|. c, (10


The conventional way of writing the solution of equation
(10,1) is

dx, (10.4)
=
^ integral of ^{x) with respect to
ori^ of t^ notation will be x. The
explained later (§ 10.6) ; at present we
It a means of expressing
^ y when the derivative
S
ofV Witt-
won nn
^
(10.2) would be witten
“Nation the solution of equi!
^
,

170 PURE MATHEMATICS [10

and this indefinite integral we have seen in (10.3) to be ut -j- lat^ -f C,


where C is an arbitrary constant.
There is a distinction between direct and inverse operations in
mathematics. Differentiation is a direct operation and can be per-
formed according to definite rules to give an unambiguous result. An
inverse operation is of the nature of a question. The operation of
integration, i.e., of finding the indefinite integral, asks what function
when differentiated will produce an assigned result ? We have seen
in the example given in (10.2) and (10.3) that if there is one answer
there are an infinite number owing to the presence of the arbitrary
constant C. To discover under what circumstances there is an answer
is beyond the scope of the present book. We shall simply state here
that there is an answer, i.e., the indefinite integral exists, for a large
class of functions and in this and the subsequent chapter we shall
discuss methods for finding it.

10.2. Standard forms


There are no infallible rules by which the indefinite integral

^[x) dx of any giv'en function <f>{x) can be found. Integration being


J
an inverse operation, we can only be guided by the results of the direct
operation of differentiation. Moreover, although the indefinite integral
exists fora large class of functions, it may not be capable of expression
in terms of functions normally employed in mathematics. An example
of a comparatively simple function where this is the case is

f sin jr

and such instances can be extended indefinitely.


standard
A of integrals is easily obtained from the list of
first list
these
forms for the derivative given on page 146. On inversion, each of
will givean indefinite integral. The student should become thorough y
familiar with this list which is fundamental.

^ (x") — nx^~'^, 1 X^ dx ^ —
« +
r + 0,
f
dx J
(except when « =—
—(sin x) = cos X, cos X dx = sinx C.
dx J

—(cos x) = — sin X, j* sinxdx = — cos X C.


dx

—(tan x) = scc^ x, sec^ X dx ^ tan x -r C,

dx J
,

10] STANDARD FORMS 171

—(cot x) ~ — cosec^ X. cosec^ X dx ~ — cot x C,


ax J

= dx ^
-^(sin'^if)
dx^ ' - x^y ,
C
-77 V
- x^) g.
= .

sin-i
,
.t + C.
V(1 J Vll
^(tan'i x) =— f dx
= tan~^ X ,
C.
^
1 +
^ -i-
dx^

In each case C
denotes an arbitrary constant.
By differentiating with respect to a; the results given on the right
of the above list by use of the standard derivatives
on the left it will
be seen that the result in each case is the function {i/ie
integrand)

included within the sign d.x. Thus, since


| .

dx + y n+l
= — + C.
v^+l
then
« + 1
_
And since — (tanA;+ C) =sec2A;,
then sec^ a; ij; = tan a: -h C,
J
and so on. It should be noted that the result given for x^ dx is
J
invalid for w =—1 ; the integral dx will be discussed later
J
(§ 13.4). It should also be noted that

r dx dx
and
J V(i- x^) ji
are conventional ways of writing integrals which should strictly be
wntten

dx and 4x,
x‘)
Since the differential coefficient
of the sum (or difference) of two
tmchons B toe sum (or difference)
of the differential coefficients of
me separate functions, it follows conversely
that the indefinite integral

L iJ'T
Pte xndefimle
of functions is the sum (or difference) of
integrals of the separate functions.
Thus

j
± X(x))dx = p(x)ix
± j x{x)dx. ( 10 6 )
.

j
1

172 PURE MATHEMATICS [10

and this result can be generalised to cover any finite number of


functions.
Again, since the differential coefficient of a constant multiplied by
a function is equal to the constant multiplied by the differential
coefficient of the function, it follows conversely that the indefinite
integral of a constant multiplied by a function is equal to the constant
multiplied by the indefinite integral of the function. In symbols, if a
is a constant,

a <fi[x)dx “^ j" (10.6)


J
The standard integrals given in this section and the rules expressed
symbolically in (10.5) and (10.6) enable the integrals of quite a large
number of functions to be written down. Some examples follow.

Example 1. Evaluate + 2 + (L.U.)


f(..

= rfjT + 2 f rfjtr
+
j* x-^ dx
j{
^+ 2x - x~^ i- C.
3
It should be noted that the integral is first expressed as the sum of three

separate integrals and that f 2dx = 2 dx=^ x^ dx = 2x. Also


2^
that the three arbitrary constants from the three separate integrals
can be combined into a single arbitrary constant C.

Example 2. Integrate {2x — 1)* with respect to x.

Since {2x - 1)=> = 8x^ - I2x^ ^ Qx - we 1. can write


%

[2x — \)^dx = 8
x^dx - 12
dx 8 \
X dx
J I J %

2x^ ^ 4x^ + 3x^ — X -\- C,

Example 3. Evaluate {26 + sin 0)dd.

(2^ + sin 6)d9 6 dO + I sin d9.


J'
6
2. — cos 6 C
2
Qi — cos 6 + C.

Example 4. Find

101 GEOMETRICAL AND DYNAMICAL APPLICATIONS 173
Since + 1)//* can be wTitten + 1//*, we have

j
^
1
^1 1
<3
- 3-/-3+C = --- + C. 1

EXERCISES 10 (a)

Integrate the following functions with respect to x :—


1. x*/^. 6
;»* + 1
. (L.U.)

2. -S/xK 6. sin X + cos x.

3. {l+;r)*. 7. 2sec*;r 4-
1 + X*

4.
(• ^ ^)’ 8 . x^ + 4x^ — 2x^ + X — Z.

Evaluate the following indefinite integrals :

9. j(3«« - + t l)dt. n.
dx~^ + bx-"^ + edx.
—8
j
10
f(^ 4. -)
.
dO. X2. f (20 + cos 0)dd.

13. By using the relation sec* 0 = 1+ tan* 0, find tlie value of

jtan*0da. In a similar manner show that

Jcot* 6 iO =C— 0 — cot 0.


14. Use the relation cos at = 2cos*i;r — 1 to evaluate Jcos* i;r rf,r,

15* If (1 + “ I* general value of y.

10.3. Some geometrical and dynamical applications


The problem a function which has a known differential
of finding
^efficient has many geometrical and
djmamical applications. The
indefinite integral gives
a general solution to this problem but often
a pa^cular ^lution is required
which satisfies some geometrical or
phyacal concEtion obtaining in the specific
problem under discussion,
ouch a condition enables the particular
solution to be selected from
a general solution
by fixing the value of the arbitrary constant in
the mdefinite integral.
Some illustrative examples are given below.
^

174 PURE MATHEMATICS [10

Example 5. At a point on a curve


the prodnct of the slope of the curve and the
square of the abscissa of the point is 2. If the curve passes through the
point X = 1, y ~ find its equation. (L.U.)

The slope of the curve at the point whose coordinates are x and v is
dyjdx so that

dy 2
Hence
Tx"" "x^*

and

where C is an arbitrary constant. 2lx + C gives, The equation y — —


for different values of C, a family of curves in each of which the product
of the slope and the square of the abscissa at a point is 2. The particular
curve which passes through the point ;r = l.y = — 1 is obtained by
selecting from this family that curve which passes through the point
in question. Since y = — 1 when x = \
- 1 = - 2/1 + C.
so that C= 1 and the required equation is y = 1 — 2/x.

dy
Example = +
6.
^ ax 2, where a is constant, express y as a function of x,

a*y
given that
^
= 6 and that y = 4 when x = 0 . (L.U.)

Differentiating the given expression for dy/dx we have


rf*y
= a.
dx*
and hence a = 6. Therefore
dy
~
dx
= (ix + 2.

and -
y = (G.r + 2)dx = 3.r* + 2;r + C.

The constant C found from the condition that y


is = 4 when x — 0, so
that C = 4 and y = 3x^ -{ 2x 4.

Example 7. A particle starts from rest with acceleration (30 — 6/) ft. /sec.*

at time t. When and where will it come to rest again. (O.C.)

Since acceleration is rate of change of velocity, if V is the velocity at


time /,

Hence (30 - 6t)dt = 30/ - 3/* +

where C an arbitrary constant.


is

Since the particle starts from rest. v = 0 when /


= 0. so that C - 0.
when y _ 0 and
Thus y = 30/ - 3/* = 3/(10 - /). The body is at rest
;

AREA AS LIMIT OF SUM 175

this occurs when < =* 0 (the beginning of the motion) and again when
i = 10 sec. If s is the distance travelled in i see.,
ds
—= = t; 30/ — 3/^

Hence s
= j* (30/ — 3/= )dt =., i5/> - /3 + C',

where C' is another arbitrary constant. Here the distance travelled s is


from the starting point so that s 0 when / =
0 and therefore C' 0. = =
Thus s = 15/* —
/* =
/*(15 —
/) and when t 10, the time when the =
particle is again at rest, the distance travelled will be 10*(15 10) or —
600 ft.

EXERCISES 10 (5)
dy
1. The gradient of a curve at any point given by —
is
^ 1. If

the curve passes through the point ;r = 1, y = I, find the equation


of the curve. (L.U.)
2. Find the equation of the curve whose gradient is 1 — and which
passes through the point x = 0,
y = h (L.U.)
3. A curve passes through the origin of coordinates and its gradient is
2x — x^/2 at the point whose abscissa is x. Find the ordinate of
the curve when = 2. (O.C.)
4. A particle is moving along a straight line with acceleration
(2 + 3<) ft./sec.* at time / seconds. At zero time its distance from
tte origin is 6 ft. at time f = 1 its velocity is 10 ft./sec. Where
is it at time f = 1 ? (O.C.)
6. A moves on a straight line OA and at time t it is distant
particle
X from O, X being taken positive when the point is on the same side
of 0 as Write down expressions for the velocity v and acceleration
.

/ of the particle at time t.

Find the distance x at time < if


/= 48/ — 24, given also that
» == 6 and =a — 1 when i = 0.
Show that the particle is stationary at 0 when / 1/2. (O.C.)
6. At time the velocity of a particle moving in a straight line is
t sec.
mcreasing at the rate (4/
+ 3//*) units/sec. When / = 1, the velocity
^10 ftysec. and at that time the particle is 4 ft. from the origin.
Where is the particle 2 sec. later and what is its velocity then ?

10.4. Calculation of
an area as a limit of a sum
As a preliminary to a second interpretation
of integration we give

tT ^ of
of the calculation of an area as the limit of a sum.
= 1 -|- a; be plotted, the graph is seen to be a
Qtm K* r
xraignt line and
a diagram as shown in Fig. 64 results. Consider the
AOBC included between the graph, the 3;-axis and ordinates
^ ^ ^ ^ = 10 respectively. The base of the trapezium
^
,^ts, OA = 1, BC — 11, the mean height of the trapezium is 6
and Its area A is therefore 60 units
of area.
« i.
176 PURE MATHEMATICS

Fig. 64

The area A might be calculated as follows. Divide the area


also
into n strips of equal width 10/n by lines parallel to the y-axis. Sup-
pose the rth strip is P'NMQ. Since for the first strip PN lies along
OA, for the second strip P'N is at distance 10/« from the y-axis, for
the third strip is distant 2(10/n) from the y-axis and so on, the

distance of PN from the y-axis for the rth strip will be (r — l)(10/w).
Similarly the distance of QM
from the y-axis for the rth strip will be
r(\Q/n). Thus for the strip shown in the figure, the ^-coordinate of
P is (r — l)(10/n), and since P lies on the graph of 1 y = +
the
ordinate PN will be given by

(10.7)
n n n
Similarly the length QM is given by
lOr
C1M =1+ (10.8)

By drawing PR perpendicular to and QS perpendicular to PNy


QM
it can be inferred that the area of the trapezium is
greater than the
area of n rectangles like PNMR
and that it is less than the area of
SNMQ. Since the base of the rectangles is 10/«,
n rectangles like

10 10r\ 10 100 lOOr


10/
area rectangle PNMR = ,

area rectangle SNMQ = — 10/,


f 1 -f —
10r\
1
—~ 10 ,
lOOr

The sum of the n rectangles of which PNMR is typical is therefore


10 ] THE INTEGRAL AS A SUM in
100 100 100 200 \
_ + + (\0 _ lot
\ n n \» n-
100 300 10 100 100«\
+ n
4“
n
_ /lO
— —~ 1
ICON .. , ,
1(1 T* 1 -f 1
,
+ . . to » terms)

+ ^«(l + 2 + 3 + • ‘
‘h ?•)

10 lOOY, 100 «(1 H- n)


'

n / n~ 2

= 10 -
n
A9
- T 50(1+^ =
+ n
jma
GO
bU - 50 .

n
Similarly the sum of the 7i rectangles of which SNMQ is typical can
be found to be 60
+ 50/«, and we have

60 - 52 < ^ < 00 + 5?. (10.9)


n n
By taking n = 10, i.e., by dividing OB into 10 equal parts, we have
< .4 < 65,
55
while if we take 100 strips (n ^ 100),

59-5 < ^ < 60*5.

*hat A lies, between 50-95 and


60 Oo and equation (10-9) shows
where e can be made as smaU as
that A lies between 60 —
e and 60 4- e
we please by taking n sufficiently

number n
of rectangles such -ls
PNMR aad SNMQ increases, their widths
(10/«) decrease and the
The area is in fact the
iMt
limh to which the sum
of the areas of the rectangles
their number approaches as
mcreases and their width decreases.

10.5, The
integral as a sum
The procedure outlined in
§10.4 could be used to Ihid
below a curve like v 1 4- -rJ Vm* = the area •

"patters we consider the function =


is y <i(x) as one which

TO AUVBr ‘ We «ish to i.v»tig,te


178 PURE MATHEMATICS [10

PNMQ is one of the n strips into which the area is divided in the
manner of § 10.4 and we take ON = x, PN = y, = bx. It is NM
assumed that the area CONP bounded by the axes, the ordinate PN
and the curve y — (j){x) is some function A{x) of x. When x increases
to X bx, the area CONP increases to the area COMQ and this we
take to be A(x) + bA{x). The area PNMQ is, by subtraction, bA{x).
The area of the rectangle PNMR is y bx, and since QM =y by,

the area of the rectangle SNMQ is (y -j- by)bx. It can be inferred


from the figure that
area PNMR < area PNMQ < area SNMQ,
or, ybx< bA{x) < (y + by}bx. (10.10)

Employing the symbol 27 to denote the summation of n such strips,

we have
27y bx < area AUVB < 27 (y + by)bx. (
10 11 )
.

Suppose we draw AF paraUel to the :):-axis and construct the


between
rectangle DEFB of Fig. 65 to be of width dx. The difference
difference between the
the two sums £ydx and E{y + Sy)dx (or the
SNMQand PNMR) can be seen, by shdmg
sums of rectangles like
rectangles like SPRQ paraUel to the *-axis until QR hes along
the area of the rectangle DEFB. This area is BF.d ,

to be equal to
dx=U V/n, the difference £ {dy Sx) between the two sums
and since
is I BF.UV)/n and this can be
made arbitrarily small by « wge t^g
if £y 5* ten^
and to
enough. Thus Z{Sy 6x) tends to zero with d*
In ^is ca^ t
the same hmit.
limit then £(y + dy)dx tends to
have the same limit,
area AUVB lies between sums which
follows that
area AUVB = lim. 27y bx.
10] THE INTEGRAL AS A SUM 179

rb
Thb limit is denoted \
y dx and is called ihe definite integral of y
with respect to x taken over the range from x = a to x ~ b. The letters
a and b, called ihe lower and upper limits of integration, indicate the
range in x from UA to VB over which the summation is made. The
symbol is specialised form of S, the sign of summation used in
J
earlier times.
The definite integral so defined isindependent of the idea of
differentiation. Except in very simple cases such as that given in
§ 10.4, it is not practicable to use it as a means of determining an area
because of the awkward summations involved. We can, however,
connect this definition of the integral with the definition of the indefinite
integral (the inverse of the derivative) given in § 10.1 and a practicable
method of calculation of area is then available.
To do so, we return to the inequalities of (10.10) which, after
division by 6x, give
dAix)
y < 6x
.
<y-\-6y. .

Assuming * that 6y tends to zero with 6x, this shows that


dA
(
10 . 12 )

y therefore the derivative of A{x) and hence A(x) is the indefinite


is

integral of y. Now A(x) measures the area CONP, and from Fig. 65,
area AUVB = area COVB - area COUA = A(h) — A(a),
Hence 1
y = fim. Ey Sx = area AUVB
Ja 0
-=A(b)-A(a), (10.13)
where A{x) denotes the indefinite integral of with respect to
y a: as
defined in § 10.1.
The argument given above a curve in which y is positive and
is for
increasing with x. When
y decreases as x increases, the inequalities
are reversed but it is stiU true that dA/dx lies
between y and y
and (10.13) still applies. If increases while x increases
5y +
y from a to A
and decreases while increases from k to 6, the integral
or area can be
found in two parts and these can then be summed.
It is also assumed

above that y positive when y


is ; is negative Zydxznd [ y dx are
negative (see §12.2).

funchons used in this book Sy always tends


to zero with 8* for the
values of ar under discussion. The student
should be warned, however toat this
IS not always the case and
equation (10.12) does not then reml
180 PURE MATHEMATICS [10

10.6. The example of § 10.4 solved by integration


The area bounded by y = I x, the x-a.xis and ordinates at = 0,
= 10 is, by (10.13),
^( 10 ) -^(0 ),

where yl (at) = indefinite integral of (1+^) with respect to x,

=X -f C. i4(10) is therefore 10 + +C= 60 -|- C and


<4(0) is simply C, so that in Fig. 64,
area AOBC = ^(10) -
^(0) = 60 + C -C
= 60 units.
This example shows that in evaluating an area or a definite integral,
the arbitrary constant of the indefinite integral may be omitted. If

the notation A{x) is used to denote the difference A(b) — A(a), a

convenient way of setting out the working of this example would be


10
area AOBC (1 -f- x)dx
=i
10
= X \x
0

= 10 + J102 — 0 — 0 = GO units.

The connection between the limit of a sum and integration has


In the
therefore enabled lengthy summation processes to be avoided.
present example, this not be a matter of great importance for the
may
when
area of a trapezium can be otherwise calculated. Nevertheless
are not
we pass to areas bounded by curves, formulae for the area
those of §10.4
generally known, summation processes similar to
complicated and integration provides a simple means o
become very
effecting the calculations.

examples of definite integrals and calculation


10.7. Illustrative
of area
illustrating a method or
In this section a few examples are given
out the calculations involved in evaluating a defoite “ '

setting
definite integial to the calculation of area
Some applications of the
are also given.
(L.U.)
Example 8. Evaluate

If I denotes the definite integral.


,

10] DEFINITE INTEGRALS 181

a’ (2ay a* 17c*
- ^ "a
” 3a
“ 6'

(L.U.]
Example 9. Evaluaie
r f-^)dx.
jr* - I
Writing the integrand in the form x j and denoting the

integral by /,

n2
r^r*

-L2+;Ji
*
2* 1 1

Example 10. Evaluate (t) 1 sin x dx, (ii) I .

Jo + *

(i) sin xdx = j^— cos ;rj


*
— cos Jir + cos 0=1.
J
~ ~ “ tan--^ (0) =
(“)
J ^ ^ X* 7r/4.

Example 11• Calculate the area between the x-axis and the curve y = x{x 4)
(L.U.)
A rough sketch of the curve shows that the area required (shown shaded

Flo. 60

in Fig. 66] is

=j (*«-4jr)*

182 PURE MATHEMATICS [1C

Vx^ n 4 43
\
~— „
2x^\ 2 (4)
is Jo 3

— —3 units,

(the negative sign occurs because the area lies below the A'-axis. see also
§ 12.2).

Example 12. The gradient of a curve at the point whose abscissa is x is


1 +^x^ and the curve passes through the point x 1, y 0. Find the = =
equation of the curve and the area bounded by it, the x-axis and the ordinates
X = 1. X = 3. (L.U.)

We are given
^ Hence
dx

-1 (1 + ix'‘)dx = X + ’L + C.
6

where C is a con.stant. Since y = 0 when ;ir=l, 0 = 1


C = — - and the equation of the curve is =—
•^6
y -j- x — 7
o 6
•3

The area required = ( (— + x — -Vx


J ^\6 6/
rx* 7 “13 81 ,
9 21 1 1,7 .

L24 2 oJl 24 2 6 24 2 C

EXERCISES 10 (c)

Evaluate the following definite integrals :

10 dO
(1 + x^)dx.
- 0 ^)’
I
J 0 V(1

2.
x*^l - ~^dx. (L.U.) 6 . sec * 0 dO.
I 1;

L x(l 4- x^)dx. (L.U.)


j0
;f/3
tan® 0 dO.

4.
x{x - l)(x - 2)dx. 8 . f' (sin + COS
p Jo (L.U.)

Find the area of the space bounded by the curve y = 1 + 10;r - 2^”'
9.
the x-axis and the ordinates for which x = 1 and x
= 5. (O.C.)

10. Find the area included by that part of the curve


y = jr® - Gx» 1 lx - -t- 6,

x = 1 and x — 2.
(O.C.)
which lies between
the
11. Find tlie area enclosed by the x-axis and that part of
y ,= 6x - a - X* for which y is positive.
10] VOLUMES 183

12 . The gradient of a curve is 6x —


the curve passes through
3x*. If

the origin find its equation and show that it cuts the ^*axis again
where x = 3, Find also the area bounded by the curve and the
;r-axis. (L.U.)

An ordinate is drawn to the curve y =


through the point
;r(l — x^}
X = 1 -f- p where p is positive. Find p so that the area between
the ;r-axis and the curve for x between 1 and (1 + /») may equal in
absolute magnitude the area between the ;r-axis and the curve for
X between 0 and 1. (L.U.)
The curve y llx — 24 — x*
cuts the ;r-axis at points A, B and
PN is the greatest positive ordinate. Show that 2PN .AB equals
three times the area of that portion of the curve which lies in the
first quadrant. (L.U.)

10.8. Volumes of figures of revolution


Another simple application of the definite integral occurs in the
calculation of the volume formed by the rotation of a plane curve
about an axis. Fig. 67 shows part of the curve y <f>{x) (again for =
simplicity shown as positive and increasing with x). That part of the
curve between ordinates aX x a and x =
b is rotated about the =
x-axis and we wish to find the volume of the solid so formed.

As in § 10.6, PNMQ is one of the n


which the area strips into
AUVB IS divided and ON =
PN =y. iVM dx. The volume =
formed by the rotation about the ;r-axis of the strip
PNMQ will be
rotation of the rectangle
formed by the rotation of the rectangle
bNMQ. The body formed by the rotation of the rectangle
PNMR
^ be a nght circular cylinder
NM =
of radius
dx and therefore of volume ny^ dx. Similarly
and length PN=y
the volume
• See
§ 20.6.
184 PURE MATHEMATICS [10

formed by the rotation of the rectangle SNMQ


be 7i{y -\- dy)^dx, will
for the radius of this cylinder is
y Sy. QM =
If we take the volume
formed by the rotation about the ;t-axis of the area CONP to be V{x),
that formed by the rotation of COMQ will be V{x) dV(x) and, by +
subtraction, the volume formed by the rotation of will be PNMQ
dV{x), Hence
Tiy- 6x < 6V{x) < 7i(y + dy)^ 6x.
The summation of n such strips shows that Sny^ Sx < volume (V)
formed by rotation of area AUVB < E7t(y + dy)^6x, and an argu-
ment similar to that used in § 10.5 leads to
V = lim. E Tty^ dx
ny^ dx = 71 (10.14)

Again the reasoning has been given for a curve in which y increases
with X. Arguments similar to those given at the end of § 10.5 will
enable this restriction to be removed.

Example 13. The portion of the curve xy % from x = = 2 to X = 4 w


rotated about the x-axis : find the volume generated. (L.U)

Here y = S/^r and the required volume is given by

jr dx = If {y,ixYdx
J

= 647r{- J = 16ff units.

Example 14. Find the volxone of a right circular cone of height h and sernt-
vertical angle a.

= ^ tan a is (Fig. 68 ) a straight line through the origin O


The graph of y

*-axis. The rotation of this 'i"® ^°ut the


niaking an angle a with the
circular cone of sem.-vert.cal angle «. The
^-axTS wiU generate a right
10] EXERCISES 185

volume of such a cone of height h is, by (10.14), given by


‘A rh
= jT j* y® jr |* {x tan a) * dx

Tj tan* a dx = -n tan* a
^"3
J ^0
iWi® tan* a.

EXERCISES 10 (i)

I'ind thevolume generated when the area enclosed by the ^-ajds and
the curve y = Zx^ — x^ is rotated about the ;»r-axis. (L.U.)

Find the volume generated when the area enclosed between the curve
y = x^/1^, the ;t-axis and the ordinate at ;r = 2 revolves about the
Af-axis.

3. Draw a graph of the curve y* = x(5 — x)*. If the loop of the curve
is rotated about the ;r-axis, find the volume generated. (L.U.)

4. Show by an argument similar to that given in § 10.8, that the volume


generated when a portion of a curve between y = c and y = is

rotated about the y-axis is ;i x* dy.


|

The curve y* = contained between the axes and the point


Iat,

where ^ = 1, y = 2 is revolved about (a) the Ar-axis, (6) the y-axis.


Find the difference bebveen the volumes of tlie solids so formed.
(L.U.)
The part of the curve Ar*y = at* + 3 behveen the ordinates x = \
and AT =s 2 is rotated about the Ar-axis. Calculate the volume
generated. (L,U.)
Find the volume of the solid formed by the rotation of the curve
y = cos ^x from = 0 to at = « about the Ar-axis.

EXERCISES 10 (tf)

1. Evaluate (i) 3Ar(2Ar« -|- Sat - ^)dx, (ii) -x)*y/x dx.


|
2 . Find the indefinite integrals wth respect to x of :

(i)(3A:-4)*. [ii) {Z + x){x - 1).

3. If (7 is a constant show that

(a* - Vt di 2(3/2^^ - C
I \^3 7 ^ 11
.

16^+^*
By using the addition formula for sin [x
+ a), show that

f sin (at -b a)dx =- - cos (at + «) -f- C,


— —
186 PURE MATHEMATICS [10

where a is a constant. Show, in a similar way, that


5.
cos {x + (x)dx = sin (^ + a) + C.
I
Use the relation cos 6 = 1—2 sin* |0 to evaluate \BdO.

6. A curve passes through the point where ;r = 0 and y = I and its


gradient at any point is ^ x — ^x^. Find the equation of the
curve and the area enclosed by the curve, the ;»r-axis and the ordinates
X = I, X = 3. (L.U.)

7. If 6 = 2 t =
n/% and if dB/dt = sin t, express 0 in terms of
when
8. The gradient of a curve which passes through the point ;r = 3, y = 1
is given by dy/dx = ;r* — 4;r + 3. Find the equation of the curve
and the area enclosed by the curve, the maximum and minimum
ordinates and the x-axis. (L.U.)

9. The velocity (v) of a particle is given by


d{v^)
= - 18;r,
dx
and t; = 4 ft. /sec. when = 1 ft. Find the velocity when x = B
and the greatest value reached by x during the motion. (O.C.)

10 . Integrate the equation = M— Wx, where E, I, and

are constants.
Given that both dyfdx and y = 0, w’hen x — determine the
value of dy/dx and y at the point ^ = 1. (Q-E.)

11. Evaluate the definite integrals:


(L.U.)

12. Evaluate

(i)
i) f*(l - y/xYdx, (ii) r (cosjf +sinx)dx. (L.U.)

Jo j «/2
13. Find the value of :

V2 dx
(i)
{x + cos x)dx.
'"'I
r04
14. If y = x^, evaluate | y + I x dy.
Jo
rn/*

15. For what values of a does I {x + i)dx — sec* X dx}


Jo
16. The equation of a curve is of the form y — ax^ + bx + c. It meets
the axis where = - 1 and ^ 3 = : also y = 12 'vh^J =
pr
and the area between it and ^
Find the equation of the curve
jr-axis.
the first quadrant bounded by the cu^e
17. Find the area in
a*y = - 4x) and the ^-axis. '
10] EXERCISES 187

18. For the curve y = — 3;r* — 12;r + 25, find the area enclosed
between the curve, the ;ir“axis and the ordinates at the maximum
and minimum points. (Q-E.)
19. Find the area between the curve — 4x^, the ;r-axis and the
ordinate at = 4. If the ordinate at = c halves this area, find
the value of c.

20. From the point where = 6, y = 27 on the curve 8^ — per-


pendiculars are drawn to the axes of coordinates. Show that the
rectangle formed by these perpendiculars with the axes is divided
by the curve into portions the areas of which are in the ratio 1 3. :

(O.C.)
21. A vessel for holding flowers is of the shape formed by the revolution
of the curve y = ^^/x about the y-axis. Its height is 8 in. Find
its volume. (O.C.)
22. Liquid in a cylindrical can of radius r which is rotating rapidly
about its axis is bounded by the surface formed by rotating the
curve y* ** 4ax about the ;r-axis (the ;r-axis of the curve coincides
with the axis of the cylinder and the origin is at the centre of the
base of the can). The liquid just clears the centre of the base of
the can and just does not overflow. Find an expression for the
height of the can and also the volume of the liquid in terms of
r and a.

23. Show that the volume generated by the revolution about the
^-axis of the area included between that axis and the curve

cy ^ a)ix - b) is
*

24. Find the volume generated when the loop of the curve 3x* = y(3 — y)^
revolves about the ^f-axis,
25. The part of the curvey/(20x — x*) between x ^ 0 and ;r = 10
y =
is a quadrant of a circle. The arc of the curve from the origin to
a point P whose abscissa is a is revolved about the ;r-axis. If the
volume of the bowl thus obtained is one-h 2Llf of the bowl obtained
by revolving the whole quadrant, show that 30a* a* 1000. —
(N.U.)
CHAPTER 11

SOME METHODS OF INTEGR*\TION


11.1. Introduction
In the introduction to the basic ideas of the integral calculus in
the last chapter, examples were chosen which involved only a few
simple integrals these were obtained as the inverses of the differential
:

coefficients given in the standard list on p, 146. The scope of the


integral calculus will, of course, be extended when other integrals can
be determined and it is the object of the present chapter to indicate
methods which help in this direction.
The student gradually realise that although he can usually
^^’ill

expect to be able to find the derivative of any function, there are many
functions which he cannot integrate. In certain cases this may arise
from a lack of experience or ingenuity but in others it will result from
the fact that many functions cannot be integrated in terms of functions
which are famiUar to the student at this stage of his knowledge of
mathematics.
A systematic study of methods of integmtion is most importmt
and the student should not regard integration as a purely tentative
judgment
process. Nevertheless it does require a certain maturity of
apply a standard method or to rely o" some
to decide whether to
will be well advised
special device in a particular case and the student
to work through many examples in his early
work. The exerci^
chapter will usu^y
given at the ends of the various sections of this
the preceding sections the ^t at tne
be on the methods given in
:

chapter not arranged in any definite order and the studen


end of the is
standard method or special cle\nce
wiU have to decide for himself what
he is going to adopt.
this chapter are by no me^
,

The methods of integration given in


methods which depend on a know-
aU that are available. There are

integral is discussed
ledge of f dx which are delayed until this
beyond
many other methods which are
in Chapter 13, and there are
the scope of this book.

of standard integrals
11.2. Generalisation of the list
of /(x) ^vith respect to x is/ [x).
Suppose the differential coefficient
definition of the indefinite integral gives
then the
11 1)
f'{x)dx = /W +
.

j"
C, (
11] METHODS OF INTEGRATION 189
where C
an arbitrary constant. The rule for the differentiation of
is

a function of a function (§ 8.6) shows that, if a and h are constants, the


differential coefficient with respect to x of J(ax + 6) is af\ax + i).
Hence

^f'(ax + b)dx = lf(ax ->rb) C. (


11 . 2 )

A comparison of (11.1) and (11.2) enables the integral of a function


of
(ax -f b) to be written down when the integral
of the same function
of X IS known. We can in fact say that if the integral of a function of
X is known, the integral of the same function
of {ax b) is of the same +
form but it is divided by a. For the special case of a
1, we have the =
rule that the addition
of a constant to x nuikes no difference in the form
of the integral.
As an example of the first of these important rules,

(2^ + 3)» dx = i{2x + 3)‘ + C,


J

i{2x divided by
integral of (2x + 3)’ willbe
3) 2, the coefficient of x in [2x
-f-

examples are v +
^ 3) Other

j(^+l)*<fA! = i{j: + l)3 + c.

(1 - *)» dx=- J(1 _ x)* + C,


j
ax __ 1
J {2x + 3)» 2(27+1)
^oilowing revised and more general list of
intJIlT
integrals to be drawn up and
it is this list that the student
should
^
tategrSiom

f (ax 4- 6)" dx = 4- C, except when w =_


J (» + l)a i,

cos ax dx = -1 sin ax + C,
.

jsmaxdx = cos ax 4- C,
J

sec® axdx = l tan ax + C, cosec® =-


I flAT rfj; icota^r 4- C.
j
( dx X
.
dx
J V(iFZD^) = ^
+ C. = ^ tan-i - + C.
I.
The above integrals aU foUow directly from those eiven in Sin 9
except the last two which ^

V(a* - **) = aV{l -


are derived^as foUo4 ShT
(*/«)>}.

V
190 PURE MATHEMATICS [H
dx dx
Iv V{1 - {x/a)^}
1 1 —1‘ ^ = sm _1 ^- H-
a (l/«)
Sin
a
+ I

C

a
I
/-•
C,

dx dx
and
1. = Mr + {x/aY
1 1
tan-' - + C = - tan-' -
+ C.
(1/a) a a a
should be carefully noted that the rules given in this section
It
apply only wlien x is replaced by (ax +
b). i.e., by an expression linear
(of the first degree) in x. They do not apply to integrals like

(x^ 4)^ dx or sin 2x^ dx where x is replaced by expressions of


J J
degrees other than the first in x. It is also important to be careful
not to omit the dividing factor a, a common mistake when the process
of integration is first begun. It is perhaps as well in the early stages
to check the correctness of an integration by differentiating the
expression obtained. This should, of course, give back the function
which was to be integrated (the integrand).
One other point may be noticed here. Earlier in this section we
found that, apart from the arbitrary constant.

(x + \)^dx = \(x 4- 1)
I
if we find this integral by first squaring (x H- 1), we have

+ 1)2 ‘ix
=^ {x‘ + 2x+ l)iix

I
= ^ + X.

This can he written l{x^ ^x‘ +


3a: 1)+ J’ + ~
two by 1/3. This apparent discrepancy can e
and the results differ
removed by including the quantity 1/3 in the — arbitrary cons an
The arbitrary constants in t eexpres
which so far we have omitted.
two methods will differ by 1/3 but all the terms
sions obtained by the
be identical. Expressions for indefinite ")tegrals
containing x will
differ at first sight "
obtained by different methods often
be found, on exammation, that aU terms
;
error has been made it will
are identical and the results differ only y
containing the variable
a constant.

(L.U,)
Evaluate + + 3) Ux.
3)|.
Example 1.
nj METHODS OF INTEGRATION
The given integral

{{x + 3)-^^ + (x +
i
r2(;^+3)V5+ l(x + 3)3«]
li\
L JO
2V4 + |V(‘i)’-2\/3-§^/(3) =
4 -f Y _ 2V3 - 2-V/3 = ^ - 4v/3.

ri/2
Example 2. Evaluate (ro5 + 2 cos 2;r}<f4?. (L.U.)
Jo
The integral
"1”^^
2
sin + A sin 2jr
^ Jo
sin - + sin w — sin (0) — sin (0) = 1.

Example 3, Find the value of (L.U.)


i:- 4;y + 13

Since .** - 4^ + 13 = ar* - 4.?


•U + 4 + 9 = (j? - 2)» + 3\ we have
dx
_ n
J_ IQ ^ f
4^+13e
J2
I
3> + (»-2)»
tan-Y'

-= i {tan-i (i) - tan-> (0) = i tan-» f }


The method used above o( expressing — 4jf -f 13 as the sum
jt*
of the
^uare of a linear expression in x and a constant should be notioed.
Integrals involving quadratic denominators without
factors can often be

made in this way to depend on the integral


f—
J
r—
a* X* +
— . When the
denominator can be expressed as the product of
linear factors, separ-
ation into partial fractions can be used
to make the integral depend on
r dx
* discussion of this case must wait until Chapter
X a’ 13. In a
J

way some
similar integrals can be made to depend on f ^^
J V{a*

EXERCISES 11 (a)

Integrate the following expressions


with respect to x •

I. {2-x)\ 3 - l)-a.

2. (l-ir)w. _
1
4.
- 7)'
Evaluate the following definite
integrals

6. sin
a.

192 PURE MATHEMATICS [ll

7. Integrate with respect to x :

(i) cos 2.;r — sin 2x, (ii) sec* (2x — 1).

8. Evaluate C'" . (L.U.)


Jo 4 + 9»>

Integrate the following expressions with respect to x :



1 1
9. 11
V(4 - x^y VIS ix - x^}'

1 '
10. 12 -
X^ X 1' •
3^2 ^ + f

11.3. The integration of products of sines and cosines


The product of two sines, two cosines or a sine and a cosine may
be integrated by first expressing the product as a sum by means of
the trigonometrical identities (§ 5.5),

2 sin ^ sin = cos — B) — cos {A -f B),


5 (.4

2 cos ^ cos Z? = cos {A — B) cos {A -f B), (li-3)

2 sin A cos 5 = sin — 5) -f sin (A + B).


(.4

Thus,

sin mx
" sin nx dx cos (m — 7t)x — cos (m -\- n)x}dx
1
_ sin (m — n)x _ sin (m +
2{m — n) 2{m «) -f-

j* cos tnx cos nx


~~ n)x}dx
^J
sin (nt — n)x sin {tn -f-
^
2(m — n) 2(m + n)

sin mx cos nx (fx =^j {sin (m — n)x + sin (m + n)x}dx


J"
cos (m — n}x cos (
m+
^ 2(m — nj~ 2(m + «)

these last three integrals should be


It is not recommended that
better to remember the method and apply it to
memorised— it is
Specific examples.

to x.
Example 4. Integrate cos Zx cos 2x with respect

(cos X + cos 5x)dx


cos 2x cos 2x dx
J
sin X + i sin 6x + C.
IIJ METHODS OF INTEGRATION 193

•n/2

Examples. Evaluate sin 2;r cos (L.U.)


|
0

p/2
sin 2x cos x dx _1 (sin X + sin 3x)dx
Jo “2J0
^
-i[- cos X — ^
O
cos Zx.1

Jo “3*

The integrals of sin^ mx, cos* inx can be found from the double-
angle formulae
sin 2mx = J(1
— cos 2mx),
cos^ mx = cos 2mx),
which can be deduced by ^vriting A B mx in the first two of = =
equations (11.3) and which have been given also in equations (5.11)
and (5.12). Again it is best to apply these identities to the particular
example under discussion rather than to attempt to remember the
general integrals.

p/4
Example 6. Evaluate I cos*xdx. (L.U.)
Jo
n/4
1 r'
COS* X dx (1 4- cos 2x)dx
1 2J0
n/4
0

p/2
Example 7. Evaluate I sin^2xdx. (L.U.)
Jo0
p/2
sin* 2x dx cos ix)dx
Jo
n
sm 4x = ~.
Jo 4

EXERCISES 11 (6)

Evaluate the following definite integrals

r«/2 fn/2
1. 1 2 sin* X dx, 4. (2 cos* 0 + 3 sin 30)d0,
Jo Jo
rti/Z
2.
[
cos* {x — a.)dx. 6 .
\
2 sin 3jr cos x dx.
Jo Jo
r»/2
p/4
3. (2 cos* 0 + 3 sin* 0)d$. 6 . 1 sin cos dx.
Jo Ja
H
194 PURE MATHEMATICS [11
ji/2 nl2
7. [ sin X cos x dx, 9. I (sin 3x -J- sin x cos x)dx.
I,0 Jo
3.1/8 2fi
8. I sin 30 cos 0 iO, 10 . sin mO sin nO dO, where m
i:n/a f
and n are integers and m ^ n.

11.4. Irrtegration bychange of variable


A very powerful method of integration isthat of changing the
independent variable, or, as it is sometimes called, integration by sub-
stitution. It will be seen from the derivation of equation (11.7) below
that this method is the converse of the method of differentiating a
function of a function (§ 8.6).
Suppose we wish to change the variable from to ^ in the indefinite
integral

=j ‘l>{x)dx. (11.4)

By the definition of the integral

(11.5)

and, if a: is a given function of /, the rule for the differentiation of a


function of a function (8.10) gives
dl _ dl dx
dt dx dt

Substitution of dl/dx from (11.5) leads to


dl . dx

and hence, if we again use the definition of an integral,

dx (11.6)
dt.
=j Tt

Equations (ll.C) together give the


important result
(11.4),

(11.7)
^{x)dx =j
j
The following are important cases .

Suppose we wisli to find ^(ax + b)dx where a and b are constants.


(i)
j
= tso that a{dx/dt) = 1 and dx/dt = 1/a. Then
Write axJ,-b
(11.7) gives
J
<j)(ax b)dx =J ^[t] .-<^1

J
11] CHANGE OF VARIABLE 195

= ^ j mu.
a S5anbolic expression of the important rule of § 11.2.

(ii) Integrals of the form dx can often be found by using


J
the substitution = t. In this case, 2x[dx/dl) = 1, giving
dx/dt = l/{2x), and from (11.7)

^<j>{x^)xdx
= ^<f>{l).x.~^dt
=Ij
so that ^{x^)x dx can be determined when (j>{t)dt is known.
J J
The presence of the **
extra a; in dx should be noted.
J
It is this term which enables the integral to be reduced to the

simpler form j* <l>{t)dL If the " extra x " were absent, the
^
corresponding result would be

m)^x = ^(o<-v* dt.


j i j
and this is not usually a specially helpful transformation.

Example 8. Integrate xy/{\ + at*) with respect to x.

By writing x* = /, we find as above that

^V(l + x^)dx = ^ I
V(1 + i)dt

= 1(1 + Ar»)3/^ 4- C,
when we replace / by x* after the integration.

X dx
Example 9 . Find j*—
+ Ar*'

Using the same substitution x* « t,

dt
1 + 1 + /» /* i tan-* t 4- C
J 2J
i tan-‘ X* + C.

For other types of integral, the choice of a successful substitution


196 PURE MATHEMATICS [11

is a matter of some judgment. It is only possible here to give a few


hints and examples.

(rt) If the integrand contains {a^ — x^), the substitutions x = asint


OT X = a cos / are often helpful.
For example let us put x = asint to evaluate the integral
C
I = —7—-, If X = a sin t, dx/dt — a cos t, and (11.7) gives
J
1

\^(a- — sin^ /)
. a cos i
-fi'"-
dl

for \/{a^ — sin^ /) = a cos t. Since x = a sin — sin“^ (x/a) and


t, t

we therefore have
dx
= sin -1^- 4* C.

^ ,

(
11 8 )
.

j -x^) a
as given in the listof § 11.2.
If we had determined this integral by means of the substitution
X — a cos t, we should have had dx/di = — a sin ^ and by (11.7)
^ = ->,'2 ^ U-l)dl=-t + C.
J
f \/la^ — cos^ /) J

Replacing t by its value cos“^ the final result would have been
dx X
= — cos * - H-
.

C . (11.9)
I \/(a^ — x^) a

The apparent discrepancy between the two results (11-8) and


(11.9) is explained by the relation cos”^ x + sin“^ x = obtained in
given
Example 12 of Chapter 5. Using this, the value of the integral
in (11.9) can be \vritten sin"^ (x/^) C' and agreement with —
(11.8) can be secured by writing C C' J.t. This is another = —
example of indefinite integrals obtained by different methods differing

by a constant quantity.
integrand, the substitution
(/)) When (a^ + x^) occurs in the
= a sec^ and
= a tan ^s indicated. With this substitution dx/dt t

dx
for
+ ij2tan2/ = a^SQc^t so that, using it to evaluate
j a^-tx^
example,
f — f 5 .a sec^ tdt =- C
x^ sec^ t ^ J
J -i- J

= - + C = ai tan-* a- + c,
a
for t = tan-* {x/a).
cos" 2 where >« and n are positive
(c\ Products of the type sin’" ;r :

can be integrated by the


odd
Sege^ and one at le^t of them is
llj METHODS OF INTEGRATION 197

following devices. If n odd, » — 1 will be even and cos’‘”^ x is


is

expressed in terms of sin x by means of cos^ x = l ^ sin^ x then :

we use sin = If m odd, sin”*”^ x can be expressed in terms of


^. is

cos:a; by means of sin^ = 1 — cos^jt and then the substitution


cos = used. Some examples follow.
A? /

Example 10. Find I sin^ x cos x dx


I"'
Here the powers of the sine and cosine arc both odd, so either of the
above methods can be used. Choosing the first (it is preferable here),
>\Tite sin x ^ t, so that cos x{dx/dl) = I and dx/dt = sec x. Using
(11.7) we therefore have

sin® X cos X dx <®,cos ;r.sec;r t^dt

= i/® +C ss
J sin< ;r + C.

Example 11. Integrate cos^ x with respect to x.


Here it is essential to use the first method for cos* is a particular case
of the product sin»» ;r cos» ar with m= 0 (even) and w 5 (odd). = We
write sin.»r /, =
and therefore, as in E.xample 10. dx/dt see x, =
cos'ar is witten as cos*;r.cosar, or {1 — sin*;r)*.cos;r, so that

cos* X dx (1 — sin* xy.cos X dx

(1 — <*)*.cos4r.scc^f

(1 -/*)»rf(

(I - 2/* -{ t*)dt

< - 1^* + +C if*


sin x^^ sin* x +
l sin* + ;r C.

Example 12. Find sin* x cos* x dx.


(Q-E.)
J
Here the sine is raised to an odd power so we set cos x
= and dx/dt = ~
=: t This aivei “
l oosoex. sm», is wriiten
ll — COS* x) sin X and hence

sin* * cos» * d,
= (1 _ <.<,5.
^
j I
“•j" (1 - **) sinar./».(- cosec*)*

= - (<• - <«)* = -
1 +C
« — i cos* X i cos* X + -f- c.
108 PURE MATHEMATICS [11

{d) There are many other substitutions which, with (11.7), are effective
in particular cases.For example, even positive integral powers of
sec X may be integrated by writing / = tan x and those
of cosec x
by using t = cot x. Examples will be found among the exercises.
As previously stated, the choice of a correct substitution is not always
evident and often depends on judgment and experience : in some of
the exercises hints are given.

11.5. Definite integrals by change of variable


In the evaluation of a definite integral when the integration is
performed by changing the variable, two methods are available. In
the first, the indefinite integral is found and expressed in the original
variable and then the hmits are inserted in the second (and usually :

preferable) method, everything, including the limits, is expressed in


terms of the new variable. Both methods are used in the first example
below the second method only is shown in the remaining examples.
:

r.i/2
Example 13. Evaluate I sin^ X dx.
Jo

Method 1. First find |* sin^ X dx, by writing

cos X = t, sin® X = sin .r(l — cos* ;r).

Then — s\nx{dx/dt) — 1, and dxfdt = — coscc Therefore

sin® X dx = sin ^(1 — cos* ;«)(— cosqc x)dt /»)(//


si

-I"-
=3 — cos ;r + J cos* x.
Hence
•.V2 r a/2 1 2
sin* X dx = — cos 4- J cos* xj
“I

^
=1—2 3
0

Method 2. Using the substitution cos x = /. the indefinite integral

can as before be reduced to -j (1 - At the lower Umit of the

given definite integral ;r = 0. so that since cos x = t, the lower limit of

the integral in / wiU be given by / cos0= 1. Similarly, since =


;r. the upper limit
of the
;r =
ff/2 at the upper limit of the integral in
transformed integral will be r cos Hence =
=- j°(l - f*)* = - - l'’]i “ 5'

as before.
X dx x — sin t
l)y meciHS of the suhsiitutxoH
Example Evaluate
14.
£ ^(1 -
_
0 and f = 0, when =
u x = sin t. dx/dt = cos /. WTien * = 0, sin I

= = w/2, so that the lower


and upper limtte of the
, = 1 sin < 1 and <
H] METHODS OF INTEGRATION 199

integral in / are respectively 0 and 7r/2. Hence


X dx f sin t
cos
j; Vli
- =j: V(1 - sin*0
. t dt.

=1 sin t dt

=
[- '] T
rn/4
Example 15. Evaluate \ see* X dx.
Jo
Put = tan so that sec^ x{dxfdt) = 1 and
i ;r dxfdt = cos*j \\Tien
^ = 0, = tan 0 = 0 and when x = v/A, =
i i tan{7r/4) = 1. Hence
n/i
sec* X dx = \ sec* x sec* x dx
I Jo
•nji
(1 + tan* x) see* x dx
j:0
l

(1 + <*) sec* ;r.cos* x dt


j:0
1

(1 + t*)dt
l:0
1 4
[+ *"]; 3

EXERCISES 11 (c)

Integrate the following functions with respect to x


1. flfcos (;r* — 1).

+ =
^ + 8]*
8 /).

8- (Hint, put = 0*

4. sin X cos* x,
6. sin* 2;r.

AT — 1
— ^
'v(l 9^*)

7. j-^_(Hint, put ** = 0 .

8. V(4 - x^).
Evaluate the following dehnite integrals

9. \xV{^+ x')dx, (L.U.)


Jo
,

200 PURE MATHEMATICS U1


.1/3

10 . sin® X cos X dx. (O.C.)


10
4.r dx
11 .

0(2 _;,2j3/2*
I
a
12 . xy/[a^ — x^)dx.
J0
p/s cos OdO ^
13.
.

® = '>•

J„ rr^n^-o
a
14. V(«* — x^)dx,
0

15. tan® 0 sec® 0 dd (Hint, put tan 0 = /)


I
J - .1/4
•**/2
COS X dx
10 .

V(siii X)
^

11.6. Integration by parts


",
A useful method as “ integration by parts
of integration, known
differential
results from the inverse of the formula (8.4) for the
This is
coefficient of the product ttv of two functions u and v of x.

d . du . dv
—(uv),
=v-- ,

dx dx dx
gives
which, with the fundamental definition of the integral,

uv - j i’t * “ih
This can be witten
dv
uv u^dx,
dx

which can be transposed into

_I
j
on the right-hand side of (11.10) is
The integral
evaluate than the one on the left. Some

when one an inverse


of the functions is
useful
^^
should be taken the
In this case this function "
right becomes
nn the left of (11 10) for then du/dx m the integral on the

algebraical function. This and some other devices


rfa rly imple illustrated m Examples 16-19
are
useM in this'^method of integration
below.
11] INTEGRATION BY PARTS 201

Example 16. Integrate x lan-^ x with respect to x


dn
Take u = tan-'^ x,
dv
— = x, so that
.

— = 1
and V — ^.T*
dx dx \ +
Then (11.10}
- -
/
gives
o

-i^dx=iz=
X tan-^ tan-**- dx. ( 11
. 11 )
J
1 f x*
The on the right, - can be written
integral
2
J
TT ,dx,

and this is — tan-*x). Replacing this in (11-11) we have

X tan“* X dx = J.** tan”' x — — tan~^ x) + C


1
« i(;r^ + 1) tan-^jf — Jx + C.

Example 17. Pniti ^jrco5*rfx.

If we take tt as cos x and dv/dx as x, dufdx =» — sin t and v = J-r*.

so that the integral on the right of (11.10) is j* \x*siTixdx and this is

more complicated than the original integral.


If, however, we take u =» x, dv/dx — cos x. du/dx = 1, v =* sin x,
and (11.10) gives

x cos X dx ^ x%\Tix —
-f.,
sin X dx
1

J
= X sin X + cos X + C.

Example 18. Find sin*xdx by the method of integration by parts.


I
Although integration by parts is not the best method for finding

sin* X dx, the working below illustrates a device which is sometimes


J
useful.
Take u = sin x, dv/dx = sin x, so that dxt/dx = cos x, w = — cos x.
Then (11.10) gives

sin* X dx ^ — sin X cos x cos x) cos X dx


J"
»» — sin X cos ^ j*
+
Since cos* x = 1 — sin* x, this can be written

(,i
sin* X dx ^ — sin X cos x + (1 - sin*
si x)dx
j
— i sin 2x + Ji*- sin*
fsi X dx.
202 PURE MATHEMATICS [11

giving, when the last term on the right is transposed to the left,

2 jsi
I sin* X dx = — ^ sin 2x + \
dx
1
i sin 2;r + + C.

X 1
Thus jsi
sin* X dx = — -^sm2x + C\
where C' (= IC) is an arbitrary constant.

Example 19. Find x^ sin x dx. (Q-E.)


^
Take « = x*, dv/dx = sin x, so that dtt/dx = 2;r. t» = — cos x. Then,
by (11.10),

X* sin X dx = — x'* cos cos x) 2x dx .

x^ cos X + 2'1 x cos X dx


\

j
Thus, one application of the rule of integration by parts has connected
the integral of x* sin x with that of x cos x. As in Example 17, a second
application of the rule gives

X cos X dx = X sin X — sin


-jsi I ar dx

and hence

X* sin xdx — x* cos x + sin t — 2


sin
si X dx.
J
Two applications of the rule have therefore related x* sin x dx to
J*

sin X dx, a known integral, and finally

X* sin xdx = — x* cos ;r + 2;r sin t + 2 cos x + C.


•r

In a similar way, if n is a positive integer, x** sin x dx can be related to


J
xn-^ 5 \nxdx and this latter integral can similarly be related to

be with either
xn-^s\nxdx and so on. We shaU finally left

dx to find in order to determine completely the


{xsinx dx or f sin ;r

original integral. integrals can in fact


Both these
integration performed in this way is
known as ^
reduction. It is a method of considerable
this book to pursue it except in very si p p
the scope of
above.
such as the example considered
11] APPROXIMATE INTEGRATION 203

In using the method of integration by parts to evaluate a definite


integral, the limits of integration can be inserted as the working pro-

ceeds. Thus if we require I x cos x dx and perform the integration


Jo
as in Example 17 above, the working could be set out
rV2 r -]n/2 pV2
I X cos xdx = \x%mx\ — I
s>\nxdx

= n- — r — cos X
2 L Jo

-I-'-
EXERCISES 11 {d)

Find the following by the method of integration by parts

J X sin X dx.

$
4.
1

f
sin“' X dx.

xsin-^x ,
cos* X dx.
^

1 V(l -
J

sin
si X cos X dx. 6. 1
X* cos xdx. (Q.E.)
J Jo
7. = 1 x^ COS xdx and = \ X” sin X dx, show that

Un = x^sin X - nF„_i.
= — ar»*cos;r + nC/„_,.
8. Evaluate
rn/2
(i) x^cos^xdx, (ii) cos“‘ X dx.
J j

11.7. Approximate methods of integration

If the value of the definite integral f{x)dx is required and we are


J
unable to find a function whose derivative is
f{x), methods of approxi-
mation can be used. There are several methods available.
Since f[x)dx measures the area bounded by the curve
| y =*/(x),
the x-axis and ordinates at x a, x = =
&, probably the simplest method
IS to draw the curve on
squared paper and to estimate the area by
countmg the squares enclosed. This is often laborious
and. unless a
very large-scale drawing isused, not very accurate.
Another method is to divide the area into a
.
number of strips and
204 PURE MATHEMATICS [11

replace each of these by a trapezium. In Fig. 69, the area PABQ


is to be found and the area has been split into strips by dividing AB
into six equal parts and erecting ordinates y^, yg. •. at each • •

point of subdivision, and y^ are used to denote the ordinates at

Fig. C3

X ~a and x = b. If C and D
are respectively the points of inter-
section of the ordinate with the curve and the ,v-axis, it is clear
from the enlarged diagram shown in Fig. 70 that the area of the
trapezium is slightly greater than the area bounded by the
FA DC

Fig. 71

that the area of the


PA, AD, DC, and
arc PC of tlie curve and the lines
area below the
trapezium could be used as an approximation to the
Now mean height of the trap^mrn is
arc PC of the curve. the
by AD
h, its area is \h{yQ -|- yj.
Treatmg
A IVn +
Vi) and if we denote
way and summing we see that
the other strips of Fig. 69 in the same
the area PABQ is approximately equal to

j/<(yo + yi) + ¥‘iyi + y^) + + jva) + • • • +


+ y,) + yi + + +
h{\(y« >-3 J-. +y>}-
AB = b - a and we have used six strips, A - i(* «)
and since
C

U] APPROXIMATE INTEGRATION 205

lu the same way, using n strips, we should find that the area PABQ
or y dx is approximate!}^ equal to
I

+>'1 "I" • . - + v„_2 (11.12)

wliere now h = {b — a)/n. The formula (11.12) can be expressed by


saying that if the range of iniegration (6 a) is divided into n equal —
parts each of width h (b = —
a)/n, called the interval, and ordinates are
erected at x ^ a, x = b and at each point of snb-division, then
b

y dx = interval x ijialf the sum of the first and last ordinates


a

+ the sum of the remaining ordinates).

This is known as the trapezoidal rule for approximate integration.


that the approximation improves as the number of strips is
It is clear
increased (or as the interval is decreased). It will also be clear that
the rule overestimates the integral when
y f{x) is a curve of the =
shape shown in Fig. 69. In other cases, for example when the curve
is of the shape shown in Fig. 71, it will underestimate the
integral.
The above method of replacing each piece of the curve = f(x)
y
by a straight line not usually the one most economical in labour.
is
If great accuracy is required a very large number of ordinates are
needed. A better method is obtained when an arc of the curve
y = Ax^ Bx -p C is used to replace an arc of the given curve
y =/w.

X
Fig. 72

Suppose in Fig. 72 that LMN is an arc of the curve y — fix) inter-


secting the y-axis at M and that LL\ NN' are ordinates at x = ~ h,
“3^0' hIO=yi, NN'~y^, Since the equation

k
+
Bx A- contains three constants the curve represented
Dy this equation can be made to pass
through three given points. If
we take these points to be L, Af and N
and if A is not too large, it is
206 PURE MATHEMATICS [11

to be expected that the area below this curve will be a good approxima-
tion to the area below the curve LMN
shown in the diagram. Hence

j* f{x)dx = j* (Ax^ Bx A-

^Ax^ -h IBx^ -f- Cx


J-h
= iAh^ + 2a. (11.13)

If the curve y = Ax^ + Rx + C passes through L, M and N, since

y = y^ when x = — h, y y^ when x = 0 and y — y 2 when x = h,


y^ = Ah^ — Bh C, -\-

= C-
y^ = Ah^ -f Bh -f C.
The addition of the fmst and last of these relations gives

yo-\-y2 2Ah^ = + 2C,


and since from the middle relation, C
2Ah^ =yo+y2~ 2^1-
Substituting for Ah^ and C in (11.13) we have

/(x)dx N ^*(>*0 + ya — 2>'i) + 2/iyi


j -h
= ^(^'o + ^yi + y^)- (ii.u)

of
This formula gives an approximation to the area of a s+xip
width 2h when an arc of the curve y f(x) is replaced by an
arc of
=
= three points on the ^ven
y Ax^ +
Bx A- C which passes through
this formu a is
curve whose successive abscissae differ by h. If
gives
applied to Fig. G9, the addition of three such strips

y dx = area PABQ
I
=. Ib’o + 4^1 + y^) +

= ^{>’0 + >’6 + 4b’i + + j’s) + 2b'2 + y*) )


number 2n of
In a similar way, if we divide the area into an even

we find that dx is approximately equal to


strips,

^’{^0 + + 4b'i + Jl'a + :i’5 + •+ • •

(n.l5)
+2(y, + v.+..-+^W 2)).

iij approximate integration «
207

where A = (6 — a)/2». This formula, which is known as Sifnpson's

rule, can be expressed thus if the range of integration {b


: —- a)
is divided into an even number 2n of equal parts each of
width
h = — {b a)/2n, called the interval, and ordinates are erected at x a, =
X = b and at each point of sub-division, then
/•6

I ydx'=ix interval X [the sum of first and last ordinates


J* four times the sum of all the odd ordinates
+ twice the sum of the remaining even ordinates).
Both the trapezoidal rule (11.12) and Simpson's rule (11.15) can
be used to evaluate definite integrals is given bywhen the integrand
a numerical table or by a mathematical formula. If the integrand is
given as a formula, a table giving values of the function (the ordinates
y) at equal intervals of x has first to be calculated.
Two examples
are given below. In the first, the integrand is only given numerically ;

the second, in which the integrand is 4/(1 x^), has been chosen so
that the approximate results can be compared with the known exact
answer.

Example 20, A function y of x is given by the following table


1

X 0 01 0-2 0-3 0-4 0-5 0-6

1
'

y 0-0000 0-0499 00095 0-1483 0-1960 0-2423 0-2867


1

ro-o
Find ydx by both the trapezoidal and Simpson's rules.
I
Jo

The working can be arranged as follows

M Fmi and last Odd Remaioiog evco


ordinates. ordinatos. ordioales.

0-0
0-1 00499
0-2 0-0995
0-3 0-1483
0-4 0-1960
0-5 0-2423
0-6 0-2867

Sums 0-2867 0-4405 0-2955

The interval A = 0-1, and the sum of the ordinates omitting the first
and last = 0-4405 + 0-2965 « 0-7360.
Hence the trapezoidal rule (11.12) gives

yd* ^ 0*l(i X 0-2867 + 0-7300) = 0-08794.


208 PURE MATHEMATICS [H
Simpson’s rule (11. 15) gives
rO-6
01
ydx •= —(0-2867 + 4 X 0-4405 + 2 x 0-2955) = 0-08799.
Jo
Thus to four places of decimals, the values of the definite integral by
the two methods are respectively 0-0879 and 0-0880. If the graph of
y against x be drawn it will be found that the curve is of the shape
shown in Fig. 71 and therefore the result 0-0879 obtained by the
trapezoidal rule must be expected to be a little low.

Example 21. Use the trapezoidal and Simpson's rules with five ordinates to

4 dx
find approximate values for p Compare your results with the
J0

exaet value of the integral.

Ordinates y = 4/(1 + X').

9 1 + X*
Fir^t and last Odd Rem.*iinif)g even
ordinates. ordinates. ordinates.
1 t

0-00 1-0000 40000


0-25 1 -0625 3-7647
0-50 1-2500 3-2000
0-75 1-5625 2-5600
1-00 2-0000 20000

Sums 6-0000 G-3247 3-2000

Here the interval h — 0-25, and the sum of the ordinates omitting the
first and last = 6-3247 -}- 3-2000 = 9-5247.

Hence, by the trapezoidal rule


4 dx
= 0-25(i X 6-0000 + 9-5247) = 3 1312.
X
J,0^-
By Simpson’s rule,

- ^?^(G.0000 + 4 X 6-3247 + 2 X 3-2000) = 3-1416.


1 + ^
3

The exact value of the integral is

n dx
tan"* X = 4 ( 77 / 4 ) =77 = 3‘14159 •

0
Simpson's rule .s
so that the result obtained by trap^l
place of decimals. To obtain similar accuracy ""‘h
and therefore more labour, would be necessary
rule more ordinates,

EXERCISES 11 (e)
riO
ydx when
Simpson’s rules to evaluate
I

Use the trapezoidal and



11 ] EXERCISES 209

y is given in terms of x by the following table :

X 00 9-25 9-5 9-75 100

y 01111 O-lOSl 0 1053 01026 01000

2 . Use eleven ordinates and Simpson's rule to find an approximate

value of

3. Equidistant ordinates of a curve are 10, 0-66^67, 0-5, 0-4 and


0‘3333 inches. Estimate the area bounded by the curve, tlic JV-axis
and the extreme ordinates which are 2 in. apart.
4. The ordinates of a curve y = f[x) are given by the table

X 0 1 2 3 4 5 6

y 0 2 2-5 2-3 2 1-7 1-5


1

Use Simpson’s rule to estimate the volume generated when the


area
bounded by this curve and the ordinates at = 0, = 6 revolves
about the ;»r-axis.
6. Use Simpson’s rule and eleven ordinates to obtain an approximate
ri/2
value of the definite integral' I Compare your result
with the exact value of the integral.

EXERCISES 11 (/)
Find the following indefinite integrals

^2x{x^ ^Z)^dx. (L.U.) -


1. 6.
6’
I 2;r« + 2;r +
2. Un ^ sec* x dx, (L.U.) 7.
I I :r«)
X ^ 2ta.n <)

>1 \/(4 - x*)' 8.


J
(sec* 0 — 1) sec* 0 dd, (L.U.)

dx
^ + + cos (x + n/2)dx,
1 2;r 6*
|
^ f cos* dd$ f
“sm*0 * \xV{x 2)dx (Hint, put
J
**
X -\-2 = f*).
Evaluate the following definite integrals :

rfi/2
11. sm
si 0ar cos ;r (L.U.) 12.
(L.U.)
Jo (6^r + 2)»‘
210 PURE MATHEMATICS [11

13. r%i
Jo
sin® 0 dO.
£ rV 2
1

o(2
X dx

sec®0rf0
14. sin® 0 dd. 18.
—a Jq 4 + tan® 0*

71/2 rn/2
15. sin® X cos® X dx. 19. I X sin* X dx.
j: J0

16. (6 + cos 0) sin 0 dO. 20 . x^{a* — x*)^^* dx.


i:
21 . By writing x = a cos® 0+5 sin® 6 show that
f" _
rn/2 r2
22 . If
I
X sin X dx = \ {ax* + 2x)dx, find the value of a.
Jo Jo
23. Find substitutions to show that each of the integrals
X* dx ri/V 2 X* dx
« and (U,
-
I V{1 X*)

f«/4
is equal to I sin® 0 dO.
Jo
24. Use Simpson’s rule and seven ordinates to find the area bounded
by the curve xy = 12, the A’-axis and ordinates at x = 1 and = 4. .ar

25. The coordinates of a curve are given in the table below. Find the
area between the curve and the ^-axis.

0 5 10
I 1
'

0 10 18
1
(Q.E.)
CHAPTER 12

SOME APPLICATIONS OF THE INTEGRAL CALCULUS


12.1. Introduction
A few geometrical and dynamical applications of the integral
calculus were given in Chapter 10. More elaborate examples of these
applications can be undertaken now that the student has studied some
methods of integration. Further applications of integration are
discussed in this chapter. These occur in finding mean values, centres
of mass, moments of inertia, lengths of arcs and areas of surfaces of
revolution.

12.2. Further examples of the calculation of area


Equation (10.13) gives the area bounded by the curve
y =f{x) the
jc-axis and ordinates x = a, x == h zs dx. If the curve lies below
y
^
the «-axis, negative and the area obtained from the definite integral
y is
appears as a negative quantity (see Example 11 of Chapter
10). If
therefore werequire the total area enclosed by a curve which
crosses
the ^-axis at points between the two abscissae
in which we are inter-
ested, we appeal to a diagram and divide
the range of integration
mto appropriate sub-ranges. Typical cases of this procedure
are given
in Examples 1 and 2 below.

Example 1. Find whole area enclosed by the curve


x-axts between
the
y = sin x and the
x => 0 and x => 2n.

A sketch ol the curve is shown in Fig. 73 and it is clear that

in
area OAB — cos X

cos n + cos 0 = 2,

^
2n
area BCD »
= 1 X (iar
sinxd^f
sin
= 1^— cos x~j
J
3 — cos Ztt 4* cos o> — 2.

“ ‘•‘"'fore * ““its. and the working could have been

whole area sin X dx sin x dx


i -f"
In
= [-cos,];-[_ooea]__ 4.
If the range of integration
had not been sub-divided
at » - w. the
211
212 PURE MATHEMATICS [12

^ 4

C
Fig. 73
result would have been
-\2ji
area
=r sin X dx -[ — cos
^J ^
= — cos 2»r + COS 0 = 0,

r2«
and although this is a correct value of the definite integral I sin x dx,
Jo
it is not a correct interpretation in terms of area.

Example 2. Find the area enclosed by the curve y — x(x — l){;r — 2) and
the x-axis between x =
0 and x 4t. =
A rough graph of the curve (Fig. 74) shows that the area required is

Now y = x{x-- l)ix - 2) = - 3x^ + 2x. so the area is

- *’ + - Q- -
[t *']o
= j_l + l— {4 — 8 + 4— i+l — l)+64 — 04 + 16 — 4 + 8 - 4
c= 10*5 units.

the *-axis has been a bounding line of the


In all our examples so far,

Sometimes the area enclosed between two different curves is


area.
and in such cases the result may be obtained as the ifference
required
between two areas each bounded partly by the x-axis. A ^typical

example follows.
12] AREAS AND VOLUMES 213
Example 3. Find the area bounded by the curves y* = ^ and x* = 4^.

In Fig. 75, the cur\-e y* = 4r is shown by a full line and the curve
= 4y by a dotted line. The two curves intersect at the origin O
and at the point B where at = 4. The required area is OA'BA and this
is clearly the difference between the two areas OCBA and OCBA', BC
being the ordinate at at = 4.

Pig. 75

For the curve y* = 4je, y = 2y/x and

area OCBA =« f 2y/xdx = t\xV':^^ units


Jo 3 L Jo 3
For the curve at* = 4y, y = Ar*/4 so that

area OCBA- = pi* i* = LlzA* = units.


4
Jq 12L Jo 3

The required area OA'BA ^ = 1? units


3 3 3

12.3. Calculation of volumes


Suppose the area of a section of a solid body by a plane
perpendicular
to the ^-axis at a distance x from the origin
is a function S{x) of x.
The volume dV of the element between planes at distances
x and
xi-dx from the origin lies between S(x)6x and S(x
ing the method of 10.6
dx)dx* FoUow- +
§ we therefore have, for the volume V of the
solid,

F — lim. i7S(^)d« = [
S{x)dx, 12 1 )
.
(
Ja
where a and are the end values of
fi
for the solid under discussion.
For equation (12.1) to apply, it is not necessary
for the solid to be
a figure of revolution about the *-axis. If it is
such a soUd, S(x) is
replaced by where y =
f{x) is the generating curve. In this case.
* See 20.2.
§
214 PURE MATHEMATICS [12

(12.1) gives

as given in equation (10.14). Volumes of solids of revolution generated


by the rotation about the Ar-axis of the area enclosed between two inter-
secting curves can be found as the difference of two other volumes in
a similar way to that sho^\Tl in Example 3 above. The second example
below illustrates the procedure.

Example 4. The top and bottom of a cistern are horizontal rectangles whose
longer and shorter sides are respectively parallel. The top rectangle
measures 16 by 10 ft. and the bottom rectangle 9 ft. by 6 ft.
ft. The sides
of the cistern are plane and the vertical depth is 8 ft. Find an expression
for the area of a horizontal cross-section at a vertical height of x ft. from
the bottom. Find, by integration, the volume of the cistertt. (L.U.)

In Fig. 76, A BCD


a side elevation with
is = IQ ft., BC = 9 ft.
AD
and ABEF is an end elevation with AF = 10 ft., BE = 6 ft. is XY
the longer side and XZ the shorter side of the rectangular cross-section

$/D£ ELEVATION £ND ELEVATION


Fig. 70

at height AT ft. from the bottom of the cylinder. BP and CS are vert^l
through B and C respectively intersecting the lines AD.
lines
XY .P.5

and Q.R as shown. From the first diagram it is clear that


AP ^ SO
and hence 2.4P = 16 - 9 giving AP = Hit. Also, from
the similar

triangles XDQ, ABP,


XQ AP XQ H
^ ” BP
XQ = RY,
X
so that
8
*

XY •= 2XQ + QR
so that XQ = Ix/IQ. Also
giving

= (t + V '*•

the same way from the second diagram we 6nd that


Working in precisely

XZ = (I
+ e) ft.

therefore
the horizontal cross-section at height * ft. is
The area S{x) of

5(z) = XY.XZ = (y + 9
A2 +
- (w + ¥ + “) -“
12] VOLUMES 215
The volume of the cistern
8
39^ \

1
J0
S[x)dx
I6 + T+
7jr> 39jr»

[ 48 8
4- 54;r
]:= 818| cu. ft.

Example 5. The area included between the parts of the two curves
4 = 1 and 4^’ 4 = 4
for which y is positive is rotated about the x-axis. Find the volume of
the solid thus formed. (O.C.)

A sketch of the curves shows that the first (full line) meets the A^-axia at
A and B (abscissae ^ 1 and 1 respectively) and the_y*axis at C (y = 1),
while the second curve (dotted line) passes through A and B and meets
the y-axis at D
(y =a 2).

The required volume is therefore the difference in


volumes of the
solids formed by rotating the areas
ABD, ABC about the x-axia. For
thecu^e ^BD, 4*» 4 y> =. 4 giving y» = — x*),
while for
4(1 ABC»
^ ^ y ^ ^ giving y* as 1 — Hence the volume required
^j* ~~ x^)dx ~ V
(1
— x*)dx => Stt >- x*)dx
^ j:.
x*li
Stt
4Tr units.

EXERCISES 12 (a)

1. Find the area enclosed by the curve =, ix - x\


y the ;r-axis an
ordinates at ;ir =» 0 and z ^ Q,

2 . Find the area bounded by y* *= 16;r and


y =1 3x,
3. Find the area between the curves a’y = and a^x = yS. (L.U.)
4. Find the area enclosed by the line « 2 and the y curve y « ;r(3 ~ x),
(L.U.)
6 . The area enclosed by the two curves y* = x*
and y* = 8(;r — 1)* is
rotated about the axis of x. Find the volume of the solid thus
formed.
216 PURE MATHEMATICS [12

6. The area enclosed by the curves y = 2 + v = 3 + and ordinates


at ^ = 0, ^ = a is rotated about the ;r-axis. Find the volume of
the solid so formed.
7. Water poured into a hemispherical bowl whose axis is vertical and
is
radius is a in. Show that at the instant when the depth measured
from the water’s surface to the lowest point of the bowl is x in.,
the area of the water’s surface is given by
S{x) = nx{2a — x) sq. in.
Find, by integration, the volume of the water at this instant.
8. A regular pjTamid has a square of side a for its base and its height
is h. Show that the area of a plane section at depth x below the
vertex is {ax/h)^. Prove also that the volume of the pyramid is
aVi/Z.

12.4. Mean values


Let jy be a function <f){x) of x and suppose that the range from x ^ a
to a; =6 is divided into n equal sub-ranges each of width dx. Let
yv y'z’ >' 3 >
•yn be the values of y at the middle points of each

sub-range. The arithmetic mean of these n values of y is

-(>'l
n
+ .y2 + ^3 + • • • + yn)t
and, since n dx = b — a, this can be N\Titten
+ y^ + + >^3+ yn)Sx
» « «

b — a

If as
limit is
n ^ 00 or dx — 0, the expression lias a limiting value, the

Fig. 78
12] MEAN VALUES 217

Figure 78 shows the curve y = and <l)(x) PA, QB are ordinates


B.t X = a and x = b. If construct on A B di rectangle of area equal
we
to that enclosed by the curve, the ^-axis and PA, QB, its height H
will be given by

H{b — a) — { y dx.
Ja

( ydx
Hence H= 0—a .

which the mean value of y over the range (b


is a). Thus in the —
geometrical representation, the mean value is the altitude of the
rectangle on base (6 —
a) whose area is equal to that included between
the curve y =
<l>{x), the extreme ordinates and the it-axis.

Example 6. Find the mean value of sin 6 over the range 0 to n.

The required mean value

sin 6 d$

0-637.
TT — 0
Sometimes a quantity whose mean value is required can be
expressed in terms of one or other of more than one variable. In
such cases it is important to state which is the variable whose range
is sub-divided in calculating the mean. Example 7 is an illustrative
example.

Example 7. A
body has an initial velocity of 80 ft. fsec. and it is subjected to
a retardation of Z2 ft.fsec.^ Find the mean value of the velocity of the body
during its forward motion.

If V is the velocity at time t we have dv/dt = — 32, so that

0
(- 32)dt =- 32/ + C,
I
where C is a constant. But « 80 when / = 0,
v so that C= 80 and
= 80 — 32/.

Forward motion ceases t» i.e., when = 0,


/ =* 80/32 when = 2-6 sec.,
and hence the mean velocity with respect to time
r 2-6 ^
2-6
vdt (80 - 32t)dt
I

“ ^-6 2^6

" “ B*®®® - 1®®) “


In the above we have worked in terms of the time /, Alternatively
wo
could have worked in terms of the distance s moved
by the body.
218 PURE MATHEMATICS [12

Instead of using dv/dt we could have used vdvjds or

for the acceleration. In this case

= - 32,
so that = — 32s + C',
and, since y = 80 when s = 0. C' = 3200. Hence
= 6400 — C4s,
and forward motion ceases when s = 6400/04 = 100 ft. Thus the mean
velocity with respect to space is
100
•v/(C400 - 64s)rfs
100

8
-
1 ^ j: v/(100 s)cis

8 r 2 "1100
= - !r(ioo - s)3''3
looL 3 -lo

8 2
.-.1000 = 63i ft./sec.
100 3

EXERCISES 12 (6)

1. Find the mean value of sin®^ for values of ^ between 0 and n.


2. The pressure p Ib./sq. in. and volume v cu. in. of a quantity of steam
are related by the law = 500. Find the mean pressure as the
volume of steam increases from 3 to 8 cu. in.
Find the mean height of the curve between jv = 1 ai^
= 3.
3.

4. Find the mean value of ;r sin ^ as ar varies from 0 to n.

6. The quantities v, s, t are connected by the relations


y = « <isin«^ 5 =
Show that the mean value of v considered as a function of /

*'

t = 0 and t = n/2n is {2an)/7t. ^


6. A stone is projected vertically upwards with velocity « and. on ite

retardation is g (constant). Show


upward journey, its
on the upward journey are (i) with respect to
values of its velocity
time, lii, (ii) with respect to space, |«.

12.5. Centres of mass


of particles
Let m,. m,. m, denote the masses of a system
at,. *3,
situated at points P„ P,. P,.
whose abscissae are
• •
. . .

on the particles are proportional to "h-


The forces acting '"f- field
gravitational It
of the earth’s
and they act through the centre
are distributed over a region whose
the particles
f
paraUel.
can be considered as being
fold the directions of the forces
;

12] CENTRES OF MASS 219

The resultant of the force system a single force proportional to


is

{nil + + ^”3 + • • •) acting at a point whose abscissa x is given by

{nil + "*2 4- ^«3 + . . (12.3)

and the point at which this resultant acts is known as the centre of
mass or centre of gravity.
Writing Wj -j- Wg + ^3 4* • • = ^» the total mass of the system,

and ntiXi nt ^2 + nt^x^ 4" • • • = N, sometimes called the x-monient


or first moment wth respect to x of the system, equation (12.3) can
be written
Mx = N, 12.4)

where M — Xm and N ^ X mx, (^2.5)


the sign X
denoting summation over all the masses of the system.
To obtain an extension from a system of particles to a continuous
body, it is natural to replace the particles by elements of the body
and to use limiting sums (i.e., integrals) in place of summations.
Consider first a rod of length I situated along the ^-axis and with
one end at the origin. Let the line-density of the rod at a point of
abscissa x he p this may be a function of x and, for simplicity, we
shall here consider that
p increases (or at least does not decrease)
with X. If dM
is the mass of the element of the rod for points whose
abscissae lie between x and x 6x, +
pSx <, dM < (p 4- ^p)dx.
Following the method of § 10.5, these inequalities lead to
dM
dx
and the total mass M of the rod is therefore given by

M= f
pdx,
(
12 6 )
.

Jo
The flc-moment 6N of
element lies between xp dx and
this
{x -h dx){p dp)dx, for lower and upper bounds for ^ are
^ and
X 4- dx, and for the mass the bounds are pdxand(p-^ dp)dx.
Hence
xpdx <6N<[x-i- dx){p -b dp)dx
leading to
dN

and, for the total *-moment of the rod,

W= px dx.
(12.7)
J
The abscissa X of the centre of mass of the rod is given by (12.4), viz
S = NJM, (12.8)
220 PURE MATHEMATICS [12

but now the values p dx, N= xp dx for the rod replace


| ^
those given in (12.5) for the particle system.
For the case of a rod of uniform density, p is independent of x
and can therefore be taken outside the signs of integration in (12.6),
(12.7). Hence, in this case,

X dx = \pl^,
and (12.8) gives x = ll, showing that the centre of mass of a uniform
rod is at its mid-point.

Fig. 70

Now consider a lamina AUVB


(Fig. 79) bounded by the curve

y = tl>{x), the 3c-axis


. and ordinates at x =
a and x b. For sim- =
plicity we shall here take the surface-density p to be uniform but
the

analysis is easily modified when the surface-density varies from


point

to point of the lamina. The total mass M


of the lamina is simply

p times its area, so that


(12.9)

PNMQ is bounded
a typical element by ordinates PN QM at abscissae
,

PR QS drawn parallel to the ;r-axis to inter-


X and x + Sx. and are
R The ^-moment dN^ of
sect QM and PN and
respectively in 5.
Sx)p{y + dy)dx
the element lies between xpy 6x and {x + PyJ^
the rectangles PNMR
and p{y + dy)dx are respectively the masses of
and SNMQ. Hence «
xpy Sx < dNx <(x d- Sx}p{y dy)dx. ,

-j-

leading in the usual way to


^ dNz
12] CENTRES OF MASS 221

The total ^t-moment Nt of the whole lamina is therefore given by


^

=J xpy dx = xy dx. (
12 . 10 )

since p is here assumed to be constant.


The mass of tlie rectangle PNMR is py dx and its centre of mass
is at a distance
\y from Ox. The corresponding quantities for the
rectangle SNMQ are p{y + 6y)6x and Hy 5y). Hence the mass of
the element PNMQ lies between py 6x and p{y + 6y)dx and the height
of its centre of mass between
Jy and 5 is they-moment
of the element

iypy dx < dNf, < J(y + dy)p{y + dy)dx,

leading to
dx
= lpy
The total y-moment Ny for the whole lamina is thus given by

N,
=j ipy^dx = i/? y^dx. (
12 11 )
.

j
If X, y are respectively the abscissa and ordinate of the centre of
mass of the lamina, equations equivalent to (12.4) give
Mx = Nt, My = Ny, (12.12)
where M, Nz, Ny are given by (12.9), (12.10) and (12.11). Solving
y and inserting the expressions
(12.12) for X, for M, Nx, Ny, we find
that the coordinates of the centre of mass of the lamina AVVB
(Fig. 79) are given by
'b rh
xydx y* dx
J
X = ~rS < y =
‘i:
rb • (12.13)

Finally consider a solid body of uniform volume-density


p. If the
area of a cross-section by a plane perpendicular
to the x-axis at distance
X from the origin is 5(a;) and if the solid is bounded
by planes per-
pendicular to the ^-axis at distances a and b from the origin, the volume
of the solid is (see
§ 12.3)
j
S(x)dx and its total mass M is given by

M = p[ S{x)dx,
(
12 14 )
.

J9
If dN is the *-moment of the element bounded
by planes distant x
and * + dx from the origin,
xpS(x)dx <dN<{x + 6x)S(x dx)dx. +
giving = ^ps(x).
g
222 PURE MATHEMATICS [12

The total jt-moment for the whole solid is therefore given by

TV =p xS(x)dx. (12.15)
J
The abscissa x of the centre of mass is then given by
Mx = N (12.16)

where M and N are given by (12.14), (12.15).


In the case of the solid of revolution formed by the rotation about
the x-a.xis of the area bounded by the curve y <t>{x), the .ar-axis and =
ordinates at x a, x — b, =
S(Ar) = 7iy\
The expressions for M and N become

AI = Tip dx, N = TTp^ xy^ dx^


^
and (12.16) gives in this case.

Example 8. The density of a rod AB varies as the distance from the end A.
Find the position of the centre of mass of the rod if its total length is 2 in.

If we take the rodto lie along the x-axis with the end A at the origin,
the density p at a point with abscissa x is hx where A is constant. The
total mass M
of the rod is given by

M=
r2
H 11
r-
X dx = k\
r T= 2A.
Jo lo L Jo
The AT-moment N is given by

f2
Fx^y Sk
N= f"
1 xpdx — k
Y'
Jo Jo L Jo
The abscissa of the centre of mass is therefore
N (8A/3) 4
^ ~ M~ 2k 3’

so that the centre of mass is IJ in. from the end A,

lamina of uniform
the coordinates of the centre of mass of
the
Example 9. Find
density bounded by the curve (1 y =
the x-axis - and
which X = 0 and x = 1.

If P is the surface density, the total mass M of the lamina is given by

M = «(* y dx =
ydx p (1
— x)* dx
\
Jo Jo
(1 - xyy __ p
[- 4 Jo 4’
12] CENTRES OF MASS 223
The ^r-moment Nx is

N.x = (>\ xydx = p\ ^(1 — xydx


Jo Jo

Zx* + Zx^ - x*)dx


Jo
3^*
_
~ -
~ L
sJo
For the ^-moment, Ny,

Ny=ip[ y^dx^yi (1- x)^dx


Jo Jo

7 Jo U*
The coordinates x, p of the centre of mass arc therefore given by

^ = (^)/(0 = B-

2
y « Ny/M «
r
Example 1 0. Find the position of the centre of gravity of a solid right circular
cone of uniform density.

As Example 14 of Chapter 10, the cone can be regarded as the solid


in
formed by the rotation of the line
y = x tan a about the x-axis for
values of X from 0 to A, a being the semi-vertical angle and h the height
of the cone.

The application of formula (12.17) gives, for the distance i of the


centre of gravity from the vertex of the cone.

x{x tan a) • dx x*dx


s = r
(x tan a) * dx x^dx
I j:

m: Zh

and, from sy ctry, the centre of gravity lies on the axis of the cone.

EXERCISES 12 (c)

1. The density of a rod varies as the square of the distance


from one
end. If the length of the rod is find the distance of the centre
of mass from this end.
2. From the point P
where ;r « 18, y «
IS drawn perpendicular
12 on the curve y» =» gr PN
to the x-axis

Py
™ find the distances from the
centre of mass of a lamina of unifonn
the x-axis and the curve,
density bounded
:

ic\n\
224 PURE MATHEMATICS [12

3. Find the position of the centre of mass of a lamina of uniform density


bounded by the curve = x^, the A^-axis and ordinates at x = I
and X = 4,

4. Find the abscissa x of the centre of mass of a lamina of uniform


density bounded by the curve ^ = 1 + lOx — 2x^, the 4r-axis and
ordinates for which x = I and x ~ 5.
Verify that
X = {y I 4- 12^2 + 5y3)/{yi + 4^2 + y^),
where yi, y2 , >'3 are the ordinates of the curve at at = 1, 3 and 5.

(O.C.)

5. Find the volume cut-off from the solid obtained by rotating the curve
= 4ax about the Ar-axis by a plane at distance 5a from the origin.
If the solid is of uniform density, find also the position of the
centre of gravity of this part of the solid. (O.C.)

6. Find the position of the centre of gravity of the solid within the
surfaces formed by the revolution of the curv'Cjy = x* and the straight
line y = 6 about the a.xis of y, (O.C.)

12.6. Moments of inertia

Let vu, w/3, denote the masses of a system of particles


. . .

situated at points P^, P^, P^^, whose perpendicular distances from


• • •

a given straight line are r^, Then the sum of the products
. . ,

is caUed
of each mass and the square of its distance from the given line
given line.
the mofnent of inertia of the system with respect to the
conveniently
This moment, sometimes also called the second moment, is
denoted by I, so that
I = m^rjf + nt^r^^ + •

If we imagine the total mass M(= m^ + wij -f WI3 + • • •)


the
®
the
system to be concentrated at a point at distance k from
this single mass has the same moment of inertia about the
such that
given line as the particle system,
Mk^ = 1 = mjTi^ + ^ ^ + 2 2
^ + • • •

is known as the
radius
calculated from this equation,
The distance k,

of gyration about the given


line.
P^rt.des +0
to
from a system of
As in the last section, an extension
continuous body is obtained by replacmg the
a VV below
place of summations. p
the body and using integrals in
a L
tion of
examples
moments of
apjucation of the
inertia
of fhe
but the scope of this bo
, ,

^ CtTforo/'lenSi^T^ situated
wh^
hne-density
the origin and let its
Xhe moment of
^
of the rod between points
12] MOMENTS OF INERTIA 225

with abscissae x and x dx satisfies the inequalities

x^pbx <61 < {x 6x)^{p -|- Sp)dx,


dl
so that = x^p.
dx
The total moment of inertia I is therefore given by

= f px^dx, (12.19)
Jo\i
V

Since, from (12.6), the total mass M of the rod is given by Af =J pdx,

its radius of gyration from the equation Mk^ = /, is given by

r px^ dx
Jo
(12.20)

p dx
j;
In the case of a uniform rod, is constant and can be taken outside
p
the signs of integration, so that

x^dx
Ml - 1 ( 12 . 21 )
dx H
Nowconsider the lamina AVVB
of Fig. 79. If the lamina is
supposed to be of uniform density, the moment of inertia 6Iy about
the y-axis of the typical element satisfies PNMQ
x^py dx < dli, <{x 6x)^p(y -f dy)5jt.
dly
Hence =
dx
and the total moment of inertia /y is given by
>6

=p dx.
(
12 22 )
.

J
The mass of the rectangle PNMR is py 6x and its radius of g^^ration
(from equation (12.21)) Its moment
is y/V3. of inertia about the
X-&XIS IS
py dx X (y/ ^3)^ or Jpy3 dx. Similarly the moment of inertia
about Ox of the rect^gle SNMQ is
ip{y
the moment of mertia about the x-axis of
6y)Hx. Hence, if dl, + is
the element PNMQ,
ipy^ 6x d7* <
Jp(y 4- dy)3 dx, <
leading to
dx *

and =
7* ipj y^dx. (12.28)
226 PURE MATHEMATICS [12

From (12.9), the mass M of the lamina is given by M=p y dx


|
and, if kr, ky denote the radii of gj'ration ^vith respect to the x- and
y-axes, (12.22) and (12.23) give

(12.24)

For a circular lamina of uniform surface-density p, the moment of

inertiaabout an axis OP through its centre 0 and perpendicular to its


plane can be found by considering the ring element bounded by circles

of radii r and r A- 6rshown shaded in Fig. 80. The m^s of the ring
is approximately 2-Tpr br and all points
within the ring lie at distances
Hence if 61 is the moment
from the axis OP of between r and r -f- 6r.

of inertia of the elementary ring

r^ifiTxprdr) < < ('' + 6r)'^i^pr dr),


o 3
Hence

and, if a is the radius of the lamina.

= 2., dr = Inpa*.
i;

Since the total mass M


of the ring isnpa^ this
moment of inertia ^bout
tbou't
This result can be used to find the
of symmetry of the solid formed by the
its axis
curve y
"=
x-a\is of the area bounded by
Ims L X «. X = .. It is=
the
left as an exercise to ^o^er
(uniform) volume-density of the solid
show that, is the
if
P
(12.25)
= i^p dx.
12 ] MOMENTS OF INERTIA 227

Example 11. The line-density of a rod varies as the distance front one end.
Find its radius of gyration about an axis through this end perpendicular
to the rod.

If the rod lies alongthe 4r-axis with one end at the origin, the line-
density p = Xx where A is a constant. If I is the length of the rod, the
radius of gyration A is given by (12.20) as

and A =
Example 12, A lamina of mass M
of uniform stirface-density p and is in
is
the form of a right-angled triangle of base b and height h. Show that the
moment of inertia about the base is ^Mh*.
The lamina (see Fig. 81) can be considered as the area bounded by the
line y *= hx/b, the x-axis and an ordinate at x = 6. By (12.23), the

moment of inertia about its base (the x-axis) is

ipj y*dx = \p^ {hx/h)*-dx

.
A*rxn6

But tte area of the lamina is \hh, hence


mertia can be written i(J/)A6).A* or \Mhy
= M {phh, and the moment of

Example 13. Show that themoment of inertia of a uniform solid right circttlar
cone whose radius of base is r and mass M
is ^Mr*.
M in E^mple 14 of Chapter 10. the cone can be regarded
as the solid
formed by the rotation of the Une
y «
^ tan « about the x-axis for values
of X from 0 to A. « being the semi-vertical
angle and h the height of tlie
228 PURE MATHEMATICS [12
From (12.25) the moment of inertia I about the axis of symmetry is

given by
•A

I = ^np y* dx = np
^7Tp {x tan a) * dx
Jo
J Jo
6~lh
Inp tan* «|_yjQ =
where p is the volume-density of the cone.
Since the volume of the cone is {Example 14, Chapter 10) \vli^ tan* a,

M= iw/j/i* tan* a.

By division, I/M = tan* a,

giving / = YoMr^, for r = h tan a.

EXERCISES 12 {d)

1 . Find the moment of a rod in which the line-density varies


inertia of
as the square of the distance from one end about an axis
through
that end perpendicular to the rod.
2. Find the moment of inertia about the y-axis of the lamina (surface-
density p) bounded by the curve ay = x[a — x) and the
^r-axis.

Find the moment of inertia of a uniform rectangular lamma


of sides
3.
about an parallel to the sides a and distant \b, §6 from
a and b axis
them respectively.
of moment
inertia of a uniform lamina in the form of an
Find the
isosceles triangle of height A about a line
through its vertex parallel
„ its base.
radius of gyration about its axis of
6. ind the square of the
by rotating about tlic
f the solid

-axis the
(of
area
uniform
between
density)
the curve
obtained
= ax and an ordinate ax.
6 . L uniform solid is formed
by the rotation “f
jb) from
and b about a line in its plane distant d (>
Find the square of the radius of gjTation
.arallel to the sides a.
solid so formed.
bout the axis of symmetry of the

'"Fis‘'8?s'h“w'.jrarc .1 a curve, -
.1;.
ci-pBciy.
!o!"p

“t:
/>i. y’2f- • *'
V;i
if t
j
r H
/
'riS
represents the angle between
the
s
-langie

subsidiary diagram shows that


3f the
p^P^^, = (^,+x-^r)secv''.
12] LENGTH OF ARC 229

Fig. 82

Now 1/.' isgreater than the angle vv at P, and than the angle v^r+i
less
at and it foUows that AP^ P^P^ + + .
+
P„_iB Ues between
. .

“ ^r) SGC^r and ^ (^r+i — secyj^+j. As the number of


points Pj, Pj, . . P,t_i between A and B increases, each of these

• f*
quantities approaches the same limit sectpdx. Since

sec^ y =1 -f tan® y) and tan y; = dy/dx,


the length of arc, which is defined to be this limit, is given by
AB =
arc
U{ ( 12 26 )
.

Example 14. Find the length of the arc of the curve Qxy ^
the points whose abscissae are 1 and
3 + between
4.

Here y - (3 + x*)/Qx = l/{2x) + x*/Q and hence

^
dx^
" 1_
**
'2'

(^y\^
1 + xix)
1 + (- - -V
\2 2.vV

^+ 4-2 + *
47

i{“ + •
+ i)

Therefore
J {'
+ ©} - K- + i).
230 PURE MATHEMATICS [12

12.8. Areas of surfaces of revolution


Consider a frustrum of a right circular cone.* If similar and
first
similarly situated polygons of n sides are inscribed within the circular
ends, by joining corresponding vertices PQ, P'Q' we obtain a series of
n trapezia of which PP'Q'Q is typical (Fig. 83). If M
and iV are the
mid-points of the sides PP' QQ* ,

area PP'Q'Q = \{PP' + QQ')MN.

Fig. 83

surface area
As the number n of sides of the polygons increases, the
formed by the trapezia tends to a Umit which is defined
of the solid
conical frustrum. The sum of the areas o
as the surface area of the
the trapezia is
, xjirAr
polygons)AZiV,
J(sum of perimeters of the
of the po'yg°"s ^nd to the c'r^mfCTenccs
and since the perimeters
of the end circles of the frustrum and tends MN
to the hmit PQ.

the surface area of the frustrum


ends) X PQ
=Afsum of circumferences of circular
its slant height.
=mean circumference of the frustrum X
by the rotation of area bounded
Now consider the solid formed
=the ^-axis and ordmates at x a x =
6 abou
by the curves

definitions.
• See Chapters 19 and 20 lor the necessary
12 ] SURFACE AREAS 231

dx sec {y> where xp is the angle between the tangent to the curve
4- Sf)

y = (l)[x) at P and the ;c-axis. Thus if dA is the area of the conical


frustrum generated by the rotation of the line PQ

^{y + i <5^ sec y) < dA C. 27i{y -}- J dy)dx sec (y -j- dtp).

The surface area A of the solid of revolution is defined as the limit


to which the sum of such areas tends as dx tends to zero.

dA
Hence = 27iy sec \p,
dx
and the total area of the curve is given by
6
A =2n ysGCxp dx.
^
Since sec^Y^ = 1 + tan®y = 1 + {dy/dx)^, this can be recast into the
form
•ft

A =2nj y^/{l + {dy/dxy}dx. (12.27)

Example 15. Find the area oj the surface formed by the rotation of the curve
y* = 4^ about the x~axis, from the origin to x Z,

Here y = 2^x, dyfdx = \/^/x, so that


r3
area required = in 2v';r.V{l + (l/V;r)*Mjr
%1 0

^ Alt 1

4 \)dx
%lo

“4 }(* + 1)H*
Jo
00
U 43/a _ is/aj

60jr

3
232 PURE MATHEMATICS [12

EXERCISES 12 {e)

1. Find the length of the arc of the curve between = 1


and X = Z.
2. Find the length of the arc of the curve x^ = r* between x = 0
and X = r (this curve is a quadrant of a circle of radius r. see
§ 16.1).
Find the length of the arc of the curve xV^
3.
+ yVZ = 4 between x = 0
and X = d>.
4. Find the surface area of the solid generated by the rotation about
the A'-axis of the area bounded by the curve =
y the
AT-axis and ordinates at ;r = 0, x r.

5. bind the area of the surface formed by the revolution about the
A'-axis of that part of tlie curve x-^^
+ y2/3 _ 4 which lies between
X — 0 and a” = 8.
(Hint, the integral expressing the area can be evaluated by writing
;tr2/3 = /.)

6. Ihe curve 3y = (3 — x) y/x between x — 0 and x = 3 rotates about


the ^-axis. Find the surface area of the solid so formed.

EXERCISES 12 {/)

1. Sketch the curve y = x{Z — x) for values of x betw’een — 2 and


4- 5. Find the area contained between the curve, the ;r-axis and
ortlinatcs at = 0 and ;r = 5. Explain why this area is not
I ;v{3 — x)dx.
Jo
2. Find the area enclosed between the two curves y^ — x and x* = 8y'
(L.U.)

3. Find the area enclosed between the two curves y* = 8(6 — x) and
y^ = 12x.
4. Find the area of the segment cut off from the curve y* = 4ax by
the line y — x. Find also the volume of the sohd obtained by
rotating this segment about the ^r-axis. (L.U.)

6, The curve y = 1 + cos x is rotated about the axis of x. Find the


volume contained by tlie surface of revolution and the planes
= -j- n/'l and x = — 7i/2. (Q-E.)

6. The capping of a stone pillar is a solid with every horizontal cross-


section a square. The centres of these squares lie on a vertical
axis and their corners lie on the surface of a sphere of radius 10
in.

whose centre is on the axis, 6 in. above the plane base of tlie solid.
(L.U.)
Calculate the volume of tlie capping.
+ over tlie ranges
7, Find the mean value of sin (»^ a) .

(i)
^ — a/« to ^ = 2mn: — (a/»),

(ii) ^ = - a/n to ^ = (^/2«) - («/«)»

where m and n are positive integers.


12] EXERCISES 233

8. The horizontal range i? of a stone projected with velocity V at angle


a to the horizontal is given by
R = (7V^) sin 2a,

where g isa constant. Show that the mean range for all angles
of projection from 0° to 90*^ is 2V^/ng.

9. Show that, a is a positive constant, the mean ordinate of that


if

part of the curve y ax — which lies in the first quadrant is


t^vo-thirds of the maximum ordinate.
10. A number n is divided at random into two parts. Show that the
mean value of the products of these parts is «*/6.
11. Find the position of the centre of mass of a lamina of uniform
density bounded by the coordinate axes and that part of the curve
{x*/a^) + = 1 which lies in the first quadrant.
12. Find the position of the centre of mass of a lamina of uniform
density bounded by the curve y = a sin [x/a) and the ^r-axis between
X = 0 and X = an.
13. Find the area of the loop of the curve y* = 4jr®(l — x). Find also
the position of the centre of mass of a uniform lamina bounded by
this loop. {Hint, put 1 — to evaluate the integrals.)
;if

14. Find the position of the centre of gravity of the solid (of uniform
density) formed by the rotation of the curve ;r>
+ y» =* a* from
X 0 Xo X = a about the jr-axis.
15. A uniform density is formed by rotating the portion of
solid of
the curve y = x^ ~ Zx which is cut off by the ;r-axis about that
axis. Find the position of its centre of gravity.
10. Find the moment of inertia about one of its sides of a square lamina
of uniform density
p and side a.
17. Find the square of the radius of gyration about the ;r-axis
of a
lamina of uniform density bounded by the curve
y = sin and the
AT-axis from x =^.0 to x ^ n.

18. Show that the moment about the ;ir-axis of a uniform


of inertia
lamina of mass M bounded by the curve y* = ^ax, the ;r“axis and
an ordinate at ;r = a is ^Ma*.
19. A uniform solid is formed by rotating the curve x*
+ y* = from
X = ~ a to X = a about the ^-axis. Show tliat the square
of the
radius of gyration about the axis of symmetry
of the solid is 2a* /6.
20. The surface-density of a circular lamina of radius a
varies as the
distoce from the centre. If the total mass of
find Its moment of inertia about
the lamina is
an axis through its centre and
M
perpendicular to its plane.
21. Find the length of the arc of the curve *• =
+ b‘ between points
where ^ « a cos a and « a cos
22. The slope ofa curve at a point whose abscissa is x is Z^J(x -h
Fmd the length of the arc of the curve from
x 1 to « a lO.
234 PURE MATHEMATICS [12

23. Find the length of the arc of the cur\'e = a2/3 for points
between x a and x = b.
(Hint, put ^ to evaluate the integral.)

24. Find the area of the surface formed by the revolution of the curve
= ^(3 — x)^ from = 0 to ;r = 3 about the ;r-axis.
25. Find the area of the surface generated by the zevolution about the
;r-axis of that part of the curve
y = x^ which lies between x ^ C
and X i.
CHAPTER 13

THE LOGARITHMIC AND EXPONENTIAL FUNCTIONS


13.1. Introduction
In reading Chapters 10 and II the student will probably wonder
why the result

I x*^dx +C
is invalid when « =— 1 and will have noticed that a discussion of

x~^ dx was postponed. We commence the present chapter by con-


J
sidering this integral and then show how it leads to two functions of
great importance in mathematics (and in some of its applications to
physics and mechanics). Once this integral has been established, many
more functions can be integrated and the gaps in § 11.2 can be filled in.

13.2. The area below the curve y s= 1/jc


Fig. 85 shows the graph ol
y = \/x for positive values of x. Sup-
pose we wish to find the area enclosed by the curve, tlie ;e-axis and

ordinates at ^ —1 and x = t. It can be inferred from the diagiam


that the area, or the definite integral
dx, exists. The area or
|
mtegral could m
fact be found approximately by
counting squares or
by using one of the approximate methods
of integration given in
236
236 —
PURE MATHEMATICS [13

§ 11.7, and the student who has worked Exercise 11 [e), 2 will have
carried out this procedure for the case of / = 2.
Wecan write
dx
— = area ANMB, (13-1)
Ii ^
and should be clear that as t increases the ordinate
it moves to BM
the right and the area and integral are increasing functions of t. The
curve y =
\/x is (see § 17.17) a hyperbola and a convenient notation
for the area bounded by the curve, the A;-axis and ordinates at =1
and X =
t is hyp (/). With this notation, equation (13.1) can be
rewritten
^ dx
- = area ANMB = hyp [t). (13.2)
1 ^
It should be noticed at this stage that when t =\, the ordinates AN,
BM coincide and the area ANMB
vanishes so that
hyp (1) — 0. (13.3)

Agraph of hyp (i) calculated by giving t various values and per-


forming the integration by one of the approximate methods would
appear, for values of / greater than unity, as shown in Fig. 86.

-t

Fig. 86 values of
So we have only discussed and shown in
far
when betw^n zero
hyp (/) when t is greater tlian unity. Values
i lies

and unity can be found as follows. If 0 < < / 1, then (l/i) > 1 an

hyp
(0 = rT * to z where ^ - 1/z
Now change the variable in this integral from
by the method of 11.5. The limits of
values of z of 1 and
§
t, and, since [dx/dz) = “‘X^‘’whe°inS
- (lA the mteg can).
.

be written
f' y_ 1 ^ _ f'
*
13] LOGARITHMIC AND EXPONENTIAL FUNCTIONS 237

Tliis is — hyp {t) and we have the important result that

hyp = - hyp (t). (13.4)

This formula enables the graph of hyp (0 to be completed for values


of t between zero and unity. For example,
formula (13.4) if / = 2,
gives hyp (1/2) hj^) (2)=— =—
0*69 approximately, if we read off
hyp (2) from Fig. 86. Other values of hyp (/) for the range 0 ^ 1 < <
can be found in the same way and a completed graph of the function
is shown in Fig. 87.

Fig. 87

13.3. Some important properties of the function hyp (t)

From the definition hyp (0 =j dx it should be clear that

|{hyp(0} = l,
and this result
can also be inferred from the diagram shown in Fig 88
This shows the curve
y =
1/x and AN, BM, CP are ordinates at
X —
I, X == t and x = t dU BD, CE are drawn parallel to the
ar-axis to meet CP and BM at D
and E respectively. The figure shows
that the area BMPC lies between the areas of the rectangles EMPC
and BMPD.
Since BM = 1//, CP = l/(t -f dt) and MP = dt, this
can be expressed as

< <r
tfst (13.6)

Since = area ANMB and hyp + dt) = area ANPC, we have,


(<)
{/
by subtraction, *

hyp {t 4- dt) — hyp (i) = area ANPC — area ANMB


• — area BMPC, 13 6)
.
(
238 PURE MATHEMATICS [13

Combining (13.5), (13.6) and dividing by dt, we have


1 ^ hyp {i -f dt) — hyp (0^1
i-\- 6t^ dt t*

showing that

lim. f hyp {I + 6t) - hyp (0 1


dt ^
\ /

or. |{hypW} = J. (13.7)

Using (13.7) and the formula for the differentiation of a function


of a function, if c is a constant,

|{hyp(c/)}=Ixc = J.

Hence
J^{
hyp (cf) - hyp (<) } =y -^ =
showing that hyp (cl) — hyp (f) = constant.
The value of the constant can be found by putting < 1, = and smce
the =
constant is hyp (c). Hence
from (13.3) hyp (1) 0,

hyp (ct) - hyp (t) = hyp (c).

Writing c = a, t = bm we have (13.8)

hyp (ab) — hyp (a) + hyp (b).


f • >

putting c = a/b. t = b, we can


show that
Similarly

hyp = hyp (a) - hyp (b).


(13.10)

(^)
13] THE LOGARITHMIC FUNCTION 239

One other property of the function is required. Writing t ^


in (13.2),

hyp (a") =
Jl ^
Changing the variable in the integral by the substitution x = the
limits of integration become 1 and a, and since {dx/dit) =
ra I
hyp (rt")
= j

du

— »hyp(a). (13.11

13.4. The logarithmic function


The properties
hyp (ah) = hyp (a) + hyp (fi).

= hyp (a) - hyp (6),

hyp (fl”) = n hyp (a),

and hyp (1) = 0,


of the function hyp (i) suggest that there is a connection with the
logarithmic function. That hypnot exactly the same as logj^ i
(i) is

^ be seen from the graph of Fig. 87 and a table of common logar-


ithms. Thus the graph gives hyp
(2) and hyp (3) as approximately
0'69 and Mwhile a table of logarithms shows that logiQ2 0*301 =
and logio 3 =
0 477.
The details of the connection between the " hyp ” and logarithmic
functions can be established as follows. From the graph
of Fig. 87
it may be inferred that there is a
value of t (which we shall denote
by e) which makes the value of hyp (1) unity the graph
shows that ;

the value of c is about 2-7. Hence

hyp (e) = 1,
and writing a=^e, n^x in (13.11),

hyp (if*) — a; hyp (e) =x


Putting =
y, this gives hyp (y) = x and a combination of these two
equations shows that
y==tf»*FP(y).
Thus hyp (y) is the power to which the number
e must be raised to
make it equal to y, and hence, by the definition
of a logarithm, the
hyp function is the logarithm to base c, or
hyp (y) = log,y.
:

240 PURE MATHEMATICS [13

The logarithm to base e is called a natural or Napierian logarithm


and an alternative notation for loge^ is \ny, the “ n " signifying the
word “ natural ",
A summary of the contents of the first four sections of this chapter
is contained in the two important results

^{Xogex} =-, (13.13)


dx X
and its inverse,

^= log* X -\- C, (13.14)


J
written
The variable in these results has for convenience been here
as ^ and it should be noted that the function log* % has
been defined
between the “ hyp " and
for positive values of ^ only. The identity
also a graph
logarithmic functions means that the graph of Fig. 87 gives
logarithmic function. The relation (13.13) shows that the slope
of the
curve very gentle when the independent variable is very large
of the is

and that it is very steep when the variable is very


small.

Example 1 . Show that common and natural logarithms are connected by the
relation log^Q x = log^ x X e.

immediately from Example 3 of Chapter 2 (page 26) by


This follows
writing ^ x, a =* 10, 6 = «.
N
x with respect to x and hence find f tan x dx.
Example 2. Differentiate log, sec

li y = log* see ;r, we can write

= = log. 1 - loE. OOS * = - log, COS *,


y log,
(-^)
\COS X
the result =
(13.13) and the formula
since log, 1 0, =
Writing » cos x,
of a function gives
for dificrentiating a function
dy^ dy du
dx d^^Tx
|^{-log,«}X~(cosx)
1 sin X
_ - X — sin X — ,
tan x.

(log, sec X) = tan the inverse relation is


Since
dx

tan X dx — log* sec X +

EXERCISES 13 {a)

to ^
1 Differentiate with respect
(i) ;^*log*^.
(ii) log, (lA).

13] THE EXPONENTIAL PXJNCTION 241

Differentiate ^(log^ x — 1) with respect to x and hence find log, X dx.

Taking e => 2*718, use the result of Example 1 of this Chapter, viz.

^ X logio e.
logio ^
to draw the graph of ^ = log, {x + 3) between x =• — 2*5 and x = Z.
With the same axes and scales draw the graph of 5y = x^ and use
these graphs to solve approximately the equation

(;p + 3)® = (L.U.)


Differentiate with respect to x :

(0 log. (Q.E.), (ii) log, (cosec + cot . (L.U.)

Find dy/dx when y


%
= log, {x + -^{x^ + 1)}, and hence evaluate
dx
(L.U.)
V(1 +
A submarine telegraph cable consists of a copper core with a con-
centric sheath of nonconducting material. The ratio of the radius
of the core to the thickness of the sheath is x and it is known that
the speed of signalling is equal to kx* log, (l/x), where A is a constant.
Show that the greatest speed of signalling is reached in a cable
for which x = 1/V««

13.5. The exponential function


x = log.t, (13,15)
X given uniquely for positive values of /. It may be inferred from
is

the graph of Fig. 87 that for any assigned value of


x, there is one and
only one value of / and that value is positive. may regard i as We
a fimction of x and this function is single-valued and
every\vhere
positive. We could write
but t as antilog, (x) it is more usual to write
the inverse relation to
(13.15) in the form

or, in a notation which particularly useful when x


is is replaced by
a complicated expression,

11 .3 .,.
f = exp(x). (13.17)
t IS
called the exponential function
of x and this function is of great
imj^rtance in mathematics and its application to
physical science.
^The graph of the function
can be obtained from that of
t —^
“ ^ interchanging the axes of Fig. 87. This is shown in
of
.
function t =
e~* has many physical applications and

f
is easily obtained from that of
ot thQ graph of c“* for
e* for c"* 1/e*. A sketch =
positive values of x is shown in Fig. 90.
Ihe exponential function can be
expressed in the form of a limit
as follows. Since

0
242 PURE MATHEMATICS [13

it follows that as h tends to zero

-{loge {t + h) — \0Qet}
tends to \/t. Putting t = 1, since logc 1 = 0, we have that

I
h
loge (1 + h)
tends to unity as h tends to zero. Writing h ^ x/n, this can be
expressed by
'i log, 1 as « -> 00,

showing that loge 1 -j- jK as n —>• CO.

Fig. 90
13J THE EXPONENTIAL FUNCTION 243

This can be written in the alternative form


n
« as 00
(- +0
or. ^ = lim. (i + A" (13.18)
nj

13.6. Differentiation and integration of e®


Uy= then x = log^y and (13.13) gives

dx _ 1 _ 1

V e®*

Since from (8.11),

we have —
dx
giving the important result that

(13.19)

This shows that the slope of the curve


3 =
e® at a point whose abscissa
^

is a; is equal to the ordinate at this point.

The formula for the differential coefficient of a function of a function


then gives, if a is a constant

-
(O = (13.20)
and the inverse relation is

e^dx^ -e^ -f C. (13.21)


J a
Suppose now that a is a positive constant and that = a*. Then
= iog, (a®) = X loge a, y
so that

y =
It follows from (13.20) that

|(a») =
= loga a(«**0B.a)
= >'Iog.a
= aMogea. (13,22)

Example 3, y = show that + 2— + ^v *= 0 CL.U.)


dx* dx~
By the formula for the differeatial coefl&cient
of a product

%= + x±(.-^) <= .-X _


— (1 — #)•-* — (13.23)
— . •

244 PURE MATHEMATICS [13

Henco
dx^ dx

giving
g *‘£— - ( 13 24 )
.

The required result follows immediately from the addition of (13.23),


(13.24).

Example 4. Show that = x and that _ I/^r*.

If y = tflog**, then logey = log, .r.loge e = log^ x, since log« e =L


Hence y = x.
If 2 = e~ 2 \og,x^ then
loge ^ = — 2 log« ;r.loge « = — 2 lo^'^ ^ = log^ (!/.»*)
so that z = 1 /x^.

EXERCISES 13 {b}

1. Differentiate with respect to x :

(i) sin 3;r. (ii)

If V e~^ cos 4;r, prove that


^
+ 4— + 20y = 0.
dx^ dx
dy vy
If y = x^e^^, show that ay = —X (L.U.)
dx
11 y — sin x, show that


dx
= -v/2.c*sin [x + 7i/A), ^
dx^
= 2c* sin {x + 7i/2). (O.c.)

Evaluate the following definite integrals : —


c"3x dx , (ii) j' (e* - «-*) * dx, (iii)
iii) (AT - I))c* dx. (L.U.)

"i: j
=
Sliow that the length of arc of the curve 2y = c* + c"*
from x Q

to X = a is ^(c® — c"**).
parts
Find the following by the method of integration by

i) f ;t2c* dx, (ii) c* cos X dx.


I
ordinate of the curve = xe * and draw a
Find the maximum y
rough sketch of the curve.

integrals depending on J.v d:i


^
13.7. Some
Now that the integral

f dx = log (13.25)

integration whidi .^re


some methods
of
is available,
can be given. first note tna We f
11
the discussion of Chapter
13] INTEGRALS DEPENDING ON J
x-i dx 245
important extension of the integral occurs when x is replaced by the
linear expression {ax +
b) where a and b are constants. By (11.2),
this is
1 , , ^
^=-los. (ax + b) + C, (13.26)
1
and for the particular case of a = 1, this reduces to

dx
— ioge “I* C*. (13.27)
\X b

Now consider the integral

.dx.
f /w
in wliichthe numerator of the integrand is the derivative with respect
to X of the denominator. To evaluate this integral write J{x) so =
thai f'(x).(dx/dt)^ I and (dx/dt) l/f{x). The rule = (11.7) for
integration by change of variable then gives

J /W J < S'(x)
dt

-I?
— +C
log* t

= log«/(^) + C. (13.28)
Hence the integral of a fraction in
which the numerator is the differential
coefficient of the denominator is simply log^
{denominator).
The important rule of (13.28) enables the integrals
of certain
trigonometrical functions to be written down. Thus

{t^nxdx={^j^c
J J cos x
sin X
dx
=-J- cos X

cos X
dx = — log. cos x + C. (13.29)

Similarly __ r cos X
cot X dx dx
I J sin X
C d
(sin x)dx
dx
sm X
log* sin x-j-C, (13.30)
246 PURE MATHEMATICS [13

Also,
dx
cosec X dx
I sm X
dx
2 sin \x cos {x
dx
2 tan lx cos^

_ r i sec^ lx
dx
J tan lx

dx
tan lx

= loge tan lx + C. (13.31)


The integral of sec x can then be found by noting that
cos jt = sin (In + x),
so that secxdx ~ ^ cosec (In -f x)dx
^
= log, tan (\7i + lx) + C, (13.32)

by using (13.31) and (11.2).


Some other instances of the important result (13.28) will be found
in the following examples.

f^ jr* dx
Example 5. Evaluate
ate I -r r. (L.U.)

Since + I) Zx*, the integral can be wTitten

=
5[>og«(^’ + i)]o

‘°e. 2-
I
since log, 1 s= 0.

Example 6. Find
ind i + 5)dx2’
(4;r

j -\-2x +
The derivative with respect to x of the denominator is 2jr 2, and if +
1. the integral can
+ be expressed
+
we write the numerator as 2(2.*- 2)

as the sum
f 2'
-i- 2j» + 2
j
X* + 2x +
2

13] INTEGRALS DEPENDING ON 247

The first integral is 2 logg {x^ + x'+ 2) and the second integral can be
written as
dx
ix-h 1)*+ 1

or by § 11.2, tan“* {x + 1). Hence finally

r {‘^ + 5)dx
I :,t^2x+ = ^ + 2.r + 2) + tan-> (^r + 1) + C.

The integration of rational algebraical fractions (i.e., fractions in


which the numerator and denominator contain only positive integral
powers of the variable and constcint coefficients) can often be made
to depend on the integral

dx
- log* {ax + 6) H- C.
I ax + b a
We consider below a few of the simpler cases. In all cases, if the
degree of the numerator is equal to or greater than that of the
denominator, the numerator must first be divided by the denominator
until the remainder is of lower degree than the denominator,

{a) Let the denominator be of the first degree. In this case the
remainder after the division will be independent of the variable and
the integral will be given as a sum of terms involving powers of the
variable and a logarithmic term.

Example 7

By division of x^ by (x - 1) we find
X* 1
T
^ ^ + 1 +
the quotient being x* + x -j- 1 and the remainder unity.
Hence

1.^=1 (** +^+ l]dx 4-


J
ix^ + 4 4r + log* (*-!) + C,

Example 3 ” Zx
8. Integrate with respect
^^ to x.

Here the division process gives


3-2* 2
2x
-1 + 2*-
r
so that

- -*4 2.ilog,{2*- 1) 4C
- —X+ log* (2* — 1) + C.
248 PURE MATHEMATICS [13

(b) Let the denominator be of the second degree and let it break up
into a pair of linear factors. In this case, we can split the integrand
by resolving it into partial fractions and each partial fraction can be
integrated by (13.33).

Thus to End
P
I
~ (ix we first write the integrand as
J ^ X

a —X a X*

where A, B
are two constants to be found. The usual method (§ 2.8)
for resolution into partial fractions then gives as the identity from
which A and B
arc to be determined
A{a x) -h B{a — ^) = 1,
By letting x — a m turn we find

A=B = 2a
1

so that
r dx _ J_ f dx ,
i_ r dx
J a-
— x^ 2a J a —X 2a J a
1
= — loge {a — x) A- 2"
1
{a X) +C
^
a +
'

= log, “h C*.
a-x)
f dx
Example 9. Find
^
j

Here the numerator of the integrand is of higher degree than the


denominator, and division until a remainder of lower degree is obtained
shows that
21x - 20
^x-l-h X - 20
xi ^ X -20
The last term on the right is resolved into partial fractions by writing
- 20 ^ A__ ^
21;r B
x2^x — 20~x+5 X - 4:’

so that the identity for the determination


of A and B is

Alx - 4) + B(x + 6) =21x


- 20.
By letting .r = - 5 and 4 respectively we find that
125
A =
9
Hence
125 f dx C4 f
l)dx + : +9 i"-
;r* + ;r - 20
'
"^J X + 5 J
J
J
~ log, {X + 5} + j log, (* - 4) + c.
13] INTEGRALS DEPENDING ON ^x-^dx 249

(c) When the denominator is of higher degree than the second, the
method of resolution into partial fractions is still often useful. A few
instances will be found among the example and exercises which follow
but a complete discussion is outside the scope of this book.
»

Example 10. Find


ind
J + 1)

Let, according to the usual rules for resolution into partial fractions,

1 _ ^ ^ Cx +D
{x - l)^{x^~-\- 1) “ {x - I)» ~x^~+ I
'

then A{x - l)(Ar2 + 1) + B{x^ + 1) + {Cx + D){x - 1)» = 1.

By letting = I, we find B s=« 1/2. Equating the coefficients of x*


and the terms independent of x gives respectively
— A+B-2C-^D = 0 and ~A+B-\-D^l.
Subtraction of these Uvo relations then gives 2C « or C = 1 1/2.
A comparison of the coefficients of x^, shows that A C = so that
0,
a= —
C s= 1/2.— Finally, putting A 1/2, =— B= 1/2 in the
relation — i4+B +
£)=sl shows that 0. D=
Hence
f dx I C dx 1 ( dx xdx
\ [
J
(;r - + 1) 2 J T - 1 2 J {x - 1)« + 2 J x^ + 1

-2log« (*-!)- — ij + I
tog, {x^ + 1) + C,

the last integral being evaluated by uTiting

\ ( xdx if (2;r) dx
2J + 1
~ 4J 1

1
dx.
1

The method of integration by


often useful in cases where parts is
the integrand contains the function log, x. If this function
is taken
as the in formula (11.10), i.e.

then du/dx in the integral on the right is simply /x and


1 this integral
can then often be found easily. As a simple example,
the integral
log, X dx can be found by taking
J*

rTv
1
log, X,
5
- = I,
dx
so that du
dx
250 PURE MATHEMATICS [13

and we have logs xdx ^ X loge X ~ x.-.dx


J ^
= log« X dx
-I
= XlogeX — X C,
Another example which depends on the work of this section is

given below.

Example 11. Find


iiid (an-^ X dx.

By \sTiting u = =
so that du/dx
dv/dx 1, 1/(1 = + x~) and
V = X, in the formula for integration by parts, we have

tan X dx — x tan - ^ x dx.


I -Ir +
d
dx
{-*•» + 1;
1
X tan*' ^ dx
^ x^ + 1

X tan*' ^ — i log< {x* + 1) + C.

EXERCISES 13 {c)

Evaluate the following indefinite and definite integrals :

X dx — e
1. 6. dx.
I
r sec*
6 dx.
2 .

J
3 +
r dt
3.
t \og~t
J
(8f - Vfdl
8
4.
j 2/* + 2/ + 1

Integrate the following with respect to x .

C;r
13.
9.
2;r - 3
+ l){jr - 2)
AT* + 2
2;r + 3
14.
10
X + 2'
1^- 1*
4;r + 3
16.
11 . {X - 3)*'
4 - 2x'
7;r + S
6 16.
12 .
- + 4)
;r> + X o’

13] LOGARITHMIC DIFFERENTIATION 251

Evaluate the following dehnite integrals :

dx . f2 dt
17. j' 18.
-

{x-\-l){x + 2)
£ |,/='(1 +0
Use the method of integration by parts to find the following integrals

19. X* log* X dx. 22. ^ log^ [x + ^)dx.


1 I
logg ^ dx
20 . X* tan~' X dx. 23.
I {x + 1)*'

21 . xsec*xdx. (Q.E.) 24. sec* X logj tan x dx.


I

13.8. Logarithmic differentiation


When
a function consists of a number of factors it is often con-
venient to take the logarithm before differentiating. Thus, suppose
that
• •
y=--

where u^, u^, Wj, , . Ug, Vg, , . . are functions of x. We have


log.^' = log* H- log* «a -f- log* ^3 .

,
— log# Vi - log* Vg - log* V - 3 . . .
so that

' ' *
y dx dx u^dx"^ u^dx^
1 dv-i
^ 1 (fvg 1 dv^
Vi dx Vj dx v^dx
A similar method can be applied to find the differential coefficient
of the function

where « and i; are functions of Taking the logarithm we have


;f.

log* 3^=sulog*M,

so that
=
ydx dx ^ + udx
Example 12. Find the differential coefficients with respect to x 0/

From log*y «
W A - x),
(i),
J log* (1 + | log* (1

so that j<(y^ I I 1

y d4r
“ 2(1 4 X) 2fl - jr)
" 1 - t*’
.

252 PURE MATHEMATICS [13

Hence y
1 - X*

(1 - ;r)3/2(l 4-
From (ii), log^y = xlog^x. so that
1 dy
-5-=l + log,*,

using the usual formula for the differentiation of the product x logg x,

Hence
^ = >-(1 + logg x) = xi{]. + logg x).

13.9. Successive approximations and Maclaurin’s series


Consider first the function 1/(1 —
x) for values of x less than unity.
It is easy to verify by ordinary algebraical processes that


I
1

^X = 1 + ^ +
—X :=

I

1

and so on. Hence


1, 1 -f- 1 X x^, • • .

are successive approximations to the function 1/(1 — x) for the

respective errors are


X x^ x^
r^' r^’ *'
' *

progressively
and, for values of ^ less than unity, these errors become
approximations
smaUer. It should be noted that the successive
1, 1 -{- X, 1 X x^, . . •

all have the


arc all equal for x = Q. onwards,
From they (1 + x)
derivative at 0 from % = x^) onwards, tliey aU + +
same first (1 ;

same second derivative for this value of and so on.


have the
following method of approximatmg to a function
This suggests the
/W- Let +
f(x) =a^x + Sa*’ +
• • • «»*• + +
Choose
= 0, the function J(x) and its first « derivatives (assumed to exist)
*
are the same as the values at * = 0 of the polynomial and its
first
. .

13] MACLAURIN’S SERIES 253

n derivatives. By this procedure, the polynomial may well be a suc-


cessively better approximation to the function as the number (« + 1)
of terms in the polynomial increases.
The first n derivatives of the polynomial are
ai 4- 2a2X +
2«2 + 6^3^ 4- . • • 4- w(w —
6^3 4- ... 4- n(n — l)(n — 2)a„x^~^,
and so on. At x — 0, the values of the polynomial and its first n
derivatives are

Equating these to the values at = 0 of /(x) and its first n derivatives,


we find
«0 =/( 0 ), «3 =|/"'(0 ),
«i=/'{ 0),

Thus we may expect to be able to write

/W N/(0) + x/'(0) + p"{0) + p"'{0) + . . . + ~/<”>(0). (13.34)

The above procedure is only satisfactory if (i) f{x) and its first
n derivatives all exist and are continuous at a; = 0, and (ii) the differ-
ence between f(x) and the polynomial on the right of (13.34) decreases
as n increases. Thus the method would fail entirely for the case
f(x) = \/x, for none of f(x) nor its derivatives exist at % =: 0, The
method would fail also for the case f(x) — 1/(1 — x) when x > I since,
for such values of x, the errors in the polynomial representations
would
increase as the number of terms in them increased.
In the example /(x) = 1/(1 — x) it was easy to calculate the errors
of the approximations at each stage. not so easy to do this for It is
the general function f{x). It is, in fact, beyond the scope of the present
book to attempt to do more than point out that there are many
func-
tions f{x) for which f{x) and all its derivatives exist and are continuous
at = 0 and for which the series

m+ xno) 4- J/"{o) 4- . .
+ ^/<">(o) 4- . .

converges. Under such conditions f(x) is the Umit of the sum of this
series and we can write

/W =/{0) + xr(0) 4- |j/»(0 ) -b . . , 4- (13.35)

the series on the right of


(13.35) being known as Maclaurin*s scries
for f(x)\
t
+ —
264 •
PURE MATHEMATICS [13

13.10. Series for c® and loge(l + x)


As examples of the expansion of functions in their Maclaurin’s
series we consider here the series for and log* (1 4- x).

Firstly, if f{x) = e^, since ~{e^) =


f(x) =f'{x) =f"{x) = . . .
=/(")W = . . . =
and /(O) =/'(0) =/"(0) = . . . = /(")(0) = . . . = «» = 1.

Hence (13.35) gives

^=^+*+li+S+---+5+---
It is clear that ^
and all its derivatives exist and are continuous at
X = 0 and it can be shown (but we shall not attempt to do so here)
that the difference between and the first (n 1 ) terms of the series
of (13.36) tends to zero as n —
oo for all values of x. The series of
(13.36) is therefore a valid representation of the function ^ for all x.
The above series for ^ is known as the expofiential series and has
many useful applications. Here we shall consider only its use in
evaluating c* numerically as a function of x. For example, if = 1,

we have

1 + '
+ + + + - • •
+^ s ^ ^
= 10000 4- 1-0000 4- 0-5000 + 0-1667 4- 0-0417 4" 0 0083
4- 0-0014 4- 0 0002 4- • • •

= 2-718 . . .

To obtain similar accuracy, for values of x less than unity less terms
of the series would be required, but for larger values of x it would be
necessary to retain more terms.
Now take f{x) log* (1 4- x) so that /(O) log* 1=0. Then
= =
/'W = r(x) = - 1
r(x) =
l+x (14-^)^' (1 +
- 3!
Mx) = (1 xy mix) ==
(_
^ zl—
i)"-i(«
i
— 1 !
L,

+
and
/'(O) = 1, /"(O) =- 1. /'"(O) = 2, /‘-(O) = - 3!, . . .

Maclaurin’s series for log. (1 + x) is therefore

108,(1 + .)-.
at
derivatives exist and are continuous
Again log, (1 + x) and all its
13] LOGARITHMIC AND EXPONENTIAL SERIES 255

x = 0 but can be shown that the above expansion is valid in the


it

sense that the difference between logs (1 x) and the first n terms of +
its series tends to zero as n —>• oo only when 1 < x <1. —
The reader may wonder why we have given a series for log« (1 x) +
rather than for loge x. The reason is that log« x and its derivatives
do not exist at =
0 and log* x cannot therefore be represented by a
series in ascending powers of x.
The logarithmic series (13.37) is only useful for calculating natural
logarithms for small values ot x in fact, the series is not valid when
^ > I and even for values of x approaching unity many terms would
have to be retained to obtain reasonable accuracy. Some algebraical
manipulation of the series can, however, permit the logarithms of
larger numbers to be calculated and a typical example is given
below.

Example 13, If the absolute magnitude of x is greater than unity, show that
1 1 \
*
*/*
3;r’
and use this series to calculate log^ 2 to three places of decimals.
We can write

If X greater than unity,


is x can be replaced by l/x and — l/;r in tho
scries (13.37) so that

T>
Putting x^Z, we
^
have

(2 ) “ ^5 + 3i!t» + slTsi + • •
•)
giving log, 2 = 2(0-3333
+ 0-0124 + 0-0008 + .,.)== 0-693 . • .

EXERCISES 13 ((f)

Use the method of logarithmic differentiation to find dy Jdx when ;

L y {I - 2x){l - Qx). 3. y«10*.


V{1 - Sx)
2. y *
(1 - x)^ 4. y (log, x)‘

Assuming the expansions given below


are convergent, show that :
% m .

x^ x*
6. sin x
^
<
' ‘ •
31 61

cos X « • «
21 4!
— —
25G PURE MATHEMATICS [13
7. tan (4r + 0) = ta.n 0 + x sec* 0 -{• x^ sec* 0 tan 0 -f- . . .

8 .
4“ • • •

9. If a so small that its cube and higher powers may be neglected


is

find the values of the constants A, B and C so that


(,x+a. = Ae^ + + Ce*+1»
10. Show that log* (2 3x) 2 + + = log* - fjt* ^x^ - . and .

state the limits between which x must lie for the expansion to be
valid.
11 . By writing 1+ x + x^ = ~ x^) /(I — x) show that, — 1 <
{1 if ;ir < 1,

log* {\ \- X + x^) = X Ix"^ — Ix^ + ^x^ {x^ — \x^ + .. .

12 . If X, p are the roots of the quadratic equation x* — px + q = 0,


show that for suitable values of
log, {l+px qx^) = (A + P)x - i{A* + /5*);r* + + P^)x^ - • •

EXERCISES 13 {e)

1 . Find dy/dx when :

(i) y = log* tan X, (ii) y = log* (see x + tan x). (L.U.)

2. Find the maximum value of {loggX)/x.


3. Differentiate the following witli respect to x :

(i) log* 4-Y iVi^ + 1) + V(^ - !)}•


+ ;r
I

4. If y = cos (log* x), prove that


d*y dy .
0 .

5 . If y = - l)/{e^ + 1). show that

*
- (^)
6 ,
Evaluate the following :

•«/4

(0 i) \ie^ -\-
*
^ 2tz.nx)dx,
^ J

7. If y = exp (tan-‘;r), show that


d*y dy
(1 + + (2- - = <’
dx
8 . Show that if a and are suitably chosen, y = cos fix satisfies

the equation
^ +A^ + By =
rf.r* dx
0.

and find a. /? in terms of


^
A and B. i. f ^
to the *-axis of the tangent
inclination
9. If V is the angle of
curve = i(e* + e-% show that
point of the y
* = i°g* V + ««= V)-
(i)
y = sec v>,
(ii)
— — — 2

13] EXERCISES 257


10. The arc of the curve y 2r/2
from to;r=s2= ;r=—
rotates about the A'-axis. Find the area of the surface so generated,
1 1. Integrate the following with respect to x :

... e* + e-^ u)
x^-'^ +
(1)
— e~ x^ e-

12. Show that

f—-
J
X dx
^ + 1
i log, (2^* - ;»r
+ 1) +^tan- '4x -
+c
13. Integrate the foUowing with respect to x
+ 1 - 3.r + 1

X— +2'
(») r-:;. (u)
ar + 1

1 4. Evaluate the following

(i)
{2t + l)dt
O + 2 /* -It'
15. Integrate the following with respect to x :

- 3;r*
1 1
(i) (ii)
Zx - X*' {^* - !)»
10. Find the following;
(10;ir* + 13;r + 9)dx
J
^* + 7^ + 10 '
J
- 2){2x + 1)*

17. Evaluate ;

Jo(l +<«)(3 +<«)•


18. Find the area enclosed by the curve
(1 + 3;r + 2;r*)y = 1, the x-axia
and ordmates at ;r = 0 and x ^ 5 .

- «) « 0, where a is a positive
nstant, ^d show that y is a maximum or a minimum when
Prove that the volume enclosed by
u
about the
r
axis of
rotating
the part of the curve which Ues
between x = 0
and X = o is ji:a«(f S - 2 log, 2). ,0 rr \
20. Show ^
that
r«/4 _ f
(i) 0sec*0(fa=7-|log,2.

(ii) {^ - 1)* log, ^ = I log, 2 -


J
21. Find dy/dx when
^(1 -hx*)
(i)
y ° = +
V(r-x*)’ (Q.E.)
22. Show that

^(sec ;r tan" x) « sec xin tan"-l + (« + i) tan"+l x}.


258 PURE MATHEMATICS [13

Use this result to show that ii


y = see x.
d^y d^y
= see x{l + 2 tan* x). = see ;r(5 tan ;r -f G tan* x).

d^y
- = see x{5 + 28 tan* x 24 tan^ x).
dx
Hence show that the first three terms of Maclaurin’s series for sec x
are given by

sec* = l+-
_
2
+ —
5x*

23. Find the coefficient of at* in the expansion of «*/(! + 2x^) in ascending
powers of x. (Q E.)
24. In the equation x'^+X -= e^, A is a small quantity whose third and
higher powers may be neglected prove that ;

X
(i) X = e.e 3+^,
(ii) X =e{l -iX 4- iA*).

25. Find the coefficient of x’' in the expansion of

in ascending powers of can be assumed that x


; it lies within
the range for which the expansion is convergent.
CHAPTER 14

COORDINATES. LENGTHS OF LINES. AREAS OF


TRIANGLES. LOCI. INTERSECTION OF CURVES
14.1. Systems of coordinates
In elementary graphical work, it is customary to specify
the position
of a point P in a plane by its perpendicular
two fixed perpendicular
distances OM, MP from
lines Oy, Ox (Fig. 91). The lines Ox, Oy are

Fig. 91

called the axes, 0 the origin and the distances OM, MP are referred
to as the abscissa and ordinate
of the point P. A
convenient method
ot relemng shortly to the
position of a point whose abscissa is ^
and
ordinate y is to say that the point has coordinates x and y and to use
the s^bol (x, y) to designate the position of such a point.
The student assumed to be familiar with the usual sign conven-
is
lons used m graphical work.
Briefly these are as foUows-for points
to the right of the axis Oy the
abscissa is positive and for points to the
260 f'URE MATHEMATICS [14

left of Oy the abscissa is negative, while the ordinate is positive or


negative according as the point is above or below the axis Ox. Thus,
in Fig. 92, the points P^, P^, P^, P^ would be denoted by (5, 2), (— 2, 3),
(—3, — 1) and (3, — 2) respectively.
The above method of specifying the position of a point in a plane
by its distances from two straight lines (the axes) was first introduced
by the philosopher Des Cartes. It is not essential for the axes to be
mutually perpendicular as in Figs. 91, 92, but it is often convenient
to use, and in this book we shall only deal with, axes at right angles.
Such a system of coordinates is known as the Cartesian system and,
when the axes are at right angles to one another as in Fig. 91, x and y
are referred to as the rectangular Cartesian coordinates of the point P .

Other coordinate systems, for example that outlined below, exist but
the Cartesian system is by far the most important.
Another method of specifying the position of a point in a plane
is by its polar coordinates. Suppose that 0 is a fixed point, called
the origin or pole, and Ox is a fixed line, called the initial line. Then

Fig. 03

the position of a point P is known when the angle xOP (- 0)


in FiE. 93,
The two quantities r ^nd 0 are
and the length OP (= r) are given.
polar coordinates of P and the position of P is conveniently
called the
this system of coordmates by
the symbol [r, 0).
denoted in

between Cartesian and polar coordinates


14.2. The relation
rectangular Cartesian coordinates of a
Sunnose that the
Then, from big. 94.
^
.reS^rnd tiiat its polar cLdinates are (r, 0).

Fig. 04
llj COORDINATES 261
X = OM = r cos 0,
y = MP = r sin 0,
(14.1)

and X, y can be found when r, B are given, Conversely, if x^


y are
given, we have
= y^
tan 0 = y/x. (14.2)

Equations (14.2) do not determine r, 0 uniquely, for


r -^ = + y^)
and B can take an indefinite number of different
values. To obtain
an unique correspondence, we take r = + V(^- + y^) and determine
B as the angle which lies between — and n satisfying the two
;r
equations cos B x/r, sin 0 y/r. = =
Example 1. Find ihe
Cartesian coordinates of the point whose
(i)
polar
coordxnates are (6, n/ 4) and (ii) ihe
polar coordinates of the point whose
Cartesian coordinates are
(— 1, 1).
(i) y = 6, 0 =s ir/4, so that
^ = 5 cos {n/4) = 5/V2 = 3-630,
y = 6 sin (,r/4) =s 5/^/2 = 3-530.
(ii) X ^^ I, y = SO that
r:= V{1*+ 1*) =* ^2= 1-414.
cos fl =— 1 / y2, sin 6 = 1/V2 giving 0 * Sn/i.

14.3. The distance between two points with


given rectangular
^
coordinates

let their coordinates be


f*
>'J' (*! yi)- Draw
(Fig. 95)
perpendiculai
.
to Ox. and draw P,R paraUel to 0;r to meet P,M, Ther
P^R = = Oil/. - OM, =
and Rl\ = M.P, - m,R = M.P. _ M,P. =
From the right-angled triangle
P.PP,,
= V{PtR‘ -f- PP,*)

Fig. 06
262 PURE MATHEMATICS [14

The proof
of formula (14.3) has been given only for the case in
which the coordinates of both the points are all positive. When due
regard paid to the usual sign conventions for the coordinates, it
is

will be found to remain true for any positions of the two given points.

Example 2. Find the distance between the points (4, — 7) and (— 1, 6).

By \\Titing = 4, yj = — 7, = —
5 in formula (14.3). or
at, 1, y, =
(preferably) working from first principles from a diagram showing the
given points, it will be found that the required distance
= v/{(4 + 1)* + (_ 7 - 5)n = V(25 4- 144) = -/(IBO) = 13.

14.4. A proof of the addition formulae of trigonometry


A compact method of obtaining a formula for cos {A — B) where
A and B are angles of any magnitude can now be given. In Fig. 96,
diagrams are given for 90® <. A <, 180®, 0 < B < 90° and 0 < ^4 < 90®,

for
180° < B270° and corresponding diagrams can be constructed
<.
OP and OQ are each of unit length, the Cartesian
angles of any size. If
coordinates P are (cos A, sin and of Q (cos B.
cases cos POQ cos =
{A B), and the —
cosme formula for the triangle

POQ gives
PQ 2 ^ Qp 2 — 20P,0Q cos POQ
oQ^
= 1 1 — 2 cos (A — B)
-j-

= 2 — 2 cos (A — B).
(14.3) for the distance
between P and Q,
But, from formula
PQ^ = (cos A — cos B)2 + (sin A —
= 2 — 2 cos A cos B — 2 sin sin
when use is made of the relations
^ = + sin=> 5= 1.
cos® A + sin^ 1, cos2 j?

Etiuating these values of PQ" we have


~ B) ~ cos ^ cos B + sin ^ sin B.
cos (A
+ B) sm(A - B) can then e
The formulae for cos (^ + B). sin (^
- B -
deduced by replacing B by
- B, 90“ B and 90 .
'

14] COORDINATES 263

EXERCISES 14 (a)

1. Plot the points (i)whose rectangular coordinates are (2, — 3) and


(- 4. - 1), (ii) whose polar coordinates are (2, 2ji/3} and (2. - 2;i/3).
2 . (i) Find the Cartesian coordinates of the points whose polar co-
ordinates are : —
(a) (4, 7t/3), (6) (5, - 71/4:),

(ii) Find the polar coordinates of the points whose Cartesian co-
ordinates are
(«) (1. 1), (-6, 12).
(6)
3. Show that the distance between two points whose polar coordinates
are (r^. and (r^, d^) is

V {»'i* + r** — 2rjr^ cos (0i — }.


4. Find the distances between the following pairs of
points
(i) (0, 0) and (a,
6),
(ii) (a, 0) and (0, b),
(iii)
(i> + ?.
and (g + r, r p). ? + r)
6 . Find the lengths of the sides of a triangle whose
vertices are the
points {5, - 6). (- 3, -
2) and (1, - 3).
6 . Show that the triangle whose vertices are the points —
f 2 21 12 31
and (—1, — 2) is isosceles.
7. Show that one of the angles of the triangle whose
vertices are the
points (6, 1), (—3,
7) and (8, 6) is a right angle.
8. ^e VomUA and C have coordinates (4. 1) and (1, 4) respectively,
ine pomt B is constructed by drawing AB
equal and parallel to OC
where O is the origin. Find the coordinates of the point B and show
that the figure OABC is a rhombus.

14.5. The coordinates of a point which divides the join of two


given points in a given ratio

(*« respectively

divides the line


-
coordinates of a point which P
1
P,P, « the ratio m ,: m,. Draw P,M„ P^„ PM

Flo. 97
:

264 PURE MATHEMATICS [U


perpendicular to the axis Ox and draw P^S parallel to Ox to meet
PM, P2M2, in R and S respectively. Then, if P is the point (x, y),
OM^~x^, OM^ — x^, OM = X,
From the construction of Fig. 97 it should be clear that
P^R = A/jM = OM — OA/j = X — x^,
RS = MM2 ~ OM2 — OM = X2 — X,
Since PR, P2S are parallel,
PiR _ PjP _ »ii
RS PPg tn 2

so that

Solving for x we find x = —^-=


ttti -j-

By drawing PiA^i, PN, P2^2 perpendicular to Oy, we could show in

the same way that the ordinate of P is given by

^
OTi + Wa
Hence the coordinates (x, y) of a point P
which divides the join
of the points P^ (x^, y^) and Pa (^2» >*2) internally in the ratio Wj Wa :

are given by

^ ^ ^ + m,y,
^
nil + ^^^2 ^2
These are called Joachimsthal’s section-formulae. As a special case,
the coordinates of the middle point of the line joining (^1, yj to (^2»>'2)
are \(Xi -f x<^ and §(yi d-ya) for here Wj mg. =
If the point Q divides the line P1P2 externally in the
ratio Wa, .

i.e., if PiQ QP wig, its coordinates would be found in


: : :
a similar
: ^,

way to be
- WjU'i
^ ^ ^ ^lya (
14 5 ) .

nil — ni2
*
nil — ?«2
Example 3. The a triangle are A(2, 4). B{- 4.
vertices oj 6). C{6, - 0).
Find also the co-
Write down the coordinates of X. the mid-point of BC.
ordinates of the points which divide internally and externally in AX the

ratio 2:1.
The coordinates of the mid-point of BC are given by
= + « y = i(- 0 -P 0 ) = - 3.
i(- 4 C) 1,

so that X is the point (1, — 3).


.
« i

The coordinates of the point dividing AX intemaUy in the ratio 2

are given by _
1_X_4 2
2xl-t-lx2 4 _2 x_(- 3) -f
^ _
14] AREAS OF TRIANGLES 265

The coordinates of the point dividing AX externally in the ratio 2 : 1

are given by
2 X 1 - 1 X 2 2 X (- 3) - 1 X 4
2 — 1 2 — 1

14.6. The area of a triangle whose vertices have given coordin-


ates
In Fig. 98, let the vertices of the triangle ABC be the points
^i). Jz) 3.nd C{Xq, ^'g). Draw AL, BM, CN perpendicular

Fig. 9h

to the axis Ox. Then


area ^ ABC
— area trapezium ALNC + area trapezium CNMB
“ area trapezium ALMB
= \LN(LA + NC) 4- \NM(NC + MB) - \LM(LA -f MB)
“ + ^3) ^*’2 ~ *3)(y3 + yz)
After a slight rearrangement of the right-hand side,
we have
area A ABC = - y^) + x^{y^ - y{j + x^{y^ - y (14.6) ^) }.
By talcing the point C to be at the origin 0 = yg = 0), the
wea of the triangle OAB with vertices 0(0, 0), A[xj_, and BixL
x x y^
^1/ y,)
\ a.^a;
IS given by
area A 0^45 = |(^iyg - (14,7)
It should be noted that for the formula (14.6) to give a positive
value for the area, it is necessary for the points
A, and C to be taken B
in a special order. This is such that in starting from the point A
Md proceeding round the perimeter of the triangle in
the order A,
B and C, the area of the triangle must always be on the
left In a
specihc example it is probably
better to draw a rough diagram and
work from first principles (see Example
4).
The above formula for the area of a triangle can
be used to obtain
266 PURE MATHEMATICS [14
the condition that three points shall be collinear (i.e., that they shall
lie on the same straight line). If the three points {xi, [x^, and y^), yz)
^ 3 lie on the same straight line, the area of the triangle formed
(^3. )

by them vanishes and, from (14.6) the condition for coUinearity is


therefore
^ib'2 — > + Xziy^ — yi) + x^iyi - y^) = 0.
3) (14.8)

The area
of a quadrilateral (or other plane polygon) with given
vertices can be obtained in a similar way. Perpendiculars are dropped
from the vertices on to the axis Ox (or on to a line parallel to Ox
through the vertex with the smallest ordinate) and the area can then
be expressed in terms of the areas of various trapezia (or triangles).
The resulting formula is rather complicated and again it is probably
preferable to work from first principles (see Example 6).

Example 4. Find the area of the triangle formed by the points (— 2, 3),
(- 7. 5), (3, - 5).

Denoting the points {— 2, 3), {— 7, 5) and (3, — 5) by A, B and C,


we drop perpendiculars AL, BM
on to a line through C parallel to the
axis Ox. Then,
area A ABC = area trapezium BMLA + area ^ A LC — area A BMC
= i(5)(10 + 8) + i(5)(8) - i(lO)tlO)
15 units.

of the triangle
Alternative^, since in proceeding round the perimeter
is always on the left^we
in the order A. B, C. the area of the triangle
could obtain the same result by writing x, = 2, 7, ^4 — 3, — —
= 3, y, = 6, yj

= 6 in formula (14.6).
x and y if the point {x. y) lies
on the
Example 5. Find the relation between
line joining the points (2, 3) and (5, 4).

for coUinearity of the three given points gives


The condition (14.8)
— 4) + 2(4 — y) + 5(y — 3) =* 0,

I.e.. ^ - 3y + 7 = 0.
:

14] AREAS OF TRIANGLES 207


Example 6 . Find the area of the quadrilateral whose vertices are the points
(2. 1). {3, 5). (- 3, 4) and (~ 2. - 2).
In Fig. 100, the four points (2, I), (3, 5). (— 3, 4). (-2,-2) are denoted
by A, B, C and D. Perpendiculars AL. BM, CN are dropped from A,

F and C on to a line through D (the point with the smaUest ordinatel


parallel to Ox. Then
area ABCD = area trapezium CNMB — area A CND
— area A DLA — area trapezium ALMB
= i(6)(6 7) + - i(l)(6) - i(4)(3) - J(l)(3 + 7)
25 units.

EXERCISES 14 (6)

1. Find the coordinates of the points dividing the line


joining the point
(7, “ 6) to the point ( — 2, 7) internally in the ratio 6 4 and externally :
in the ratio 3:2.
2 Find the distance between the points
.
- (-2. 3). (3, 1) find also
the coordinates of the point of trisection
that lies nearer to
(
— 2» 3).
3.
angular points A, B, C, D oi a, quadrilateral
ABCD are respectively
are the middle points of the sides
(2, 6), (8. 7), (10. 3) and (0. 1). E. F, G, H
AB, BC, CD. DA respectively.
Show that the middle points of EG and FH coincide.
4. U X and y are the coordinates of the middle point of the line
joining
the pomts (2, 3) and (3, 4), show that
y 1 = 0. — +
6 . Find the areas of the triangles whose
vertices are
(i) (0, 4). (3. 6) and (- 8. - 2).
(“) {a, c + a), {a, c) and (—a, — c — a).
Show that the four points (2. (-
9). 3. 12). (- 8. 15) and (7. 6) all
he on the same straight line.
Find the area of the quadrilateral
whose angular points are (1 1)
1, - 6).
(3* 6). (- 2. 4) and (-

triangle whose vertices are the points with polar


coordmates ( 1, 30*). (2. 60*) and (3. 90*).
^
268 PURE MATHEMATICS [14

14.7. The equation to a locus


a point moves so that it always satisfies a given condition, or
If
conditions, the path traced out by it is called its locus. For example,
if a point always lies on the straight line joining the points
(2, 3)
and (5, 4) we have seen in Example 5 of this chapter that its coordinates
{x, y) always satisfy the equation

^ - 3y -j- 7 = 0. (14.9)

The locus of the point here the straight line joining the two
(x, y) is

given points and equation (14.9) is the equation of the locus. (It will
be shown in the next chapter that when the locus of the moving point
is a straight line, its equation is of the first degree in x and y.)

Again, suppose that a point P moves in a plane so that the lines


AP, BP joining it to two fixed points A and B in the same plane are

Fig. 101

always at right angles to each other. Then we know that P


lies on

101 in which
the circumference of a circle on AB as diameter (see Fig.
of the point P). The circum-
P\> ^
2 - ^3 show three
possible positions
ference of this circle is the locus of when P
its motion is restricted

so as to fulfil the above condition.


plane so as to satisfy some condition such
If a point moves in a
will describe in general a definite curve or locus. An
as the above, it
general be found which relates the coordmates (x, y)
equation can in
will be true only for points lying on
of the point and this relation
equation relating ‘
the locus. Conversely, to every *
locus. Thus equation ^ above
(14.9)
in general, a definite geometrical
Many other examples otU occur to the
represents a straight line.
equation
rcLer who will famiUar ^vith the concept o an
afready be
from his previous work on graphs and curve
representing a curve

method of formation of the equation to


''"'a lew examples on the
a locus are appended.
14J LOCI 269
Example 7. Find the equation to the locus of a point which is always equi-
distant from the two points (0, 0) and (3, 4).

In Fig. 102, O is the point (0, 0) and A the point (3. 4). P,, P^, Ps are
three possible positions of a point P
on the required locus. Let P{x, y)

Fig. 102

be any point on the locus. Then


OP* = jr* + y\
PA> = (X ~ 3)‘ + (y ~ 4)».
Since OP is always to be equal to PA,
^r*
+;V* = (^ - 3)* + (y-4)K
This equation reduces to

6x -h8y=^ 25,
which the required equation to the locus of P. On
is
geometrical
pounds it is clear that the locus is the perpendicular bisector
of the line
joining the two points O, A and the
equation 6.r 8y 25 therefore + =
represents this bisector.

Example 8. Find the equation to the locus of a point which is always at


distance a from the origin.

If P is the point {x, y) and 0 is the origin


OP» = + >*.
Since OP = a, the required equation to the locus of P is

X* + = a\
the equation representing
a circle centre the origin and radius a.

Example Find the equation to the locus of a point which moves


9.
so that its
distance from a line parallel to
the axis Oy through the point (-
r x 1, 0)f is equal
t ,
to its distance
from the point (1. 0).
In Fig. 103, AB is the line through C{- I, 0) parallel to the axis Oy.

i
^ >')• the distance of P
aTfta ^
, crHencr'"
PJI/ = + 1.
Also, since P and S are respectively the points {x. and
y) (1. 0)»
PS* -(X- 1)1+^.,
270 PURE MATHEMATICS [14

Fig. 103

Since PS « PM, we therefore have


{,_ = (^+ 1 )
1
.

or, y* = 4r.
This is the required equation to the locus of P. The locus Is shown
dotted in Fig. 103 and its exact shape could be obtained by plotting
the graph of y* —4jf.

14.8. The intersections of two curves whose equations are


known
The coordinates of the points of intersection, or common points,
of two curves will satisfy simultaneously the equations of both curves.
The problem of finding the coordinates of the points of intersection
of two curves therefore reduces to that of finding the solutions of a
pair of simultaneous equations.
As an example, consider the points of intersection of the straight
Une X 3y — 1 =0
(equation (14.9)) with the curve

_f_
3^2 — I8x — 20y + 57 = 0.
The coordinates of the points of intersection are given by the solutions

of the simultaneous equations


_ 3^ + 7 = 0,
^
Sx ^ 3jy2 _ 18^ _ 20y + 57 = 0.

Substituting the value x = 3y-l, obtained from the first equation,


common points are given by
in the second, the ordinates of the
_ 7)2 4. 3jy2 — 18(3y — 7) — 20y + 67 = 0.
3(33,

This reduces to 3_y* — 20_y + 33—0,

or, (3^ - ll)(y


- 3) = 0.
= and from the relation x = 3y — T, the corres-
Hence v 11/3 or 3,
coordinates of
ponding values of are 4 and 2. Hence the required
and (2, 3). If the grap s o
the points of intersection are (4, 11/3)
14] INTERSECTION OF CURVES 271
line and curve are plotted and the coordinates of the points of inter-
section are read from the diagram, they will be found to
agree -with
these values.

Example 10.
Fiiid the length of the line joining the points of intersection
of
the straight line x
y I ^ +
0 and the curve y= 2x* x 1. = ^ +
From the first equation, y x =
1 and substitution in the second gives
for the determination of the abscissae of the points
of intersection
X -i- 1 = 2x^ ~ X + 1,
or, 2x* ~ 2x = 0.
This can be witten x{x — 1) =: 0,
so that ^ = 0 or 1. The corresponding ordinates, found from = x + 1,
y
we therefore y = I and 2 and the points of intersection
are therefore
(0, 1) and (1, 2). The length of the line joining these points
= V{(0 - 1)»+ (1-2)*} « ^/ 2 .

EXERCISES 14 {c)

1. Find the equation to the locus of a point


which moves so that its
distance from the point (3,
4) is always 6 units.
A and B are respectively the points (1.
moves so that the angle
0) and (7, 8). A
point P
is a right angle. APB Find the equation
to the locus of P.
3. cooperates of the points are {I, 6) and (5, 3), FindA and B
the equation to the perpendicular bisector
of the line AB.
4. Find tlie equation to the locus of a point
such that the areas of P
A. B.C, Dane the points
1)* (0.
^
{0; 7), (3, 0), (5, 0).

- r«
6 . Find coordinates of the point of intersection of
the two lines
2^r 3y 6 and 3.^ + y + 2 « 0.
6. Find the coordinates of the points
of intersection and length of
the common chord of the two
curves y = 2x* and y* = 4x.
7. Find the coordinates of the common
points of the line — 2 =* 0

8.
^ tte curve *• +
ates of the middle point
- 1(^ + 4 = 0. Find also the coordia-
of the line joining the common points
equations to the sides AB, BC, CA of a plane triangle are respec-
^ - 3y + 4 , 0 and y X D mid-pLt
Tbc^ ; hndmg the coordinates of A, B,
is "the

C and D
thaf A
that AB* verify
^
+ AC* ^ 2{AD* + BD*).

EXERCISES 14 (i)
1.
^rdmates, the points (0. 0), (2, n/6), (4 . :t/2).
2 .
sides of a triangle whose vertices ara the
P^ts^2
pomts 2
( , 3), (4. - 6), (-3. - 6}.
272 PURE MATHEMATICS [14

3. Find the coordinates of a point equidistant from the three points


(2, 3), (4, 5), (6, 1).

4. Show that the four points (— 2, 3), {0, — 1), (6, 7) and (8, 3) arc
the angular points of a rectangle.
5. Find the coordinates of the centre of the circumscribed circle of
the triangle whose vertices are the points (— 2, 2), (1, — 2), (1, 3).

6 . Find the radius of the circumscribed circle of the triangle the polar
coordinates of whose vertices are (0, 0), Oj), O^).

7. The line joining the points (—6, 8) and (8, — 6) is divided into
four equal parts. Find the coordinates of each point of section.

8 . A and B are respectively the points (5, 6) and (7, 2). If the line

AB is produced to a point C such that BC — hAB, find the coordin-


ates of C.
9. By finding the coordinates of the middle points of the lines joining
the middle points of pairs of opposite sides of a quadrilateral the
coordinates of whose angular points are given, prove that the lines
joining the middle points of the opposite sides of any quadrilateral
bisect one another.

10 . The polar coordinates of three points A, B and C are given in the


following table :

S = 71 /O, n/Z, 3a:/4,


r = 100, 100, 160.

Find the lengths of the sides and the area of the triangle ABC.
(QE)
11 . Find the area of the triangle whose vertices are the points (2, 1),

(3, — 2) and (—4, — 1).

area the quadrilateral whose vertices are the points


12 . Find the of
(1, 1), {2. 3). (3. 3) and (4. 1).

13. A the point (2, 3) and B is the point (0,


is
- 1). The angle
is a right angle and BC is 5 units
in length. Find the coordinates
of the two possible positions of C.
(

respectively. Find
14. A and B are the points (-3, 2), (6, 8)

the equation to the locus of a point which moves so tha P


PA^ - PB^ = 50.
^

to the locus of a point which moves so that .ts


15. Find tlic equation
is always equal to
lour times
distance from the point (a, 0)

distance from tlie y-axis.


squares
10 A point moves so that the sum of the „
equal to
is constant and
.

the two fixed points (a, 0) and ( - a, 0)

2c2. Show that the equation to its locus is


+ ya = c* - fl».

coordinates of two of the vertices of a !>'-<=


17. The verte.x if the area
of the third
and (6, 6). Find the locus
triangle is 12 units.
U] EXERCISES 273
18. The coordinates of points A, B
are respectively (2, 6) and (5, 8).
P is a point such that AP ** 2BP. Find the equation to the locus
of P. (O.C.)
19. Find the equation to the perpendicular bisector of the Kne joining
the two points (4, 1)and (— 2, 3). (O.C.)
20. A variable point P moves so that the square from
of its distance
the origin is equal to the area of the triangle PAB where A and B
are the points (13, 0) and (0, 1). Find the equation to the locus
of P,
21. Find the lengths of the sides of the triangle formed by the three
straight lines x — = 6, 3jt — 2y + 1 = 0 and x + y = 2.
22. Find the coordinates of the middle point of the line joining the
common points of the line 2;r — 3y + 8 = 0 and the curve y* = Sx.
(O.C.)
23. Find the values of a and b the straight lines ax
if
5y 1 and ^
4jr + = 5 intersect at the point (2, — 1). If the lines meet the
ar-axis at A and B, find the length of AB. (O.C.)
24. Show that the coordinates of the common point of the line
y =mx -h a/m and the curve y* = 4ax are {a/m*. 2a/m).
25. Show that the length of the line joining the common points of the
line y = mx + c and the curve y* ^ 4x is

^,(1 +m*J*(l -mcjl.

8
CHAPTER 16

THE COORDINATE GEOMETRY OF THE STRAIGHT


LINE
15.1. An equation of the first degree in x and y represent^ a
straight line
The most general equation of the first degree in ^ and y is

Ax^By-\-C = 0, (15.1)
where A B and C
are constants. Let P,{x„
y,), P,(x,. y,). P,{x„ y,)
De any three points lying on the locus represented by the equation
(15.1). Since lies on the locus, its coordinates
must satisfy yj
equation (15,1) and hence
Axi Byi -f- C — 0,
Similarly Ax^ q. + C == 0,
Axj + Rya + C = 0.
Subtracting the second of these equations from the first and the third
from the second we obtain
^(^1 — x^) + B(y^ -yj) = 0
and A (x^ - x^) + R(y, - y^) = 0 .

By equating the values of the ratio A/B obtained from these two
equations we have

giving {x^ - x^)(y^ - y^) = (x^ - Xs)(y^ - yj.


This can be rearranged as
Xi{y2 - yti) + x^iys — yi) -f - >'2 ) = 0.
From formula (14.6) we see that the area of the triangle formed by
the three points and P3 is zero. Since the points P^, P%r P3
are any three points on the locus represented by equation (15.1), the
locus must be a straight line for a curved line could not be such
;

that the area of the triangle formed by joining any three points on
it should be zero.

15.2. The equation to a straight line which is parallel to one


of the coordinate axes
In Fig. 104, AB be a line parallel to the axis Oy. Let AB meet
let
the axis Ox in C and let OC = Let P be any point on the line
c.

AB and let its coordinates be (x, y).


274
15] SPECIAL FORMS OF EQUATION TO LINE 275

y
1
A
<

0 C

8
Flc. 104

Since the abscissa of the point P is always c, we have


JC = c.

This relation true for every point on the line


is and for no other AB
point. The relation x =
c is therefore the equation to the line.
Similarly the equation to a line parallel to the axis
Ox and at
distance d from it is

y = d.
The coordinate axes are special cases of these lines for
The equation of the axis Ox is therefore
which c d = 0. =
= 0 and' that of the axi^
Oy IS x = 0.

15.3. Special forms of the equation to a straight line


It is often useful to be able to write the general equation
Ax By C == 0
of a straight line in a form in which
the constants A, B, C, or rather
the ratios of two of them to the third,
are related to some geometrical
properties of the line. Such properties
might be the slope of the line
and the coordmates of a point on it. the
intercepts it makes on the
coordinate axes and so on. Special forms
of the equation to a line
m terms of v^ous properties possessed
by it are developed
^ in the
paragraphs which follow.

(a) The equation to a line in terms of its slofe and its intercept on the

T>,
^ intercept c on the v-axis

® = ”• the coordinates of
p^^t
coordinates (*, y), the figure
shoil^hat"""'
CR = OP' = *,
PR = PP’-RP' = PP’^OC=y-e,
276 PURE MATHEMATICS [16

Fig. 105

m= tan ^
0 = PR = y — c
_ -i. ,

CR X
Hence y =. mx -j- c (15.2)

is the relation between the coordinates of any point {x, y) on CP and


this is the required equation to the line in terms of its slope m and
intercept c.
By writing the general equation Ax By C 0 in the form =
A C

and comparing it with (15.2) we see that the ratios A/B, C/B of the
constants in the general equation can be expressed in terms of the
slope mof the line and its intercept c on the y-axis by
A
d _ C= -
= «. £ c. (15.3)

For the line of slope m passing through the origin of coordinates

Of c = 0 and the equation to such a line is

y mx. =
slope and the coordinates of a
(6) The equation to a line in terms of its

point on it

The equation to any line of slope m is, from (15.2),

y = mx + c.

point whose coordinates are (^„ yj lies on the line, these values
If the
so that
of X and y will satisfy the above equation
y^ = mx^ 4- c.

Bv subtraction we have ,^sa\


^ y_y^ = nt(X~Xy) (15-4)
the point
line of slope m which passes
through
as the equation to the
15] SPECIAL FORMS OF EQUATION TO LINE 277

Example 1. Find the equation to the straight line which passes through the
point {1, 2) and makes an angle of 45“ with the x-axis.

Here m= tan 45“ = \, x^^ I, y\ = 2 and the required equation is

-2= {l){x - 1),

or, X —y I = 0.

(o) The equation to a line passing through two given points


one of the given points has coordinates
If (ati, the equation to
a line passing through it is (equation (15.4))

y —yi = ni(x — *,).


If the second given point has coordinates (x^, and the above line
passes through it, its equation will be satisfied by y —y^, x =
Hence
y2-yi = ‘^(xz-x^-
By division, we have
x-x^ ^ y-y\
(15.5)
X^-Xs yz - jvi

as the required equation to the line passing through the two points
y^» (xt. y*).

Example 2. If B and C are respectively the points (—« 1, — 3) and (5, — 1)


find the equation to the line BC.
From (15.6) the required equation ia

- (- 3) y
6 - (- 1)
" - - (- 3)' 1

* + 1 +3
ie '
0 2
giving X - 3y 8.

((/) The equation to a line in terms of its intercepts on the coordinate axes
Let the line AB (Fig. 106) makeintercepts of lengths a and h on
the coordinate axes Ox^ Oy respectively. Then the line passes through

Fig. 106
278 PURE MATHEMATICS [-JS
the points (a, 0) and (0, b) and formula (15.5) gives for its equation
X~ a ~y — 0
0-a
or, ^ f _y
-a b'
This, by cross multiplication, gives

bx — ab = — ay^
or, after division by ab and a slight rearrangement,

y^ i

=
J+1 ^- (15.6)

The general equation Ax -f- By -|- C = 0 can be written


^ -1
i-C/A)^ {-C/B) '

and a comparison with equation (15.6) shows that the line


^y + C = 0 makes intercepts on the coordinate axes of
“ C/A and — C/B.

(e) The equation a line in terms of the length of the perpendicular from
to
the origin and the angle this perpendicular makes uith the x-axis

In Fig. 107, let the line cut the coordinate axes in A and B, let
p be the length of the perpendicular OP drawn from the origin 0 on

io A B and OP make an angle a


let with the axis Ox, Then the right-
angled triangle OAP gives
OA =^seca.
and, since the angle BOP is 90“ - a, the right-angled triangle BOP
gives
OB =p sec (90® — oc)=p cosec a.
cosec a on the
The line AB therefore makes intercepts p sec a and p
15] SPECIAL FORMS OF EQUATION TO LINE 279

coordinate axes and, from (15.6), its equation is

^— = 1.
p sec a p cosec a
or, cos a + ^ sin a = p, (15*7)

In working problems on the straight line, a correct choice of the


form of the equation to the line can often reduce the algebraical
work. We conclude this section with some illustrative examples.

Example 3. Two parallel lines AP, BQ pass


through the points A[5, 0) and
B(— 6, 0) respectively. Find the slope of these lines if they meet the line
4x + = 25 in points P
and Q such that the distance PQ is 6 units.
(L.U0
Let the slope of the parallel lines AP, BQ be m. Then, by (15.4), the
equations of AP, BQ are respectively y m{x 6) and y m{x 6). « — »
The coordinates of P, the point of intersection of and the line AP
<jLir +
3y s
25. are given by the values of x and y which satisfy the
simultaneous equations

y » mix — 6 ),
4x Zy 25.
Substitution of y from the first equation in the second gives
4x + Zm{x — 6) = 25

giving
+ I6m
25
4 + 3m
Since y = m(x — 6) the ordinate of the point P is

/25 + 16m \ 6m
^ - 4 +^-
and the coordinates of P are therefore
^25 + 16m 5m \
\ 4 + 3m '
4 -f 3m/*
The coordinates of Q are similarly found from the solution of the
simultaneous equations y m{x +6), 4x 3y =25 to be + »
/26 — 16m 45m \
\ 4 + 3m *
4 + 3m/*
The distance PQ is therefore given by
PQ* ps + I5m 25 — 15m / 6m 45m
+ 3m
y y
\ 4 4 + 3m / \4 -J" 3m 4 + 3m/
2500m«
“ (4 + 3m)
•*

Since PQ = 6, we therefore have, for the determination of the slope m,

2600m*

(4 + 3m)»
giving eim* — 24m - 16 » 0.
or. (7m - 4)(13m + 4) - 0.
Hence the required values of the slope are 4/7 or — 4/13.
280 PURE MATHEMATICS [16
Example 4. If O is the origin, A the point (8. 0) and the point (0. ^).find B
the coordinates of the points P and Q, where the line Zx 2y c meets = OA
and AB respectively. If the area of the triangle OPQ is one-half that of the
triangle OAB, find the value of c. (L U )
The line OA the ;if-axis and its equation is
is
y 0. The coordinates =
P therefore given by the solution of the simultaneous
equations
y = 0, 3x -j- 2y ^
c. Hence P is the point (c/3, 0).
The line AB makes intercepts of 8 and 6 units on the coordinate axes
and, from (15.6), its equation is
X y
8 6 ^
or, 3;»f + 4y = 24.
The coordinates of Q are given by the solution of the simultaneous
equations 3.r + 4y = 24 and 3.r + 2y = c. These are easily found to be
2c
.= - 8 ,
3
Since, from (14.7), the area of the triangle formed by the points (0, 0),
y\)> ('*'«. yt) is the area of the triangle OPQ is given by

= 2 =-^'.
The area of the triangle OAB is that of a right-angled triangle of base 8
and height 6 and hence is 24 units. If A OPQ = } A OAB,
24
12 “ 2 *

or. — 24c -f 144 = 0,


giving (c — 12)* = 0 or c = 12.

Example 5. the length of the perpendicular from the origin on to the


Find
straight line passing through the two points (6, 4) and (9, 8).

By (15.5) the line through the two given points is

X Q y 4 - -
9 - 6
“ 8 - 4'

or, 4x —
3y = 12.
This can be compared with the form x cos « + y sin « = ^ by
dividing

through by V(4» + 3*) or 6. for 4/5 and 3/5 can be taken the -
cosine and sine respectively of the angle tan
*(-3/4). Thus it ^
of the line as
a = tan-* (— 3/4) we can wite the equation
X cos a + y sin a = 12/5,
from the origin
showing that the required length of the perpendicular
is 12/5.

EXERCISES 16 (a)

I. Find the equation to the straight line which


of 3 units from the negative y-axis and
(i) cuts off an intercept

makes an angle of 120® with the x-axis,


- and 3 respectively on the axes of
(ii) makes intercepts of 6
X and y,
and -
passes through the two points (—2,
(4, ).
3)
(iii)

liQ ANGLE BETWEEN TWO LINES 281
2. Find the slope of the straight line cutting off intercepts of - 3 and
4 from the axes of x and y respectively.
3. Find (i) the intercepts on the axes of coordinates, (ii) the slope
and {iii) the perpendicular distance from the origin, of the line
hx — \2y = 65.
4. Find the coordinates of the point of intersection P of the
two straight
lines 4^ + 3y = 7, Sat — 4y = — 1 Find also the equation to the
* .

line joining P
to the point (— 2. 3).
5. Find the distance bebveen the t^vo parallel straight
lines
2r -!-
y = 4 2y = 2.
and +
6 . Find the equation to the straight line passing through
the point of
intersection of the Uvo lines 24r +
3^ *= 4 and Sat - 2y « 5 and also
through the point of intersection of 3:r —
4y = 7 and 2x + 6y = 2.
7. Find the equation to a straight line which passes
through the point
(3. 5) and makes equal intercepts on the
coordinate axes.
8. Find the equation to a straight line which passes
through the point
(4. 4) and forms with the coordinate
axes in the fourth quadrant
a triangle whose area is 4 units.
9.
OA.OB =
coordinate axes at A and B and
c* where 0 is the origin. If AB is parallel to
the line
X cos a -h y sm a s= p, find its equation.
10 . Find the condition that the straight
line (x/a) -f (y/b) = 1 should
he at unit distance from the origin.
11 . Find the equations to the diagonals of
the parallelogram whose sides
have the equations :

+ y =. 1
3^ 3y + 3, .

+ y *= 15, 3y 6^ — 11
3^ *=>
(L U
^e straight line y = w(, _ 2a) through a fixed point .

(2a, 0) meete
)

the lines * = a and


y = 6 in P and 0 respectively. If O is the
equations to the lines OP,
AQ and the coordinates of their point of intersection
R If m
2i"-ay
* ^
-"aft

15.4. The angle between two straight lines


equations to the two straight lines
mtersecting at P AB CD
be respectively
y = «,* + y = c*'

Then If AB, CD make angles 6,. 0, with the axis 0;e as shown,
“ Wg. QK o\
Since the external imgle PBO of the triangle PDB is equal to the sura
S!.i iTZ'by'*"
angle APC = angle BPD
= angle PBO — angle PDB
= Oj — 0,.
282 PURE MATHEMATICS [15

Fig. 108

Hence
tan APC = tan — 62
{Oi )

tan $1 — tan 62
1+ tan di tan 0,
Wj —
1+ WjW,
when use 5s made of the relations (15.8). The angle between the two
lines y = ntiX c^, y = is therefore

^1 —
tan-1/ (15.9)
U + WjWa/*
It should be noted that if the quantity in brackets in (15.9) is
(the
positive, it is the tangent of the acute angle between the lines
tangent
angle APC of Fig. 108) if this quantity is negative, it is the
;

of the obtuse angle (the angle DPA).


If we require the angle between the two straight
lines

Aix
A^ + Cj = 0,
their respective slopes are, from (15.3), given by
Ax
“ b;
Substitution in (15.9) and a slight reduction shows that the angle
between the two lines is

(15.10)

Finally, if the two lines are given in the form


X cos a + y sin a =
X cos ^ y sin ^ = pt*
the origin
we know that the perpendiculars from
— )

IS] ANGLE BETWEEN TWO LINES 283


make angles a and p respectively with the x-axis. The angle between
the lines is clearly equal to the angle (or its supplement) between the
perpendiculars, so that the required angle is. in this case, either —a
ot 71 — — a).
Example 6, Find the angles between the following pairs of lines :

(«) > = 2x + 6 and 3;r+ y => 7,


(i») 3x — y + 7 = 0 and x — 3y + $ = 0.
(i) The slope Wj of the first liae is 2 and by writing the equation to
second line in the form
y =- 3;r -f 7 we see that its slope m, =— 3.
Hence, from (15.9) the angle between the lines is given by

3) 1
tan
U + 2(- 3)/
-Un-M- 1) « 135*.
(ii) Comparing the given lines with A^x + B^y + Cj 0 .

C, +0. /Ij - 3.= = - 1. = I. — 3, so that


by (15.10) the required angle is

tan“*
f(l)(- 1) - (3)(-. 3)1
l(3)(l) + {- 1)(- 3)/

M tan”^ - »'•
G)
Example 7. Find
the equations to the lines through the point
(2, 3) which
make angles of 45® with the line jr 2>^ — ™
1. {L.U
In Fig. 109, P is the point (2, 3) and AB the line jr - 2y 1. This
can be written
y ^ ^ and its slope is therefore 1/2.

It is clear from the diagram that there are two possible


w^ch make angles of 46-> with
lines PM PN
AB, and the tangents of these angle’s are

1
»» - 1 /2( )

I + (m/2)'
284 PURE MATHEMATICS [16
or m= 3. Similarly if m' is the slope of

_ - (1/2)
1
1 + (m'/2y
or m' = — 1/3.
The required lines are therefore those which pass through the point
(2, 3) and have slopes 3 and — 1/3. By (15.4) the equations are
—3=
y 3 {t — 2) or 3^r — y = 3,
and y —3= — lix — 2) or T + 3y=Il.
15.5. Conditions for parallelism and perpendicularity
If the two lines v = m^x -f y = -f- Cg are parallel, they have
the same slope and hence
Wj = Wj- (15.11)
If the lines are given by AiX + B^y Ci = 0, A^x B^y -f = 0,
their slopes are — ^i/^i and — A^/B^ and in this case the condition
for parallelism becomes .

(15,12)

the two straight lines y


If ntiX Cj, y ni^ 4- c, are per- = + =
pendicular, the angle between them is 90®. Since the tangent of an
angle of 90° is infinite, formula (15.9) shows that 1 tn^m^ 0. + =
Hence the condition for perpendicularity is
mjWj =— 1, (15.13)

i.e., product of the slopes of two perpendicular straight lines is


the
1. —
This is a most important result it can be expressed in a slightly —
different way by saying that if the slope of a given line is m, the slope
of a line perpendicular to it is 1/w. —
the
In a similar way, by using formula (15.10) we can deduce that
condition for perpendicularity of the two lines A^x -f B^ + Q =
A^ + B^y 4- Cj =0 is

AiA^ -f- B^B^ — 0. (15.14)

Example 8. Find the equation to the straight line which passes through the
point (—2. 3) and is parallel to the
line 7x — — = y 6 0.

The equation to the given line can be written y = 7x - 6 w that its

slope is 7. The equation to the required


line will therefore be ^t to
(- mis is
a line with slope 7 which passes through the point 2. 3).

_ 3 = 7{x 2) or 7x — y + 17 = 0
y +
1, 3), -
Example 9. The coordinates of three points are -4(1, 2),
— Find the equation to and the equation to the line through
BC
C(5, 1).
{L.U.)
A perpendicular to BC.
The equation to BC is
x + l +3y
6 + 1 - 1 4 3‘
i.e., X — 3y = 8.
y :

IS] DISTANCE OF POINT FROM LINE 2S5


The slope of this line is 1/3, so that the slope of a line perpendicular to
it is — 3. Thus we require the equation to a line of slope — 3 which
passes through the point (I, 2). This is
—2 — 3(^ - 1) or 3^ + :y = 6.

EXERCISES 15 (5)

1. At what angle are the lines whose equations are ax by c = 0


and (a — b)x + (a + b)y d 0 inclined to each other ? (L.U.)
2 . The vertices of a triangle are the points ^(1, 4). B{5. 1). C{— 1, — 1).
Find the equations to its sides and the values of tan B, tan C.
(L.U.)
3, Find the equations to the straight lines passing through the point
(3, — 2) and making angles of 60“ with the line = 1.
The base of an isosceles triangle lies along the line 3x
2y = 2
4.
and one of the equal sides lies along
y ^ 2x. Find the equation
to the other equal side if it too passes through the origin.
Write down the equation to the straight line which
(i) is parallel to the y-axis and
passes through the point (2, 3),
(ii) is parallel to the line 4x
+ 3y + 8 = 0 and passes through the
point (2, - 3),
(iii) isperpendicular to the line 4;r + Sy + 8 == 0 and passes through
the origin.
A triangle ABC is formed by the lines 3jr — 4y + 3 =» o {AB)
jr
+y _
3 ^ 0 (BC)
and 4^r _ 3y - 6 = 0 C). Find the equation
to the straight line through C perpen4icular to AB. (L.U*)
7. Find the equation to the join of the points
{1, 2) and (3, 4). Find
^so the coordinates of the middle point of the join and
hence \vrite
down the equation to the perpendicular bisector of the join.
(L.U.)
8 . P, Q, R are three points with coordinates
(1. 0), 2 4 ( ,- — 6' —
( )* ^ 21
respectively. Determine
the equation to the line through
(i)
perpendicular to OB, P
(u) the equation to the line through
Q perpendicular to PR*
(m) the coordinates of the point of intersection
of these lines.

8 . Find tte coordinates of the point of intersection


of the perpendiculars
from tte vertices to the opposite sides
of a triangle whose sides have
equations — 3y=.0, 4;r— = 0 and x + ^ 20.
y
10 . Prove that the straight line joining any
two of the four points
{ami, o/mi), (am„ a/m,), (am,, a/m,), (am,, a/m,)
IS perpendicular to the straight line joining
the other two
mxm,m,m, = — f.
if

15.6. The perpendicular distance of a point


from a straight Une
In Fig. 110, AB is the straight
]me Ax C 0 meeting the + =
coortoate axes in ^ S
By
is the point (A.
AB and we require to find a formula for thepg k
distance PQ
P
a.
V pe^TnS
=
;

280 PURE MATHEMATICS [15

Fig. 110

Two shown in Figs, [a) and (&), arise according as P is or is


cases, as
not on the same side of the line AB diS the origin. PN, PM are drawn
perpendicular to the axes Ox, Oy respectively.
The equation to the line AB can be written in the form
^
^ 1
- C/^ - C/B
showing that it makes intercepts on the coordinate axes given by
OA^- C/A, OB = ~ C/B,
From the right-angled triangle OAB we have
AS = -^{OA-^A-OB^) = V{(- C/AyA-{-C/B)^}
= C^/{A^ + B^)/AB.
Since PM = h, PN ^ k, PQ = p,
A OAP = \OA.PN = — —
Ck
area

Ch
area A BOP = \OB.PM = — 2B’
^ CpVi^^ + B^)
area A BPA = ^AB.PQ =
C2
area A OAB = 10 A. OB = 2ZB
From Fig. 110(a),

area A OAP + area A BOP + area A BPA = area A OAB,

C^ Ch CPViA^A-B^)_ C^
SO that 2AB
2A 2B 2AB
Solving for p we find

= Ah + Bk + C
P
BISECTORS OF ANGLES 237
From Fig. 110(6), we should have
area A OAF -f area A ^OP — area A ^FA = area A OAB.
This would lead in a similar way to
_Ah-\-Bk-\-C
~V(A^ +
To sum
up, the perpendicular distance of the
point F(/t, k) from
the straight line Ax By C + +
0 is given by =
AhA-Bk + C
’ (15.15)

° '"agnitude of the distance irrespective of

tViA ^ *'^0 poults calculated from (15.15) without


s?defof The
th^th
r
^
that the two points are on opposite
formula is best remembered by obse^ing
^at the nunaerator is obtained by writing the coordinates of
the given

is“thTsi T ‘he den^in-


‘ho coefficients of
TTid
X ana y m the equation.

L*+ (2. - 1) ond (1. 1) from the

+ *>--« =0 . the distance ol

3(2) + 4(_ 1) _ 6 4
V{(3)*+(4)*) "~6-
The distance of the point (I, 1) from the line is

nus
3(1)

the reqnired distances would

5r.-“.Tr.r
+ 4(1)
V{(3)* + (4)*} ~6'
-6


1

usually be quoted as 4/5 and

“S'"*
eH;;
»•»•» »o
1 /a

liir
In Fig. Ill the given
lines
A^x + B^y 4- Cj = 0, A^ + B^y + C. = 0
P°«‘ <“ intersection being
^a i? If p

^ + C,
^
a/Mi' + B,*) VW + B^) •
288 PURE MATHEMATICS ri5

If the point {x, y) on the bisector of the angle ARD, Q will


5 is

be equidistant from both lines and will lie on the same sides of the
lines as the origin. Hence the coordinates {x, y) of Q will satisfy
A^x + Q + ^2

The equations to the bisectors of the angles between the two lines
-\- B^y -{ +A^x -\’C^ =
0 cBJi therefore be written
AiX + B^ + Cl L A^ +
B^y H-
(15.16)

Example 1 1 . Write doum the equations to the bisectors of the angles between

the lines 3;r + 4>' = 12 and 4jr - == 6.

forms
Writing the equations to the lines in the standard
2x 4y — 12 => 0 and
— 6 = 0,
are
the required equations to the bisectors
3jr 4^ 12 + 4r - 3y - — 6
"^ ,

V{(3)* + (4)*) VfW* + (- 3)*)‘


These reduce to jr - 7y + 6 - 0 and + y - 18 = 0.
EXERCISES 15 (c)

Findthedistanceo£thepoint(2,3)from theline5*
- 12y + 39 = 0
1. (i)
from the hne 3* + 2>- = 8
(ii) Find the distance of the point (2, 1)

and explain the result.


from the line .r cos «+ J-
sm « - ^
Find the distance of the point
(A. k)
2 .

equidistant from the Unes 3Ar + 4>. = 12.


3 .
Show tlrat the point (1, 1) is

whether the point the centre


By LwS a“rougt fi'gu^e.'d^ide
is
15] LINE THROUGH INTERSECTION OF TWO LINES 289
of the inscribed or one of the escribed circles of the triangle formed
by the above lines. (O.C.)
4. Find the equation to the straight line which lies mid-way betsveeo
the point (2, — 1) and the line 3x ~ 2y -\- 5 = 0. (O.C.)
5. Find the equation to the locus of a point which moves so that its
perpendicular distance from the line — 3y = 7 is rivice its per-
pendicular distance from the line 6;r -f- 12y = 8.

6. Find the equations to the bisectors of the angles between the lines
4y -f- 3y = 7 and 2^ -f 7y = 31.
7. Find the equations to the six bisectors of the angles between the
lines X ly =» Z, llx — 7y-{~3 = 0, x-y-i-l = 0 and show that
three of them pass through the point (1,
1). (O.C.)
8. Q the foot of the perpendicular from the point P{h. k) on to
is
a
line RS whose equation is Ax -i- By
+ C = 0. Kind the equations
to the bisectors of the angles between the lines PQ and
RS.

15.8. The equation to a straight line passing through the point


of Intersection of two given straight lines
Suppose the two given straight lines are represented by the equa-
tions A^x 4- -f- Cl 0, = A^ =0 and be the
let P
point of intersection of these two lines. Consider the equation

AiX + H- Cl -f- X{A^ + ^ay + C*) = 0, (15.17)


where ^ is an arbitrary constant. The equation (16.17) represents
some straight line for it is of the first degree in and y. Further, the
equation is satisfied by the coordinates of the common point of the
two ^ven lines since these coordinates simultaneously satisfy the
equations Biy -f Cl 0 and >4^ = C, +
0. Hence + =
equation (16.17) represents a straight line passing
through the point
of intersection of the given lines.
The method used above of deducing the equation
to a line passing
tmough the common point of two other lines is an
example of a device
of great use in coordinate geometry
and should be studied with care
That there is an infinity of lines passing through
the common point
be clear on pometrical grounds : it is also
apparent from equation
(16.17) m
which the constat X is quite arbitrary.
It will gperally be required to find
the equation to a line which
besides passing through the point of
intersection of two given lines'
satisfies a second condition such as
passing through a second given
condition wiU enable us
tn
to fiv ih
^the value of X and to pick out from
wjch fuiais aU the required conditions. Some
the infinity of lines that one
iUu^strative e.xamples

T
I
290 PURE MATHEMATICS [15
Example 12. Find the equation to the line
which passes through the point
(6. 2) and the point of intersection of the lines 2;r —
3_y l = o and +
Any line through the point of intersection of the given lines is
3y - 1 + X[Zx _ 4y -
2^r + 6) = 0.
If this line passes through the point (3, 2),
2(3) + 3(2) - 1 A{3(3) - 4(2) - 6} = 0,
-f-

or, 5X = 11. Writing A = 11/5 in the equation we have


2.r + 3y - 1 + {n/5)(3^ _ 4^ _ 6) = 0,
or, — 20y = 71 as the required result.

Example 13. Find the equation to the line


through the intersection of the lines
3^r — 2>' + 14 = 0, a: -f-
y = 6 and perpendicular to the line 5x Gy 0. ~ =
Writing the equation to the second given line in the standard form
^+^ — 6 = 0, the equation to any line through the intersection of the
two given lines is
- 2y + 14 + A(:r + - 6) = 0
3jr

or, (3 + X)x - (2 - A)^ + 14 - 6A = 0.

The slope of this line is (3 + A)/(2 — A) and the slope of the line
5x — 6y = 0 is 6/6. Hence for the lines to be perpendicular

leading to A = 27. Inserting this valne of A, the required equation is

(3 + 27);r - (2 - 27)y + 14 - 6(27) = 0,


or, 30;r + 25:y = 148.

15.9. The equation to a pair of straight lines


Consider the equation
— 4xy = 0. (16.18)
This can be written
(x-y){3x-y) =0,
and the equation is satisfied by the coordinates of all points which
make either x — y = 0 or 3x — y = 0. In other words the equation
is satisfied by the coordinates of points lying on either of the straight

linesy = xoiy = 3x. Hence equation (15.18) represents two straight


lines which pass through the origin and are inclined at angles of
45®
and tan”^ 3 respectively to the ^-axis.
Consider now the more general equation
ax^ + 2hxy + by^ = 0. (15.19)

Multiplying it by a, it cnn be written in the form


{aV 2ahxy -f- A^^) — (/>^ — ab)y- = 0
or, {
[ax + hy) + ){ («* + *3')
- -«*)}= <>•
15] EQUATION TO PAIR OF LINES 291
Tlius the equation represents the two sttaight lines
ax-\- {h -\- ^/(h^ — ah)}y = 0,1
{h^ — ab)]y =
(15.20)
ax -\- (h ~ 0,J
each of which passes through the ori^ for equation (15.19) is satis- :

fied by the coordinates of all the points which lie


on the first of the
lines given by (15.20) and also by those of all
the points which lie
on the second line.
It should be noted that the two lines given by
(15.20) are real and
different if A® >they are real and coincident ii ab and they^
=
are imaginary ii ab. <
The angle between the two lines represented by equation
(16.19)
be found as follows. Suppose the separate equations of the two
lines are y =5 m^x and
y =
Then equation (16.19) must be
equivalent to
— m^x)(y — m^) = 0
{y
wpwjx* - (wii m^)xy + y* = 0. -1-

Comparing this with equation (15.19),

^ _ w, -f _ 1
ffla
~V
a 2h
so that WjWj = a/b, + w, = - 2A/6.
By formula (15.9), the angle between the two lines is given by
— m.
tan-> Y
-f
Since (Wj — =: (jttj __

= (4AV6*) - (4a/6)
..
= 4(A» - a6)/6*,
the required angle is

J 2V(^« - atm
1 i + M) ;
or. ; v(A«
-
(16.21)
1 o +b y
°f ,an angle of 90» is infinite,
ch formula (16.21)
shows that the two straight lines given by the
equation
ax^ -h 2hxy -f- 6y* =0
are perpendicular if « + J = 0. (15 22)
The general equation of the second degree
^* + !ihxy + by^ + 2ex + 2fy + e^0 (16.23)
which contains terms in
*,y and a constant as well as the second
degree
1

292 PURE MATHEMATICS [15


terms of the left-hand side of equation
(15.19) can, in certain circum-
stances. represent two straight lines.
the expression It ^viU do so if
on the left can be broken into two
factors, each of the first degree.
It IS, however, rather beyond the
scope of the present book to obtain
between the constants a, b, c,f,
/ f g, h in equation
for this equation to represent two straight
lines and we restrict
the discussion to pairs of lines which pass through
the origin.
Example 14. Calculate the acute angle between the pair
of straight lines repre-
sented by the equation lO^y -f 7y* as 0.— (L U )
Working from first principles, the equation can be written in the form
{X ~ y)(Zx 'ly) -0, =
showing that the equation represents t^vo lines through the origin of
slopes 1 and 3/7. By (15.9), the angle between them is

Alternatively, since here a = 3. 2A = — 10, 5=7, formula (15.21)


gives the required angle as
21 )
|
^ ^
EXERCISES 15 {d)

1. Two lines pass through the point of intersection of the lines 2x + iy ^ 1,


X 3y = 2. The passes through the point (2, 1) and the
first line
second is parallel to the line x 2y 3 0. —
Find the equations =
to the two lines.
2. Find the equations to the two lines which pass through the point
of intersection of the lines x -i- y = 2, 2.*’ — 3y + l= 0 and are at
unit distance from the origin.
3. The sides AB, AC oi a. triangle are the lines y = 0 and 3x y = 3.
B is the point (5, 0) and C the point (3, — 6). Find
(i) the equation to the line joining A to the middle point of BC,
(ii) the equation to the line through A perpendicular to BC.

4: Find the equation to a line which cuts off equal intercepts from the
coordinate axes and passes through the point of intersection of the
lines 5x -j-
y = 1 and 3.*' — 4y + 1 = 0.

5. Find the equation to a straight line passing through the point of


intersection of the lines 2x — y = 8, 4x + y = 10 and also through
the point of intersection of 2^ — 3y = 4, 4x + y = 3.
6. Find the angle between the two straight lines 2x* — 7xy + 3y* = 0.
7. Show that the two straight lines x* — 2xy sec 0 + y* = 0 make an
angle 0 with one another.
8. What two straight lines are represented by the equation
4x* — 24xy 4- Hy* = 0 ?
between these
Show that the equation to the bisectors of the angles
12y* = 0.
two lines can be written 12x* — 7.*y
)

15] EXERCISES 293

EXERCISES 15 (e)

1. Find the equation to a straight line which passes through the point
(3, 3) and forms with the coordinate axes a triangle in the first
quadrant of area 1$ units.
2. A{4, and B( — 2, — I) are two vertices of an isosceles triangle
7)
ABC having a right angle at B. Find the length and equation of
AB hence write down the equation to BC and find the coordinates
:

of the two positions of C.


(E U )
3. Find the equation to the line joining the two points
(1, 5/3)
(5/4, - 2) and the equations to the two lines parallel to
it and
distant 2 units from it.
U )
4. Find the equations to the two straight lines which
are parallel to
the line + 1 = 0 and at a distance 2 from it, and also
the areas of the triangles which these two lines
respectively make
with the coordinate axes. /r\ ^ \
5. The vertices of a triangle are A(3, B(- _
3). 1, i) and C(- 1, 5).
Fmd the angle made by the median from B with the side AC.
6. Find the equation to a line at right angles to the
Une(;r/a) - (v/L « 1
through the point where it meets the ^^-axis.
7. Find the coordinates o£ the foot of the
(1, 0) upon the line
perpendicular from the point
y =* mx + (1/m).
^
8. Write down the equation to the straight
line passing through me
gu the
two points ®
(3, 2). (6. 6).
Find where line cut by each of the two straight
is
dra^ through the pomt Unes
(1, 4) with slopes of 0-6 and -
IS the area of the triangle enclosed by
1. What
these three straight lines ?

9.
= ^C; the eqnatioS^t’o
^B and BC are respectively - -
Prove that
2;»r
y =, i and ;r 2v 4. i - o
.<4 C is parallel to the line 2^r + Hy
10 . A and B are the
0, (O C 1
points and
^mt whose ordmate
(2, 0) (4. 0) respectively P is a variable
:

is always positive, such that the angle APB


IS 45 . Show that P lies on the curve
+ y® — 6:r — 2y -f 8 = 0
11 . Find the equations to two straight
which make angles rf'S’
lines
^ = 21 and which pass th^h
12 .
St (l! -T tte

on the
perpendicular from the'S
W
(5, 7) line which 30ms the
points (6, - 1). n r) fO r
13. Fmd the equations to the lines through 1

the point (2 (\\


dicnlar to the lines ar -
3y + 8 = 0. * - 4y
^
‘o’
tte equation to the line joining
the feet of thes^e
perpendicula2,“to C
14 .
,
of the quadrilateral“so formed
S =T the origin to the linel

Si ??
fU.C.)
294 PURE MATHEMATICS [15
perpendicular to the line joining Ai2
JnH 3)

produced to R so that PQ = QR. Find


P and prove that the area the coordinates
of R
nf
of the quadrilateral A PER is 45.
(L-U.)
A A •

triangle is formed by the three straight lines

— a
y H
m , y — y = m^x +
m. m
Prove that its orthocentre always lies on the line X + a = 0.
(L.U.)
17. Find the equation to the straight line which is such
that the jr-axia
bisects the angle between it and the straight
line 2x 5y = 18.
(O.C.)
18. Prove that aU pomts on the line ll;r -
3y + 11 = 0 are equidistant
from the lines 12.r + ^
+ 12 = 0 and Sjt — 4y + 3 = 0. (O.C.)
19. Prove that the straight lines which join the point (—2,
3) to the
points (6, 7) and (0, — 1) are perpendicular. Calculate the coordin-
ates of the fourth vertex of the rectangle which has these points
as three of its vertices.
20. Two perpendicular lines are drawn through the origin so as to form
an isosceles right-angled triangle with the line /;r -|- my -|- n = 0.
Show that their equations are (/ — m);r + {/ + m)y = 0 and
(/ + m)x + (w - l)y = 0. (O.C.)
21. Find the equation to a line perpendicular to the line 3;r -f 4y -I- 5 = 0
which passes through the point of intersection of the two lines
3.r —y = 1 and x +y = 3.

22. Find the equations to the diagonals of a parallelogram who.se sides


are the lines 3^r — 2y = 1, 4jr — 5y = 6. 3^- — 2y = 2, 4;r — 5y = 3.
23. Show that the angle between the lines 6x* — ;ry — y* = 0 is 45®.
24. Find the equation to the bisectors of the angles between the lines
3;r* 4jy — 5y* = 0.
25. Express in a single equation the pair of perpendicular straight lines
through the origin, one of which is the line ax -f iy = 0.
CHAPTER 10

THE COORDINATE GEOMETRY OF THE CIRCLE


16.1. The equation to a circle with given centre and radius
Suppose that the radius of the and that its centre is the
circle is R
point C(a, b). The circle is the locus of a point which moves so that
its distance from the point {a, b) is always equal to R, Hence if P
is any point {x,
y) on the circle,
(x-a)^-\-{y~bY = R\ (16.1)
the left-hand side of this equation being the square of the distance
between the points (x, y) and (a, h).
Equation (16.1) therefore represents a circle of radius R and centre
the point (a, 6). By writing a = 6 = 0, we find that the equation
to a circle of radius R and centre the origin is

x^^y^ = RK (16.2)

Example 1. Write down the equation to the circle (t) with centre the origin and
radius 2 units, (ti) with centre the point (3, — 2) and radius 3 units.

(i) The required equation is x* -f- y* = 2*, or x* + y* = 4.


(ii) From (16.1), the equation is {x — 3)> -H (y -f 2)* = 3*. which can
be written in the form x* y* — 6x -h 4y -|- 4 = 0.
16.2. The general equation to a circle
The general equation of the second degree in x and y is

ax^ + 2hxy + fry® + + 2^ + c = 0, (16.3)


where a, b, c, f, g, h are constants. From the way in which the equa-
tion (16,1) to a circle was formed it should be clear that the coefficients
of and y* in its equation must be equal and there must he no term
containing the product xy. In other words, equation
(16.3) will repre-
sent a circle if a 6 and A = 0, =
There is no loss of generality in
taking a to be unity and the general equation to a
circle can be written

+ 2g^ -f 2yy -I- c = 0. (16.4)


The radius and coordinates of the centre of the circlegiven by
the general equation (16-4) can be found as follows.
Writing
® it in
the form

+ iy -h/)* =g" +/> - c.


and comparing it with equation (16.1). which
represents a circle of
radius R and centre the point (u. b), we see
that the radius of the
circle given by equation
(16.4) is

Vte* + /* “ c), (
16 6 )
.

295
2^6 PURE MATHEMATICS [16
and that its centre is the point

(16.6)

Example 2. Fmd the radius and the coordinates of the centre of the circle
Zx — iSy = 5.

Working from first principles, the equation can be written in the form
(^ + |)= + CV - 3)> =5+^+ 9
= ¥=(|)^
showing that the point {x, always at a distance of 9/2 units from
y) is
the point {— 5/2, 3). Hence the radius is 9/2 and the centre is the
point (— 5/2. 3).
Alternatively, in the given equation
g = 5/2, f = _ 3, c = — 5 and
(16.5), (16.6) give

radius = s/{(5/2)> + (- 3)» - {- 5)} = 9/2,


centre. (~ 5/2, - (- 3)) or (- 5/2, 3).

16,3. The equation to the circle whose diameter is the join of


the points y,) and yz)
In Fig. 112, A and B
are the points (x^, yi), (X 2 ^ 2 ) respectively ,

and P
is any point (x, y) on the circle. Since is a diameter, the AB

angle APB is a right angle. The slopes of AP, PB are respectively

and
X — Xi X — X2
Since AP, PB are perpendicular, the product of their slopes is — 1,

so that
=- 1,

or, {x - x,)(x - X2) + (y- yi){y - y^) = O-


of any pomt P
This relation is satisfied by the coordinates (x, y)
and therefore the required equation to the circle whose
on the circle is

diameter is the join of the points {x^, y^, {x^, >' 2).
)

16] CIRCLE THROUGH THREE POINTS 297

16.4, The equation to a circle through three given points


Suppose that the three given points are y^), {x^, y^), {x^, y^).
Let the equation to the circle through these points be

+ 2gx 4- 2/jy 4- c = 0.
Then if the point (x^, y-^ lies on the circle, these coordinates must
satisfy the equation and hence

+ yx^ -h 4- 2/yi + c = 0.

Similarly x^^ -f y^^ + %gx^ 4- 2/^^ + c = 0,

and ^3* + ^ 4- 2g;r3 4- 2/^3 4- c = 0.


3
^

These three equations are sufficient to enable the constants


/, g
and c, and hence the equation to the circle, to be determined.

Example 3, Find the equation to the circle which passes through the origin
and the points (2, 0), (3. — 1). (O.C.)
Let the required equation be
^* + y* + 2gx 4- 2/v 4 c « 0.
e ^ 0.
If the origin (0. 0) lies on the circle.
If the point (2, 0) is on the circle
(2)* + 2(2)g + c«0,
and if the point (3, - 1) is also on the circle.
(3)* + (- I)» 4 2{3)g 4 2(_ 1)/ 4c= 0.
Hence the equations determining
/. g and c are
c = 0. 4 4 4g = 0. 10 4 6g - 2/ = 0.
Thus c=i0, — 1, / =5,2 and the required equation is

+ y* — 2Ar 4 4>' = 0.
EXERCISES 16 (a)

1. Find the coordinates of the centre and radius of the circle


^ 4 y* - 10.r 4 12y =. 0,
^O.C.)
2 . Write do\vn the equation to the circle which :

(i) has its centre at the


point (-6, - 6) and whose radius
is 10.
(a, — b) and
(u) has its centre at the point
whose radius is V(«* — 6*)
3.

^
4.

6
.

.
“ "rS",
TST,
“,r

Find the equation to the diameter of the


which, when produced, passes
circle -i-ys _
through the point (8 -
’sz
gs
2) {O C )
7, Find the equation to the circle whose centre
and which passes through the points
Ues on the line y « Sat 17
(1, 1) and - (2, 1) (l U
298 PURE MATHEMATICS [16
S. If O the origin and P, Q are the intersections of the circle
is

+ 4^ 2y — 20 = 0 and the straight line — 7y + 20 = 0,


+
show that OP and OQ are perpendicular. Find the equation to the
circle through O, and Q. P (L.U.)

16.5. The equation to the tangent to a circle at a given point


Suppose that we require the equation to the tangent at the point
(^ 1 - yi) to the circle
2gx + 2/v + c = 0. (16.8)

Differentiating the equation with respect to x

2x + + 2/^ = 0,
dx dx
so that the gradient of the circle at the point {x^, is given by
(dy\
^ _ xi+g
The tangent the line through the point
is y^) with slope equal to
the gradient of the curve its equation ;
is

This can be written


xxi + vVi + g{x — ^i) H-/(y — >'i) = + yi^- (16.9)

Since the point {x^, yj lies on the circle

+ >' ^
1 + ^fyi + ^
+
and we can replace x^^ -j- y^ by — {^gXi + 2Jy^ + c). Hence equation
(16.9) can be written
xxi + yyi + g{x -f ^i) +/(>' + 3^i) +c (16.10)

point (^i, yj.


This is the required equation to the tangent at the
to the circle by replacing x"^
It can be obtained from the equation (16.8)
This is a par-
by XX,, y2 by yy^, 2x by {x -f- ^i) and 2y by (y
the equation to the t^gent
ticular case of a general rule which enables
down at sight for any of the curves in this
at (ati, yi) to be written
and the next chapter, x . - ^

particular, the equation to the tangent at (x^, yi) to the circle,


In
centre the origin and radius R, i.e. the circle x®
y" ,
is
+ =
xxi +yyi = (16.11)

the equation to the tangent (i) at the


point (1, - 7)
Example 4. Write down
the circle x* + y* =
to the circle
to 60, (*») at the point (2. 1)

^ H" 6y “ 23.

(i) The required equation is

;r(l) y(- 7) - 60. or x ly ^ 60.


16] INTERSECTIONS OF LINE AND CIRCLE 299
(ii) Here the required equation is

4z(2) + 4>'(l) - + 2) + f{> + 1) = 23,


or, 15^ + IZy = 43.

16.6. The points of intersection of a straight line and circle


Here we consider the points of intersection of the straight line
y = mx + c and the circle R^. The coordinates of the =
points of intersection of the line and circle will satisfy the simultaneous
equations

y = mx c.

Hence writing y = mx
the equation to the
-f- c in circle, the abscissae
of the points of intersection will be given by

+ c)8 = R\
-h {mx
(1 + m^)x^ + 2mcx -f c* — R* = 0. (16.12)
This quadratic equation has real, equal or imaginary roots according as
(2mc)2 - 4(1 + w2)(c2 - R%
is positive, zero or negative i.e., according as
; is less than, equal
to, or greater th^ R2(l + w®).
The three possibilities are shown in Fig. 113. The
three lines (a),
(6) and (c) all have slope m
and make successively greater intercepts
on the y-axis. The line (a) corresponds to a value of c* which is less

FTg. 113

than R2(l -f m^) meets the circle in two real points A,


it
B. The
(c), corresponding to a
value of c* which is greater than
does not meet the ckcle at aU, or
RHl 4*
rather, it meets the circle
two J
corresponding to a value of c*
K (1 w ), meets the circle of
+ m
two coincident points T it is tha
^ •
* is the
tangent to the circle at T,
PURE MATHEMATICS [16
The equation to the line for which = 2
i? (6) [i q, ^^ 2 ) j^g
VVTl

y = mx ± R\/{1 + »i2). (16.13)


These lines always touch the circle ^R 2
of them is ^ One
shown as line (Z>) of Fig. 113 the other is the
tangent to the circle :

whose point of contact is diametrically opposite


the point T.
The length of the chord intercepted by the circle on
the line (a) of
Fig. 113 can be found as follows. The abscissae of the end-points of
the chord are given by the roots of equation
(16.12). If the roots of
this equation are denoted by x^ and x^.

= — 2 tnc
X2
I -f
X1X2 = -

1 H-
Hence
{Xi ~ x^)^ = {Xi + ^ )* — ^1X2 2

4
(c2-R2)(l -f w2)}
(1 -f W2)2
4
(1 + W2)2
^{R*(l + m2)-c*}.
If y2 are the ordinates of the end-points we have, since the points
>'1,

lie on the line y = +


mx c,
yi—y2 = ^xi + — <^ ^ m{xi — Xi),
The square of the length of the chord
= + -Ts)’
= (1 +
4
1 w + «») - c»}.
Example 5. A coordinates of whose untre are both positive, touches
circle, the

both axes of coordinates. If it also touches the line 3;*r 4y -f 6 Q.find — =


its equation and the coordinates of its point of contact with this line.
(L.U.)

Let the required equation to the circle be x* + y* + 2gx + 2^ + c = 0.

Tlie circle meets the 2r-axis (the line y = 0), where


jr» + 254: -f- c = 0,

and touches it if This requires that


this equation has equal roots.
^3 _ c. Similarly the circle touches the y-axis if /* = c. Hence
c = =5 / 2 , and the equation to the circle can be witten
+ y^ + 2gx 2gy + g* 0. =
To find where the line 3;r — 4y 4- 6 = 0 meets this circle, we write

^ = {3;r 4- 6)/4 in its equation, giving


{3;r + — 6)*
2gx +
. ^ .
2^{33r 4- 6)
16
+
16] INTERSECTIONS OF LINE AND CIRCLE 301

or. 25;r« + 4(9 + Ug)x + 4(9 + 12^ + 4^») = 0. (16.14)


The line touches the circle if this equation has equal roots. The con-
dition for this is

+ Ug)^ = 4. (25). (4). (9 + 12^ + 4^«).


16(9
This reduces to 2f * — f — 3 = 0 giving g = — or 3/2. 1 The positive
value excluded because g must be negative if the centre (— g,
is
/) —
is to lie in the first quadrant. Hence the required equation to the
circle is obtained by using g =s — 1 and hence is

X* + y* - 2x - 2y 1 = 0.

With g —
the equation (16.14) giving the abscissae of the points of
1,
intersection of line and circle reduces to 25x* 20x -|- 4 = 0. This —
has two roots each equal to 2/5 (we should expect equal roots for the line
is a tangent). Hence the abscissa of the point of contact is 2/5 and,
since y (3x= 6)/4, the ordinate is 9/5.

Example 6, Awhich passes through the origin, cuts off intercepts of


circle,
lengths 4 and 6 toii/s on the positive x- and y-axes respectively. Find the
equation to the circle, and the equations to the tangents to the circle at the
points {other than the origin) where it cuts the axes. (L.U.)

2fy + c = 0.
Let the equation to the circle be -f y* -f- 2gx Since
the circle passes through the origin, its equation is satisfied by x =
0,
y = 0 and hence c » 0. The x-axis (the line y = 0) meets the curve
where
X* -t- 2gx = 0.
i.e., where x =
0 (the origin) and where x 2g. The second point =—
of intersection is at 4 units from the origin, so that
2g 4 giving — =
g =— 2. Similarly the y-axis (the line x == 0) meets the circle where

y* + 2/y = 0.
and this equation must have roots zero (the origin) and 6.
Hence
2/ =s — a giving / =
— 3. Hence the required equation to the circle is
^* +
y* — 4x — 6y =3 0.
The equation to the tangent at the point (4, 0) is
x{4) 4- y(0) - 2(x + 4) - 3(y -H 0) 0.
o*"* 2x — 3y =r 8 ; the tangent at the point (0, 6) is
x(0) 4- y(6) - 2(x 4- 0) - 3(y 4- 6) = 0.
or, 2x — 3y 4- 18 =» 0.

EXERCISES 16 (6)
Write down the equation to the tangent to the
circle
X* 4- y* - 4x — Cy 4- 3 = 0
at the point (6, 4) and that to the tangent to
X* 4- y* _ 2x - 3y 4- 3 « 0
at the point (1, 2).

(i) Write down the equation to the tangent


at the origin to the
2/y « 0.
circle X* 4- y* 4- 2gx 4-

(ii) Find the equation to the tangent at the point (3 21 to the


circle (^ - I)? 4* (y 4- 2)* « 20. .
302 PURE MATHEMATICS [16
Find the equations to those tangents to
which are parallel to the line
the circle a:* + v» = 25
~ Zy = 0.
Show that the line 2;t - 33/ + 26 = 0 is a tangent to the circle
^ +y - 4^ + — 104 = 0 and find the equation to the diameter
^
through the point of contact.
Find the equation to the circle which has its centre
qj ^
at the point
(2, — 1) and touches the line Zx + = 0.
y (O.C.)
Tangents are drawn to the circle
+ y^ - Qx ~ 4y + 9 =. 0 from
the origin. If 6 is the angle between them, find
the value of tan 0.

Show that y = mx is a tangent to the circle


2fy -he = 0,
*2 + ^2 4. 2gx +
if
{g +
mf)^ = c{l + m*).
Find the equations to the tangents from the origin to the circle
y^ — 6x — 3y -h 9 = 0 and the coordinates of their points of
contact. (L,U.j
Find the length of the chord joining the points in which the straight
line {x/a) -f (y/b) — 1 meets the circle x* -h y* = R*.

A touches the ;r-axis and cuts off a constant length 2a from


circle
the ^-axis. Show that the equation to the locus of its centre is
the curve y* ^ x^ = a*.
10 . The point (a, b) the middle point of a chord of the circle
is
x^ -h y* R*. Show that the equation to the chord is
ax + by a* =
bK +
16.7. The length of the tangent from a given external point to
a circle
In Fig. 114 r is a given point with coordinates (a, b) and TQ is

one of the tangents from T to the circle 2gx 2fy -f c + — 0,

Fig. 114

itspoint of contact being Q. If C is the centre of the circle, TQ is

perpendicular to the radius CQ and hence


TQ^ = TC^ - CQ\ (16.15)
16] ORTHOGONAL CIRCLES S05
Now T is the point (a, h), and from (16.6), C is the point {— g, —f),
so that
TC« = (a+5)2 + (i+/)2.
Also, from (16.5), the radius CQ of the circle is given by

Substitution in (16.15) then gives for the square of the length of


the
tangent
+ {b +/)2 _p _j_ c
= 2g(i + 2fb + c. (16.16)
Thus the square of five length of the tangent drawn to the circle from tlve
point (a,obtained by writing a for
b) is x aftd b for y in the left-hand
side of the equation to the circle.

Example 7. Find the length of the tangent from the point ^


(5, 1) to the
Circle {x — ^)a
+
The given circle is one of radius 6/2 with centre C at the point {I 01'
If T is the point - (5, 1).

Cr> - (5 - + (_ 1 _ 0)« =V+ = 1


V.
i/

Fig. 115

tangent from T, the angle CQT is QQ’*


^d he^t
TQ* = cr« - CG«.
But the radius of the circle CQ is 6/2, so that
.
re* =¥- ¥= 16 ,
givmg TQ = yi6 units.
.
,

AlUmatively^tbe equation to the circle rvritten


in standard form
r y X o M 0« Hcqcc (16.16) gives is

^G*=(6)*+(- 1)*-(5),8«16.
16.8, Orthogonal circles
Two circles are said to be orthogonal when the tangents
points of intersection are at at their
right angles.
+ +
304 PURE MATHEMATICS [16

Fig. 116

In Fig. 116 one of the points of intersection of two orthogonal


circles is P and 0^, 0^ are the centres of the circles. Since the tangents
to the two circles at P are perpendicular, the radii OjP, which are 0^
at right angles to the tangents, are also perpendicular. Hence the
angle 0^0^ is a right angle and therefore
= OiP2 + OgP^,
i.e.,the square of the distance between the centres of two orthogonal
circles is equal to the sum of the squares of the radii.
If the equations to the circles are

^2 _j_ ^2 ^ + c = 0,1
^2fy (16.17)
and 2g'x 2/> + c' = 0,J
the centres of the circles are (— g, —f) and (— g', — /')• The square
of the distance between the centres is

and the squares of the radii of the circles are and


g* 2 _|_ y '2 _ c'.
.
The circles therefore cut orthogonally if

i-g-h gv + {-/+/0* = g’*


+/" - c g'^
-\-n - c'.

i.e., if, 2gg' + 2//' = c 4- P. (16.18)

Example 8. Find the equation to the circle which passes through the origin
and cuts both of the circUs
_ 6;r + 8 = 0 and ;r> -f y* - 2;r - 2y = 7 orthogonally.

The equation to any circle which passes through the origin is

+ y* + 2gx + 2/y => 0.

This circle cuts the first of the given circles orthogonally if


2g(— 3) + 2/(0) = 8, i.e.. if 6g 8 —
or g =
4/3. = —
second of the given circles orthogonally if
It cuts the
2g(- 1) + 2/{- 1) = - 7.

This gives / “J — g=i + 3=V*


and the required equation to the circle is

X* +y* 0.

or, 3;r* + 3y> — 8^r + 29y = 0.


16] RADICAL AXIS 305

EXERCISES 16 (c)

1. Find the length of the tangent dra\Mi from the point (2, 5) to the
circle ~
3y = 1. 2x —
2. The length of the tangent from the point (I, 1) to the circle
+ * = 0 is 2 units. Find the value of A. (L.U,)
3. Given the three circles,
+y^ - +
IGx 60 = 0,
— +
I2x 20 = 0 .

X* - iGx - Uy + 84 = 0,

find the coordinates of the point such that the lengths of the tan-
(i)

gents from it to each of the three circles are equal, (ii) the lengtli
of each tangent. (L.U.)
4. Show that the circles jr* + >-* — 2a;r + c* = 0, -f ;/* — 2iy — c* =0
are orthogonal.
6. A centre C, cuts the circle
circle, -h 4 at right angles and =
passes through the point (1, 3). Find the equation to the locus of C.
(L.U.)
6. The line x +y =
3 meets the circle -I- y* -f ;r 5y 4- 4 0 at — =
points Q. PFind the equations to the tangents at P,
,
Q to the circle
and the coordinates of their point of intersection R. Find also
the
equation to the circle centre R which cuts the given circle
orthogonally.
(L.U.)

16.9. The radical axis of two circles


The radical axis of two circles is the locus of a point which moves
so that the lengths of the tangents drawn from it to the two circles
are equal.
Suppose the equations to the two circles are

2gx + 2/y + c = 0 and 2g'x 2fy 4. c' := q.


The square of the length of the tangent from a point (x, v) to the first
circle IS, by (16.16). ^ 2gx + 2/y +
while that of the tangent c.
irom the ^me point to the second circle is
It the point {x,
^2 2g'x 4- 2fy c' + 4 4
y) lies on the radical axis, these quantities are equal
so that ^ *

x^+y^ + 2gx + 2fyJr0^x^+y^ + 2g'x + 2/> + c' (16.19)


~ + 2(/ — /> + c - c' = 0. (16,20)

V't
It IS ^
of the ^t r
degree in * and
> it is a straight line.
and since
The equation (16.19) to the radical axis can
be written in the form
*2 -1-^2
4 + 2/^ -f- c - (x* 4 ^-= + 2g^x 4- 2fy + c') = 0.
In this form apparent that any point which lies on
it is
both the given
crclesjies also on the radical axis.
Hence the radical axis corresponds"
300 PURE MATHEMATICS [18
in the case of two circles which intersect in real points, to the line
through the common points.

Example 9. Find the equation to the common chord of the two circles

+ \0x + 8;' 4- 32 = 0 and — 4j; — Gy 12 = 0


and show that this line is perpendicular to the line of centres of the circles.

The equation to the common chord (or radical axis) is obtained by equat-
ing to zero the expression corresponding to the difference of the left-hand
Sides of the equations to the circles. In the case of the circles given here
it is

lAx + 14_y + 20 = 0, or lx + ly + \G = 0.

The coordinates of the centres of the two given circles arc respectively
(“ — 4) and (2, 3). The slope of the line of centres is therefore
3 -(-4)
2 - (- 5)
while the slope of the common chord is — 1. The product of these
two slopes being —
1, the common chord is perpendicular to the line of
centres.

16.10. The circle through the points of intersection of two


given circles
Consider the equation
^2 +^2 2gx + 2/^ -f c -f X{x^- + 2g’x -f- 2fy + c’) = 0, (16.21)

wliere X is Since this is an equation of the second


any constant.
degree in which the coefficients of x^ and are equal (they are both
1 + and there is no term in xy, it is the equation to a circle. Further,
the equation is satisfied for points whose coordinates (x, y) simultane-
ously satisfy the equations •

y^ + 2gx + 2/y + c = O.'l


22 )
x^-hy' + + 2fy + c' = O.J
Such points are those which are common to the curves represented by
equations (16.22). Hence equation (16.21) represents a circle passing
through the points of intersection of the circles given by equations
(
10 22 ).
.

in
. ...
mentioned
The foregoing another example of the device
is

15:8. Equation (16.21) represents a family of circles, aU of which


§
pass through the common points of the circles given by
(16.22).

of this family correspond to different values of /.


Individual circles
wiU be required to find the equation to a circle which,
Generally it
through the intersection of two given circles, fulfils
besides passing
condition such as passing through another given point
some additional
given A further condition of this sort enables
or touching a line.

of X to be fixed for the particular circle required.


the value
16] EXERCISES 307

Example 10. Find the equation to the circle through the origin and through
the points of intersection of the circles
4- _ 2^ — —4= 0 and + 8^ — 4y + 6 = 0.
(L.U.)
Any circle through the common points of the given circles is

+ y* - - 4y - 4 + A(x2 4- ya 4- gjr - 4y + 6} = 0,
or, (1 + X){x^ + y») 4 (8A - 2)x - {4A 4- 4)y + 6A - 4 = 0.
If this circle passes through the origin, its equation must be satisfied
by ;r as 0,
y = 0, so that
6A —4= 0
giving A = 2/3. With this value of A the equation is

(1 + f)(^" + y*) + (Y - 2)^ - (1 + 4Jy = 0,


i-c., + y* + 2.ar — 4y = 0.
EXERCISES IG {d)

Find the radical axis of the circles x*


5y = Q and
1.
ix
+ y* + 5.« + 4y = 9.
2. Find the equations to the common chord and the line of
centres of
the two circles
+ y“ + 6.r - 3y + 4 =0
and 2a' + 2y* - 3 a: - 9y + 2 = 0.
3. Show that the length of the common chord of the circles
X* + ^ + c = 0,
y* ax
+ y* + OAf + ay + c = 0
is
{H« + 6)»-4c}l.
4. Show that the circles Ar*+y* + 4Ar+y = 3, x*+y*~-x—
and Af* + y* + 14Ar + ^ = 7 are co-axial (i.e., they y = l
have the same
radical axis).
6. Find the equation to the circle which passes
through the point
{1, 2) and through the points of intersection of the
circles
4- y» 4. 2Ar + 3y = 7 and at*
-f y* 4- 3;r — 2y = 1.
6. Find the equations to the circles passing
through the points of
intersection of the circles
^* + y* - ISat - 2y -f 8 = 0.
Af* + y* - 26Ar 4- Gy = 24
and touching the straight line y * 10.
(L-U.)

EXERCISES 16 (n)

1. (i) Find the radius and coordinates of


the centre of the oirole
A^* 4- y» _ 2Ar -
6^ 4. 0 «= 0.
Ifthe line A?
4-y* - Sat J- <w _ 15 = 0
(ii)
2y meets the circle
at the points P.
Q, find the coordinates of
equation to the circle passing through P,
and Q and the P
Q and the point l). (1,

2.

from -the point (3, 4) is proportional


^ its diaS
to its distance from the line
308 PURE MATHEMATICS [10
^ + ;y = 0, one point on the locus being the point (1, 2). is a circle
and find its centre and radius. (O C.)
3. Find the equation to the circle of which the points (8, -
2) and
(—2, 6) arc ends of a diameter. {O.C.)
4. Find the equation to the circle which passes through the points
2, 2), (2, 4), (5, — 5). Show that this circle touches the circle
2;r2 + 23^2 _ \~x + l6y + 65 = 0 at the point (5, - 5). (L.U.)
5. Show that the circle ^ 2ax — 2ay + = 0 touches the
axes of X and y.
Also find the equation to the circle which passes through the
points (2. 3), (4, 5), (6, 1). (Q.E.)
6 . Find the equations to two circles which touch the ;r-axis at the
origin and also touch the line \2x + 5y = 60. (L.U.)
7. Find the coordinates of the centre and the radius of the circle
x^ + y^ —
2x ~ Sy + I = 0. Show that this circle touches the
;f-axis and that the point {h, h) of contact of the otlier tangent
from the point (3, 0) must satisfy the condition h — 2k = 1. (L.U.)
8 . A circle of radius 5 centre in the positive quadrant, touches
has its
the A'-axis and intercepts a chord of length 6 on the y-axis. Show
that its equation is x^ + y^ — 8-r — lOy + 16 = 0. If y = mx is
a tangent from the origin apart from the A-axis, find m. (L.U.)

9 . Prove that the points (3/2, 6), (— 9/2, — 2) lie on the circle

4 a2 + 4y 2 -j- 12 a — lOy — 76 = 0, and that the tangents at these


points arc parallel. (O.C.)

10 . Find the equation to the tangent at the point (3, — 4) to the circle
x 2 ^ yi — 25. W’hat are the equations to the two tangents parallel
to the y-axis ? Show that the first tangent intersects these two in
points which subtend a right angle at the origin. (O.C.)

11 . Find the equations to the two circles each of which touch tlie three
circles a 2 -f- y* = 4a2, a* + 2^3A -f y* = 0. a*
- 2aA -f y* = 0.
(Q-E.)
the pomt
12 . Find the equation to the circle touching the A-axis at
(5, 0) and passing through
the point (7, 4). What are the coordin-
the tangent at
ates of the point on the circle other than (5, 0).
(L.U.)
which passes through the origin ?

in the first
13. Prove that the equation to the circle whose centre lies
passes through the
quadrant, which touches the A-axis and which
y® - 24 a - 30y + 144 = 0. Find
]>omts ^(0, 6), R(0, 24) is a* H-
equation to the other chord through the origin whose
length
also the
AB. (L.U.)
is equal to that of the chord
the coordinates of the centre and the radius of the circle
14. Find
q. 2y2 + 4 a - 12y + 15 = 0. Find also the equation to the
the origin. Calculate
tangent to the circle which is furthest from
length of the chord intercepted by the circle on the line a + y = 0.
the

that touch the circle +>’’ = •*

15 Find tlie equations of the circles


.

and the straight lines y = 0 and a = 7. («• ^'1


EXERCISES 309
16. Verify that the circle - ly \2 0 passes through =
the point (2, 0) and calculate the coordinates of the other points
of intersection of the circle with the axes. Find the equation to
the tangent to the circle at the other end of the diameter through
(2, 0). Calculate the length of a tangent to the circle from the
point (9. 2).
17. Show that the line x Zy = \ touches the circle
+ 3'® - 3^ - 3y + 2 = 0
and find the coordinates of the point of contact. Prove, by cal-
culation, that the point P{3. 2-5) lies outside the circle
and calculate
also the length of the tangent drawn to the circle from
P. (L.U.)
Find the length of the tangents from the point
(5, — 4) to the circle
IS.
X -f y* 8 = 2k{x -\- y 1). IC in this equation k be given
various values, show that all the circles so obtained
have the same
radical axis.
19. Find the equation to the circle which cuts orthogonally the circle
+ y* — 4;r 4- 6y - 7 = 0, passes through the point (0, 3) and
touches the ;r-axis.
20 . -4(3. 0) and B(0.
are two fixed points. Find the equation
2)
to
the locus of a point P
such that 2PA = ZPB. Show that the locus
IS a circle passing through the origin
and find the equation to the
tangent at the origin. Show also that this circle
cuts at right angles
the circle + y* 3.r 2y 0. - - =jL y.)
Show that the circles
-t-y* + - 2y - II = 0 and + y* - - =
a:* 4r 8y + 11 0
intersect at right angles and find the length of their common chord.
(L.U.)
Prove tliat the circles
4- y* + 2;<r - 8y 8 « 0 ,

4- y* 4- lOx — 2y 4- 22 = 0
touch one another. Find .(i) the point
of contact, (ii) the equation
to the common tangent at this point,
(iii) the area of the
taiangle
enclosed by this common tangent, the
Une of centres and the y-ajds.
I'- j .V (L.U.)
23. Find the equations to two circles
which pass through the ooint
(4. 1) and touch boUi the lines * =
6. = 6. Prove tirat tte /
equation to the common chord of
24.
_ the circles = 6.is » +
(
through the point - 2. _
and through the two points of intersection 0)*
of the circle
- 3* + 4y - 2 = 0 and ** y*
25 F-
+ + 6* - 3y = 8.
. Find the radius and the coordinates
of the centre of a circle which
of the circles +y«
a^T» - o''®
^ = 0 “d 'vWch touches the str^ght Une
Tl 2y to.
CHAPTER 17

THE PARABOLA, ELLIPSE AND HYPERBOLA


17.1. Introduction
The equation of the second degree in x and y represents, in certain
circumstances, a pair of straight lines or a circle. For example, if
the equation contains only terms in x^, and xy it represents a pair
of lines through the origin (§ 15.9), while, if the coefficients of the
terms in x^ and y^ are equal and there is no term in xy it represents a
circle (§ 16.2). Other curves can be represented by an equation of
the second degree and these are the subject of the present chapter.
Consider the locus of a point P which moves so that its distance
from a fixed point (the focus S) is always in a constant ratio (the
eccentricity e) to its perpendicular distance from a fixed straight line
(the directrix AB), We shall show in the sections which follow that
the locus is given by an equation of the second dcgi*e in x and y.
A plot of the locus reveals that its shape, which depends of course
on the eccentricity e, changes significantly when e passes through
unity. The locus is called a parabola when £ = 1, an ellipse when
£ < 1 and a hyperbola when £ 1. >
can be shown, but we shall not attempt to do so here, that the
It
parabola, ellipse and hyperbola are, like the pair of straight lines
and
circle, all sections of a right circular cone by a
cutting plane. For

this reason, the equation of the second degree is


sometimes said to
represent a conic section.

17.2. The equation to a parabola

equation to a parabola takes its simplest form


when we take
The
and the directrixyB as the hne
the focus S to be the point (a. 0)

X = — a If in Fig. 117, P is the point {x, y) and is drawn PM


curve on which the point P
perpendiMlar to AB. the parabola is the
lies when it moves so that the distance PM
is always equal to the

and PM
C is the point (— a, 0)
If the line AB meets the :r-axis at C,

sum of the abscissa of P and the length CO. Hence


s clearly the
P S respectively the pomts [x, y)
^x + a, and since and are

and (a, 0),


PS2 ^(x- a)2 H-y*.
Since PS = PM, we have
[x-aY-Vy^ (X + a)\
^ax, (17.1)
or,
310
171 THE PARABOLA 311

and this is the required


equation to the parabola with focus the point
{a, 0) and directrix the line
x a 0. =
To trace the curve, we first observe that
y is imaginary when at
• • _ no part of the curve to the left of the
origin 0. If y be zero, so also is a;, so the
x-axis meets the curve only
at the origin 0. This point is caUed the vertex of the
parabola. If
be zero, =
0. showing that the >--axis meets the
curve in two
comcident pomts at the vertex i.e., the y-axis is the tangent
;
to the
parabola at its vertex. For every
positive value of x. equation (17.1)
shows that y h^ two equal and opposite
values, so that corresponding
to any point P
on the curve there is another point
P', the image of
possesses
svmmetr^'^
symmetry about
h*’ t the x-axis, andthis line is often referred to
as the
Finally, as x increases in magnitude
and th so docs y
“1 the diagram.
The double ordinate LSL' (Fig.
117) drawn through the focus S
IS knoTO as the latus-rectum of the parabola.
Since LS is the value

knrth of the latus-rectum of the


length
-d hencet:
parabola = LSL' = 2LS = 4a.
parabola at a given point
Differentiatmg the equation y» = 4ax with respect to x

2,1 - 4 .,
•o thM th. gradient of the prabola
at the point
(«„ ,J 1, riven by
/M 2a
312 PURE MATHEMATICS [17
The tangent the line through the point
is {x^, yj) with slope equal to
the gradient of the curve its equation ; is

2a
{x - Xj) f

yi
OT,
yy^ — _ 2ax — 2aXi.
Since the point on the parabola, y^^ can be replaced by
[x^, Vj) lies
4aAri, and the equation to the tangent becomes
= Mx + x^). (17.2)
It should be noted that equation (17.2) can be written down by observ-
ing general rule that ihe equation to the tangent at the point (Xi, y^)
tlie

is obtained from the equation to the curve by replacing y^ by


yy^ and
2x by {x +
JTi).

normal to a curve at any point P is the straight line which


'I'he

passes through P and is perpendicular to the tangent at P, Since


the slope of the tangent at the point (^i.yi) to the parabola is2a/yi,
that of the normal is —
yJ2a and the equation to the normal is
therefore

- - **)• (17.3)

Example 1. Find the equations to the tangents and normals to the parabola
= ICx at the points (16, 16), (1, - 4). intersect at the
The tangents
point T and the normals intersect at R. Prove that the line TR is parallel
(O.C.)
to the axis of the parabola.

Here 4a = 16, so that a = 4. For the point (16, 16). = y\ = 16

and equations (17.2), (17.3) give for the tangent and normal
16y = 8(;r + 16),
and y — IQ = — ^^(x — 16)
forms
respectively. These equations can be written in the simpler
_ 2y + 10 = 0 and 2x + y — 48.
For the point (1, - 4). ;ri = I, yi = - 4 and
equations (17.2), (17.3)

give, after a little reduction, the


equation to the tangent as
2.r y 2 0 + + =
and that of the normal as ^ — 2y = 9.
are T
given by solving the simultaneous equations
The coordinates ot
= - 4.
*• _2v 4- 16 =
0, 2 x y 2 + + =
0, and are found to be ^
of
= The coordinates of R are similarly found, from the solution
r 6.
- =
the simultaneous equations 2x y 48, x 2y =
9, to *

R height 6 above the x-axis and the line


V =6 Thus both T and are at
the
to the ;r-axis which is also the axis of
TR is therefore parallel
parabola.

intersection of a straight line and parabola


17.4. The points of
points of intersection of the straight line
The coordinates of the
c +
and the parabola y- iax are =
the values of a: and y
^
17] INTERSECTIONS OF LINE AND PARABOLA 313

which simultaneously satisfy both equations. Writing y mx c in =


the equation to the parabola, the abscissae of the points of intersection
are therefore given by
{mx q- c)® = 4ax,
or, viV H- 2(mc — 2a)x -b = 0. (17.4)
This quadratic equation has real, equal or imaginary roots according as
{2{mc ~ 2a)Y ~
is positive, zero or negative ; i.e., according as c is less than, equal to,
or greater than a/m.
The three possibilities can be illustrated as was done for the circle
in Fig. 113. When c <a/m, the line intersects the parabola in two
real points. When c'> a/m, it does not meet the parabola at all,
or ^ther, it meets the curve in two imaginary points. If c = a/w,
the line touches the parabola.
Substituting c =
a/m in the equation y = mx c io the line, we
find that the line

y^mx-\-t, (17.5)
m ' '

touches the parabola y- = ^ax for all values of m.


Example 2. Find the ordinates of the points in
=
which the line ;r + 2y c
meets the parabola y* IOt, = and find the value of c when this line is a
tangent to the parabola.
(q q \

Since we here require the values of


y at the points of intersection, we
substitute x = c ^
2y from the equation to the line in the equation to the
parabola. The ordinates of the points of intersection arc
then given by
y* = I0(c — 2y),
y> + 20y - 10c := 0.
— 20 ± V(400 + 40f)
y ^ ^=^-10±^/^10(10 + c)}.

The line touches the parabola if this quadratic in


y has equal roots
This IS so when (20)* = 4(- 10c), i.e., when c « - 10.

Example 3. Show that the point of intersection


of two perpendicular iangenis
to a parabola lies on the directrix. *

tnx +
^'*'***'5
y^1/w m
ia/m) is a tangent to the parabola
place of tn. the perpendicular
line
1 “ u
the abscissa
^•
r ^ tangent to the parabola. By
subtraction
of the point of intersection of these two lines
is given by

(”‘+i)^ + £ + “’" = 0'


i.e., by X +a= 0, and this is the equation to the
directrix.

EXERCISES 17 (.i)

1. Find the equation to the normal to the


parabola
r v»
y Sx at the
point (4-6. - 6). .
(0,C,)
314 PURE MATHEMATICS [17
Find the equations to the tangents to the parabola y* = 144;r at the
points (144. 144), (9, — 36), and show that they are perpendicular.
Find also the coordinates of their point of intersection. (O.C.)
Find the coordinates of the points in which the line y = 8;r — a
meets the parabola = iax. Find the equations to the tangents
to the parabola at these points and the coordinates of their point
of intersection. (L.U.)
Find the equation to the normal at the point P(l, 2) to the parabola
y _ 4^
2
normal meets the ;r-axis in G and is the mid-point M
of PG. A line through M
parallel to the y-axis meets the ;r-axis in
N and the parabola in Q. Prove that QN = PG. (L.U.)
5. A point P moves so that its distance from the line y = 2 is equal
to its distance from the point jr = 2, y = 4. Find the equation to
the locus of P and the equations to the tangents through the origin
that touch the locus of P.
C. Find the angles at which the parabolae y* = ax and x* = by intersect,
(QE)
7. If the tangent at the point P to the parabola y* = iax meets the
parabola y^ = ia{x b) at points Q and R, prove that P is the
middle point of QR. (O.C.)

8 . Prove that the normals to the parabola y* = iax at its points of


intersection with the straight line 2;r — 3y -f 4a = 0 meet on the
parabola. (O.C.)

17.5. The parametric equations to a parabola


Tlie point {x^, y^) lies on the parabola y*the relation = 4a^ only if

yj
2 _
4aXi between its two coordinates is satisfied. It is often con-
point which
venient to be able to write down the coordinates of a
coordinates
always lies on the parabola. Such a point is one with
{ai^, 2at) for it is clear that if

^ = ai\ y.= 2at, (17.6)

then, y2 =
(2a/)2 4a(aP)= = iax
for all values of t The equations
are called the parametric equations to the parabola y^
(17.6)
the coordinates of a point on the curve in terms of the
They express
point with coordmates
parLieter t, and we shaU. for brevity, refer to the
" i”,
(at^, 2ai) as the point . o « . \ (at^
^ ^ 2 o.m ^

the points (at^~, 2at^, ^at^)


The equation to the chord joining ,

on the parabola is, by equation (15.5),

X — at^^ y— 2fj/i

at^ aU 2aL — 2ati

which reduces to n7
2x — (^1 -f- ^ 2
v .

)^' "f"
a j
— a0. ( •
7l1

parabola coincide
If we write = i. so that the two points on the
1

171 IMPORTANT PROPERTY OF PARABOLA 315


at the point {at^, 2at), the chord (17.7) becomes the tangent at this
point, and its equation reduces to
x~ty + at^=-- 0. (17.8)
Alternatively, the equation to the tangent at the point “ t" can
be found as follows. The parabola can be written

X = at-, y = 2at,

and the gradient at any point is

dy _ {dy/di)
_ 2a _1
dx [dx/dt) 2ai T
Hence the tangent is the line passing through the point (at^, 2ai) with
slope \/t and its equation is

y-2at= !(* - at~),


or, ^ — /y -f a/2 = 0
as in (17.8). The normal at the point t is
the line through the point [at^, 2at) with slope
and its equation -
is therefore

y - 2a^ = - t{x - at\


y ix = 2at at\
(
17 9 )
.

Many problems onthe parabola are best solved by using the


parametric equations. The above equations to the
chord, tangent and
normal are very useful in such work. The student
should either
remember them or (preferably) be able to derive them quickly.

Example 4. Show that the tangents to a parabola at the extremities of a focal


chord are at right angles to each other.
If we take the parabola as y* = 4ax,
and the extremities of the chord
as the points “ ",
" ", the equation to the chord is, by (17.7),
2^ - (h + ti)y 4- = 0.
If the chord passes through the focus, the
point (a, 0), the above
equation shows that 2a +
=. 0. or 1 But equation = - .

(17.8) shows that the slopes of the tangent


at the extremities of the
chord are l/q and 1//,. Since =
- 1, the product of the slopes is
1 and the tangents are therefore
at right angles to each other.

17.6. An important property of the parabola

foe Jr IS
focus iff' PT
= 4"* 'Vhose
S. the tangent at
IS P
meeting the *-axis at Q. Since
the coordinates of P
and S are respectively (at*, 2a/) and (a,
0)
= (a/s - a)2 •+ 2a/ - 0)» = a*(/* - 2 -u 1 4-r-u)
PS 2
( 4 /^

= + 1)2,
fl«(/*

.
/>l

so that PS = b(/
+ The equation to the tangent PT
2 1).
from is
quation (17.8), x-ty
+ = 0. This meets the *-axis ’where
ai<‘
316 PURE MATHEMATICS in

X ai^ = 0, so thatQO = ai^. Since OS = a,


QS = QO + OS = ai^ + a = a(t^ + 1).
Hence QS = PS and the triangle QSP is isosceles. Hence
angle PQS == angle SPQ,

If PM is drawn through P parallel to the A;-axis, it follows that

angle TPM = angle PQS = angle SPQ,

so that the lines PS and PM are equally inclined to the tangent at P.


If PN is the normal at P, it follows immediately that the lines PS and

PM are equally inclined to the normal at P.


It means that if
This is an important property of the parabola.
starting from the focus 5 of a parabolic mirror strikes
a ray of light
P. the reflected ray, which makes an equal mgle with
the mirror at
be parallel to the axis of the mirror. Since P
the normal, will
focus
incident rays from a source at the
any point on the parabola, all
axis.
as rays which are all paraUel to the mirror s
will be reflected
parallel rays from a distant source striking such a mirror
Conversely,
pass through the focus. Motor-car head-
will all be reflected so as to
fires are often designed so as
to
lamps, searchlights and some electric

make use of this property.


EXERCISES 17 (6)

the points (a/.*, 2al ). (at,y on the parabola


If the chord joining
1
the focus 0). find , -
terms of /. P0
= 4«;r passes through («,
to the a-xis of tee
is a focal chord
and PL. QM
are perpendicular

parabola. Prove that PL.QM is constant.

of parallel chords of a parabola, prove that


o If PO is one of a series
mid-point of PQ always on a line parallel to the axis of
lies
lire
parabola.
17] THE ELLIPSE 317
3. P the point (nP, 2at) on the parabola y* = 4ax.
is I'rom a fixed
point Q{h, k) a line is dra^vn perpendicular to the tangent at to P
meet, at R, the parallel through to the x-s^xis. P
Find the equation
to the locus of R. (L.U.)
4. A linefrom the point (2. 0) perpendicular to the tangent at the point
(2/*, 4/) to the parabola y* = 8x meets that tangent at the point

(/j, k). Express h and k in terms of / and deduce the equation to


the locus of the foot of the perpendicular from the point (2,
0) on
to any tangent to this parabola. (L*U.)
5. P is the point and Q the point [at*. 2aU) on the parabola
2ai^)
^ tangents at P and Q intersect at R. Show that the
area of the triangle PQR is (L.U.)
6. P isthe point {aP, 2al) on the parabola y* = 4ax, If is the PN
perpendicular from P
to the ;ir-axis and M
is the point where the
normal at P
meets the ^-axis, prove that the distance is inde- MN
pendent of /.
(L )
U
7. P is a point on a parabola whose focus is
5. D
is the foot of the
perpendicular from P
to the directrix. Show that the tangent to
tlie parabola at P
bisects the angle SPD. (L.U.)
0 . The normal to the parabola y» =
4ax at the point P{aP. 2at) meets
the axis of the parabola at G and GP is produced,
beyond P, to O
so that GP = PQ. Show that the equation to the
locus of O is
y* ^ 16a(.*r + 2a).
{L.U.)

17.7. The equation to an ellipse


The equation
to an ellipse of eccentricity c (less than unity)
takes
Its simplest form
when we take the focus S as the point (- «£ 0)
and the directrix as the line x a/e. In Fig. 119, =- is the P
A'

C'

5 *

point and P.l/


^ is perpendicular to AB. The ellipse is the curve

meets the *-axis at C. C is the


and PM point (- a/e 0)
is the sum of the abscissa of P and the len^h CO. Hence
318 PURE MATHEMATICS [11
PM — X -{ (a/e), and since P and 5 are respectively the points (x y)
and {— ae, 0) •

P52 = + a£)2
Since PS — e.PM,

or. (1 - -\-y^ = a^l - e^).

This can be written


X
- ^ a2(l - £2) = 1 .
^2
and writing, = a^(l — e^), (17.10)
the equation to the ellipse becomes

(17.11)

To trace the curve we observe that, since only even powers of


X and y occur, the curve is symmetrical about both the coordinate axes.
From this symmetry we can deduce the existence of a second focus S'
at the point (ae, 0) and a second directrix A'B' along the line x = a/ e.

The curve cuts the ^-axis where (x^/a^) = 1, i.e. in the points (± 0)
and it cuts the y-axis where (y^/b^) — 1, i.e., at the points (0, ± b).

These points are denoted by H, H\ K, K' in Fig. 119. By writing


equation (17.11) in the form

it is clear thaty* is negative, and therefore there is no part of the curve,

for values of x greater than a or less than — a. Similarly, there are

no parts of the curve for which y is greater than b or less than b.

The general shape of the curve is shown in the diagram.


The points H, W
are called the vertices of the empse. The lines
HH' and KIC are called its axes since e < 1, equation (17.10) shows
;

that and the axes HH\ KK' are referred to respectively as the
b<a,
the ellipse,
major and minor axes. The origin 0 is called the centre of
The
and a chord passing through the centre is called a
diameter.
latus-rectum and
double ordinate LSV through the focus 5 is the
be a second latus-rectum through the second focus 5.
there will
Since LS is the value of y when x
— — ae in equation (17.11),
“ -
‘ji'-
= b^/a,
Thus the latus-recttfm is of
when use is made of equation (17.10).
length 2b^/a.
171 TANGENT AND NORMAL TO ELLIPSE 319

To sum up, the curve


X“ V*

is an ellipse of eccentricity e, less than unity, given by


£2 = - 1 (
62 /^ 2
). (
17 12 )
.

The foci are


the points (± tie, 0), the directrices the lines x =± (a/e),
the semi-axes a, b and the centre the origin of coordinates.

Example 5. Show that the sum of the focal distances of any point on an
ellipse is equal to the length of the major axis. Deduce a simple mechanical
method for constructing the curve.
I

Using Fig. 119 and the dednition of the ellipse, if X is the abscissa of P,

PS = e.PM = -f = a + tr.

Similarly, if PM' is drawn perpendicular to the second directrix A'B',

PS' = €.PM' = - x) = a _ er.

The sum of the focal distances PS. PS' is therefore equal to 2a, the length
of the major axis.
By 6xing rivo pins at S, 5' and keeping stretched by a pencil point
an
endless piece of string passing round the two pins, the pencil
will describe
an eUipsc with 5, S' as foci.

17.8. The tangent and normal to the ellipse at a given point


Differentiating the equation (17.11) to the ellipse with
respect to x,
^x 2ydy_
a
so that the gradient of the ellipse at a given point {*i, _y,) is given by
_ b’‘xi

\dx)x~z^ a^y.
The tangent at the point the line through this point with
{x., y.) is
slope equal to the gradient of the curve
; its equation is

^^XX, + a^y, =
Dividing by a’‘b‘, and using the relation
IS the condition that the
KVa”) which + (y, = 1
point {x„
y.) lie on the eUipse, the
shall equa-
tion to the tangent at the point
[x„ y.) can be written

^+fr = l-
(17.13)

Again it should be noted that th« equation to the tangent at the point
'

320 PURE MATHEMATICS [17

(Xi, can he obtained from the equation to the curve by replacing x'^,y^
yy\ respectively.
The normal at the point (jtj,
y^) is the line through this point at
right angles to the tangent. Its slope is therefore and
its equation is

y - yi = r-j ' ^
This can be written in the more symmetrical form
^ __ r - Vi
(17.14)
ar,/a2 y^/b^

Example 6, Find the equations to the tangent and normal to the ellipse
5-s’* 4- 3y^ = 137 at the poifit in the first quadrant whose ordinate is 2.

The abscissae of points on the ellipse at which the ordinate is 2, ar:^


given by WTiting y =
2 in the equation to the ellipse. This gives
5x^ =
137 —
3{2)*. or .r 5. = ±
The point in the first quadrant with
this ordinate is therefore the point (5, 2). The equation to the ellipse
gun be written in the form

(137/5) (137/3)
so that a* =
137/5 and 5^ 137/3. = Equation (17.13) then gives lor
the tangent at the point (5, 2),

(137/5) ^ (137/3)

or, 25.r + Cy = 137. The normal is given by equation (17.14) as


X - 5 y - 2_
6/(137/5)
“ 2/(137/3)'

i.c., 6x - 25y + 20 = 0.

of a straight line and ellipse


17.9. The points
of intersection
the points of intersection of the straight line
The coordinates of

y = c and +the ellipse (3'V6“) 1 are the vdues of


+ =
simultaneously satisfy both equations. Wntmg
x: and y which
the abscissae of the pomts
y =
,„x +
c in the equation to the ellipse,

of intersection are therefore


given by
*2 ,
(h« + c)“ _ ,

b-

or, (a^m^ + b^)x‘ + 2 a‘»icx + a^c- - b‘) = 0. (17.15)

as
quadratic equation has real, equal or imaginary roots according
Tl'iis

{2ah)icy - 4{ahn^ + b^)a\c^ - 5 “)

according as c* is less, equal to or


positive, zero or negative ;
i.e.,
is

was done for the


be illustrated as
^‘^Agak "heireetossibilities can
THE ECCENTRIC ANGLE 321

circle in Fig. 113. When c- < the line intersects theellipse -f*
in two real points. When + 6^, the line intersects the
ellipseonly in imaginary points. If c® = the line is a
tangent to the ellipse.
Writing c —
^/{aHx- +
b^) in the equation ^ = ;«;c + c to tlie line,
we find that the Line
y = mx-\- ^{ahn^ + ^.
2
)^ (17.16)
always touches the ellipse. Further, since the radical sign on the
right-hand side of equation (17.16) may have either positive or negative
signs attached to it, we see that there are two tangents to the ellipse
having the same m. In other words, there are two tangents
parallel
to any given direction.

Example 7, Findthe locus of the point of intersection


of tangents to an
ellipse which are at right angles to one another.

Taking the eUipse in the usual form (17.11), the line

^ = mar + ^/(ahn* + *>),


always a tangent. A perpendicular tangent
IS
is obtained by replacing
^
m by 1/m, and its equation is

These equations can be vo^itten

y — mx + 6*),
my + X = ^/{a* + 6*m*).
The coordinates of the point of intersectionof the tangents
simultaneously
satisfy these two equations. If therefore we eliminate m between
the
equations we shall obtain the locus of the
point of intersection. Sauar-
ing and adding we find ^
(1 + m*){x* +y*) =» a*m« + b* + a* +
o**- .ar*
+ = a* + 6*.
The required-locus is therefore a circle with centre
coincident with the
centre of the ellipse and with radius
the director circle.
V(a* i-»). +
This circle is caUed

17.10. The parametric equations an


to ellipse. The eccentric
angle
As with the parabola, it is often convenient to express the
coordin-
variable. It is easy to
“ satisfied by a point whose coordinates
arr^ven by'**'

x = acos^, y = 6sm<^. (17.17)


322 PURE MATHEMATICS [17

In Fig. 120, 9 is a point on the circle wliose centre coincides with


the centre 0 of the ellipse (x^/a-) 1 and whose diameter + =
is equal to the major axis (2a) of the ellipse. Such a circle is called
the auxiliary circle of the ellipse. The line QP is drawn perpendicular
to the ;t-axis to meet the ellipse at the point P shown. If the angle

QOx =
<f>,
it is clear that the coordinates of Q are (a cos a sin <j>).
The abscissa of P is equal to that of Q and so is also a cos Substitu-
tion of this value of x in the equation to the ellipse gives the corre-
sponding value of the ordinate to be h sin ^ and hence P is the point
(a cos 4>. b sin </>). The angle defined in this way, is known as the

eccentric angle of the point P.


To obtain the equation to the tangent to the eUipse at a point
whose eccentric angle is we have
X = a cos <f>, y — b sin
and the gradient of the ellipse at this point is given by

dy_
^ (dym) ^ bcos<l>
^ ^ ^
dx (dx/d<f>) — sin ^ a

The tangent therefore the line through the point (acos^, bsin<f>)
is

having a slope —
(b/a) cot <l>. Its equation is

^ b sin (j)
= - cot <f>{x
— a cos (f>),
a
which reduces to
fcos^+^m^ = l. (17.18)
a b

point is the line through the point {a cos tf>,


The normal at this
tan-^. Its equation will therefore be
hsm<i>) with slope (a/b)

Jy
— sin (A
T
= - tan i}>(x - a cos<^).
^

Tliis reduces to
ax sec ^ by cosec — o (17.19)
<l>
,

17] THE ECCENTRIC ANGLE 323


Example 8. Q is the point {a cos a sin
on the auxiliary circle of the
ellipse (4rVa*) {y^/b^) + 1. =
P is the point on the ellipse with co~
ordinates {a cos b sin <f>). If S is a focus of the ellipse show that the
length of the perpendicular from S on to the tangent at
equal to SP.
Q to the circle is
^L.U.)
The equation to the auxiliary circle
the tangent at the point (a cos
is x* + y* = a* and the equation to
a sin is xa cos ^ ya sin = a*, or,
X cos ^ sin ^ = a.
If S is the focus (ac, 0), where e is the
eccentricity of the ellipse, the
length of the perpendicular from S on to this
line is
ae cos ^ —a
= ^ ~
V(co5>^ + sin»^}
The distance SP between the points (at. 0). {a cos b sin is given by
SP< = — a cos ^)> + (0 — 6 sin
{ae
*= a^{e - cos
+ a*{l - e*) sin* if,,

when use is made of the relation 6* = o=(l - «»), This, by use of the
^ t ^ = "(e cos - 1). showng
that SPequal to the length of the perpendicular from
IS
S on to the
<f>

tangent at Q to the circle. A similar proof


holds if we take 5 to be the
second locus (— ae, 0).

EXERCISES 17 {c)

1. Find the distance between the the eccentricity and the length
foci,
of the latus-rectum of the ellipse 3^»
+ 4y* =, 12.
2. Find the equation to an ellipse whose
centre is the origin, whose
la^s-rectum is 10 and whose minor axis is equal to the distance
between the The axes of the elUpse
&X65»
foci.
lie Lng the coordinate

"
3, Find the locus of a point which moves
so that the sum of its dis-
tances from two fixed points 6 units
apart is alwa>^ 10 units Your
to axes of

4.

point (16/6. 9/6) to the elUpse


(xVl6) + (y*/9) = 1 are equal.

6. Fmd the equations to those tangents


which are parallel to the Une
to the ellipse *« + 2v‘ - 8
y = 2x.
-^(O C
6 . Fmd the equations to the normals
at the points (6 4 ) and (8 81

® tmddle point o{ the chord joining


r. these hvo points

mtS" n"otL*°no^al^^^
7, Two diAcnctcrs of &n gIUdsc a.re ^ • j ^
^ ^

bis^cu au c_horfs pa^neMo


the other. ShowZftoe'dWtem
U 1 “"jugate
324 PURE MATHEMATICS [17

8 . P is cos <j). b sin <f>) on the ellipse


the point (<7 = 1.
The normal at P
to the ellipse meets the ;r-axis at Q. Show that
the locus of the mid-point of PQ is an eUipse whose semi-axes are
{2a^ — b^)/'2a and 6/2. (O.C )

9. Show that the ellipse (x^/a^) -f (y^/b^) = 1 at


tangents to
points whose eccentric angles differ by 90® meet on the ellipse
{x^/a^) + (y^'62) = 2.

10 . Show that the equation to the chord joining the two points whose
eccentric angles are <f>, <f>' on the ellipse {x^/a^) {y‘/b^} = 1 is

- cos -f- -h ^0 sin = cos ^{<f>



a
Deduce the equation to the tangent at the point “ ^

17.11. The equation to a hyperbola


to a hyperbola can be derived by a similar method
The equation
to that used in § 17.7 to obtain the equation to an ellipse. Again
(Pig. 121) we take the focus S as the point (- ae, 0) and the directrbe

^'
^1 \y 1

Fig. 121

AB as the line * a/e. Since=-the eccentricity e is now greater


relative positions of the focus S and the point of inter-
than unity, the
interchanged— here C is nearer
section C' of with tlie AB
x-axis are

to the origin 0 than S. n ^


Ifthe point [x, y) on the curve,
P is
PM is
,

again the sum e


of di

abscissa of P
and the length CO, i.e., PM = x+ («/£>• J^^e lengt

PS is again given by o
P52 = (a: + ae)^+y%
hyperbola, P is a point such that
and since, by the definition of a
PS = e.PM
{x -|- aey + =
17] THE HYPERBOLA 325
This can be written
- l)x^ ~ II
or.
a- ~ 1)
Writing *2 == _ 1), (17,20)
the equation to the curve becomes
.ir2
-^v2 = 1 (17.21)

In tracing the curve, we first observe that


equation contains its
only even powers of x and
y and it is tlierefore symmetrical about both
the coordinate axes. From this symmetry we can deduce the existence
of a second focus S' at the point
(ae, 0) and a second directrix
A'B'
along the Ime x^a/e. The curve cuts
the x-axis where (x^/a-) =:
m the points (± a, 0) shown as H' and H in Fig. 121. By writing
i.e.,
1

^-0 in equation (17.21), the points in which the curve cuts


the
~ that such points are
imagmary. By writing the equation in the form

_ 1
'
6= a®
It IS clear that y® is negative, and therefore there is no part of the
the equation can u
"" between ± «. On the either hand '
be written

a= ^ b‘‘
showing tliat
points exist on the curve for aU
values of v. The above
orois of the equation to the
hyperbola also show that'y
* mcre^es and vice versa. The increases as
0 which extends in an infinite
curve consists of two
direction towards the positive
pXnTo^
dire’cUon
* directioh towLds the negative
side of Th
Sde^of the axis anda is shown
in the diagram.
The points H, H' are called
the vertices and the line HU' the.

hrough the focus 5 is the


latm-rectum and there win a
^ ^
latus-rectum through the second
since iS is the value of
for 7 I
/ whej ^ -
yc bv{e^ 1) r
a
when use is made of equation (17.20)
The length of the latus-rectum
IS therefore 26*//i.
326 PURE MATHEMATICS [17
To summarise, the curve

__Z!- 1
(17.21)

is a hyperbola of eccentricity e, greater than unity, given by


£2 = 1 -j- (17.22)
The foci are the points {± the directrices the lines x
ae, 0). (a/e), =±
the transverse axis is of length 2a and the centre is the origin of
coordinates.

Example 9. Show that the difference of the focal distances of any point on
a hyperbola is equal to the length of the transverse axis.

Using Fig. 121 and the definition of the hyperbola, if x is the abscissa of P,

PS = c.PM = £.1: -f- a.

Similarly, if PM' is drawn perpendicular to the second directrix A'B\

PS' = e.PM' = — cx — a.

The difference of the focal distances PS, PS' is therefore equal to 2a,
the length of the transverse axis.

17.12. The tangent and normal to the hyperbola at a given


point
Differentiating the equation (17.21) to the hyperbola with respect
to X,

62 dx
so that the gradient of the hyperbola at a given point (x^, is given by

(t) =
this point with
The tangent at the point y,) is the line through
(x^,

slope equal to the gradient of the curve ; its equation is


b^x^, .

or, b^xxi — a^yyi = b-x^^ —


Dividing by and using the relation {xi^/a^) —
CViV^‘) ^ =
isthe condition that the point {x^, yj shaU lie on the hyperbola, the
equation* to the tangent at the point {x^, yj can be written

xxi _yyi __ j ' (17.23)


62

tangent at
to the the point (x^, yj can be obtained
Once again ihe equation
curve by replacing x\ by xx^, yyj respectively.
from ihe equation to the
-
17] intersections OF LINE AND HYPERBOLA 327
The normal at the point
yj) is the line through this point at
right angles to the tangent. Its slope is therefore — {a^yi)/(b%) and
its equation is

This can be written in the more easily memorised


form
_ y yi
(17.24)
yi/{~b^y
Example 10. Write down the equations to the tangent and normal to the
hyperbola 9x* -
4y> =
36 at the point (4. 3v/3).
The equation to the hyperbola can be written as

(X 3 Vsfis''

T
or. 3x - V3y *= 3. Equation (17.24) gives the normal at this point as
the line

4y - 3v/3
4/4 -3V3/(-9)'
or, X + V3y =* 13.

17.13. The points of intersection of a straight line and hyperbola


equation to the hyperbola only differs
... from that to the
ellipse in havmg - 6* many of the results derived for
in place of b^,
the elhpse can be used for
the hyperbola if the sign of i* is changed.
Tor example equations
(17.23), (17.24) for the tangent and nomal
soonHi
sponding
^
i!
fro"' ‘he corre-
results (17.13), (17.14) for the ellipse by replacing b> by -
sam^ 'vay, the analysis given in
17.9 for the points of inter-
§
section nf
M
==mx +c and the ellipse
the sign wiU apply if
throughout. We shaU find that the
fine meets th^
^erbola
^voerbolX m real, comcident or imaginaiy
points according as e> is
also tod that

y = mx-\- V(a ^2 __ ^ 2
)^
always touches the hyperbola.
we ShaU
find that the locus of the point
off ‘nte^ertKin of
perpendicular tangents to the
hyperbola is the Lie
k j iT"
1
°
T IS again called the
director circle n
quation (17.20)) no corresponding limitation in the case of the
328 PURE MATHEMATICS [17

hyperbola. The radius


of the director circle being -v/(a® the —
circle is real whenever *2 2
^ , if ^2 < radius of the circle is ^
zero and it reduces to a point circle at the origin and in this case the
centre of the hyperbola is the only point from which perpendicular
tangents can be drawn to the curve. If the radius of the
director circle is imaginary and no perpendicular tangents can be
drawn to the hyperbola.

17.14. The parametric equations to a hyperbola


An ordinate of the hyperbola docs not meet the auxiliary circle
on HH* as diameter in real points (see Fig. 121), There is thus no real
eccentric angle as in the case of the ellipse. however, often useful It is,

to be able to express the coordinates of any point on the curve in


terms of one variable. Since sqc^<I> 1 tan 2 i^, the equations = +
X = a sec (f>, y = b tan <j>, (17.25)

may be used, for these expressions clearly satisfy the equation


(x^/a^) — (y V^") = 1 to the hyperbola.
a geometrical definition of the angle ^ used
It is possible to give
above but it is not very important to do so. The parametric equa-
tions (17.25) are, however, very useful in solving some problems and
we shall derive the equations to the tangent and normal to the
hyperbola at the point “ ”, i.e., the point whose coordinates are
<f>

given by equations (17.25).


From (17,25),

— asec<t) tan ^ b scc^<j>,

so that the gradient of the hyperbola at the point {a sec <j>, h tan <f>

is given by
dy (dy/d<j>)
__
b sec^<f> _ 6 sec

^~ (dx/d<l>) a sec ^ tan ^ a tan ^

The tangent is the line through the point (a sec <^, b tan ^) with slope

(^ sec <f>)/{a tan <^). Its equation is

b sec
V — ,
5 tan <t>
,
= <j>,
j{x — jv
a see 9 ),
^ a tan <f>

which reduces to
^sec<h — y tan^ = 1. (17.20)
a b

hnc through the point (a sec 4, Man <!>)


with
The normal the
is
will therefore be
slope - (a tan 4>)/(b sec Its equation

V - 6 tan ^^ = —
6 sec 9
17j PARAMETRIC EQUATIONS TO HYPERBOLA 329

This reduces to
axsm<^ by — {a^ -f b^) tan (j>. (17.27)

Example 11. 1/ S, S' are the foci and P is any point on a hyperbola, show that
SP, S'P are equally inclined to the tangent at P. (L.U.)

In Fig, 122, S is thefocus (— at. 0) and S' the focus {ae, 0) of the hyper-
bola. P is the point (a sec <1>, b tail 4>) and the tangent at cuts the P

Fig. 122

x-axis at the point T. The equation to the tangent at P is


{x sec ^)/a — (y tan ^)/6 = 1,
and the abscissa of T is obtained by writing
y = 0 in this equation.
The abscissa of T is therefore given by x = a/scc ^ acos
Hence <f> ij>.

"ST = a« -J- a cos and TS' = ae — a cos Hence


ST ^ at + a co s ^ cos ^
TS' at < — — a cos ^ cos ^
But we have shown in Example 9 that, if x is the abscissa of P,*
PS =s cx + a, PS' = «x — a. Here x = a see so that
PS at sec ^ + a « -1- cos 6
PS' at sec ^ — a e — cos 6'
ST
Hence ^ PS
TS'
and it follows from a well-known geometrical
the angle SPS'.
theorem that PT bisects

EXERCISES 17 (d)

1. Find the coordinates of the foci, the eccentricity


and the length of
the latus-rectum of the hyperbola 4x* —
9y* = 36 .

2. The centre of a hyperbola is at the origin


and its transverse axis
hes along the x-axis. Find the equation
to the h>T>erbola if the
distance bet^veen its foci is equal to 4a. where 2a is the lengtli
® of
the transverse axis.
3. Find the equations to the tangent and normal
at the point
r f — 3, — ij
n
to the h5q)erbola x* — Oy* =* 3.
v

4. Find the equations to those tangents to


which are parallel to the line
the hyperbola
' 4x* - 9v» = 1
4y = 6x -f 9.
5. Two diameters of a hyperbola are said to
be conjugate when each
c

330 PURE MATHEMATICS [17


bisects all chords parallel to the other. Show that the diameters
y = mx. y =
m'x of the hyperbola {x^/a^) — {y^/b^) = 1 are conju-
gate if mm' = b^Ja^.
P a point on the h>T)erbola {x^fa'^) — (yV^*) = 1 and
is is the N
foot of the perpendicular from P on the ;r-axis. The tangent to the
hyperbola at the point P meets the ^--axis at T. Show that, if 0
is the origin, OT .ON = a*.

Show that the coordinates of any point on the hyperbola


(ArVa®) - (yVfc'*) = I

can be represented by lx = a{t i"*), 2y = b[t — t-^) and find the


equation to the tangent at such a point.
Show that the equation to the chord joining the points (a sec b tan ^),
[a sec <f>',
b tan <f>') on the hj’pcrbola {x^/a^) — {y^/b^) = 1 is

— cos h{<i>

~ <!>')
^b sin ^{<i> + <f>')
— cos ^{<f>
“! ^ )•
a
Deduce the equation to the tangent at the point “
^

17.15. The asymptotes of a hyperbola


The abscissae of the points of intersection of the straight line
y — mx 4- c the hyperbola (x^/a^) (yV^^) 1 given by — —
writing y = mx + c in the latter equation. This gives

x^ (mx + ^

or, arranged as a quadratic equation in }/x,


aHc b^)
'
,
2a^mc ,
« ,
— 6* = n0.
-| -j-

x^ X
If — 62 and a^mc are both zero, this equation in l/x has two
zero roots. In other words, if

m=± h/a and c = 0,


of
the line = mx
meets the hyperbola in two points at each
y two points
which X/x is zero, i.e., the line meets the hyperbola in
of the cur\^.
situated at an infinite distance from the centre
meet a hyperbola in two points both of which are
Lines which
distance, but which are not themselves altogether
situated at an infinite
asymptotes. The asymptotes of the hyperbola
at infinity are caUed
(x^/a^) — (y'^/b^) == 1 are therefore the lines

_L (17.28)

TI,o» t,.o lines pa« “'“/V “ °Sl


“'“f
nj ASYMPTOTES OF HYPERBOLA 331

Written as a single equation, the asymptotes {17.2S)


would be given by

y ~-x)(y =»
or, after division by and sb'ght rearrangement,

a (17.20;

Example 12. P is a point on the hyperbola (x^/a^)


- 1 and the
tangent atP meets the asymptotes at
Q and Q\ Show that P is the mid^
point of QQ'.

tangent at P is. from equation

X y
^
sec ^ ~ tan ^ = 1.

The asymptotes of the hyperbola are


X* y*
^™M“
and the abscissae of the points
Q. Q' are given by eliminating y from
these two equations. Writing

y =
from the first equation in the second, we have
(X
7i
~ cot* ^(^sec^-lj
P[~ sec P =0,
This, using cosec*
^ - 1 + cot* p and division by cot* p. reduces to
X* 2x
— sec ^ + « 0 1 .

If the roots of this quadratic equation are Xi, jr„


+ x^ = 2a sec p,
+ a sec p
*^i) = =
abscissa of point P.
Siim^Iarly, by eliminating
;r from the equation to
the tangent and that
to the asymptotes, we
shall find that half the sum o£
the ordinates nf
nUd^pohit of 00'. P”-* -P
332 PURE MATHEMATICS [17

17.16. The rectangular hyperbola


the hyperbola {x^a^)
If in (y^b^) 1- =
the quantities a and b
,

are equal, the equations to the asymptotes are, by


(17.28), v x. =±
The asymptotes are therefore inclined at angles of
AT-axis and are perpendicular to each other.
45° with the ±
The equation to the
hyperbola can be written

and, because of the perpendicularity of its asymptotes, it is often called


a rectangular hyperbola. Because of the equality of a and b, this
hyperbola is also sometimes said to be equilateral.

17.17. The equation to a rectangular hyperbola referred to its


asymptotes
The equation to a rectangular hyperbola takes a very simple form
when the axes of coordinates coincide with the asymptotes. If in

Fig. 124

Fig. 124, a point on a rectangular hyperbola and the coordinates


P is

y) of P are measured from lines OX, OY


bisecting the angles

between the two asymptotes Ox, Oy, X


and Y are related by
— y2 = (17.31)

respectively and ML,


PK, PM drawn
areperpendicular to OX, Ox
MN are drawn perpendicular to PK, OX as shown. Since the angle

XOx is 45°,

OK = NK + 0N ^ ML -f- ON
= PM cos 45° + OM cos 45° = (PM + OM)/ \/2.
and PK^PL-KL^PL-MN
= PM sin 45° — OM sin
45° = (PM OM)/V--
If Ox, Oy are taken as coordinate axes, OM : X, PM = y. and
" )

17] RECTANGULAR HYPERBOLA 333


since OK = X, PK = Y, the above results give
= (y x)/^/2, Y^{y~ x)/^2.
-I-

Substitution in (17.31) yields

(y + x)’^
_{y ~ x)^ _
2 ^ *
2
reducing to xy = 0 ^/ 2 .

Writing 2c^ = a~, the equation takes the very simple form

xy = cK (17.32)
Whena rectangular hyperbola is referred to its asymptotes as
axes, a point whose coordinates are given
by
x^ct. y = c/t, (17.33)
always on
The
lies for these coordinates satisfy the equation xy
it,
c\ =
equations (17.33) give a parametric representation to a rectangular
hyperbola and the point given by them may be called the
point " /
If we differentiate equations
(17.33) with respect to /
dx _ dy c

Md the gradient of the rectangular hyperbola xy


= c* at the point
t is given by

dx (dx/dt) c
The tangent to the hyperbola at this
point is therefore the line

= 2c<.
(17,34)
riie normal is the line through the point (c/, c/l) with slope <2 and
Its equation is

y - - = t^(x - cl),

or.
l-x~y = c{i^-l/t). (17.35)
Example 13. Find the equation to the normal at the point
(3. 4) to the
rectangular hyperbola xy
=
12. and the coordinates of its second
point
of intersection with the curve.
jO C
Here c* 12 and c « 2^3. The parameter " t » of the point (3
^ 4)

or, 3jr - 4y -f 7 = 0.
334 PURE MATHEMATICS [17
Let the second point of intersection of
the normal be the point
{•\/3/, 2\/3//), Since this point lies on the line Zx — T ^ 0

61/3/ - ~ + 7 = 0.

giving 6 v/ 3/= 4- 7/ - 8-V/3 = 0.This can be written in the form


(2/ -
v/ 3)(3V3^ 8) +
0. = so that t = ^y/3 or -
8/(3-v/3). The first
value of I corresponds to the- point (3, 4). so that the
coordinates of the
second point of intersection are found by using t
8/(3 v/3). With = —
this value of /, the point (2^3/.
2\/3//) -s the point (- 16/3. 9/4J. -

EXERCISES 17 (e)

1 . A tangent to the hyperbola {x^/a^) - {y^/b^) = 1 meets the asymp-


totes in Q and Q'. If O is the centre of the hyperbola, show that
the area of the triangle OQQ' is ab.
2. S the focus on the positive ;r-axis of the hyperbola {;rV9) —
is = 1
and y is the foot of the perpendicular from S on that asymptote
which lies in the first and third quadrants. Prove that Y lies on
the directrix and also on the circle whose centre is the origin and
whose radius is the semi-transverse axis.
3. Find the eccentricity of a rectangular hyperbola.
4. Show that the equation to the line joining the points {ct, c/i),
{ct', c/V) on the rectangular hyperbola xy = is x ti'y — c{t + /').

Deduce the equation to the tangent at the point '* t


The normal to the rectangular hyperbola ;ry — 8 at the point (4, 2)
meets the asymptotes at M and N. Find the length of MN.
6 . PN is the perpendicular to an asymptote from a point on a rectangular
hyperbola. Prove that the locus of the mid-point of PN is a rec-
tangular hyperbola with the same axes. (O.C.)

7. TP, TQ
arc the tangents at two points P, Q on the rectangular
hyperbola xy = c*. Show that the line joining the centre of the
hyperbola to the point T bisects the chord PQ.
8 . Find the locus of the mid-point of a straight line which moves so
that it always cuts off a constant area A’ from the corner of a square.

EXERCISES 17 (/)

Write down the coordinates of the focus 5 and the equation to the
directrix of the parabola y* = 4x.
PQ is a focal chord of this parabola and PR is a chord perpen-
dicular to the axis. The tangents at Q and R meet at T. Prove
(L.U.)
that ST is parallel to the directrix.

The tangent 4ay at the point for which y =


to the parabola x^ =
meets the axes of and y at the points P and Q. Obtain an
expres-

sion for the area of the triangle POQ in terms of t,


O being the
'
origin. ' ^

Find the coordinates of the point of intersection R of the tangente


2ati). Q{at^ ,
to the parabola y* = 4ax at the points P{ati*,
17] EXERCISES 335
Ifthe tangents at P, Q are inclined to one another at an angle ol
45®, show that the locus of R is the curve y* ^ax + <2*. = +
(L.U.)
4. P and Q two points on the parabola y- = Aax whose coordinates
are
are (a^l^ 2a/,) and (a/g*, 2a/2). O is the origin of coordinates and
OP is perpendicular to OQ. Show that /,/, + and that the 4=0
tangents to the curve at P and Q meet on the line + 4rt = 0.
(L.U.)
5. P the point (a/*, 2a/) on the parabola y* = 4a;r.
is is the foot N
of the perpendicular drawn from the origin to the tangent at P.
P
Showthat, as varies, the locus of AT is the curve ;r(;r* +7®) +«>•* = 0.
(L.U.)
G, The normal at the point P(a/*, 2a/) to the parabola y^ = 4a-r meets
the parabola again at the point R[aT^, 2aT), Prove that
T= -- t —
Prove also that,
2//. if the normal at Q{a/i“, 2a/,)
passes through R, then /, = 2//. (O.C.)
7. The normal at the point P(a/^ 2a/) to the parabola y* =
Aax meets
tliecurve again at the point Q{aV*, 2a/'). Find /' in terms of /
and hence, or othenvise, prove that the lines joining the origin to
P and Q are at right angles if /* = 2
8. From a point P on
a parabola with vertex A and focus S, the line
PN is drawn perpendicular to the axis ^5. The tangents at A
and P intersect at Q. Prove that PQ* ^ AN .SP.
9. A variable chord through the focus of the parabola y^ = Aax cuts
the curve at P
and Q. The straight line joining to the point P
(0, 0) cuts the line joining
Q to the point ( - a. 0) at P. Show
that the equation to the locus of the point R is y*
+ 8;r* + Aax = 0.
10, Find the equations to the tangents to the parabola = 9.^ which
pass through the point (4. 10).
11. Find the equations to the tangents to the curves
X* +y* 9, Ax* -\-y* = 10
at one of the points where the curves intersect, indicating on a
sketch which point of intersection you have chosen.
Calculate the angle at which the curves intersect.
(Q-E-)
12 . AB and BC are two rods each of length a jointed at B, A is pivoted
to a fixed point and C can move in a
straight slot, which passes
through A. P is a point on BC such that BP =
b. Find the
coordinates of P referred to as axis of x and a perpendicular
^ough A to as axis of y, when each of the angles BAC, BCA
IS 8, and show that, as the rods move, P
traces out an ellipse whose
semi-axes aie (a + and \a - hi. ^O.C.)
13, Show that the tangents to the ellipse x*
+ 2y* = 18 at the points
3), (- 72/17, - 3/17) intersect on the normal
{0, --
at the point

14 . ^
The equation
.

to a chord of the ellipse x* + Ay* =


(O.C.)

Find the coordinates of its middle


260 is ;r + 6y 60
point. (O.C.)
336 PURE MATHEMATICS
15. The tangent at the point P{a cos b sin to an ellipse centre C <f>)

and semi-axes a. b meets the major axis at T. N is the foot of the


perpendicular from P to the major axis. Show that CN .CT = a*.
(O.C.)
S, S' are the foci of an ellipse of semi-axes a and b. The normal
at a point P on the meets the minor axis at G.
ellipse Show that
the square of the distance of G from either focus is {a* — b^)SP.S'P/b*.
(O.C.)
17. Find the ratio of a to 6 for which the ellipse [x^/a^) -f {y^fb^) = 1
and the parabola = Aax cut at right angles. (Q-E.)
18. The centre of a hyperbola is the origin and its transverse axis lies
along the axis of x. If the distance between the foci is 16 and the
eccentricity is \/2, write down the equation to the hyperbola.
19. If and k are the intercepts on the coordinate axes
h of any tangent
to the hyperbola (x^/a^) — (y^/b^) = 1, show that
{aVA") - = 1-

20 . The normal at any point P on the hyperbola (x^/a^) — ^

meets the ;r-axis at G. Q is a point on either asymptote such that


PQ is parallel to the y-axis. Show that GQ is perpendicular to the
asymptote on which Q lies.

21 . P the point (5/4, 3/4) on the rectangular hyperbola


is — y* = I.
The normal at P
cuts the axes of x and y at G and g respectively.
Prove that, if O is the origin, PG = Pg == PO.
22 . Show that the equation to the chord joining two points {x^ yi),

ya) on the rectangular hyperbola xy =


{X 2 , c* is

+ = 1 .

-I + ^3 yi +^8
a diameter of the rectangular hyperbola xy = c*. The
is

ingent at P meets lines through P' parallel to the


asymptotes
and Q'. Prove that P is the middle point of QQ'
and that
Q
le equation to the locus of Q is xy -h 3c*
= 0.
he perpendicular from the origin to the tangent at
a point P
le rectangular hyperbola xy = c*
meets the curve at Q
he chords PQ and PR meet the ;r-axis at U and
F. Prove that
from P cO
mid-point of UV is the foot of the perpendicular
x-axis. '

1C tangent at to P
the rectangular hyperbola xy c* meets the =
and =B. and A denotes the
,cs ;r y = 0 and ;r + y
- 0 at .4
The normal at B
ea of the triangle OAB
where O is the origin
at D. If A. denotes the area
nets the ;r-axis at C and the y-axis
the triangle ODC show that — he*.
CHAPTER 18

SOME THEOREMS IN PURE GEOMETRY


18 . 1 . Introduction
The student is assumed to be familiar with the usual theorems
on angles at a point, parallel straight lines, the congruence of triangles,
areas of triangles and simple rectilinear figures and Pythagoras’
theorem. Such theorems form one part of what may be called
" school ” geometry
and, except in so far that they may be appealed
to in the solution of some examples and exercises,
will not be further
considered here.
A
knowledge of the usual theorems on the circle and similar
tri-
angles will also be assumed. This part of
the subject is also usuaUy
studied at the school stage but a statement
of the more important
theorems and a few revision examples may not
be out of place here.
I'or proofs of these theorems of
elementary geometry the reader is
referred to one of the usual texts.
The remainder of this chapter is devoted to
theorems on similar
rectilmear figures, further properties of
the triangle, concurrency of
lines .and collinearity of points.
It is assumed that the reader has
not previously studied such theorems,
proofs are given and rather
more examples and exercises are provided.

18 2 . Statements of
.
some theorems on the circle
important theorems on the
foU^s^'— circle can be stated as

(a) A straight line drawn from the centre of a circle to bisect a chord
which IS not a diameter, is at right angles to the chord.

from Uie ,„d tta

a"g|e Which an arc of a circle


subtends at the centre is double

^ 337
338 PURE MATHEMATICS [18

(/i) The opposite angles of any quadrilateral inscribed in a circle are


supplementary, and the converse.
(f) If a straight line touch a circle, and from the point of contact a
chord be drawn, the angles which this chord makes with the tangent
are equal to the angles in the alternate segments, and the converse.
[j) If two chords of a circle intersect either inside or outside a circle,
the rectangle contained by the parts of the one is equal to the
rectangle contained by the parts of the other, and the converse.
{k) If, from any point outside a circle, a secant and a tangent are

drawTi, the rectangle contained by the whole secant and the part
of it outside the circle is equal to the square on the tangent, and
the converse.

18.3. Some revision examples on the geometry of the circle


In this section two examples on the theorems of § 18.2 are given.
Further problems will be found in Exercises 18 [a).
Example 1. A line DE parallel to of a triangle ABC cuts AB,
the base BC
AC in D and E
respectively. The circle which passes through D. and
touches AC at E, meets AB
at F. Prove that F, E, C, B lie on a circle.
(L.U.)

Join EF (Fig. 125). By theorem (*) above,


angle CED — angle EFD.
Since DE is parallel to BC. the angles CED. BCA are supplementary.

Hence the angles BCA, EFD are supplementary, and, by the con-
verse of theorem {h) above, the points F, E, C, B lie on a circle.

ABC. the side AB is greater than the side AC


Example 2. In a triangle
AC:
and D is a point in AB such that AD is equal to the mternal
p^nU
B and C n.eel in I. Show that the four
lisectors of the angles
B, D. I. C He on a circle.

Since AD = AC. the ADC is isosceles and (Fig. 126),


triangle
angle ADC = angle DC A.

sum of the angles of the triangle ADC is 180*


Since the
2 angle ADC + angle CAD
= 180®,
.
^
giving
. angle
^ ADC — 90® — ^A, -
18] THEOREMS ON THE CIRCLE 339

Fig. 126

w^re A = angle CAD. The angle CDB is the supplement of the angle
ADC so that angle CDB = 180® — (90* — ^A) =s 90“ ^A. +
Since angle IBC = JB, angle BCI =, JC, the triangle IBC gives
angle CIB = 180* -
= 180 '' •
*(180* - A) 90'
+ IA,
Since A +B -i- C => 180*.

Hence, angle CDB = angle CIB and the converse of theorem (f)
shows that the points B, D, /, C Ue on a circle.

EXERCISES 18 (a)

1 O is a fixed point andP is a


variable point on a fixed line t. 0 is
a point on the line OP such
that OP.OQ is constant. Show
Siat
the locus of a
0 is circle passing through O.
(L u
j
2 . The points A.B, C. D lie on a circle in that
order. From B per-
^ndic^ars BX,BY are drawn to AD and CD
respectively. Prove
that the angle BVA" is equal to the angle BDA,
(L u )
altitudes AD. BE of a triangle ABC meet at
the point H. If
AD produced to meet the circumscribing circle
IS
of the triangle at
K. prove that HK
= 2HD. Prove also that the three alti “dt
^1
pass through the point H,
4. With a point A on the circumference of
(L ) U
a circle S, as centre, a chcle
-Si at B and C. A straight line through A

In a triangle ,4 BC the angle C is greater than the angle B ^Tbl


aircumcircle of the^triang Jtt

7. AB a diameter of a
is
circle whose centra ic ^ v *
on the circumference. P is the

340 PURE MATHEMATICS [18

from A' to AB. H and K arc points on the circumference such that
XH = XK = XP. HK meets XP in M and XC in N. Prove tliat
the points M, N, C and P lie on the same circle. (L.U.)
8. A chord AB oi a. circle, wliose centre is C, is produced to a point
T, and a point X is taken on CT such that TC.TX = TA.TD.
Prove that the angles AXC, BXT are equal.

18.4. Statements of some theorems on proportion and similar


triangles
The more important theorems on proportion and similar triangles
can be stated as follows :

(/) a straight line is drawn parallel to one side of a triangle, the


If
other two sides are divided proportionally, and the converse.
(m) If two triangles are equiangular their corresponding sides are pro-
portional, and the converse.
(?i) If two have one angle of the one equal to one angle of
triangles
the other and the sides about these equal angles proportional, the
triangles are similar.
{o) If a perpendicular is drawn from the right angle of a right-angled
triangle to the hypotenuse, the triangles on each side of the per-
pendicular are similar to the whole triangle and to one another.
{p) The internal bisector of an angle of a triangle divides the opposite
side in the ratio of the sides containing the angle, and hkewise
the

external bisector externally. The converse is also true.

18.5. Some revision examples on proportion and similar


triangles
given
Here a few worked examples on the theorems of § 18.4 are ,

some problems of this type are given in Exercises 18 (6).

Example 3. Any line parallel to the base BC of a triangle ABC cuts AB,
is any point on a line through A
parallel
AC in H and I< respectively. P
to BC. If PH, PK produced cut BC at and R respectively, prove th^
Q
BQ = CR.

Fig. 127
18] PROPORTION AND SIMILAR TRIANGLES 341

Since (Fig. 127) AP, BC are parallel, the angles HAP, arc equal HBQ
and so also are the angles HPA, BQH. Hence the triangles APH,
HBQ are equiangular. By theorem (m) above,
BQ BH
~ AH'
'AP
Similarly the triangles APK, KCR are equiangular and
CR CK
AP ~ AK‘
Since HK is parallel to BC, theorem (/) above shows that
BH CK
AH ~ AK*
BQ CR
and hence
AP~^ AP
leading to BQ = CR.

Example 4. In a triangle ABC, the angle BAC is a right angle and


AB = 2AC. AD is the perpendicular from A on to BC. Show that
BD = 4Z)C.
By theorem (o) above, the triangles (Fig. 128) ABD, ABC are similar
and therefore
BD AB
giving BD *= 2AD.

Similarly, the triangles ADC, ABC are similar, so that


DC AC I
AD^ AB^2"
giving AD ^ 2DC.
BD ^ 2AD =z 2{2DC) * 4DC.
ptrptniicula, i

AL SN -CK KA ® ^^ ^ Prot'o the


(L«U
Applying theorem (p) above to the triangle ABN (Fig. 129),
AL BA
LN “ BN'
Similarly, from the triangle ABC,
CK BC
KA ^BA'
342 PURE MATHEMATICS L18

Bn Fig. 129
C

But theorem (o) shows that the triangles ADN, ABC are similar and
therefore
BA BC
BN ^
Hence AL LN = CK KA.
: :

EXERCISES IS (b)

Two circles A BP, PDC intersect at P and A PD, BPC


are straight
lines. Prove that, if the radius of the circle APB
is twice the radius

of the circle PDC, then the chord AB


is twice the chord CD. (L.U.)

In a right-angled triangle ABC prove that the perpendicular AD


from the right angle A to the hjT^otenuse BC is a mean proportional
between the segments BD, DC of the hypotenuse.
A variable tangent to a given circle meets two fixed parallel
tangents at P. Q and touches the circle at R. Prove that the rect-
angle PR.RQ is constant. (L.U.)

Two straight lines OAB, OCD cut a circle ^.t A, B, C, D. Through


O a line is drawn parallel to BC to meet AD (produced) in X. Prove
that OX^ = AX.DX. (L.U.)

Two circles, centres A and B, touch externally at a point C. A


common tangent touches the circles at P and Q respectively and
meets AB produced at S. If T the point in PQ such that
is

PT-.TQ =PS:QS. prove that (i) the triangles PAT, QBT are
the internal bisector of the angle A TB passes
through C.
similar, (ii)
(L.U.)

and P is any
Tangents are drawn to a circle at the points A and B,
otlier point on the circle. Prove that the product of the perpen-
to the square qj
diculars from P to the tangents is equal
pendicular from P to A B.
') •
(

Lines drawn through A


A triangle ABC is inscribed in a circle.
and E respec-
parallel to the tangents at B and C
meet BC in
tively. Prove that (i)
BD/CE
AD = =
AE. AB-/AC
(ii)
(L.U.) .

Two equal circles intersect at A and B. A line


through

circle at C and the other at D. and bT


the circle BCD ents^ BA. nw
p^-
at T. Prove tliat TC/TD = AC /AD. (L.U.)
duced if necessary,
18] SIMILAR RECTILINEAR FIGURES 343
8. X the mid-point of the base BC of a triangle
is
ABC. The bisectors
of the angles AXB, AXC
meet AB, at AC H and K, Prove that
HK IS parallel to BC.

18.6. Two theorems on similar rectilinear figures


Polygons which are equiangular and have tlieir corresponding sides
proportional are said to be similar.
If also their corresponding sides
are parallel they are said to
be similarly situated (or homotkeiic).
Theorem I . The
ratio of the areas of similar triangles
[or -polygons)
1 $ equal to
the ratio of the squares on
correspofiding sides.

Fig. 130

In Fig. 130, ABC, PQR are


similar triangles and
PS are their AD
altitudes. Since the angle ABD
equals the anele PO*? Lnd ft
BDA equals the angle IsP, both^h^a4^£^^^^^
triangles ABD. PQS are equiangular. Hence
^ '

AD _ AB BC

2e SiS-!' Thuf ^5C, PQR


A AB C _ \AD BC . BC^
/\PQR iPS.QR~~^2^

of s^ar polygons is equal to the ratio of


the^^- srcoSe^LX^
Theorem 2. 0 is any fixed point and ABCD
and %f Points A*
. , , P
7>4<^
is any polygon,
j. i

iLLr
^ ^B' C'
^ ‘ Pd'
are taken on OA, OB OC ni
(nr
{or these lines produced either way), such ' OP
that
OA'/OA = OB'/OB = — opyop = X . . .
then the polygons ABCD
similarly situated,
•p A'B'C'D'
^ ^
. ,
P are •
similar and
_

344 PURE MATHEMATICS [18

(<3.;
(b)
Tjc. 131

Since OA'/OA = OB' /OB, AB is parallel to A'B' and the triangles


OAB^ OA'B' are similar. Hence
A'B' _0A' .

~AB ~'0A~~‘
Corresponding sides of the two polygons arc therefore proportional
and parallel and the two polygons are therefore similar and similarly
situated.
In Fig. 131, 0 is said to be the centre of similitude of the
two
polygons. If corresponding points of the two polygons lie on the
same
side of 0, the polygons are said to be directly homothetxc
with respect
similitude (Fig. (a)).
to 0 and 0 is said to be the exUrnal centre of
If corresponding points lie on opposite sides of 0,
the polygons are
said to be inversely homothetic with respect to 0 and 0
is in this case

called the internal centre of similitude (Fig. (&)).


a ^nare
Example 6. PQR is an acute-angled triangle. Show how io constructPR.
one vertex on
uith two vertices on QR, one vertex on PQ and

Fig. 132
18] RATIO AND RECTANGLE PROPERTIES 345

On QR describe a square QHKR externally to the triangle (Fig. 132).


Join PH, PK and let these lines meet QR at B and C respectively.
Draw BA. CD perpendicular to QR to meet PQ, PR at A and D respec-
tively. Then ABCD is the required square, for regarding P as a centre
of similitude, ABCD is similar to QHKR and is therefore a square.

18.7. Some ratio and rectangle properties of the triangle and


quadrilateral
Theorem 3. Two triangles ABC, A'BC have a common base BC. The
line AA' joining their vertices meets the base BC at P then
;

A ABC//S A'BC = AP/A'P,


Fig. 133 shows the two cases in which A and A' lie on the same

Fig. 133

Md opposite sides of the base BC. AH, A ’!< are dra\vn perpendicular
to BC. Then
AABC _iAH,BC AH
AA'BC ia'k.bc~a1c

A'P'
for the triangles APH, A'PK are simUar.
Theorem 4.B are two fixed points and P is a moving Point
1/ A,
such that the ratio PA/PB is constant,
the locus of P is a circle.

In Fig. 134, AB is divided internally


at C and externally at Z>
m the given ratio PAfPB. Since

PC and
PB CB “ W
PD are the internal and external bisectors of the angle APB.
346 PUKE MATHExMATlCS [18

Fig. 13-1

Hence the angle DPC is a right angle and P therefore lies on the circle
whose diameter is CD. This circle is called the circle of Apollonius.

Theorem 5 (Ptolemy’s theorem). If ABCD is a cyclic quadri


lateral, then AS. CD -f- BC.DA = AC.BD.
In Fig. 1 j5, AX is drawn to meet BD at X so that the angle XAD

Fig. 135

equal to the angle BAC. Since these angles and the angles
is
ADX,
ACB are equal (angles in the same segment), the triangles ADX,
CBA are similar and
DA _AC giving BC .DA = AC.DX,
BC
Since the angle CAD = angle CAX + angle XAD — angle CAX
= segment
+ angle BAC angle BAX, and the angles in the same
DC A, XBA are also equal, the triangles ADC, are^ similar so that AXB
CD XB giving AB.CD = AC.XB.
AC AB'
By addition, AB.CD BC .DA — AC(DX -f XB)
= AC.BD.
B

J8] RATIO AND RECTANGLE PROPERTIES 347


Example 7. If P is a point on the^ smaller arc BC of the circumscribed circle
of an equilateral triangle ABC, show that PB PC = PA.
Applying Ptolemy's theorem to the cyclic quadrilateral ABPC (Fig. I3C),
PB.CA + PC.AB = AP.BC.
A

Fig. 13$

Since the triangle ABC is equilateral,


CA ^ AB = BC,
and the result follows immediately.

EXERCISES 18 (c)

Give the steps of a purely geometrical construction for an equilateral


triangle whose area is t\vice the area of a given quadrilateral, (L.U,)
Prove that the areas of the rectangles formed by the external and
internal bisectors, respectively, of the angles of a parallelogram
are
in the ratio {a + 6) * (a — 6) *. where a, b are the lengths
; of the
sides of the parallelogram.
(O.C.)
Show how to trisect the area of a triangle by means of two lines
drawn parallel to one of the sides,
(G C )
A chord of a circle (when produced) meets the tangent at a point
P to the circle at the point T. Show that {PA/PB)* TA/TB. =
The base BC of a triangle ABC is fixed and X, Y are the mid-points
of the sides AB, respectively.ACIf BV = 2CX and BY, CX
intersect at P, show that, as the point A
P always
^
moves, the point
on a circle.
lies

A and B are two fixed points and P is a point such that PA /PB
where the ratio x/y is constant. The circle on which
= x/y
AB in X, AB produced in Y and O is the mid-pointP lies meete oi A If
^
ol.oy = oB>
^BCD is a cyclic quadrilateral. The AB. CD
12 m. r^pectively and are parallel.
sides axe 6 and m
If the length of the diagonal
BD IS 11 in., show that =. AC BD and that AD
= BC = 7 in
AC a .hameter of a circle and B. D are trvo
is
points on the chcle
one on either side of AC. If the angles
CAD. BAC are denoted by
«, p respectively, use Ptolemy’s theorem
to show that
sin (ot + ^) = cos a sin ^ + sin a cos B,

348 PURE MATHEMATICS [18

18.8. Some further properties of a triangle


The two elemental properties :

{a) the perpendicular bisectors of the sides of a triangle are concurrent


and their point of intersection 0
is equidistant from the vertices of
the triangle,

(6) the internal bisectors of the angles of a triangle are concurrent and
their point of intersection I is equidistant from the sides of the triangle,

should be already known. Proofs of («) and (6) are omitted here as
they form part of every elementary’ course in geometiy\ Property {b)
above can be extended to include the property (c) that the internal
bisector of one angle of a triangle is concurrent ivith the external bisectors

of the other two angles and the three points of concurrence /j, 1 2 Iz ,

each equidistant from the three sides of the triangle. This too should
be already know’n and again a proof is omitted here. The points 0,
I, /j,I 2 and /g are knoum respectively as the circumcentre, in-centre
and ex-centres of the triangle. Other properties of a similar character
are considered below.

Property (d). The altitudes of a triangle are concurrent.

In Fig. 137, AD, BE, CF are the altitudes of the triangle ABC.
YZ, ZX and XY are lines through A, B. C parallel to BC, CA and
AB respectively. Since ZBCA is a parallelogram, ZA =
BC and since

Fig. 137

= BC. H.nc. ZA ^ AY and since


ABCY is a paraUelogram,
Thus ADjs the
AD perpendicular to YZ.
YZ BC are paraUel, is
BE. <=
Similarly
perpendicular bisector of AD.
the Imes
and A'T and hence, by property
(a),
bisectors of ZX
of the altitudes AD. BE. CF is knm^
"^rafpdntTofTncurrence
of the triangle ABC. and the triangle DEE is called
IS the onhocentre
pedal triangle of the triangle ABC.
the
18] FURTHER PROPERTIES OF TRIANGLE 349
Property (e). The Hues joining the vertices of a triangle to the mid-
points of the opposite sides are concurrent.

In Fig. 138. A\ B\ Care the mid-points of the sides SC, CA. AB


of the triangle ABC. The lines AA', BB' meet at the point G. Since
CA —\CB and CB' = \CA, A'B' is parallel to AB and A'B' iAB. =
Since A'B' is parallel to AB, the triangles
GA'B', GAB are similar and
A'G_A'B' 1
~
AG AB ?
Thus, A'G = IGA, leading to =
^AA'. Hence BB' meets A A
at the point of tnsection of
A A' nearest to A'. Similarly it can be
shown that CC meets ^^4' at the same
point.
The three Imes AA', BB', CC' are known
as the medians of the
triangle, and the pomt G of
concurrence of the medians is caUed the
centroid oi he triangle ABC.
It can be sho^.m that the point G
coin-

smSerdens^y^"'^'^^
^ triangular lamina ABC of uniform

Property C are the mid-points of the sides BC, CA, AB


of the perpendUulan fro\,t A.
t
P
P. 0 R of triangU ABC a, id
\!^^^olhemipoinU of HA. HB a, id HC. The nine points A',
^ ^ E, F, p, B He on a circle.

A' 0
Fig. 139
.

350 PURE MATHEMATICS rig


Since in Fig. 139. AP = PH and AC' = CB,
PC' is paraUel to
BH. Since A'. C' are the mid-points of BC, AB respectively
A'C' is
parallel to AC. Hence the angle PC'A' is a right angle. Similarly,
the angle PB'A' is a right angle, and the angle A' is given as a DP
right ^angle. Hence the circle on A'P as diameter passes through
C', B' and D. In other words, the points P and D lie on the circle
through A', B', C' Similarly, the points E.
Q and F, R lie on the
circle through A', B\ C', and hence the nine points A', B',
C', D,
E, F, F, Q, R all lie on the same circle.
The circle through these nine points is called the nine-point circle
and its centre N
referred to as the nine-point centre.
is It should be
noted that the nine-point circle is the circumscribed circle of the
triangle A'B'C'. Since the sides of this triangle are each half those
of the corresponding sides of the triangle ABC, it follows that the
radius of the nine-point circle of the triangle ABC is where R is
the radius of the circumscribed circle of the triangle.

Property (g). The circumcentre, orthocentre, centroid and nine-point


centre of a triangle all lie on a single straight line (the Euler line).

In Fig. 140, 0 and G are respectively the circumcentre and centroid


of the triangle ABC. A' and C’ are the mid-points of BC, BA, and
OG is produced to H so that OG = ^GH. The first stage of the proof
consists of showing that H is the orthocentre of the triangle ABC.

AH (produced) meet BC at D. Since G lies at the point of


tri-
Let
section of A A' nearest to A', A'G

IGA and the triangles OA G,
contain equal angles A’GO, AGH and the sides
about
AGH AH
similar and therefore
angles are proportional, the triangles are
is parallel to OA'. But OA' is perpendicular to BC and hence is AH
perpendicular to BC. It can be shown similarly that is per- CH
pendicular to AB
and hence that H
is llie orthocentre.
perpendicular
Since A'O DH
are both perpendicular to BC, the
the perpendicular bisector of
bisector of A'D bisects OH. Similarly,
But A'D and C'F are chords of the mne-point
C'F bisects OH. both
circle the mid-point of
and therefore OH
is the nine-point centre
.
B
18] FURTHER PROPERTIES OF TRIANGLE 3M
P a point on the circumcircle of a triangle ABC
is
and L, M, N are the feet of the perpendiculars from P to BC, CA,
AB,
then L, M, N he on a straight line (the Simson or pedal line with of P
respect to the triangle ABC).

PMC, CLP are right angles, the points


P Af^r^
w; fh the angle
‘he
MCP.
angle B-tfthe
‘f’ angles PNB PLB are both right anglel so that

supi^emernSy ’

Hencf"
angle MLP = angle MCP =: angle ABP
= 180° - angle PLN,
srighVL!""'" “Sles. Thus

Example 8, lis ihe in-centre, O


a triangle ABC. Prove that
the circumcentre and H the orthocentre of
AT bisects the angle HAO.

OAT is pe^«.d,tl«
F F^najcuiar to SC
BC Ind
^’
fa
circle at AT. Since

and is therefore paraUel to A

X
F^g. 142
3d2 PURE MATHEMATICS 1J8

Example 9. H is the orthocenlre and DEF the pedal triangle of a triangle


ABC. Show that H is the in-centre of the triangle DEF.
The points B, D. H. F (Fig. 143) are concyclic since BDH, HFB are
both right angles. Therefore
angle FDH = angle FBH
= angle ABE = 90® — angle CAB,

Fig. 143

Similarly, angle HDE — 90® — angle CAB. Hence the angles FDH,
HDE arc ccpial and HD bisects the angle FDE. It can be shown
s-milarly that HE bisects the angle DEF and it follows that H is the
in-centre of the triangle DEF.

EXERCISES 18 (d)

1. The vertices Z?, C of a triangle ABC are fixed and tlic vertex A
moves so that the angle BAC is constant. Show that the in-centre
I of the triangle ABC lies on a circle passing through B and C.
(L.U.)

2. In a triangle ABC, AD \s perpendicular to BC and H is the orthocentre


of the triangle. AD meets the circumcircle of the triangle ABC at
A'. Prove that HD = DX and that AD. HD = BD.DC. (L.U.)
3. O is the circumcentre and H the orthocentre of a triangle ABC.
Tlie line AO (produced) meets the circumcircle at L. Prove that
BHCL is a parallelogram.
from B, C on to the opposite
4 E E are the feet of the perpendiculars
sides of a triangle ABC. X
is the mid-point of EF and is a

median of the triangle ABC. Show that the angle XAB = angle

CAA'.
orthocentre of a triangle ABC. show that the triangle
6. If // is the
as the triangle ABC.
HBC has the same nine-point circle

the three escribed circles of a triangle


6. If h
are the centres of
circle of the triangle JJih is
ABC. prove that the nine-point
circumcircle of the triangle ABC.
respectively the circumcentre. centroid, nine-pomt
7 O. G. N. II arc
centre and orthocentre of a triangle.
Show that
OG:GN :NH = 2:1:3.
'
CEVA*S THEOREM 353
8. ABC is a triangle in which AB ^ AC, and P, Q are the mid-points
of the arcs AB, AC respectively of the
circumcircle. Prove that the
Simson lines of P and
Q intersect at an angle of 90 — ^A, °

18.9. The theorems of Ceva and Menelaus


Suppose that A, B and P, Q are pairs of points on the same or
parallel straight lines. Then the segments AB, PQ are said to liave
opposite senses according as the
displacements from
AXoB and PtoQ are m
the same or opposite directions. It is con-
venient to take account of the sense
of a line as in coordinate geometry.
^ ^ parallel to the jr-axis. the length AB
negative according as the point B lies to
the
right^Meh
With this convention. AB = ~ BA, or AB ^ BA ^ 0, and if
'''' ^ ^0 (Fig. 144). then
AO AO — XA m all cases.

Fig. 144

Two useful theorems on the concurrency of lines and coUineanfv


is; “ iS -s?
Ceva’s theorem.
of a triangle ABC
If three straight lines through the
vertices ABC
the triangle at X,

^C^AZ_
Xe'YA'^-'^'
4

Fig. J45
z
354 PURE MATHEMATICS [18

In Fig. 145 (rt), the point P lies inside the triangle ABC and in
diagram (5), P lies outside the triangle. In diagram (a), each of the
ratios BX/XC, CY/YA, AZ/ZB is positive, while in diagram {b),

the ratios BX/XC, CY/YA are negative and AZ/ZB is positive. In


either case, the product of the three ratios is positive.
By theorem 3, § 18.7,

in diagram (a),

BX _ A BPA CY _ A BPC AZ _ A CPA


XC ~ A~CPA’ YA ~ ABPA' ZB~ a BPC
in diagram (6),

BX A BPA Cy _ - A BPC AZ _ A CPA


YC^ A CPA* YA A BPA ' ZB A BBC*
In either case, by multiplication,
BX CY AZ
XC'YA'ZB
The converse of Ceva*s theorem. If X, Y, Z are points cn the

sides BC, CA, AB of a triangle ABC such that


BX CY AZ
XC'YA'ZB
then the three lines AX, BY, CZ are concurrent.

If AX, BY, CZ are not concurrent, let BY, CZ meet at P and let

AP (produced if necessary) meet BC at X' (Fig. 146).

Then, by Ceva's theorem,


BX'CY AZ
YC^YA'ZB
BX CY AZ
but. YC'YA'ZB
X and AX, BY, CZ are concurrent.
so that X' coincides with
356 PURE MATHEMATICS [18

This can be proved by a “ rediictio ad absurdum " method similar


to that used in proving the converse of Ceva’s theorem. The proof
is left as an exercise for tlie reader.

Example 10. Use the converse of Ceva's theorem to prove that the medians
of a triangle are concurrent.

Fig. 148

If (Fig. 148) A A'.


BB'. are the medians of a triangle ABC,
CC A\ B\
C' are the mid-points of BC, CA, AB respectively.
Hence BA' — A'C,
CB' = B'A, AC' = C'B and therefore
BA' CB' AC' _ ,

~^‘B'A'C'B
The converse of Ceva’s theorem then shows that A A', BB' and CC' are
concurrent.

Example 1 1 The tangents to the circttmcircle of a triangle ABC at the


vertices
.

A.B.C meet the opposite sides at P. Q. R. Show that P. Q, R are


collinear.

Since the angle PAC (Fig. 149) equals the angle ABC in the alternate

BPA common, the triangles PAC. A BP are


segment and the angle is

similar. Hence
BP AP _AB
AP CP AC’

DP AP _ A^
giving Jp"CP AC*'
DP __ AB*
or. ZP " AC*‘

Thus
BP
^ ^ AC*’
18] EXERCISES 357

and CQ _ RC* Alt CA*


similarly.
QA s'
^ “
CB*
By multiplication.
BP CQ AR _
PC QA RB
and the converse of Menelaus' theorem shows that P,
Q. R are collinear.

EXERCISES 18 {e)

1 . Use the converse of Ceva’s theorem to show


that the bisectors of
the angles of a triangle are concurrent*
the points of contact of the inscribed
circle of a triangle

andCZ are concurrent.


respectively. Prove that AX BY
^
^ j

P°^*® *" sides BC, CA, AB respectively of a triangle

Sr
5r^’ "I

OY AZ BY CY
•• “
OZ ^ AY' (I-U.)
4.

CA Ab’.?^ the sides BC.


circumcircle of the
trtn^l!xV
triangle AT 7 meets the sides BC, CA, AB
again at X' Y’ Z' re
pectively. Show that By' and C2' afe concu^re’nt
6. t**® “'"'erse of
Menelaus' theorem to show that
f the noints at
" '"“S'" «'*
opposite :idefarfco“'
6.

’LiL p" S
distinguishing between the
”^
BX,CY.AZ ±XC,YA,ZB
Uvo cases

BX^ s: 3 cm, show that X^X^


^
3 cm,
7, The ms^ribed circle of a triangle ABC touches the sides BC CA

8 .

fn'^P Z'cP^ZuATtg" Ihow SAQ.

exercises 18 (/)
1. Points P, Q. R
are taken on the
sides BC r A at>
of a triangleABC. If the circles A OR respectively
prove that the points
2 .
^
C P n Q concyclic. n tt
O,
\

“• ““SI- -ux:
Sr
B
35$ PURE MATHEMATICS [18
AB is a. fixed diameter of a circle and PQ is a variable chord parallel
to AB. R is the mid-point of PQ and X is the foot of the per-
pendicular from P on to AB. Show that AP^ + PR^ - AX^ is
a constant.
4. a chord of a circle and
is is the middle point of AB. N
The
diameter of the circle through is PQ. N
Prove that AQ is a tangent
to the circle through A, P and N. (L.U.)
6 . Two chords AC, BD of a circle int^sect in E within the circle and
AB, DC intersect in F. Prove that EF is the common chord of
the circles circumscribing the triangles ABE and DCE. (L.U.)
6 . ABC a triangle and the bisector of the angle BAC meets the
is
circumcircle of the triangle again at and BC at D. Prove that X
the triangles and ABD AXC
are similar and deduce that

AB.AC = AD. AX = AD^ + BD.DC. (L.U.)

AX and BY are parallel lines and AY, BX intersect in P. A line


through P parallel to XA meets AB in Q. Prove that
(i) AX.BQ = BY.AQ,
1 1 1
(L.U.)
AX'^ BY~ PQ'
8 . T a point 6-25 in. from the centre of a circle of radius 5 in. The
is
tangents from T touch the circle at A and B. Show that the length
oi AD is 6 in. (L.U.)

9 . The triangle ABC right-angled at C and O is the middle point


is

of AB. If the internal and external bisectors of the angle ACB


meet AB and AB
produced in X
and Y. prove that OC is a tangent
to the circle XCY.
C is the centre of a circle, X is any point within the and
circle

CX is produced beyond Y so that CX.CY =


X to CA*, where A
is the point of intersection of CX and the circle.
If P is any point
on the circle, prove that PX PY = CX CA. : :

circumference,
AB is a diameter of a circle and C is any point on the
P, Q are the feet of the perpendiculars from A, B respectively on

to the tangent at C to the circle. Show that


A ABC ^ A^CP+ A BQC. (L.U.)

Points D. E, are taken in the sides BC, CA,


F A respectively
of a triangle ABC such that BDjDC = CE/EA
= AF/FB = njm.
AD, BE meet in P; BE, CF meet in Q and CF, AD meet in R.
Prove that the areas of the triangles BQC, CRA, APB are each
equal to
ntn^
n* + nm + m**
(L.U.)
where A is the area of the triangle ABC.
fixed points and P is a point which moves so
A and B are two
Show that the radius of
that PA/PB = A (greater than unity).
B «
18] EXERCISES 359
the circle on which P moves is

— X
14 . If ABCDE a regular pentagon inscribed in a
is
pomt on the minor arc of AB, show that
circle and P is a

PA + PD PE
^
~P^TpD PC’
15 .
quadrilateral and the bisector of the angle
meets AD m E. Show
ABD
that the angles ACE. DCE are unequal.

16 .

fit
orthocentre of the triangle ABC and L, M. N ar^tte
‘’‘^'""WcaUy opposite to B, C, prove
thatl/t
that HL. HM, HN bisect BC, CA. A respectively. /Q c )
17 .
oircumcircle of a triangle, prove
the locus of the orthocentre ^ that
is a circle.
18 ,

escribed circle touching


AB *tc tvtf
externally for a ABC and
triangle ifthe line All, meetl
the circumcircle of the triangle ABC at P.
show that
PI =PB = PC = PI^.
19 .

20 .

the^Smison line of P .vith respect to the


^iangk ABcT;J^^
21 .

^
tl^erticifj“Murtb°^ the joins of toP
Either or both the
jotas^d”^^*^^^''^ “ f
' ^ ^“Pootively.
to give section.
Sh^w that. where^tV^Tay" bT
tllen^
_
^
•rx
*

CP
2.

.n.=«o™
tBe Bumarator aad
IB B'Buence of lettera
denoBiiBator are opwsfe
wse, show that, it
AD
^ DP. then BP/CP 1
22 . On SiHac R/^ ^a a A (L«U*)1
one on each side ^BC lie the points P,
Q. R.
respectively^
and Al^. BQ'. CR'
are drav^ pLnefto OP oo'op*^®
to meet BC, CA, AB ‘'ospectively
ia P'» O'
Q, R respectively. Prove that
^ + 00 +
Op
AP' Be'
OR
CR'
1.
23 .
3G0 PURE MATHEMATICS [18

BO intersect in H and CH meets AB in R. Show that AR = ZRB


aiid that 2AH =9HP. (L.U.)

24. ABC, A'B'C' are two triangles such that AA'. BB\ CC' meet at
a point O. If BC, B'C' meet at L, CA, C'A' meet at M
and AB,
A'B' meet at N, show that L, M, N are collinear. {Desargues'
theorem.)
25. If a straight line cuts the sides AB, BC, CD, DA of a quadrilateral
at X, y. Z. IV show that AX.BY.CZ.DW = XB.YC.ZD.WA.

CHAPTER 19

ELEMENTARY GEOMETRY OF THE PLANE AND


SPHERE
19.1. Introduction and definitions
This chapter is concerned with the elementary geometry of the
plane and sphere and, in particular, with the angles made by planes
and straight lines with one another. Considerations of space neces-
sitate a brief treatment and certain results which are usually given
as theorems " are relegated to the exercises.
The following, with most of which the reader may be already
familiar, may be regarded as a set of preliminary definitions.

(i) A surface such that straight lines through every pair of points
in it lie wholly in the surface is a plane,
(ii) If two planes, a straight line and plane, or two
straight lines
in the same plane have no point in common,
they are said to
be parallel,
(iii) A fi^re bounded only by plane polygons is called a poly-
solid
hedron, '^e bounding polygons are faces,
consecutive faces
intersect in edges and consecutive edges meet in vertices of the
polyhedron. Some particular cases are :

(a) the prism, in which two faces are congruent


polygons and
the remaining faces paraUelograms the paraUeUpiped, in
;
which all the faces are paraUelograms if every face of a
;

parallelepiped is a rectangle, the figure is


a rectangular
parallelepiped or cuboid and if every face
is a square it is
a cube) ^
one face
a polygon and all the
»
rem^ing
• . .
is
faces are triangles with a common
vertex: a
special case is the Uirahedron, in
which aU the faces are

Some typical examples are shown in Fig. 160 below.

eag^ OA.BC of tte tetrahedron of Fig.


160 and the diaeonaU
a paraUdepiped are the lines
joining opposite vertices
ie
^
men
lines, AC. CA\ BD'.
B'D of the paraUelepipcd of
the facw of a polyhedron are
all congruent regular
po^y^s
thSr^’
Md the numbers of edges which
meet at each vertex are ^unl
the polyhedron is said ’ '
to be regular.
sei
362 PURE MATHEMATICS [19

O'

TRIANGULAR PRISM PARALLELEPIPED TETRAHEDRON


Fig. 150

(iv) A straight line which intersects a fixed line (curved, straight, or


made up of segments of curves and straight lines) and remains
parallel to another fixed line generates a cylindrical surface and
is a generator of the surface. The space bounded by a
called
cylindrical surface and two parallel planes is a cylinder, one of
the parallel planes being the base.
(v) A straight line which intersects a fixed line (curved, straight or
made up of segments of curves and straight lines) and passes
through a fixed point generates a conical surface and is called a
generator of the surface. The space bounded by a conical surface
and a plane is a cone, the plane being the base.

TRUNCATED PYRAMID FRUSTRUM OF CONE


Fig. 151

prism, pyramid, cylinder or cone intercepted between


fvi) The part of a
other plane is knov^m as a truncated
pnsm,
the base and any
cylinder or cone. The part of a cone or pjramid mter-
pyramid,
plane is called a
Lpted bet4en the base and a parallel
moves so that its distance from a feed pomt (the
fvii) A point which the space mside
on a spherical surface
centre) is constant lies ;

called a sphere.
a spherical surface is
19] THEOREMS ON PARALLELS 363

19.2. Some axioms and further definitions


The
following axioms are usually taken as the basis on which
elementary solid geometry’ is built.

Axiom 1. There is one straight line, and only one, passing through
two given points.
Axiom 2. There is one plane, and only one, passing through three
given points which are not in the same straight line.
Axiom 3. If two planes have a common point, they have also
a
common straight line.
Axiom 4. Through any point in space there is one, and only one.
straight line parallel to a given straight line. {Playfair's
axiom.)

It is convenient here to list


some further definitions. Lines lying
in the same plane are said to be coplanar and
lines not lying in the
same plane are called skew lines. The angle between two skew
straight
lines is defined as the angle between two coplanar
lines to which they
are respectively parallel. ' ^

Example 1. Jf three planes intersect, two by two, show that their lines of inter-
section are either concurrent or parallel.

A
convenient notation for working this type of problem is
to denote the
three planes by a, p,
y and to use ap, py, ya. to denote the lines of inter-
section of the planes a and
/5, ^ and y, y and a respectively.

Since the straight lines ya and a/3 lie in the


same plane a, they either
intersect or are parallel. If they intersect, their common
point lies on
all three planes (it can be denoted
by the point a/8y) and it lies on the
line ^y. Hence the lines a^. ^y, ya are concurrent. If the
lines y« and
a/3y and neither ya nor ap cuts fly.
Q
afs, Py both lie in the plane
But
j8 and therefore they are parallel. Similarly
ya and gy can be shown to be parallel. ^

19«3« Some theorems on. parallels

Theorem 1 . If a straight line is parallel to one straight line in a plane


it IS parallel to the plane. ^
Suppose (Fig 162) AB is paraUel to CD
in the plane EDCF.
the hnes AB CD Then
coplanar. The planes ABCD,
EDCF meet in
mpl fb 1
and hence AB does not
to the

if a s\iieM li^e? converse that


y
364 PURE MATHEMATICS [19

Fig. 152

Theorem 2. If two straight lines are each parallel to a third, they are
parallel to one another.

Suppose (Fig. 153) the lines AB. EF are both parallel to the line
CD. Let the plane BAE cut the plane CEFD in the line EG. Since
AB is parallel to CD, theorem 1 shows that the plane BAE is parallel
to the line CD and that the plane DCE is parallel to the line AB.

Fig. 153

EG is parallel to CD.
And
EG
Since is in the plane DCE. the line
BAE. EG parallel to AB^ Hence, by
s^e EG is in the'plane
AB.
Axiom 4, EG coincides with EF and is parallel to

two parallel planes, the lines of inUrseetion


Theorem 3. If a plane cuts
au
are paraUel and are met by the
^ y
p
f .inel
19J THEOREMS ON PARALLELS 365

Thus a^, are coplanar lines which do not meet and they are therefore
parallel to each other.

Theorem 4. If iwo straight lines are cnt by thru parallel planes


their intercepts are proportional.

^ parallel planes, intersecting the lines


. T,?
at fi, C and D, E, F. The line meets the plane B
r‘
lines BP, CF respectively, so that
A y in the paraUel

AB:BC::=AP: PF,

Hence AB:BC = DE\ EF.

EXERCISES 19 (a)
ABCD is a face of a cube. A plane
passes through the diagonal
AG of the cube and bisects the edge BC
at L. Show that AL »•
LG,
366 PURE MATHEMATICS [19

2. Show that the lines joining the vertices of a tetrahedron to the cen-
troids of the opposite faces are concurrent.
3. m and n are two fixed skew lines and A and B are two fixed points.
Find the locus of a point P which moves so that PA intersects m
and PB intersects n. Show that one position of P is on m and
one on n. (L.U.)

4. n and m are two skew lines. Show that a plane can be drawn through
m parallel to n.
n, m and p are three straight lines, no t^vo being coplanar. Show
that a straight line can be drawn intersecting m and p and parallel
to n. (L.U.)
6.
5. ABCD is a tetrahedron. E, F are. points in AB and CD respectively.
If H, K are points in AC, BD such that
AE AH BK
CF
'eB~ FD~‘HC^ KD~
'

prove that EF and HK in the same plane.


lie If EF and HK inter-
sect in G, prove that HGfKG = EG/GF = X. (L.U.)

I, m. n are three non-intersecting straight lines in space, no two of


which are parallel. Prove that through any point on / just one
straight line can be drawn intersecting both m and n. If /, m, n

are all parallel to a given plane and PQR. P’Q'^' are


two straight
lines meeting / in P and P', tn in Q and Q n in R and R prove
, ,

that PQ/P'Q' = QR/Q'^'-

19.4. Normals
a straight line is perpendicular to every straight line in a
plane,
If
it is said to be normal (or perpendicular) to the plane.

Theorem 5. A a plane if it is perpefidicular


straight line is normal to

lines in that plane.


to each of two intersecting straight
plane a at A and let
In Fig. 156, let the line PA meet the given
two given intersecting straight lines. Let
AB, AC be parallel to the

Fig. 166
19] NORMALS 367
AD any other straight line in a and draw some line in a
parallel to
through D to cut AB, AC at B, C respectively. Produce PA to
Q
so that PA ~ AQ. Since PA is perpendicular to AB, the triangles
PAB, QBA are con^uent for the angles PAB, BAQ are right angles,
DA = AQ and AB is common. Hence PB = BQ and similarly it can
be shown that PC = CQ. Because of these equalities and since BC is
common, the triangles PCS, BQC are congruent and hence PD = DQ,
The triangles PA D, QDA are congruent because PA ~ AQ, PD DQ =
and AD is common, so that the angles PAD, DAQ are equal. Since
the sum of the angles PAD, DAQ is two right angles, the angle PAD
is therefore one right angle and we have
shown that PA is perpendicular
to any straight line in the plane a and hence is a normal
to the plane.
Theorem 6, Planes which are normal to the same straight line are
parallel to one another.

In Fig. 157, the line CD is normal to the planes a and


/S. If a point
common to the two planes exists, let it be P. Then in the triangle

PCD, ^th the angles PCD, CDP would be right angles. As this is
impossible, the planes a, have no common point and they are therefore
parallel.
It foUows as a coroUary that through any point there is one and
only one plane which is normal to a
given straight line.
Example 2. SAotw that straight lines which are normal to the
same plane* uro
are
parallel to one another,

Flo. 1G$
368 PURE MATHEMATICS [19
Let (Fig. 158) AB. PQ be perpendicular to the plane BXYQ.
BX is
drawn in the plane BXYQ perpendicular to BQ. Then AB.
BQ, PQ
are all perpendicular to BX and hence are coplanar.
But AB, PQ are
both perpendicular to BQ and hence they are parallel.

Theorem There is one, and only one, straight line cutting


7. at right
angles each of two given skew straight lines and its length is
the shortest
distance between them.

In Fig. 159, let AB, CD DE be


be the given skew lines and let
a line through some point D on CD parallel to AB. Then AB is
parallel to the plane EDC. Through any point A on AB draw AR
normal to the plane EDC and let the plane BAR cut CD in Q. Since
AB is parallel to the plane EDC and AB, RQ are coplanar, AB is

parallel to RQ. Draw PQ in the planeBAR parallel to AR to meet


AB at P. Since PQ is parallel to AR and AR is normal to the plane
EDC, PQ is normal to the plane EDC. Hence PQ is perpendicular to
CD and DE. But DE is parallel to AB, so that PQ is perpendicular
to CD and AB.
To show that the common perpendicular PQ is unique, let P*Q'
be another common perpendicular. Then P*Q' would be perpendicular
to the plane EDC and thereforePQ, P’Q' would be parallel. This is
impossible as AB, CD are not coplanar and hence there is one, and
only one, common perpendicular.
As AR is perpendicular to the plane EDC, AR is perpendicular to

CR. The triangle ACR is therefore right-angled at and R AR


is less

than the liypotcnuse AC. PARQBut is a parallelogram so that

= AR PQ than AC. Simil^ly we can show th^


PQ and hence is less
CD. PQ
PQ is less than the length of any other line joining AB and
therefore the shortest distance between the given skew lines.
is
plane can be drawn through one of two skew lines
It follows that a
19] SKEW LINES 369
parallel to the other and that the perpendicular distance of any point
on the second line from this plane is equal to the shortest distance
between the skew lines.

Example 3. One end of a rectangular box of


edge a. AP
length 2a is a square ABCD of
If ts a diagonal of the box,
find the length of the shortest
distance between APand DC. (L U )
From Fig 160. it is clear that the edge BC is parallel to the plane
diagonal AP, The shortest distance between
A
and BC is therefore equal to the distance of
AP
C from the plane ADP and

Fic. ICO

+ CT-P* == a« -f (2(j)* = 6a*,

^ CX 2a

and the required shortest


distance = CJf = 2a/ ^5.
It foUows also that paraUel planes
can be drawn through two
skew
370 PURE MATHEMATICS [19

lines. Use can be made of this fact to show that a tetrahedron can
be inscribed in a parallelepiped with one edge of the tetrahedron lying
on each face of the parallelepiped. Thus in Fig. 161, the edges of the
tetrahedron ABCD are the diagonals AB. BC, CD, DA, BD and CA
of the faces AB’BA\ BCCB\ CCDD\ AA'DD\ BA'DC and
CD'AB' of the parallelepiped.
In working problems on the tetrahedron, it is often useful to sketch
in the circumscribing parallelepiped. In particular cases, concealed
properties are often revealed by a figure such as 161 and instances of
its use occur in Example 4 and in Exercises 19 {b), No. 6, 19 (e),
Nos. 1, 5 and 12.

Example 4. In a Ulrahedron ABCD, BC is perpendicular to and AD CA is


perpendicular to BD. Prove that AB is perpendicular to CD.

Referring to Fig. 161, it is clear that the diagonals B'C', of the AD


parallelograms B’BC'C, AA'DD' are parallel to one another. Since BC
is perpendicular to AD. BC is perpendicular to B'C' and the parallelo-
gram B'DC'C has diagonals which are perpendicular to each other and it
is therefore a rhombus.

Similarly, since CA is perpendicular to BD, it can be shown that the


parallelogram BC'DA' is also a rhombus. Hence
BB' = BC' = BA',
and the parallelogram B'BA'A is also a rhombus. It follows that its
diagonals D.4, B'A' are perpendicular and, since B'A' is parallel to CD,
BA is perpendicular to CD.
EXERCISES 19 (6)

The ABC is right-angled at


triangle A point P not lying in the
plane of ABC is equidistant from A, B and C. Prove that the line
PN ioining P to the middle point N of BC is perpendicular to the
(L-U-)
plane ABC.
The triangle right-angled at A. Any point P is taken on
ABC is

the perpendicular through A to the plane ABC, and a


point Q is

taken on CP (or CP produced) such that BQ = BC.


Show that
(N.U.)
4Q = AC.
3. AB. CD are two skew lines equal in length and P. Q are the mid-
points of AC. BD. Prove that PQ is less than AB.
are three concurrent straight lines
each of which is
4, OA OB OC
the perpendicular from
perpendicular to the other two. The foot of
9 to the plane is ABC
H. Prove that H
is the orthocentre of the

Tiangle ABC.
vertex V, in which .4 B —
.

4BCD is the square base of a pjTamid.


6 .

md VA = VB = VC = VD = 5a. Prove that the shortest distance


pctwcen AB and VC is
(3V7«)/2.
a tetrahedron are equal in pairs, show that
6 . [f the opposite edges of
join of the
;he shortest distance bet^veen any pair is the
)oints.
19] ORTHOGONAL PROJECTION 371
7. Two equal skew straight lines AB, CD are inclined to each other
at an angle of 60®. The shortest distance BC between the two lines
is also equal in length to AB
and CD. Show that AD^ = 2AB^.
8. A square lamina ABCD, of side a, is rotated about through a AB
right angle to take up the position ABC'D'. Show that the shortest
distance between AC' and BD is a/VS. (L.U.)

19.5. Orthogonal projection and dihedral angles


The foot of the perpendicular from a point to a plane is called the
orthogontd projection of the point on the plane. When
a point traces
out a given figure, its orthogonal projection on a
given plane will
trace out a figure in this plane. This second figure is called the
orthogonal projection of the first figure on the
given plane.
Theorem 8, The orthogonal projection of a straight line on a plane
iS either a point or a straight line.

the Une
If AB
is normal to the plane a (Fig.
162 (a)), its orthogonal
projection is dearly the point C in which
the line meets the plane.

If the line is
not normal to the plane, let A. P. B
(Fig. 162 (M)
toee points on the line .IB and let A P',
B' be their orthogonal
^
projections on the plane oc. Since
AA', PP', BB' are all normal to
“fie another and therefore coplanar.

a straight line and its orthogonal iro-


Hne in the pLZ
372 PURE MATHEMATICS [19

In Fig, 163, A'B is the projection of the line AB on the plane a.


BC is drawn equal in length to BA' and paraUel to any line m in the
plane a.

Fig. 163

Since A A' is normal to the plane a, the angle CA'A is a right angle
and the largest angle of the triangle CA'A. Hence
is is less than

AC. In the triangles ABC, ABA', BA' =


BC, BA is common and
AA' is less than AC. Hence the angle ABA' is less than the angle
The angle between a straight line and its orthogonal projection
on a plane is defined as the a)\gle between the line and plane.

Suppose (Fig. 164) that is any P


point m
the line of intersection

of UTO planes a and p. PA. PB are lines, one m


each plane which
of intersection a/? of the two planes.
are perpendicular to the line
as lines of greatest slope. 1 he mutual mchnation
Suclf linM are known

,L<ed and the ,nctina,ion of AC to the hor,eontal m


JaZte is

position.

the initial and the final position of


ABC
XDC is
since the triangles ABC. XBC are both
booi
^ i, the mid-point of BC. and
191 ORTHOGONAL PROJECTION S7d

isosceles, AD and XD are both perpendicular to DC. A' is the orthoff


onal projection of A on the plane XBC and. from symmetry, it lies
on
DX. Since AC ^
2a, DC ^ \BC \a, and the triangle ADC «
o id
right-angled at P,

AD^ = AC^ - PC* 15a*


= {2a)* - {^a)* =*

AD. DX are lines of greatest slope in the planes ABC. XBC and AA'
^ « perpendicular to the plane XBC
and therefore perpendicular to
A D. Hence the angle through which the triangle is rotated,
or the angle
between the planes ABC and XBC. is the angle
from the right-angled triangle ADA' hy
and this is given ADA\

sin = AA'
(ADA')
a 2
=
AD (V15a/2) VIS
0-6 165.

so that the angle ADA' = 31® 6'.


The mclmation of AC to the horizontal
the plane XBC, and this is the
is the angle between and AC
projection A C. Since
angle between and its orthogon^ AC
is normal to the plane
ATBC, the angle
^ CA'A
IS a right angle and
the triangle CA'A gives

a
®
sia {ACA') = ^

AC 2a 2
*

so that the angle ACA' is 30®.

is 0, the
length of the orthogonal
projection of AB on a is AB cos (

Let (Fig. 166) A'B' be the


orthogonal projection of on the

Fig. 166
374 PURE MATHEMATICS [19

plane a, so that AB and A‘B' are coplanar. Draw AC parallel to


A'B' to meet BB' in C.
Then AA'B'C is a rectangle and A'B* = AC. Since AC is parallel
to A'B’, the angle BAC is equal to
the angle between the line
0, AB
and the plane a, and the right-angled triangle BAC gives
A'B' ^ AC ^ ABcosO.
It follows as
a corollary that the ratio of lengths along the same
or parallel lines is unaltered by orthogonal projection.

Theorem 11. The area of the orthogonal projection of a plane figure


of area A is A cos p, where p is the angle between the plane of the figure
and the plane on to which it is projected.

In Fig. 167, the plane figure has been divided into strips, of which
ABCD is typical, by lines AF, BC of greatest slope. Lines AE, CF
are drawn parallel to the line of intersection XY of the plane of the
figure and the plane a on to which it is projected.

Fig. 167

Since are parallel to one line (the line XY) of the plane a.
AE, CF
they are (theorem 1) parallel to the plane a and therefore
make zero
orthogonal projections
angles with this plane. By theorem 10, their
A'E', CF' are respectively equal to and CF, Since AF, CE are
AE
lines of greatest slope, they each make
an angle p with the plane a
and their orthogonal projections are of lengths AF cos p, CE cos p.

Hence the orthogonal projection A'F'CE' of the rectangle


AE and AF cos and its area is equal to dA cos p,
a rectangle of sides P
where SA is the area of the rectangle AFCE.
By increasing sufficiently the number and decreasing
sufficiently

the difference between


the width of the strips such as ABCD,
sum of that of the rectangles such as AFCE
of the given figure and the
as we please, and since each rectangle is dimin-
can be made as small
in the ratio cos^ 1, the projected area will, by
ished by projection
:

a limiting process, be A cos p.


19] ORTHOGONAL PROJECTION 376
Example 6. XY the line of intersection of two planes a, jS nuitually
is
inclined at an angle 8. Z
is a point in the plane a such that 1 and XZ =
the angle ZXY = Prove that the length of the orthogonal projection of
the line XZ on the plane ^ is \^{cos* <f> + sin* <f,
cos* $).
Z' isthe orthogonal projection of Z on the plane
^ (Fig. 168), and PZ.
PZ are Lines of greatest slope in the two planes. From the right-angled

Fig. 168

triangle ZXP, XP = cos ZP «= sin since XZ = 1. The right-


angled triangle Z'ZP gives
^'P = ZP cos 0 = sin ^ cos 8.
Finally the right-angled triangle Z'XP gives
XZ’ = ‘s/{XP* -I- Z'P*) =3 ^{cos* p -h sin* cos*
^ 0),

EXERCISES 19 (c)

1, ABCD the floor of a room. A'B’C’D' is the ceiling,


is
A’ being above
A and so on. If = 18 ft.. AB12 ft.. AA' AD = =
14 ft., fed the
angle beUveen the diagonal AC’ and
the floor of the room (O C )
2.
Jih
*
0 /?
-4/1'. BB’. CC of a triangular prism are

^4^!^ rL
ttA C to the plane BA C.
^ inclination of ^ plane
(O C )
3.
^“^^f/^l'ectangnlar box of length 2a is a square ABCD of
^ calculate
AP^a I
the plane ABCD, (U) the angle between
(i) the angle bebveen
AP and SC.
4. base ABC oi^ tetrahedron OABC is a right-angled
isosceles
mngle Md the length of the hypotenuse BC is
10 cm. OA = 1 2 cm

6.
Sic- iS z x'”c:;r„Tir
(i) cos S^C. cosCBA. cosACB
....
{n) .
the angle between the
K+^»*)(6> + 6»)(C»+«1)»
faces VBC and ABC is
be
COS” 1 ,f

^
V(6*c* + c*a* -b a*6«) f {L.U.)
376 PURE MATHEMATICS [19
6. y is the apex of a pyramid on a square base A BCD.
VA ^ VB = VC = VD = 13 in.
and the side of the base is 10 in. Find
(i) the angle between a slant face and the base,
(ii) the angle between adjacent slant faces,
(iii) the angle between opposite slant faces. (L.U.)
7. If A the area of the normal section of any prism, show that A sec 0
is

is the area of an}'’ section inclined at 0 to the normal section.


A
cylinder whose normal section is a circle of radius a is cut obliquely
by a plane. Prove that tlie area of the oblique section is nab where
2b denotes the longest diameter of the section. (L.U.)
8. The vertices A, B, C of a plane triangle ABC are at heights 5, 13
and 25 ft. above a horizontal plane. The orthogonal projection of
the triangle ABC
on to the horizontal plane is a triangle A'B'C'
(A' being the projection of A, etc.) and B'C' = 16 ft., C'A' = 21 ft.,
A'B' = 15 ft. Find the lengths of the sides of the triangle ABC
and the cosine of the angle which its plane makes with the horizontal.

19.6. Some geometrical properties of the sphere


This chapter concludes with a consideration of a few of the geo-
metrical properties of the sphere. These are all of an elementary
character and no attempt is made here to discuss what is usually
called Spherical Geometry, the geometry of points and lines lying on
the surface of a sphere.

Theorem 12. Fvefy plane section of a sphere is a circle.

Let be a section of a sphere by a plane and let 0 be the centre


PQR
of the sphere (Fig. 169). Let be the foot of the normal from 0
N

OP, NP. Then, since the angle ONP


on to the plane PQR and join

is a right angle,
^
point on the boundary of the plane section.
Similarly, if Q is any other
19] PROPERTIES OF THE SPHERE 377

we can show that


NQ^ = OQ^ - om,
and since OP = OQ = radius of sphere, it follows that NP = NQ.
Similarly all points on the intersection of the plane and sphere are
equidistant from N and hence lie on a circle.
Theorem 13. The curve of intersection of two spheres is a circle.

Let 0, 0' be the centres of the two spheres and let P


be any point
on their curve of intersection (Fig. 170). Draw PN perpendicular to
00\ Let the spheres be cut by a plane through 0, O' and P and let

Fig. 170

the semi-circles bounded by the diameters along 00' revolve


the line
about 00'. Th^e semi-circles will generate two
spheres and their
point of intersection P will generate a circle of radius NP and centre N.
The curve of intersection of the spheres is therefore a circle whose
plane is perpendicular to the line of centres of the spheres.

Theorem 14. A sphere can be drawn through four points not in the
same plane.
In Fig. 171 let A, C, D be the four points, let Z be the middle
^mt of BD and let X, Y be the centres of the circles through B,
L, D and A, B, D respectively. In the plane XYZ, draw XW, YW
perpendicular to XZ, YZ respectively to intersect at the point W,
4

Fig. 171

S perpendicular to the

plane BCD. Similarly YW as perpendicular to the jnn


378 PURE MATHEMATICS [19

Since Y is the centre of the circle through A,B,D,AY = BY = DY,


the angles AYW, BYW, DYW are all right angles, and WY is common
to all three of the triangles AYW, BYW, DYW, so that these three
triangles are congruent and therefore ^ = DW. AW BW
Similarly it can be shown that the triangles BXW, CXW, DXW
are congruent and therefore BW
= CW = DW, Hence
AW = BW = CW = DW,
and W is the centre of a sphere through the points A, B, C, D.

Example 7, I/O
be a point outside a sphere and if two secants draum from 0
cut the sphere in points A, B
and C, D respectively, show that
OA.OB = OC.OD.
Since the lines OAB, OCD intersect at O, they are coplanar and the
section of the sphere by their plane is a circle (Fig. 172). Hence OAB,

Fig. 172

OCD are two secants drawn from O to meet a circle in A, B and C, D


respectively and from theorem {j) of Chapter 18 it follows that
OA,OB = OC.OD.

Example 8. //middle points of the edges of a tetrahedron lie on a sphere,


the
show that the opposite edges are at right angles to one another.
edges BC,
In Fig. 173, let P. Q. R. X, Y, Z be the middle points of the
CA, AB,DA, DB, DC of the tetrahedron. Since X, Y are the middle
points of DA, DB, XY is parallel to AB and equal to {AB. Similarly

C
Fig. 173
19] EXERCISES 379
QP is parallel to and equal to ^AB. Hence XY
and QP are parallel
and equal and therefore XYPQ is a parallelogram. The points X, Y,
P and Q are therefore coplanar and, since they lie on a sphere, they lie
also on a circle. Hence XYPQ is a cyclic parallelogram and therefore
it must be a rectangle. XY is therefore perpendicular to YP. But
%ve have shown that XY is parallel to A B and we can show similarly
that YP is parallel to DC, so that AB is perpendicular to DC. It can
be shown in a similar way that BC is perpendicular to and that CA
is perpendicular to BD.

EXERCISES 19 {d)

1. Two planes, inclined at 60^, intersect a sphere in equal circles of


radius a. If the circles have two common points whose distance
apart is a, show that the radius of the sphere is {a^/5)/2. (L.U.)
2 . A, B, C are such that the angle ABC is a right angle
If three points
and AB = BC, prove that the locus of points at which AB and BC
both subtend a right angle is a circle of radius AB/{2y/2). (L.U.)
3. Prove that the points of contact of tangents from a point P to a
sphere centre O lie on a circle. If OP equals d and r is the radius
of the sphere, calculate the radius of the circle and the distance of
its plane from P. (L.U.)
4. The radu two spheres are a and 6. and the distance between their
of
centres is c{< a -f 6). Show that the radius r of the circle of inter-
section is given by
2cr = b c){b 4- c — a){c 4- o — b)(a 4-6 — c) ).
6 . A sphere rests in a horizontal circular hole of diameter 2 in. and
the lowest point of the sphere is
i in. below the plane of the hole.
Find the radius of the sphere.
6 . Show that the locus of points in space whose distances from two
given points are in a given ratio is a sphere.
7. A and B are points on a diameter of a sphere, of radius a, at
equal
distances 6 from the centre. A straight line through A perpendicular
to meets the sphere in P. A
straight line through B perpen-
dicular to A B and inclined to i4P at an
angle 26 meets the sphere
in g. Prove that Pg» = 4a> sin* 6+46* cos*
6. (O.C.)
8 . Show that the radius of the sphere circumscribing
a regular tetra-
hedron, each of whose edges is of length
2a, is V6.

EXERCISES 19 {e)

1, Prove that the common perpendicular


to two opposite edces of a
re^lar tetrahedron is incUned at an
angle of 46** to each of the
other four edges.
2 .
are two given skew lines and a
^
third Une cuts them
at X
and y. Fmd. for different positions
of X
and Y, the locus
a point Z dividing XY
internally in a given ratio.
(L.U 1
3. A rectangular swimming bath is 60 ft.
lone and la ff a
'

380 PURE MATHEMATICS [19

6 ft. at the other. Find


(i) the length of the diagonal joining opposite bottom corners at
tlie deep and shallow ends,
(ii) the angle which this diagonal makes with the diagonal of the
deep end wall which it meets. (O.C.)

4. Three edges AB, AC, AD of a cube are produced to P, Q, R


respectively so that AP = AQ = AR — ZABj2. Show that the
plane PQR is parallel to the plane BCD and that the section of
the cube by the plane PQR is a regular he.xagon.
5. Prove that tne line joining the mid-points of one pair of adjacent
edges of a tetrahedron is equal and parallel to the line joining the
mid-points of tlic opposite pair of edges. Prove also that the lines
joining the mid-points of opposite edges are concurrent and bisect
each other.
6 . If a straight line is parallel to t^vo planes, prove that it is parallel
to their line of intersection.
7. Each of three concurrent straight lines OA, OB, OC is perpendicular
to a fourtli line. Prove that the lines OA, OB, OC are in the same
plane. (O.C.)

8 . OA. OB, are straight lines mutually at right angles, OD is


OC
perpendicular to BC and OE to AD. Show that OE is perpendi-
ABC. (O.C.)
cular to the plane
9. ABCDEF a regular hexagon of side a and is the base of a hexag-
is

onal pyramid, vertex V. If each of the edges VA, VB,


VF . .

is of length 2a. show that the


shortest distance between AB and
VE is (2V15«)/5.
10 ,
AB the common perpendicular to two skew lines
is
AC, BD making
an angle of 0 with one another. Show that
CZ>2 = AB^ + AC* -b BD* - 2 AC.BDcosQ.
the floor of a rectangular room and
A' B'C'D' is the ceiling,
11 A BCD is
AB = 34 ft., BC - 16 ftand
A' being vertically above A, etc. If

find the shortest distance


between AB and DB .

AA' = 12 ft.,

which AD = BC = a. BD = CA = h.
12 ABCD is a tetrahedron in
distance between
CD = AB = c. Find the length of the shortest
AD and BC. . - •

a plane A.
n PN is a line perpendicular to ^ ^
and the angle PBN.
;

pendicular PN , ,
, , .
.

edges of a cubical bloc ® ® *

14 OA OB. OC are adjacent


these edges. The corner O
middle points of
P n are the

16. Z Zc of a pyranud is a square ABCD of side 4 in. The height


19] EXERCISES 381

of the vertex V above


the plane of the base is also 4 in. and all
the edges VA, VB, VC, VD are equal. Find the angle between
the edge VA and the base. (O.C.)
1C. A rectangular hoarding 10 ft. high and 30 ft. long faces due north.
Calculate the area of its shadow on tlie ground when the sun is
(i) due south at an elevation of 50^,
(u) south-west at an elevation of 30®. (O.C.)
17. The hypotenuse Z?C of a right-angled triangle BClies in a horizontal
.<4

plane and its sides AC. AB arc inclined to this })lane at angles
a and ^ respectively. Show that the inclination of the plane of
triangle ABC to the horizontal plane is sin“*{v'(sin* a + sin* ^) ).

18. Show that the diagonal of a cube is equally inclined to all the edges
of the cube and find the angle of inclination.
19. OA, OB, OC are three mutually perpendicular lines and OA = a.
OD = b, OC = c. Show that the angle between tlie planes OBC
and ABC is

tan-> (L.U.)
be
20 . The corners A, B, C ot an inclined plane are at heights
z. triangle in
C, 2 and 11 ft. respectively above a certain horizontal plane
and
the sides are a = 18 ft., 6 = 16 ft., c = 21 ft. Find the lengths
of the sides of the projected triangle and thence find, to three
figures,
the cosine of the angle between the two planes. (L.U.)
21 . Two straight lines »», n are skew. Show that the centre of th®
sphere which touches tn at a given point P and n at a given
point
Q may be obtained as the intersection of three planes. (L.U.)
22 . Three spheres, each of radius a, rest on the horizontal base
of a
cylindrical tin. Each sphere touches the other two and also the
curved surface of the tin. Find the radius of the tin.
If a fourth
sphere, also of radius a, resting on the three spheres
just touches
the top of the tin, find the height of the tin.
(L.U.)
23 A tumbler of height 6 in. is of diameter 3 in. at the top and 2 in.
at the bottom. Obtain the radius of the
sphere which (a) just
touches the bottom of the tumbler and the
sloping sides, (b) fits
into the tumbler exactly half-way
down the slope. (L.U.)
24. » spheres form a ring on a horizontal plane, their
centres being at
the corners of a regular polygon,
and each sphere touches two
others. Anotlier sphere touches all the spheres
of the ring and
rests on the same horizontal plane.
If r denotes the radius of a
sphere of the ring and R the radius of
the other sphere, prove that
r = 4fl sin* {«/«).
25 .
^
Though a fixed point O
at disUnce c from the centre of a
sphere
planes are drawn at right angles to one
so another
tl>ree planes intersect the
sphere. Show that the sum
of
——

CHAPTER 20

MENSURATION OF SIMPLE SOLID FIGURES


20.1. Introduction
It has been assumed (Chapterthat the student is familiar with
4)
the idea of the circular measure of an angle and we assume here that
the following formulae in the mensuration of the circle are also
known :

(i) for a sector of a circle of radius r, in which the circular measure


of the angle between the bounding radii is 0, the length of arc = rO
and the area of the sector = \r'0 ;

(ii) for a circle of radius r (the particular case of (i) above in which
0 = 27i), the circumference = 2rrr and the area = nr\
It is also that the area of a trapezium, in which a, h are
assumed
the lengths of the parallel sides and h is the height, is known to be
J(fl +
b)h. Two particular cases of this formula to be especially
noted are :

(i) \i a = the trapezium


h, is a rectangle of sides b, h and area hh,
(ii) if a — 0, the trapezium is a triangle of base b, height h and

area \bh.

of this chapter include the mensuration of the simpler


The contents
solid figures such as the prism, pyramid, cylinder, cone and
sphere.

The methods of the calculus are used where these seem to be suitable.

20.2. The volume of a right prism


The unit of the cube whose edges are of unit length.
volume is
lengths a, 6, c can
A rectangular parallelepiped whose edges are of
subdivided into abc cubes with edges of unit length
and the volume
be
of such a parallelepiped therefore abc.
is

whose end faces are congruent polygons and


A right prism is one
are rectangles. Since any polygon can be su
whose remaining faces
number of triangles, such a prism can be considered
divided into a
triangular prisms, and it is funda-
to be composed of a number of right
to obtain a formula for the volume of such a prism.
mental
elevation of a right triangular prism, A
Fie 174 shows the end
from A to the base BC of the triangular end
being the perpendicular
ABC Suppose that the prism is cut by
face
side and
rectangular face of which BC is one
of two such planes with the end face ABC, K
of intersection
382
20 ] VOLUME OF RIGHT PRISM 583

Fig. 174

P, Q draw PS, QR perpendicular to GH


to meet it at S and 7? and
^rough G, H HE
draw GF, perpendicular to PQ to meet it at
F and
E. Let PQ, GH meet AD at L and M, and let AM = x, AL —
x ~\- dx.
From the similar triangles AHM, ABD, AM/AD = AH/AB, and
similar triangles AHG, ABC, AH/AB = HG/BC. Hence
HG/BC == AM
/AD, giving, since AM ^x.

Similarly, PQ + Sx) AD
The element volume dV of the prism of which PQGH is the end
of
elevation between the volumes of the rectangular paraUelepipeds
of which FFG/f and PQRS are
end elevations. We have therefore.
It « IS the length
of the prism,

HG.ML.h <6V < PQ.ML.h,


Substituting for HG, PQ and noting that ML = dx, these ineguaUties
can be wntten ^

dx < dV .{x + 6x)dx,

fr^om which it follows by arguments similar to those used in Chapter


that 10

dV BC ,

dx AD‘^‘^*
The total volume V is therefore given by
BC
h.x.dx
AD'

\BC,AD,h
— area of end face x length.
By subdividing any right prism
into its component triangular
384 PURE MATHEMATICS [20
prisms, follows that the volume of a right prism is equal to the product
it

of its length and the area of its end face. This result has been used in
the derivation of formula (12.1). viz., the volume K of a
solid body,
the area of whose cross-section by a plane at distance x from the
origin
is a function 5{;t) of x, is given by

(
20 . 1
)

where a, b are the end values of x for the solid under discussion.

20.3. The volume of an oblique prism


175 shows an oblique prism whose end faces are of area B
Fig.
and whose altitude (measured perpendicular to the planes of the end
faces) is h.

1‘aKing the Ar-axis perpendicular to the planes of the end faces, the
cross-section of the prism at any altitude x for which 0 ^ A is < <
also of area B, and (20.1) gives for the volume V of the prism,
ft

= 1Jo B dx = Bh. (
20 2 )
.

Hence the volume of any prtsm is equal to the product of the area of
the base and the altitude.

20.4. The volume of a pyramid


In the base of a pyramid
Fig. 176, V is the vertex and ABCD ...
which A'B'C'D’ ... is a plane section parallel to the base. VP
of
is the perpendicular from V to the plane ABCD
cutting t le . .

... at P'. By similar triangles we have


plane A'B'C'D'
A'B' VA' _
VP'
~AB~ VA VP’
20 ] VOLUME OF A PYRAMID 385

and then, by theorem 1 (§ 18.6),

area A'B'CD’ _ 7P'2


. . .
~ (20.3)
area A BCD . . . ~AB^

taken at V and the a;-axis along VP, the area 5(:c)


If the origin is
of a cross-section of the pyramid at distance VP' x from V is given =
by (20.3) as

(20.4)

where B
the area of the base A BCD ... and h is the altitude
is
VP of the prism. Substitution for S(x) in (20.1) gives for the volume
V of the pyramid
B
p x^dx -
?[?]
^ (20.5)
Hence volume of a Pyramid is equal to the product of one third
the area of the base and the altitude. Since a tetrahedron is a pyramid
with a triangular base, the same formula
applies for its volume.
Example 1. A pyramidstands on a square base and its top is
cut away by
a plane parallel to the base and 6 cm.
from it. If the area of the top of the
/rw/rttm M one-quarter of the area
of the base and thYvolume
of the frustrum is 350 c.c., find the length
of the edge of the base. (L.U.)
the pyramid, A'B'C'D' is the top
is
of the frustrum
^ perpendicular to the plane
mooting^A'B'C'D' ABCD a^
We have shown m
equation (20.4) that the areas of
paraUe! sections of
vert^xTo tS^ V^tJvpT^ f distances from the
and hance yp. since
If o IS the length of
^ “ft
an edge of the square base, its area
KP f ,1
volume of the pyramids VABCD. a*
is and the
VA'B'aO' are respectively pp
/a«\
and 4 yP' or 4a* and
*\I/* y when we substitute VP a 12, VP* ^0
BB
386 PURE MATHEMATICS [20

Fig. 177

The difference in the volumes of these pyramids is the volume of the


frustrum A'B'C'D'ABCD, so that
a*
4a> -j= 350,

leading to a = 10 cm.

Example 2. T/ie base of a tetrahedron is an equilateral triangle whose sides


are each 8 in. in length. The remaining edges of the tetrahedron are each
12 in. in length. Calculate (i) the height, (n) the volume, (fit) the total
surface area. (L.U.)

Let ADC be the base and V


the vertex of the tetrahedron (Fig. 178).
VP is perpendicular to the base and, from symmetry, P is the point of

Fic. 178

ABC, If T> is the mid-pomt


intersection of the medians of the triangle
of the edge AB,
DC' — AC' — AD' =* 64 — 16 = 48,

2 ^ 8\/3
so that DC = 4^/3 in., and PC = ^DC = .
m.

From the right-angled triangle VPC,

(

8v3\*
j
= T’
3o8

80 that the height VP of the tetrahedron is V(368/3) in*



20] EXERCISES 387
The volume V ol the tetrahedron is givenby
y = J.area A yiBC.VP = i.i.AB.DC. yp
1 1 16v/3C8
= -.-.8.4V3. r cu. in.

Since VJB is an isosceles triangle, the line joining V to the mid-point D


of AB is perpendicular to AB and the right-angled triangle VAD gives
KZ)« = VA^ — AD^ = (12)* — (4)* = 128,
so that the altitude VD
of the triangle is in. VAB
The three
sloping faces are triangles of base 8 in., height Sy/2 in. and therefore
each is of area 32^/2 sq. in. The base of the tetrahedron is a triangle
of base 8 in., height 4^3 in. and therefore of area 16^3 sq. in. Hence
the total surface area
16^/3 + 3(32^2) = (10^/3 + <iG^/2) sq. in.

EXERCISES 20 (a)

A rectangular swimming-bath is 60 ft. long and 18 ft. wide and


the bottom slopes uniformly from a depth of 3 ft. at
one end to
6 ft. at the other. Find the volume of the bath, (O.C.)
2 . OA, OB, OC
are adjacent edges of a cubical block of side 4 in.
P, Q, R are the middle points of these edges. The
corner O is
removed by cutting through the plane and the other corners PQR
are treated similarly. Find the volume of the remaining solid.
(O.C.)
3. A pyramid with vertex O
stands on a triangular base ABC, A
plane parallel to the base cuts the edges OA, OB,
OC in A', B\ C'
respectively. If = 0.^4 /3, find the ratio of (i) the surfaces, (ii)
the volumes of the pyramids OA'B'C', OABC,
(O.C.)
4. The three edges of a tetrahedron OABC meeting
at the vertex 0
^ve the same length a and make equal angles B with each
other.
Prove the following results :

(i) AB a= BC = CA =
2a sin
(u) if
p is the perpendicular from O to the plane ABC then
Zp* = (1 + 2 cos 0)a>,
'

the volume of the tetrahedron is


Ja*{l — cos0)(I
(iii)
-h 2cos0)^.

^ the same lias


^ diagonal of the base is t\vice
tne height of the pyramid,
C^culate, in terms of a, the volume, (i) (ii) the total surface area
wcludmg the base, of the pyramid.
(Ija!
6 . A right pyramid stands on a square base of
side 2a, any sloniny

fSh cutting the pyramid


Fmd the volume of the new pyramid
formed.
The base of a tetrahedron is an
(L 1 .
U
equilateral triangle of side
4a V3 ;
388 PURE MATHEMATICS [20

the remaining three edges are equal in length. A sphere of radius a


touches each of the four faces internally. Calculate the volume of
the tetrahedron. (L.U.)
8. The areas of the top and bottom of the frustrum of a pyramid are
respectively 24 sq. ft. and 54 sq. ft., and their distance apart is 10 ft.
Find the volume of the frustrum. (L.U.)

20.5. The volume and curved surface of a cylinder


The surface generated by a straight line which intersects the circum-
ference of a circle and which is always perpendicular to the plane of
the circle is known as a right circular cylindrical surface. The space
bounded by such a surface and two planes perpendicular to the
generators a right circular cylinder and its axis is the line joining the
is

centres ofits circular ends. In what follows the word cylinder will
be used to denote briefly such a body.

Fig. 179

Taking the origin at the centre of the circular base and the ^-axis
perpendicular to its plane, the area of any cross-section of the cylinder
is nr^, where r is the radius of the circular base.
Hence, if the altitude
of the cylinder is h, equation (20.1) gives for its
volume V,

Tir^ dx = nr^h. (
20 6 )
.

equal to the product of the area of its


so that the volume of a cylinder is

base and its altitude. » t t


curved surface of
has been shown in § 12.8 that the ^ea
It
of the
by
a frustrum of a right circular cone is given
ends) X the slant height.
i(sum of circumferences of the circular

a case of such a frustrum in which the circum-


The cylinder special
is
2nr and the height is h r A being
ference of each circular end is

radius of the base and the altitude of the cyander.


respectively the
curved surface of a cyhnder is given by
Hence the area S of the
S = 27trh. (20.7)
a

20] VOLUME AND SURFACE OF CONE 389


Example 3. Tke height of a cylinder is h and the radius of its base is r.

If V is its volume and A the total area of its surface, show that

~rA + 2V = 0. (O.C.)
The area of surface
total is that of the curved surface and the two
plane ends, so that
A = 2nrh + 2nrK
Also V = irrVt, giving h * V and substitution in the expression
for A gives

from which the required result follow's immediately.

EXERCISES 20 (6)

Three solid spheres of radii 2a, a, a are placed inside a right


circular
cylinder of radius 2a whose lengtli is such that each
sphere touches
the other two and touches one of the plane circular ends of the
cylinder. Find the volume of the cylinder. (L.U.)
2. The height of a cylinder is 21 in. and its volume 1055 cu.
is in. Find
the area of its curved surface.
3. The axes of three cylinders are the sides of a plane triangle
and
each cylindrical surface passes through the
opposite vertex of the
triangle. Show that the curved surfaces of the three cylinders are
equal in area.
4. A cylinder of height 9 in. is inscribed in a sphere
of radius 6 in.
Find the total surface area of the cylinder.
5. A rectangular piece of paper, 11 in. by 6 in., is curved so as to
form
the curved surface of a cylinder.
Find tlie volumes of the Uvo
cylmders which can be so formed.
6 .
height of a cylinder are respectively r
and h. The radius and length are increased
by small amounts p and
A re.«ipectively. Show that the volume
of the cylinder is increased by

of itself approximately.
(O.C.)

20.6, The volume and curved surface of a cone


The solid generated by the revolution of a right-angled triangle
about one of the sides containing
the right angle is knotra as a riht

XcS 7: rotation t£s


cone to don
denote
t \
briefly such a body.
® ""6 the word
In Fig. 180, the vertex 0
of the cone taken as origin and its
is

is Thrcentr^f^
*’ 7^ semi-vertical angle of the cone is
a and $
«di
radius Fe of. such
distance 00 =x below 0 The
a section is clearly * tan «, so"" that UiT^rof the
390 PURE MATHEMATICS [20

Fig. 180

section nx^ tan* a. If h


is is the height of the cone, equation (20.1)
gives for its volume F,

K = j" nx^ tan* (xdx = 7t tan*

= ^.TA*tan*a. (20.8)

This formula can be cast into an alternative form by observing that,


if r is the radius of the circular base of the cone,

r = h tan a, (20.9)

and, elimination of tan a between (20.8) and (20.9), gives

F = j7ir*A. (20.10)

The formula given in § 12.8 for the surface of a frustrum of a cone


can be adapted to give the surface of the complete cone by observing
that the circumference of one of the circular ends of the frustrum is,
in this case, zero. Hence, if I is the slant height (see Fig. 180), the
curved surface 5 is given by
S=^nrL (
20 11 )
.

From the diagram I = k sec a, and this, together with (20.9), enables

the surface area to be expressed in the alternative form


5 = Tth^ tan a sec a. (20.12)

plane parallel to
Example 4. cone of height h is cut into two portions by a
A
product oj
the base. Find the distance of this plane from the vertex, if the
the volumes of the two portions is to be a
maximum. ( •) •

distance from the


Let a be the semi-vertical angle of the cone and the
the complete cone
vertex of the cutting plane. Then the volumes of
t&n a.
and the upper portion are respectively ^ 7rA»tan*a and inx
are therefore
The volumes of the two portions into which the cone js cut
irrx^ tan» a and - x^) tan« a. The product of these two volumes
will be a maximum when the function
F{x) = x*ih* - X*)
201 MENSURATION OF THE SPHERE 391
is a maximum. This occurs when
dF
s = 0,
dx
or when, x — A/*y'2.

EXERCISES 20 (c)

I. The altitude of a cone ec^ual to the


circumference of its base.
is
Find expressions for the volume and total surface area in terras of
the radius r of its base.
2. The faces of a regular tetrahedron are equilateral triangles of side a.
A cone is inscribed, having the same vertex as the tetrahedron and
base the inscribed circle of the opposite face. Calculate tiie curved
surface of this cone. (L.U.)
3. A cone of height h is inscribed in a sphere of radius R. Find an
expression for its volume. Hence show that the greatest volume of
a cone which can be inscribed in tlie sphere is (O.C.)
4. A cone cut into two parts of equal volume by a plane parallel
is
to its base. Find the ratio of the curved surface area of the part
which contains the original vertex to the curved surface area of the
other part. (L.U.)
5. The radii of the ends of a frustrum of a cone are a —b and a
and
+ 6
its slant height is 2h. Show that

(i) its volume = -r-(3a* + 6*) — 5*),


(ii) its total surface area =* 2n{a^ -f 2ah + 6*).
6 . A bell tent consists of
a conical part above and a cylindrical part
near the ground. Show that, for a given volume
and given radius
of the circular base, the area of canvas
used is a minimum when
the semi-vertical angle of the cone is cos-^
(2/3).
7. IfR, r are the radii of the larger and smaller faces
respectively of
the frustrum of a cone of height h,
show that its volume is
nh{R* + r*)/3 and its curved surface is
n[R^r)y/{h^ + (R
8 . A cone is inscribed in a sphere of radius a, If the vertical angle
of the cone is 26. prove that its volume is

sin* 0 cos* 0.
By putting x « cos* 6 find the value of x, and
hence the value of
6, for which the volume of the cone
is a maximum. (L.U.)

20,7. The mensuration of the sphere


®^ the centre of a sphere of radius r. 04 is a
vertical
radms which is taken as the *-axis, and PQR is a horizontal
circular
^ right-angled

NP» = OP* — ON* =r* — X*.


392 PURE MATHEMATICS 120

Fjg. 181

Hence the area of the circular section PQR is n{r^ — x^), and, by
(20.1), the volume K of a frustrum of the sphere bounded by parallel
planes at depths h, k (h < k) below 0 is given by

= nr^(k - A) - (20-13)

By \vriting /i = 0, A = r in this formula, the volume of a hemisphere


is given by
Inr^,

and hence the volume of a complete sphere is

(20.14)

Formula (20.13) applies, of course, to the volume of a frustrum


sphere.
whose limiting planes are on the same side of the centre of the
but on opposite
If the planes be at distances h, k from the centre
point, the frustrum can be divided into two by a plane
sides of this
The volume of the first frustrum is then
through the centre.
_ 0) - J7r(^3 - 0") = Tir^k -
of the second similarly
and the volume is

nrVi —

Fig. 182
20] MENSURATION OF THE SPHERE 393

Hence the volume of the whole frustrum is

nr‘^(k 4- /;) - Jvr(/t3 ^ (20.15)

The volume of a sector of a sphere can be found by dividing it


into a cone and a spherical cap. Thus, Fig. 182, if r is the radius of
the sphere and k is the height of the spherical cap, the height of the
cone is — k. Since OP =
clearly r r, the right-angled triangle ONP
gives NP^ = OP^ — ON^ = r 2 _ gQ volume of the
conical portion is
- {r - k)^}(r - k).
By writing k = r, h = r~km (20.13), the volume of the spherical
cap is

nr^{r - (r - k)} - - (r - k)^},


and, by addition and some reduction, the volume of the sector is
found to be
(20.16)
The area of the curved surface of
a spherical frustrum or zone of
radius r in which the perpendicular distance between the parallel
plane
ends is h is the area of the surface of revolution obtained by rotating
the arc of the circle ^2 _j_^2 _ ^2 included
between the points for
which X = a, X = b, where h — a h, about the jc-axis. =
By equation
(12.27) of § 12.8, the surface area 5 is given by

5 = 271 1 -f- {dy/dx)^]dx


J

y/{y^ + (ydy/dxY}dx. (20.17)

Since + jys = 2x + 2y{dy/dx) = 0 and

Hence (20,17) gives

S = 27r tdx^ 2jir{b - a)


j
= 2jirh, (20.18)
since b-a = h.
For a complete sphere, h = diameter = 2r and the surface area is

(20.19)

^ ^/aw« cuts a sphere of radius r into two segments


whose cumed
/ the sphere. Prove that the volume of the larger segment is Onr* /8.
(L.U.)
y ^ « distance of cutting plane from the centre of the sphere.
.

Then
304 PURE MATHEMATICS [20
the width of the two zones arc respectively r
-t- x and r —x Hence,
by (20.18),
277r(r + x) 3
2nr{r — X) ^ I
leading to = Jr.

The larger segment consists


hemisphere and a fmstrum whose
of a
bounding planes are at distance 0 and r/2 from the centre. By (20.13),
the volume is

Ittt^ + 7rr2(Jr — 0) — — 0®).


or On-r^/S.

Example 6. A
cylindrical hole is bored through a solid sphere of radius r,
the axis of the cylinder coinciding with a diameter of the sphere. Prove
that, if I is the length of the resulting hole, the volume of the remainder of
the sphere is nP/Q. (L.U.)

Fig. 183

Fig. 183 shows a section through the axis


of the cylindrical hole. EF
O is the centre of the sphere, ABCD
the section of the hole and the X
mid-point of AD. The right-angled triangle gives AOX
OX^ = AO^ - AX* = r» - f!,
4
so that the volume of the cylinder of which ABCD is a section is

g.
The volume of each of the spherical caps AED, CFB is given by writing
k = r, h = 1/2 in equation (20.13). The volume of each is therefore

ITT { - -D - i- - 3-
Hence, the required volume remaining

:7rr
3
jtP/0.

EXERCISES 20 (d)

1, Prove the equivalence of the two formulae


nh*{ZR - h) nhjh* + Sr*)
'
3 6
radius R.
for the volume of a cap of height h cut from a sphere of
20 } EXERCISES 395

r being the radius of the plane base of the cap. [Equation (20.13)
may be assumed.] (L.U.)
A 10 in. is divided by a plane into two parts,
solid sphere of radius
the volume of one part being half that of the other. Find the dis-
tance of the plane from the centre of the sphere, correct to the
nearest tenth of an inch. (Q-E-)
A spherical iron shell of outside diameter 14 in. weighs 150 lb. Cal-
culate its thickness, assuming it to be uniform, if the density of the
iron is 480 lb. per cu. ft. (Q-E )

A sphere is inscribed in a cone of height h and base-radius a. Show


that the volume of the sphere is

3{a + + /<*)}’*

A top is in the form of a solid piece of wood so constructed that the


portion OABC is a cone and ABCH is part of a sphere of radius a.

The cone is such that its generators OA, OB, OC, tangents
etc. are
to the sphere at A, B, C, etc. Prove that the area of the surface
of the top is
7ta*{l + sin 6)* cosec 0,
where $ is the semi-vertical angle of the cone. (L.U.)
A hollow cone of height 3a and vertical angle 60® has its axis vertical
and vertex downwards, A sphere of radius 2a rests on the cone.
Prove that the surface area of the part of the sphere ^vithin the
cone is one-quarter of the total surface area of the sphere. (L.U.)
A cone of height A and a hemisphere are on the same side of their
common circular base of radius r (A > r) Prove that the area of .

^at part of the surface of the hemisphere which is outside the cone
IS Ur^h/(h* + r*). ^^.U.)
A sphere rests in a horizontal circular hole of radius 4 cm.
and the
owest point of the sphere is 2 cm. below the plane of
the hole. Cal-
culate the area in sq. cm. of
the part of the surface of the sphere
below the hole and the volume in cu. cm. of this
part of the sphere
each to three significant figures.
(L.U.)
S9G PURE MATHEMATICS [20

20.8. Summary of some mensuration formulae


Some of the more important formulae are here collected for easy
reference.

Circle.
Circumference = 2.Tr. r = radius.
Area =
Length of arc = rO. 6 = angle between extreme radii.
Area of sector =
Trapezium.
Area = \(a -h h)h. a, b the parallel sides,
h — altitude.
Prism.
Volume = Bh. B = area of base,
h = altitude.
Pyramid.
Volume = \Bh. B= area of base,
h = altitude.
Cylinder.
Volume = nrVi. r = radius of base,
Area of curved surface = 27irh. h -•= altitude.

Cone.
Volume = ^nr% = tan^a. r = radius of base,
Area of curved surface = nrl h = altitude,
= Tth^ tan a sec a. / = slant height,
a = semi-vertical angle.
Sphere.
Volume — i7tr\ r = radius.
Area of surface = 4^r\
Surface of zone = 27irh. h = height of zone.

EXERCISES 20 {e)

by a chord into two portions whose areas are


1 A circle is divided
Find an equation for the angle 6 subtended
in the ratio 2 1.
:

the chord at the centre. , j


the
2. Two wheels of diameters 8 ft. and 6 ft. are in length of the^bel
centres are 9 ft. apart. Calculate the least
their
that will go round the wheels. ^
circ /
If the area of the
chords of a circle.
3 ^ P and /I e are equal
the area of the triangle A Q.
enclosed between ^ P and ^0 is twice

prove that
sin 20 -i- sin 0 = 0,

is the radian measure


of the angle PAQ.
where 0
® ))

20 ] EXERCISES 397
Determine an approximate value for 6 by drawing graphs of
the two sides of this equation for values of 0 between .^/3 and a/2.

(L.U.)
4. Show that the volume of a regular tetrahedron whose edges are
all
of length a is (v'2a*)/12.

5. A wooden block in the form of a cube of edge 26 has each corner


cut off by saw cuts through the middle points of
the tliree edges
in that corner. Prove that the total surface area of the block of
wood that remains is 6®(12 + 4^/3).
(L.U
OA, OB. OC are adjacent edges of a cubical block of
G.

are the middle points of the edges OA. OB


side fl. L M
respectively. The tetra-
hedron OCLM is removed by cutting along the plane
LMC. Find,
m terms of a, the volume of the solid remaining.
(L.U )
7. Show that the volume of a frustrum of a pyramid
xs

+ BiRj -H Bj*).
where A is the height of the frustrum and B.*
are the areas of
Its parallel ends.

8 .
1°''
a of radius
surface of the cylinder including
i^ts planf^ndr^
9.
Carved a^ohd
and a cylinder of the same
raZs
dius ttr circular
their bases coinciding and the axis of
the solid
hemisphere tou°te
* opposite end of the cylinder
hesn +
on
lies the cu^ed surface of the cone,
1,
find the volume of the
and express it as a fraction of the solid
volume of the cone (L U
10 .

volume TSSi viS’-vi" tri.? s


32/243 of the volume of the
IS
sphere. (L.U )
II. The radu of the circular ends of a frustrum of a cone are a and
6

12 .

portion a conical hole


the vertex of thA T w ^ is drilled out

13 .
“ “^®“‘ into a solid
con“''ZdrLtfiTo\\^“^
is as smaU **® '“"®'l “nrface
as possible
(L.U.)
398 PURE MATHEMATICS [20
14. A cube stands on a plane and is enclosed by a hollow cone which
also stands on the plane. Prove that the volume of the cone is
a minimum when the angle at its vertex is 2 tan-^ {1/(2
•v/2)}.
(L.U.)
15. Two floating buoys are made, one spherical and the other in the
form of a cone. Both are 5 ft. high and the diameter of the base
of the cone is also 5
Calculate the ratio of the surface areas.
ft.
The conical buoy is arranged to float with vertex upwards and
base horizontal, and half the volume is submerged. Find the height
of the vertex above the surface. (L.U.)
16. The semi-vertical angle of a cone is tan-' (3/4) and the radius of
its base is r. If its curved area is 8/15 of that of a hemisphere
of radius R, find the ratio R : r. (L.U.)
17. Three spheres each of radius a rest on a horizontal plane with their
centres at the vertices of an equilateral triangle of side 2a. A
fourth equal sphere rests symmetrically on top of the other three.
Find the height of the highest point of the fourth sphere above
the horizontal plane, and prove that the ratio of the volume of
the tetrahedron whose vertices are the centres of the spheres to the
volume of a sphere is 1 : tc\'2. (L.U.)

18. The figure represents a lens which is bounded by two equal spherical
surfaces of radius 6 ft., and whose nia.ximum thickness is 0-5 in.
Calculate,

Fjg. 185

the diameter of the lens,


(i) i.e., the length AB, (ii) its total
(L.U.)
surface area, (iii) its volume.
area of
19. Two spheres of radii a and b cut orthogonally. Find the
the first sphere which lies outside the second.
The distance between the planes of two circles is A times ^e
sum
20.
of their radii. Show that the surface areas of the spherical and
conical zones bounded by them are in the ratio V(1 +
A*) L :

Prove tot
21. AB a diameter of a sphere and iV is a point on AB.
is
spheres on AB, AN,
the volume of the space included between the
NB as diameters respectively is equal to half the
volume of a cylinder,
the radius of whose base is NU
and whose altitude is AB. where
AU is drawn perpendicular to to meet the outer sphere in U.
(L.U-)

inscribed in a sphere of radius R. Prove


22. A regular tetrahedron is

that
(i) the height of the tetrahedron is 47?/3,
plane of the base of tlie tetrahedron cuts the sphere into
(ii) the
(L.U.)
segments having volumes in the ratio 7 20. ;
20] EXERCISES 399
23. A wine glass has the shape of a cone with semi-vertical angle 30'’
and vertical depth a. The glass is completely filled with liquid
and a spherical ball is then gently lowered into the liquid until it
rests in contact with the inner surface of the cone. Prove that
the greatest overflow occurs when the radius of the ball is a/2.
(Assume that the ball is tiot completely immersed.) (L.U.)
24. A sector is cutfrom a piece of paper of radius r and formed into
a cone of semi-vertical angle a. For what value of a will the volume
of the cone be a maximum ?
25. Assuming that the earth is a sphere of radius 4000 miles, find the
area of the portion of it visible to an observer at an altitude of
10,000 ft.
;

ANSWERS TO THE EXERCISES


Exercises 1 (o). p. 16
1. (i) 3/4, - 1/2. (ii) 2-G43. 0-757.
3. ^ = 1, 2-5 < .r < 4. 4. 2q^ = Opr.
5. 4- b{b- ~~ 3ac}x -f- a® = 0,

Exercises 1 (6). p. IS
1. 0194. 1-357. O __ A
3.
2. 1. - 3. - 1 ± V5. 4 2. - 1 ; - 1/3, - 17/3,
5 - ± 2 , 4: 1 ; ± 1 . ± 2. 6. 5/2, - 1/2; 4, 1.

Exercises 1 (c). p. 20
1. ± (\/3+ V2). 2 . (^/12 ± VG ). -
3. ± {V(a + ib) + Vm). 4. 69/578, 37/578.

Exercises 1 (d). p. 20
;r > < < — 2.
0 and —
3. 3, — 1/2.
C .r

4. 2x= - Ux 7 = 0. 2.
1.
7.
3. g^x^ - (/>* - 2q){q^ + l).r + {q^ + 1}» = 0.
6.
8. - 11. 33/25. 10. ± ^/3/2. 1

11. x^ + a(p -f q)x + b{p^ -f q^) + (a* — 2b)pq = 0; 5, 1/5.


x^ + 9x + Qi = 0. 14. - 2, - 1. 1/3, 4/3.
3. 4.
12.
15. 9. 16.
8. 4. 7.
17. 4. _ 2/3 - 10, - 23/24. :
18. - 1.7 1, 5 5. 1 ; ;

19. ± ± 2 ± VQ. ± V(3/2).


1. :
20. a. a.
21. 5/12, 1/12 7/12. - 5/12. ;

22. 8. -8/3; 2,4; 4.2; - 1 i V(n/3). - 4= V(H/3). 1

24. (7/2) + 2v/3. 25. 1/2, 1. 1, 2.

Exercises 2 (o). p. 24
(i) 1/(45='..^). (ii) y. (i) 1/512. (ii) 3/2.
x^yyz\ 6. V-^/y-
4.

Exercises 2 (6). p.
(i) 0-3557. (ii) 0-0305
3-2.

6. (a) 2. - 4. (6) -

0-603, 6. 1-708.

7. — 4.
2-71. 1-71.

Exercises 2 (c). p. 31
- - - -
^ = - 5, ^
2. 3(rt b){b c)(c rt).
8 == 4.
I.
= — 39.
:

a = 3 - 6 m, 6 = 3m* - 3« -f 1.
A = 20. /i
4.
3.
5, A =0. B = - 2/5. C = - 3/5.

Exercises 2 (d). P- 34
3 2
2. X — ^ + -
- 5 + x'
3(:r + 2) 3{x
6

m 1

+
2

1)
‘ {;r + !)»•

400
4. 1 -
+ 1 + 1)
3

ANSWERS TO THE EXERCISES 401


2x- 13^
6 .

+ 1 X -4' C.
9{x* + 9)
+ 9(^ - 3)
+'

0{x + 3)*

3x 1 5 5 1
7.
+ 2x — X - 3’ 8 .

- 2) + +
x* 5 0{y 9Cv 1) 3(y + 1)*'

Exercises 2 (c). p. 34
1 . 0 .
3. 3n
4. (i) 3/2. (ii) 3. 6. (i) 0 06424. (ii) - 0*3107.
8. I. 2. 9. 0. 1.
10. a = 5/2. 5 = 9/2. 11. 512.
13. a = = - 12.
31. 5
15. - (6 - c){c - a){a - 6)(a* + 6* + c» + 6c + « «6). -j-
16. a = 1, 6 « — 6. c = 7. = - 1.
17. a *= 2. ^ ~ 3 a « - 5. ^ = 4. ;

18. a = - 5, 6 «= 6 - l)*(jr ^ 2)(.r ^ 3). ; (.r

10 . 2 .
20. X* + 2x + 2.

21 . +
1
lU + 8 1

24? + + 1 (X + 1)»'
1 ;r
22 +
12(4? - 2) ^ 12(4?* + 2.4 +
.

4)*
3 2 1
23.
X - 1 4? + 1
+
^ 4?* + r
2 1
21 .

3(1 +4:)« + 3(1 - X+X^y


1 3;r + 2
25.
4?
-f- 1 4? - 1 ^ + 4*
Exercises 3 (a), p. 39
1- (i) - I. 3. 7; 27. (ii)-l. 0; - + -
3. 220. -
3, 2187. (iii) 1» 1. I. + 1.
4. 3/2.
5. 5120.
6. 3. 76.
8. 1/3. 2.

Exercises 3 (6). p. 43
1. (a) 20. (6) 14*2. 2, 78-7.

3.
X* — 2x 3’ 4. 3.

6. 6. - 1/2.
^nverges when a < - 2 or a > 0. limit of
6. sum = I + a.
Also converges when a = 0, limit
of sum =* 0.
8 a = — 3, 6 = 2, c = 6
.
sum « Cm/ 2)(11 — n — 2«»). ;

Exercises 3 (c). p. 48
1. 1365.
2. 2520.
(i) (u) 6040.
3. 6.
4. 1024.
6. 266. Sum a= 711040.
6. 182.
V. (i) 60. (ii) 00.
8. 37 to 6.
9. 1/4.

Exercises 3 (d). p, 53
2. ^ 6V{2».3>-‘). 3. 607/16. eOi.
4. 8.
6 7.
.

00
402 ANSWERS TO THE EXERCISES
^ 1 2
- ~ + 4^* + 4^* + +
r+T2 + (1
4^r« 6x\
8. 0-7930, 0-7929.

Exercises 3 (e). p. 54
1. (m — «) {a + ^(m + n — 1)(& a)}; 77/75.
2.
16.
(3rt — c)/(a -f c). 5.
17.
25.
6. /I071. 7. ;^I47. ;^1193.
9. 11. 12. 26» X I0».
13. 186. 14. (i) 360. (ii) 144.
15 to 7. 16/21.
19. 1001, 2002. 3003. 20. a = - 1. - 80. 432.
23. 1 +
24 •
—_^- _ ^-
(1 ;^)2 I X I
!_
-{- X’

Exercises 4 (a), p. 62
1. cos $ = ^15/4. tan 6 — l/\/l5. cosec $ = 4, sec 0 = 4/%/15, cot 6 -v/16.
2. sin 0 = ± 3/5, cos ^ = ± 4/5.
3. (i) sin 6 = 4/5, tan 0 = — 4/3. (ii) sin ^ =— 4/5, tan 0 = 4/3.

Exercises 4 (6). p. 67
1. (i) - 0-5299. (ii) - 0-3420. (iii) 2-1445. (iv) - 2-5593.
2. sin 2A = — 0-96, cos 2^4 = 0-28.
3. 0*. SO*’; 60^ 90®; 0®. 150®.
4. 69® 39'. 6. 1-166.
6. 21® 28', 90®. 158® 32'.

Exercises 4(c). p. 71
1. n X 180® + (- 1)" X 18®, n X 180® - (- I)" X 54®.
2. 14® 2', 123® 41'. 194® 2'. 303® 41'.
3. 37® 55'. 25® 37'; 154® 23', 142® 5'.

18.

5. n X 180® + 35®, n X 180® + 45®. 0. « X 360® + 210®.

Exercises 4(d). p. 71

1. cos 0 = ± 2ab
, tan 0 ‘
a* + 5* 2ab
2 0
3 .
. .

smO = t/Vil +0 .
cos 0 = 1/V(1 + '•)- 9 = ^/(l + OA
sec 9 — V(1 + /*), cot 0 ^ \/t‘

4. - 1/2, - S/VS. 6.± 1/V2.


13. _ 4®, 60®. 14. 0. 7r/4. »r/2.
360®.
57® 54'. 122® 6'. 16. 0®. 63®, 135®, 281®.
15. 300®.
60®, 70® 32', 289® 28',
17. — 1-27, 0-11 radians.
19. (2« + l)7r/R>.
20. n X 360® ± 149® 21', n 360® + 78® SO'.
22. 30®, 45®, 150®, 210®, 225®, 330®.
21. r = 4-717, 0 = 148®.
23. n X 180® + 63® 26'.
not of the form 6r + 2 where r is zero or an
24. (2n + l)7r/10. provided is

integer.
25. 7r/6. 6ir/®»
ANSWERS TO THE EXERCISES 403
Exercises 5 (6). p. 80
, (1 + 0(3 + 0

i + /»

Exercises 5 (c). p. 84
3. cos (^ - B) = 1 _ sin

4. 0^ 90®. 120®. 135®. 240“. 315“, 360“.


6. 2n7T, (2n - l),7/2. (2» + 1)^5.
6. (4w ± l)77/(/> + y). {4« ^ l)jr/(/> — a).
7. 36“ 52'. 126“ 52'. ^ gogo
Exercises 5 (d). p. 87
1- w/4, 677/6. 77/3, 77/6.
1^2
Exercises 5 (c). p, 9l
28*41'.
2
2. 9.5
0-5
0 ft
5 ft.
6. 0-00776 ft.

Exercises 5 (/). p. 91
2. 45“.

4.
+ tan B + tan C — tan ^ tan B tan C
1 — tan B tan C — tan B' tan C tan tan A
It' ?*o 135“. 170“ 180“
lo. 63 26', 161“ 34',
243“ 26', 341“ 34'
16. 139“ 48'. 287“ 35'. ‘17 „ ^ o^o
18. 131-48', n-48'; j.-
^ +,
9 . 45 - » x ISO*.
1G3- 12' 48”
19. 60“. 03“ 20', 243“ 26'.
300“.
20. 38“ 23', 111*37', 218*23'.
291*37'
21. 22- 38', 36- 50';
« x 300” + 22” 38', n x 360”
+ 36” 50'.

Exercises 6 (o). p. 96
2- (4a5)/{fl + 6)».

Exercises 6 (d). p. 109

f

1 ^= ««“ = 27-35 in.
.4 =
.
4. 33 16', C= 44” 10'. a =60-21 ft
^ "“ “ = ^
« ^4?!°;

20', B= ,34” 20'.

?d ^ ' = «-25 »•
f- A - i'm”
108 B = 37“ 69', C = 34“ i'
.

8- c = 7-93 in.,
R = 4.39
Exercises 6 (€), p, 112
1. 14*11' 98* 26'.

3. d = 66“ 24', B« 22“ S' r ia,*,


8', 08“ O'.

Exercises 6 (/),
p. 114

2. tan-i
\ ainO /• 3. H-8 ft., 67“ 4r.
404
4. ANSWERS TO THE EXERCISES
7.
2I-9* (approx.). 6. 90-8 ft.
15-32 miles, S. 34® 25' E. 8. 1011 yd.

Exercises 6(g). p. 115


9*79 in. =
2. sin 0 0-8126, a = 95-7.
3. 180® - 2A, 180® - 2B. 180® - 2C.
6. 26. 30 in. 12. b = 7-22, c = 5-55.
13. A = no® 11® 51', a O', B=
17-09. b = = 3-91.
14. 108® 42', 48® 40'. 22® 32'. area 25950 sq. = in.
16. 624 sq. ft. 17. A = 75® 43'. c = 17-76 in.
18. 74®
49® 29'. 7'.

19. C = 28® 30', ^ = R = 75® 45'. sides 32, 65, 65 ft.


20. Angle ABC = 75® 3T, area =
(16v/15)/15 sq. in.
21. 6-61 miles, S. 68® E. 22. 114-7 ft.
23. 4913 ft. 24. 467-9, 784-7 ra.

Exercises 7 (a), p. 123


I. 9. - 1, 0. 2. 0, -3: 1/2, -2.
3. 1/2, - 1/2; «7r+ (- l)"(7r/6).
4.
6.
(i) - ± Vi* - *^)/2. y={-l± V5n*/2h (ii)

6. 3. 2-5, 2-25, 2-125 ft./scc.


6. 2 - 4;t - 2 2 - 4.r. 7. 3.r«.
8;r,
2.

Exercises 7 (b). p. 126 7.


8. 9.
1 . 15 ^:*. 2. 4x^ 2x. -
3.- l/x^. 4. 2 cos 2x.
5.-3 sin 3.r. C. 1 +
cos X.

Exercises 7 (c). p. 123


1. 3/» — 21, 0. 2/3 see. 15.
2. 32 ft./sec.>
3. 13 sq. cm. /see. 4. 0-4 sq. in.
3. n
0-2 per cent., 100-2. G. sq. in.

Exercises 7 (d). p. 129


1. 0. 8-402. 29-01. y = {2x*)/{4x* + 3), .0-842.

3. ax* -j- {2a A- b)x + a A- b V c. 4. /(x) = - lx* + \-lx* + + 9.

6. \/{2V*)- 2a;r + 6 — b/2a.


,

- 3, 2. —0. 8.
10.
10. 5. 11 . 1 .

12 . - 1, /2 1 . 13. (i) SxA- 1. (ii) - l/^**

14. (i) acosa;r. (ii) — sin 2.r. \l{x - 2)*. +


16. 6.r + cos X, 1. 17. 14-33 cu. ft./min.
432. 20. 1 sec., 1 ft./sec.*
18. (i) 300. (ii)

21. 75® 58'. 23. - 3 ft./sec.. ± 6 ft./sec.*

25. 8-3 ft., 83 ft./sec.

Exercises 8 (o). p. 134


1. 12;r* — cos;r. 2. 10 cos 2x.
— 2x*) cos X A- ** sin x,
4. 1 - 2x.

6.
(1
- 2.r(l + 4x*). 6. ;r*(3 cos ;r — ;r sin x),

7. 81;*-> + 108.r + 30.


2 sin X cos x.
8. ;r{ 2 sinxcos;r + ^(cos*^ -- sm*.ar)). 9.

40;r* — 2 sin ;r cos x + cos r — sin x.


ANSWERS TO THE EXERCISES 406
Exercises 8 (6)» p. 136
1 - - 5
1.
2
(1 '
2x)»’
— 5x 8x
3.
(1 + 2x^)^' (3 - 2x*)*’
2 cosx
5. ?
(I — sin x)^‘
6 .

(sin X + cos x)*’


7. cotx—x cosec* x. 8. 2 sec* X tan x.
cos^ X — sin* X
g
{sin X + cos x) * 10. — 2 cot X coscc* X,

Exercises 8(c). p. 140


I. 12(4r 5)». _ 5x*{x +
3. 2 cos 2{x a). —
2. 3)*(2x + 3).
4. 2sec*2x.
5. 3 sec 3x tan 3x.
7.-3 sin* (2 -
6. G tan {3x + I) sec* (3x -f 1).
x) cos (2 - x). 8. 2x{l x)(l - - 2x).
0. 3 sin* X sin 4x.
10. 4 sec* X tan x.
11 . sin«-» decs"-* 0{ut cos* 6 — n sin* 0).
12 . ~ (a//*) sin (2a//).
13. sinm-* X cos
(i)
x. (ii) cos (x™). (iii) _ sin x cos (cos x).
Exercises 8 (d), pr 142
1 •
+ ?)• 2. 30x*(2 - 6x>)-*,

3.
1
- 1
2V(1 + ;r)’ 4.
2(1 +.r)*/**
^- 1 - 2.r*
S. cos \/x
VH + •
6 .

~2y/x •

Exercises 8 (e). p. 144


6. sin-*x
+ x/^/(i_;r*)
Exercises 8 (/). p. 147

i.L- 3x*y*
- cos 2x
2x^y ' 2.
4/
.

2x
y
3 . - -{•
y f
X + 2y 4.
^
tan

Exercises 8(g). p. 148

1: (i)
' +« (-) - 29- + 2»-v.,
3. (.) cos / cia 3/ + 3 sin / cos 3<. (ii) 2/ sin- / + ./Vd - <•).
,

* (i) *. sin sr. (ii) i tan * (i _ i) sect


+
' -
: IVl 7 29 -u 2J).
8. (i) ?^L+|iL+l. (ii)__2*
+ 3V^
+ 1 )* 2(* + 2Vx + !) (“0
2x - vJfnjTyJ)-.-
7. (i)
-4 ;r> ** "
+ 10
(ii)
(a -3)* (iii) — ®’“ * 4- cos »
(1 + *)•
/
406 ANSWERS TO THE EXERCISES
fi
sin ^ — AT cos ^ AT — sin AT cos AT 2 cosa^ +
^ 3
®- (0 . (n) . (iii)
sin* AT AT* cos *Ar (2-1-3 cos at)*'

^at cos 2
9. (i) «Am-*{tan MAT + Af sec* mat), (ii)
sin* (at* — ^

1)

10. (i) y* + 2xy^. (ii) Qi3y 2)^.

(iii) (iv)
x{dyfdx) -y
y^ Xa
11. (i) 6{Ar* - x)^{2x - 1). (ii) 3 sin (2 - Sat), (iii) 2a: (sin 2x 4- ^cos

(”) - (1 + «)-'(20 +
^
(iii) 2Ar{A:» — 2)(;r* - !)-»/».

sec* (Ar/2) cos (Ar/2)


14. (i) (ii)
2V{1 -
. '
tan* {x/2))' '
2{1 + sin* (^2)}*

15- (i) —
V(1 - /*)
(ii)

- 2
16. (i) psin0. (ii) 2;rsec*{:y*). (iii)
rr^ *•

21. 2x/{x + y).


23. (i) (ii)
^
y >
Sy-\-x
2x sin y y sin x cos v — y* cos x
24. (i)
cos X — rrTTTT:-
a:* cos y
(»)
X siny -f 2y sin X
25. 1.

26. (i) Ar(6 — X*) sin a: -f 6 a:* cos at. (ii) (iii)
(1 + x^)^- (1 +;r)»-

X cos a: — 2 sin AT (6 — a:*) sin a: — 4 a: cos x


28.

Exercises 9 (o). p. 154


1. 38*2 in./min.
2. 6*28 sq. in. /sec., 65-41 cu. in. /sec.

3. 2 cm. /min.

Exercises 9 (6). p. 158


1. 3 (min.), 7/3 (max.). 2.-4 (min.), 0 (max.).
3. - 3. 1/3.- 4. - 3 + 2V2. “ 2^2
- 3.

6. — 3, 1 (min.), 3/2 (max.).


6. 25.

Exercises 9(c). p. 161


2. Each 6 in. 3. 432 cu. in.
7. 1188 sq. in.
6. 33 yd.
8. 6/(4 + 7t) in.

Exercises 9(d). p. 166


2. (2n 4- 1)"“. n an integer.
1. 0, 2/3.
2, X 2 gives a minimum, x = 2 gives a point of inflexion.
6. (i) ± 1* infinite, (ii) ± 1/^2.
ANSWERS TO THE EXERCISES 407
Exercises 9 (c). p. 166
1. (i) 25Tr/l6. (ii) 9ff/4 cu. in./in. 1-44 times.
2. 2 per cent./sec. 3. i/jg in./gec.
4. After 2 aad 7/2 sec. 0 ft./sec.. -- 6 ft./sec.»-
9/2 ft./sec., 12 ft./sec.» Rlin. vel. - 3/2 ft./sec.
5. 8 ft./sec.*
^ ~ y ~— c*/4a6. A mimmum.
7. 1 (min.), — 1 (max.).

10 .

\n + 1/
H. Length = = (256/x) + 2x>.
64/x*, area
12. Sum of volumes « (/* — 3/»x + 3x*/)/1728.
Mimmum volume = /V6912.
13. 4 in.
14. 7-875 cu. ft., 4 09 ft.
15. 4-8 m.p.h., 4*2 miles.
16.
18. Square of side ry'2.
19. 1 ft.
20. ^2 = 1-26.
21. — 17 (min.), 1*519 (max.), x = 4/3.
22. - 6/3a. 25. — 1/2 (min.), 1/2 (max.).
Exercises 10 (o). p. 173
I. (3x7/3/7) + C.
2. (- 3/x) 4- C.
3. X + X* + (xV3) + C. 4. (xV3) + 2x - (1/x) +
6. X - (1/x) + C. 6. sin X — cos X 4- C.
C.
7. 2 tan X + tan-* x + C.
<2"V3) + (,V2) - 3, + C.
9. - (/V2) 4- 7/ 4- C.
II. {ax'/2) +
10. C 2d - {l/B) - (I/2d»).
13. tan B ^ ^
(6*»/3) + („./4) ^ c. 12. d* 4- sin d 4- C.
d C,
14. (x 4- sin x)/2 4- C.
16. tan-* X 4- C.

Exercises 10(6). p, 175


L y==^x»~-x+l.
3. 8/3.
2. y^l^x^ {2xV3).
4. 13 ft. from the origin.
6.» =- dx
/ —
d*x
. X == 8;» - I2<» 4- 6/ - 1.
6. 30-33 ft. from the origin, 27-33 ft./sec.

Exercises 10(c). p. 182


1. 343}.
2. 4/3,
3. 81/10.
4. 16.
6. ir/2.
6
7. 1 - CV4).
. 1.

9. 41}. 8- in/Z) 4- (1/2).


11 10. 1/4.
. 1 /8.

13. V2 - 1.
12. y = 3x* - x\ 6}.

Exercises 10(d).
p. 185
1. 729w/36,
2. tt/U.
3. e26w/12.
6. 593jr/40.
4. Sn/5,
6. irV2.
Exercises 10(e),
1 /'\ ifk A P. 185

2- (i) 3/. -’m.®:;! + (iuv./,) + c.


408 ANSWERS TO THE EXERCISES
7.
8.
5. i{9 — sin 6) + C. 9.
6. 6>' = 6 + 9:r + 3^» _ 9. —
(5/2) cos t.

3y = - Gx^ ^ Qx -{• 3, 10/3. 5 ft. /sec., 5/3 ft.


2M - IV 3M - IV
2EI ’ “ 6£/ 11. (i) 29/6. (ii) 30057/1120.
15.
12. (i) 1/6. (ii) 0.
13. (i) 7t/Q. (ii) ^(6^ _ fljj sin 6 sin a.
14. 256. 2, ~ 3.
1.
16. >' = 9 + C.r - 3x*, 32. 2.
17. 27flV04.
18. 55-5. 19. 12S/5, c = 3 032.
21. 128Tr/5 cu. in.
22. Height r^/^a, vol. = = nr^/Ga, 24. 977/4. .

Exercises 11 (a), p. 191


- i(2-;r)*+ C. - *(1 - + C,
~ i[2x - 1)=
+ 4. |V(5^ - 7) + C.
3. 1/4.
5. 6 . 1 /2 .

7. (i) i(sin 2x + cos 2x) + C. (ii)


i tan {2x 1) C. - +
8. 7t/24. 9. sin-* ix/2) + C.
2 (2x + 1\
11.
)
+ C.

^ 1 (3x ~ 2\

Exercises 11 (&). p. 193


1. 77/2. 2. i« 4* i sin 2*.
577/4. 4. 1 + 2
( 77 / ).

5. 9/8. 6. 1/4.
10.
7. 1/2. 8. V2/4.
9. 5/0. 10 . 0 .

16.
Exercises 11 (c). p. 199

1. i sin {x* - + C.
1)
9(.r® + 8)®

3. 2 sin -^/x + C. 4. — {- cos* X C.


5. I cos® 2x — i cos 2x + C.
C. - i\/(l - 9-v*) - i sin-* 3x + C. 7. i tan-* X* 4- C.

8. -x^~) +2 sin-* (1) + C. 9. 106.

0-2165. II. M72.


12. a®/3. 13. 77/4.

14. 15. 2/3.


77aV^*
2.

Exercises II (d). p. 203


— 4- i sin 2x 4- C.
1. X cos ;»r + sin ;r + C. 2.

3. i sin® X C. 4. X sin-* X 4* V(1 •**) +
5. —y'(i — x^) sin-* X + X + C. 6 . — 277 .

8. (i) (77® + 877 - 32)/(2V2). (ii) 1.

Exercises 11 (e). p. 208


2. 0-6931.
1. 0-1054, 0-1054.
3.
ANSWERS TO THE EXERCISES 409
M sq- in. 4. 74.84,
6. 0-5236.

Exercises 11 (/). p. 209 4.

1. - 3)* + C. 2. ^ tan* X + C.

3. - V(4 - X*) + C. i tan-i (^-^) + C.

6. — cosec 6 — sin S C. 6. i tan-i +C


^
Vli + z‘)
c + 8. i tan» ^ + C.
18.
9. — I (cos 2x + ^3x) + C. 10. -^(.r + 2)vi(3;r - 4) + C.
II. 1/6. 12.
2. 435/237160.
13. 4/3.
14. 0.
16. 8/105.
17. 1/12.
16. (4V2 +2- ffV2)/8.
7r/4.
19. (tt* + 4)/10. 20. 2^735.
22. - 9/8.
24. 16-65.
2.
23. (i) .*r =s tan 6. (u) ;r = sin 0.
26. 677,

Exercises 12 (o). p. 215


3.
1. 64/3. 4.
128/Sl,
3. 7aV9. 4. 1/0.
6» 27r«
6. w(5a -f fa*).

3.

Exercises 12 ( 6 ). p. 218
1. 1 /2 .
71-5 Ib./sq. in,
3. 1-978.
4. 1.

Exercises 12 (c). p. 223


1. 3//4.
^ = 5.2 . ^ = 10-8. 9 =» 4-6.
RA
635/217, y
^
1276/498 ^ OO/Oi*
85/31.
6.
• eOTra a V*
C. of G* is on ^r-axis at distance
.

On j;.axis at distance 3-6 from origin. lOa/3 from origin. ^


Exercises 12 (d). p. 228

2
2. /)aV20.
^ ^ rod.

4. Mh*/2. where Af » mass.


.A/6*/9, where M » mass.
6. aV3.
6, d* + (6V4).

Exercises 12 (c). p. 232


1. 12 -
1.
2. frr/2,
3. 12.
4. 2Trr».
6. 241-2.
6. Stt.

Exercises 12
(/), p. 232
part of curve lies below #-axis.
2. l%^‘
4. 3. (96-v/6)/6,
8a V3. 32ffaV3.
6, 2260 cu, in. (3irV2) + 4tf.
7- (i) 0. (U) 2/n.

00 *
+
410 ANSWERS TO THE EXERCISES
11.
15.
S = i^a/Zrr, y = 46/37r. 12. X = a,r/2, y = aV8.
13. 16/15, X = 4/7, = 0.
16.
14. X = 3a/8. = 0.
X = 3/2, y = 0. paV3.
17. 2/9. 20. 3MaV5.
21. a(a - /3). 22. 108.
23. - fl}. 24. Sir.
25. 7r(10%/10 - l)/27.
6.
Exercises 13 (a), p. 240
1. xil(i) 2 1og,;r). + (ii) _ 1/x.
2- logcX, x{\ogfX 1) — + C. 3. 2 (approx.).
4. (i) 2x/{x* — 1). (ii) — cosec ;r.

1/V(^* + 1). logs {x + + I)} + C.

Exercises 13 (6). p. 244


1. (i) e2*(2sin3;r + 3 cos 3^). (ii) 2xe‘‘\
6. (i) (1 -
«-3)/3. (ii) i(e3 _ g-*) - 2. (iii) 2.
{x^ -2x + +
7. (i) 2)e* C. (ii) ie'(sin ;r + cos;r) + C.
8 .

Exercises 13 (c). p. 250


1. i loge (x^ 4- I) 4- c. 2. i log, (4.r» 4- _ 24r + 3) + C.

3. 3 iogg 2. 4. i log, (j:* + o') + tan-i (?) + C


6. loge (c* + e-*) + C. i (3 + 4 tan x) + C.
7. loge (loge 0 + C.
8. 2 loge (2/* 4- 2^ + 1) - 7 tan-» (2< 4- 1) 4- C.

o + {2^ - 3) + C. 10. 2;r - log, (t + 2) + C.


i
x^ x^
17.
II.
6 2
2x — 4 log, (4 — 2;r) + C.

> -
12. log.
^ + D 13. log, {(;r+l)*(;>r-2)*} +a
» 3
-2+2 i (^ + 1) +
15
16. 4 log, (.*r — — + c*
3)
;r - 3

16. 2 log, (.r - 3) - log, (*“ + 4) + tan-> + C.


^ (|)
log, (4/3). 18. (1/2) + log, (3/4).

19. (log,;r - -) 4- C.
Y
20.
Y
tan-1 :r
—^ 4- i log, {x* + 1) + C.

21 . X tan X + log, cos x + C.


22 &- •) log,
(4r 4- 4)
—— 4- 2^ + C.

23 ^ - log, (^r + 1) + C.
(^^n)
24. tan x (log, tan ;r — 1) + C.
answers to the exercises 411
Exercises 13 (d). p. 255
1. - 4{3 - 22;^ + 3Qx*).
3(1 - 5^)
2 .

2(1
3. lO^log, 10.

(logc X)^0ge (log, X)


+ J^^}.
9. A =2. B = C 9. _
8, 10. 2/3 < X < 2/3.
Exercises 13 (c). p. 256
1- (i) I/(sin xcos^r). (ii) secx. 2. e-K
1 1
3. (i)
(»)
X{1 + xy - 1)* 2y/{x*
6. (i)
+ 2 log, (^ - + C. i(l _ «-«/*) + log^ 2.
1) (ii)
8. a = _ a/2, = V(B ^ A^/i), 10. 2n{e* - «-»% 4).
(0 log, (e* — «-*) c. ff-i logj
^ c, (ii) (;yfl

13. (0 X - log, (x + 2) + C. 1 _ 4;y + 5 log^ (^ + 1) + C. (ii)

(i)
? log. 2. } log, - 1) - * log. + 3) - i log. + C.
(ii) (/
(/ /

15. (i)
i log, - 3)«) + C.
{;r(jr*
i log, - (ii)

2{x» 1)
+ C.
16- (1)* + f log. (r + 2) - V log, + 6) + C. (.r

(ii) 3 log, (* - 2) _
i log, (2* + 1) + +c
<“> * '"s*
Jg log,
18. P (U/6).

21. (i,
(U) + «^*^)
(5‘ sin 2x.log, 5.
(l~x»)V* •

23. 147/40.

25.
ti.O"

Exercises 14(a), p. 263

«: wtx tnl-
W5, V17. 6.
8. (5. 6)
+ »• + '•- - 2M.
Exercises 14 (6). p. 267
1. (2, 6/3). (- 20. 31).
6- (0 1.
2. V41, (- 1/3, 6/3).
(ii) a».
7. 21.
8 (8 - 3V3)/4.
Exercises 14(c).
p. 271
1. + y* — 6;r ^ gy ^
3. 2x -
y - 2 = 0.
2 x>
.
+ yt _ 8x - 8y + 7 - 0.
4. y s> ± 3x.
6- (- 1/11, - 19/11).
8-
2 (0. 0). (1.
( , 0). 2
( . 10 ); (
2 , 6 ).
2) ; V6.
Exercises 14(d).
p, 271
1. 6 + 2V3.
2. V08. V60, VIO8.
;

412 ANSWERS TO THE EXERCISES


3. (13/3, 8/3). 6. (1/6. 1/2),
V\>1* + - cos {$1 - »2)}
6 •
2 sin (^1- &t)
7. (1. I). (~ 5/2, 9/2), . (9/2, - 6/2).
8. (8. 0). 10. 51-8, 165-3. 209-5, 2500.
11. 10. 12. 4.
13. (4. 2), (0, 4). 14. 6x 6y = 63.
15. 15x^ — -f 2ax = a*.
17. 2x — y ~ 16 = 0, — y 4- 8 = 0. 2;»r

18. + y^ - 12x - 18y + 109 = 0.


19. 3;r - y — = 0. 1
20. 2x^ 2y^ ±x ± 13y 4 13 = 0. :

21.
6.
(lW2)/5. (11V13)/10. (11V17)/10.
22. (5. 6).
23. a = 6. 6 *= 3. AB = 1/12.

Exercises 15 (a), p. 280


10.
1. (i) V^x + y + 3 = 0. (ii) 3;r - 5y + 15 = 0. (Ui) 3;r + 2y = 0.
2. 4/3.
3. (i) 13, - 65/12. (ii) 5/12. (iii) 6.
4. (1, 1). 2.r + 3y = 5. 5. 3/-s/5.
5.*r
+y= 9. 7. -f- y = 8,
8. 2;r —y= 4.
9. X cos a 4- y sin a = 4: c\/(sin a cos a).
a» 4- fc* = 11. ;r - 2y 4- 2 =. 0. 4:r -y =
/ db — fttctb \
12. y + = 0. (a + bMy - i, + a6 = 0

Exercises 15 (5). p. 285


1. 135 ^
2. AB. 3x + 4y19 =
BC. x 3y ;
- = 2; CA, 6x ^ 2y ^ 3 = 0,
tan B = 13/9, tan C 13/11. =
3. y 4- 2 = 0, y —
V3x 4- 3^3 4* 2 = 0.
4. 29y — 2;r = 0.
5. (i) = 2. ;r 4.r 4- (ii) 3y 4- 1 = 0. (iii) 3^r - 4y = 0.
6. 4^ + 3y = 11.
7. ;»r_ y 4- 1 = 0 (2, ; 3) ; ;r +y = 6.

8. (i) - 2v = 7.
7;*r (ii) 3.V 4- = 2. (iii) (11/13, - 7/13).
9. (7. 7).

Exercises 15 (c). p. 288


1. (i) 1. (ii) 0 (point lies on line).
2. h cos a 4* A sin a. p.
3. Point is centre of an escribed circle.
4. - 4y = 3.
C.r

5. 2x - 169y = 11. 34;r 4- 27y — 67.


6. - 2y + 1 = 0. 24r 4- = 3.
;r >»

7. 4;r — 7y 4- 3 = 0. 49;r 4- 28y = 12 2^: ;


4- 3y - 5, 15^ — lOy 4- » =
_ 3y 4- 2 = 0. 3.r + y 4- 1 = 0.
8 . B(x -h)~A{y-k)= ± {Ax + By + Q,
Exercises 15 (d). p. 292
I, x+9y= 11. 2^r - 4y 4- 11 = 0. 2. ;r = 1, y “ 1.

3. (i) ;r 4* y = 1- (») ^ + 3y = 1. 4. 23;r 4- 23y =» 11.


;

ANSWERS TO THE EXERCISES


6. \8x + 29y + 4 = 0. 6 . 45 ®
8, 2x = 0, 2x ~ lly = Q.

Exercises 15 (e). p, 293


1. X - -
i y ^ Q.
2. AB = equation 4r — 3^
10, 4. 5 -= 0 ; equation to BC. :ix 4- 4v 4- 10 =
C is point (- 10, 5) or (6, - 7).
3. 4r + 3>' + 1 = 0, 4»r + 3^ + 1 1 0, 4t + 3>' = 9.
4. 4r + 3>' = 9, 4jr 4. + 11 = 0; 27/8, 121/24.

8. 3>/ - 6 0 4- = ; (6-6, 6-8). (3. 2) ; 8-4.


1 1. 4. 7>f 4- 20 0. = 7.r -> = 10.
12. (139/74, 353/74).
13. 3.r 4-
^ = 6, 4Ar 4-
y = 8 ; ;r = 1.

• <“ “>• <-• *>• (- 2- 1). (- 1/3, 13/3)


Irla
13. * + 2y = 8, 2jr - >, + 9 = 0 ; (-5. - 1).
~ (3- 3).
ol'

24
24. ^~ r
" 13^- = 9, 54->, = 0.
ll.v
0. 25. aHx‘ - y ’) + (6. _ J)xy ^ 0.

Exercises 16 (a), p. 297


1- (5, - 6), V61.
2- (i) x*-\.y»^ lox 4- 12>' - 30 = 0.
(ii) X* y* 2 ax 4- 2by 4- 25> = 0.
3. 4- — 24r — — 3 = 0.
4, 4- - 10^ - 0. 6. 4r» 4- y» — ?>- 4- 30 = 0.
11*
6. 4- 6>» 4- 2 *= 0. yt 6jr —
8. ** -
i- y * + 5;f ^ ^
7. X* 4-
^ 4- 4 = 0.

Exercises 16(6). p. 301


1. 3* 4.
y = 19, y = 2 .

2- (>) gx
+/y = 0. (ii) 4- 2v = 7.
_ 3^ ^ 25, 4x - 3;- + 25 =
3.
^
4. 3.r 4- 2^ =B 0.
(i.

4'>- -
Exercises 16 (c). p. 305
1. 3.
2 . 12 .
3* (i) (10, 2). (ii) 2.
6. 3 x~y = 1^ * — 3y 4- 6. X 4- 3>» = 7.
13 :
0; (2, 6); x»4-;.»-4r - 10>» 4* 10 - 0.

Exercises 16 (rf). p. 307


I- * y = 3.
2. 6* + y + 2 = 0. 2* - \0 y 4- 21 .

6. X* 4. y* _ 20x *= 0. X* 4- -18^+ 64-0


414 ANSWERS TO THE EXERCISES
Exercises 16 (e). p. 307
1- (i) 2, (1, 3).
(ii) (6. 3). (- 2, — 1) : - 23x + 36y - 15 =a 0.
2. (13/3. 16/3), {10\/2)/3.
3. + y^ — 6x — 4y ^ 28 = 0. 4. + y» — + 2y - 20 = 0.
5 Zx* + 3y® — 26;c — 16y + 61 = 0.
6. + y2 + I5y = 0, 3{x^ + y=) — 20y = 0.
'?•
(1. 4). 4. 40/9.
8. —
10. - 4y = 25. x = ± 5.
3;P
+ ± 8ay + II. 3x^ 3>* ia* = 0.
12. + y2 10;r - 5y + 25 = 0. (3. 4).
13. 40y = Ox.
14. (- 3), VlO/2
I. _ 3y + 15 = 0 ^2. : ;e

{x - 4)2 + iy ± 3)2 = 9. (^ + 8)2


:

15. + (y ± 15)* = 225.


16. (6. 0). (0. 3). (0. 4) 4r + 7y = 73 VH. ; ;

17. (1.0): \/3/2. 18.7.


19. 3x^ + 3y2 6.r_ — lOy + 3 = 0.
20. 5x^ + 5y2 -{- 24r ~ 3Cy = 0 ; 2x ^ 3y.
21. 24/5.
22. (i) (- 3-4, 2-2). (ii) 4.*: + 3y + 7 = 0. (iii) 289/24.
23. x^ y^ 8x lOy - 59 = 0. ;r2 + y* - 8;r - 6y + 21 = 0.
24. ;r2 + y2 - ll;r + Hy +4 = 0.
25. (1/2, 1). ^5/2.

Exercises 17(a). p. 313


1 . - 4y = 51.
2 . :r- 2y + 144 = 0, 2.r 4- y + 18 = 0 : (- 36. 54).
3. (a/lO. — a/2), (a/4, a) lC.r ; -f 4y + a = 0, 4.T — 2y + <*

(— a/8, a/4).
4. .r + y = 3.
6. 4y — ^2 _ 4. 16 ; y ^ ^ 3^ = 0.

6 . 90®. tan-i
rs/ a>'36»'2 M
l2U2/2T“«/2Vvr

Exercises 17(6). p. 316


1. — 1//|. o. xy = y{h — 2a) + 2aA.
4. /» = 0, /< = 2; ; X = 0.

Exercises 17 (c). p. 323


1. 2; 1/2; 3. 2. ;r» + 2y* =* 100.
v*
+ = 6. 2jr — y = ± 6.
25 IG
X y
- ^ =
*
6. 8.V 3y = 36, 3;r - 2y = IS, 10. - cos ^ 4- T
0
.

sin
,
1.
a

Exercises 17 (d). p 329


1. (± V13. 0) : V13/3; 8/3.
2. 3;r2 - y» = Sa*.
3. ;r - 2y + 1 = 0. + y 7 = 0. 2;«r
-f-

4. 30.r - 24y ± \/16l = 0.


7 .
(/> + l){x/a)
- (/2 - \){y/b) = 2t.
8. - sec ^
a
^ tan ^ = 1. ^
ANSWERS TO THE EXERCISES
Exercises 17 (e). p. 334
3. \/2. 4. ;r + t*y = ict.
5. 3x/5.
8. A rectangular hyperbola with the edges of the corner as asymptotes.
Exercises 17 (/). p. 334
1. (1. + 1 = 0.
0) :
;r 2. a®/*/2.
3. (aV*. + <*})• 7. - ~ (2//). r

10. 9;r - 4y + 4 « 0. T - 4y + 35 = 0.

n. V(7/3)x + V(20/3)y = 9. 4v'(7/3);r + V(20/3)y = 15 ;


36* 20'.
12. ( {a 4- cos 0, {a — 6 sin 0).
} }
14. (5. 15/2). 17. 1/V2.
18. ;r* -y« = 32,

Exercises 19(c). p. 375


1. 32* 54'. 2. 60*.
3. (i) 54*44'. (ii) 65* 54'. 4. (i) 77* 59'. (ii) 81*26'.
6. (i) 65* 23', (U) 100*. (iii) 49* 14'.
8. AB = 17. DC = 20, CA = 29 ft. {5v/3)/12.

Exercises 19(d). p. 379


3. r^/{d* - r*)/d. (<i> - r^)/d. 6. 6/4 in.

Exercises 19(c). 379


p,
2. Plane parallel to AB, CD.
3. (i) 63-23 ft. (ii) 70* 12'. 11. 9-6 ft.
12. s/{(i* + c*-a*)/2 }. 13. 1-465. 20* 56'.
15. 64* 44'.
16. (i) 261-7, (u) 367-4 sq. ft.
18. 64*44'.
20. 20-62. 16-2, 15-69 ft.. 0-843.
22. a(3 + 2^3)73. 2a(3 + V6)/3.
23. (a) M06 in. (6) 1-26 in.

Exercises 20 (a), p. 387


1. 4060 cu. ft.
2. 160/3 cu. in.
3. (i) 1 : 9. (ii) 1 27,
6. (V3a»)/2.
:
6. (v'2a»)/6.
(i) (ii) (1 + V3)a*
7. 18-6 a».
8. 380 cu. ft.

Exercises 20 (5). p. 389


1. 4(3 + 2V2)7Ta» 2. 527*9 sq. in.
4. 9(7 4 6'/7)(ir/2) sq. in. 6. 67-8, 31*6 cu, in.
Exercises 20 (c). p. 391
1. (2ffV»)/3; {1 4 V(47r» 4 l))^.!,
2. 7ra*/4.
4. 1-7.
3. (2R-*)(ffAV3).
8. X = 2/3. $ =• 35* 15'.
Exercises 20 (d). p. 394
2. 2-3 in.
3. 1-02 in.
8. 62-8 sq. cm.. 64-6
c.c.

Exercises 20 (e). p, 396


1. sin 0 - (2,r/3). 2. 40-1 ft
3« I«37 radians.
6. (23a»)/24.
416 ANSWERS TO THE EXERCISES
8. 27r(7 *4" 6 \/ /) scj. cm.
9. - a)/24 ; (3A - a)/(8A).
12. 27Tr^r + x)/3. 13. 20 cm.
15. 4: (1 + V5): 5/ e/2. 16. 5:4.
17. a(6 + 2v/C)/3.
18. (i) 12 in. (ii) 226 sq. in. (iii) 28-2 cu. in.

19. 277«2|i +
24. 54® 44*'. 25. 15M0:r sq. miles (approx ).
;;
;

INDEX
Addition theorems, for sine and cosine, Circles, orthogonal. 303 ;
radical axis
74 ; for tangent, 76 ; general proof to, 305.
of, 262. Circumcentre, of triangle. 348.
Ambiguous case, in solution of tri- Combinations, 44.
angles, 105. Common logarithms, 26.
Amplitude, of periodic function, 60. Compound interest, 40.
Angle, between line and plane, 372 ; Cone, surface and volume of, 389.
between two lines, 281 of depres-
; Conic section, definition of, 310.
sion. 112; of elevation, 112. Conical surface, definition of, 362.
Angles, multiple, 77 negative, 58
; Convergence of geometric series, 41.
of triangle in terms of sides, 101 ; Coordinates, of point dividing join of
small, 88 submultiple, 79.
; two points, 263 polar, 260
; rec- ;

Answers, to exercises, 400. tangular, 259 systems of, 259.


;

Apollonius, circle of, 346. Coplanar lines, definition of, 363.


Applications, of differential calculus, Cosine, addition theorem for, 75 dif- ;

151 ; of integral calculus, 211. ferential coefficient of, 125 formula ;

Appro.ximate methods of integration, for triangle, 94.


203 :Simpson’s rule, 207 ; trape- Cuboid, definition of. 361.
zoidal rule, 205. Cun’C sketching. 164.
Approximations, 127 ; successive, 252. Cylinder, surface and volume of.
Arc, length of, 228. 388.
Area, calculation of. 180; further Cylindrical surface, definition of, 362.
examples, 211 ; of quadrilateral,
266 ; of sector of circle, 382 ; of sur- Definite integral. 180; evaluated by
face of revolution, 230 ; of trape- change of variable. 198.
zium. 382; of triangle, 96; of Dependent variable, 119.
triangle with given vertices, 265. Depression, angle of, 112.
Arithmetic mean, 36. Derivative, 124; higher. 14Ch
Arithmetical progression, 30. Derived function, 124.
Asymptotes, of hyperbola. 330. Desargues’ theorem, 360.
Auxiliary circle, of ellipse, 322. Diameter, of ellipse, 318.
Differential calculus, some applications
Binomial theorem, for fractional and of. 151.
negative indices. 62 ; for positive Differential coefficient, as rate mea-
integral index, 49. surer, 127, 151 ; of cos x, 125
; of
Bisectors, of angles between given cosec X, 136; of cot jt, 136; of «»,
lines, 287. 243 ; of function of function, 136 ;
of implicit functions. 144 ; of inverse
Cartesian coordinates, 259 ; relation functions, 142 ; of product, 132 of
with polar, 260. quotient. 134; of sec x. 136; of
;

Centre, of ellipse, 318 of gravity, 218


; sin X, 125 ; of sin-t x, 143 ; of sum,
of hyperbola, 325; of mass, 218 z 131; of tan x, 135; of tan-^x.
of similitude, 344.
143 ; of X". 124, 140.
Centroid of triangle. 349.
Differentiation, from first principles.
Ceva's theorem, 383 converse of, 354,
; 123 ; logarithmic, 251.
Change of variable, integration by, 194, Dihedral angle, definition of, 372.
ld8«
Director circle, of ellipse, 321.
Circle, Apollonius,346 ; auxiliary, 322 Directrix, of conic section, 310.
director, 321 ; escribed, 99 general
; Distance, between two points with
equation to, 295 ; geometrical theor-
given coordinates, 261.
ems on, 337 ; inscribed, 97
; inter- Dynamical appUcations of
sections with straight lino, differential
299; calculus. 162 of integral calculus.
t^gentto,298; throughintersections ;
173.
of given circles. 308 through three
;
given points, 297 ; whose diameter is
Eccentric angle, 381.
join of given points, 296
; with given Eccentricity of conic section. 810.
centre and radius, 296.
Elevation, angle of. 1 12.
417
; ; ; ; ;

418 INDEX
Ellipse, auxiliary circle of, 322 : centre straight line, 327 latus-rectum of, ;
of. 318; definition of, 310 director ; 325 normal to, 327
; parametric ;
circle of. 321; equation to, 317; equations to. 328 rectangular, 332 ; ;
loci of, 319 intersections with
; tangent to, 326 transverse axis of,
;
straight line. 320latus-rectum of,
; 325 vertices of, 325.
;
318 major and minor axes of, 318
:
;

normal to, 320 parametric equa-


; Implicit function, definition of, 119;
tions to, 321; tangent to, 319; differentiation of, 144.
vertices of, 318. In-centre of triangle, 348.
Equation a cos d -|- 6 sin 0 = c, 82. Increment notation. 122.
Equation to locus. 268. Indefinite integral, 169.
Equations, in which unknown occurs Independent variable, 119.
as index, 27; miscellaneous, 16; Indices, fractional, zero and negative,
simultaneous, 17. 22 ; fundamental laws of, 22.
Equations, quadratic, 13 roots of, ; Induction, method of, 50.
13 theory of. 13.
; Inertia, moment of, 224.
Equations, trigonometrical, solution Inflexion, points of, 162.
of. 68. Inscribed circle, radius of, 97.
Escribed circle, radius of, 99. Integral, as a sum, 177 ; definite, 178 ;

Euler line, of triangle, 350. indefinite, 169.


Ex-centre, of triangle. 348. Integral calculus, some applications of,
Explicit function, 119. 211 .

Exponential function, 241 ; difieren- Integrand, definition of, 171.


tiation and integration of, 243 Integration, approximate methods of,
series for, 254. 203; by change of variable, 194;
by parts, 200 ;
of pro-of c*. 243 ;

Factor formulae, 80. ducts of sines and cosines, 192 of ;

Figures of revolution, volumes of, 183. rational algebraical functions. 247


Focus, of conic section, 310. some methods of, 188.
Fractional indices, 22. Interest, simple and compound, 40.
Fractions, partial, 31. Intersections, of curves with given
Frustrum, of cone and pyramid, defin- equations. 270.
ition of, 362 spherical, volume of,
; Inverse functions, differentiation of,
392. 142.
Function of function, difierential co- Inverse notation, 84.
efficient of, 136.
Functional notation. 119. Joachimsthal’s section formulae, 203.
Functions, derived, 124; explicit, 1 19 ;
implicit, 119; many-valued, 120; Latus-rectum, of ellipse, 318 of
single-valued, 119. hyperbola, 325 of parabola, 311.
;

Length of arc, 228 of sector of circle,;

Generator, of cone, 302 ; of cylinder, 382.


302. Length of tangent to circle from given
Geometrical applications of integral point, 302.
calculus, 173. Limit of sum of series, 41.
Geometrical mean, 38. Line of greatest slope, 372.
Geometrical progression, 38 ; conver- Locus, equation to, 208.
gence of. 41. Logarithmic function, 239 differen-
Geometry of circle, 295. tiation of, 240 ; series for. 254.
Gradient of curve, 120. Logarithms, common, 26 ;
theory of,

Graphs of trigonometrical ratios, for 25.


acute angles, 62 ; for the general
angle, 65. Maclaurin’s series, 252.
Gravity, centre of. 218. Major axis, of ellipse, 318.
Gyration, radius of, 224. Many-valued functions, 120.
Mass, centre of, 218.

Heights and distances, 112. Maximum and minimum. 154.


Mean, arithmetic, 36 geometric, 38.
Higher derivatives, 146. ;

Hyperbola, asymptotes of. 330 centre ;


Mean values. 216.
of. 325 definition of, 310 director ;
Medians, of triangle, 349.
;
Menelaus’ theorem, 356 converse ol,
circle of, 327 equation to, 324 ;
;
;

foci of, 326 intersections with 355 .


;

INDEX 419
Mensuration formulae, summary of, Quadratic equation, roots of. 13 sum ;

396. and products of roots of, 14 theory j


Methods of integration, 188. of, 13.
Minor axis, of ellipse, 318. Quadrilateral, area of. 266 ; ratio and
Miscellaneous equations, 16. rectangle properties of, 345.
Moments of inertia, 224. Quotient, diflerential coefficient of, 134.
Multiple angles, 77.
Radical axis, 305.
Natural numbers, series involving, 42. Radius of gyration, 224.
Negative angles, 58. Ratio and rectangle properties of
Negative indices, 22. triangle and quadrilateral, 345.
Nine-point circle, 350. Ratios, trigonometrical. 57 graphs of, ;
Normal, definition of. 312 to ellipse.; 62 of some related angles. 00,
;
320 to hyperbola, 327 ; to para-
; Rectangular hyperbola. 332.
bola, 312 to plane, 366.
: Regular polyhedron, definition of. 361
Numerical integration, 203. Relations between sides and angles of
triangle, 93.
Orthocentre, of triangle, 348. Remainder theorem, 28.
Orthogonal circles, 303. Right circular cone, definition of, 389.
Orthogonal projection, 371. Right circular cylinder, definition of,
388.
Pair of straight lines, equation to, 200. Roots of quadratic equation, 13
Parabola, definition of, 310 ; sum
equation ; and product of, 14.
to, 310important property of. 315
;

intersections with line, 312: latus- Sector of circle, area of, 382 length
rectum of, 311: normal to, 312 • ;
of arc, 3S2.
parametric equations to, 314; tan-
Sector of sphere, volume of, 393.
gent to. 311: vertex of, 31 1.
Series. 36 for e*. 254 for log,
Parallelepiped, definition of, 361. ; ; ( I -f x),
254: involving natural numbers.
Parallelism of lines, condition for,
284. 42 : Maclaurin's, 252.
Parametric equations, to eUipse, 321
Shortest distance between skew lines
to hyperbola, 328 : to parabola.
Su! 368.
ParUal fractions, 31 ; use in iategra-
® Similar rectilinear figures, theorems
tion, 248. on,
343«
Parts, integration by, 200.
Simil^ triangles, theorems on, 340.
Pascal's arithmetical triangle,
60. Similitude, centre of, 344.
Pedal line, of triangle. 351.
Simple interest, 40.
Pedal triangle, 348.
Simpson’s rule, for approximate inte-
function. 66 ; amplitude of,
oo»
gration, 207.
Permutations, 44.
aimson une, of tnangle, 351,
Simultaneous equations, 17.
distance of point from Sine, addition theorem
^*Unr?85 for, 74 dif-
ferential coefficient of,
Perpendicularity of lines, condition 125; formula
for. for tnangle, 93.
Plane, definition of, 381. Single-valued functions, 119.
Playfair's axiom, 363.
Skew lines, angle bebveen, 363
: defin-
Points of inflexion, 162. ition of, 363 shortest
; distance
Polar coordinates, 260; between. 308.
relation vn Slope, of line. 121.
(^tesian coordinates, 260.
^lyhedron, Small angles, 88.
definition of, 381.
Solution of triangles, 104.
inverse function, 8
Sphere, geometrical properties
381 volume of 376 •

^82.' ; c mensuration of, 391. *

Probability, 47. Square root of (a y/b). 18.


ftoduct, diflerential Standard forms, differential
coefficient of, 1 coeffi.
l^ogr^ssioiis, Cleats. 146 integrals. 170; gentS-
axithmcticftl ' 3fi • ;
'
metrical, 38. integrals. 188.
on, 340. intercepts on
Ptolemy s theorem, 346. ^es, 277 , m terms of length and
; VO “ “"Sin-
278 in terms of slope
,°n
,
and coordin-
ates of point on lino.
276 ; in terms
;;

420 INDEX
of slope and intercept on >''axis. 275 ; Triangle, numerical solution of, 104;
intersections with circle, 299 with ; radii of escribed circles, 99 radius ;

ellipse, with hyperbola, 327


320 ; ; of inscribed circle, 97 relations ;

with parabola, 312; parallel to an between sides andangles, 93.


axis. 274 passing through inter-
;
Trigonometrical equations, solution of,
sections of two given lines, 280 68 .

passing through two given points, Trigonometrical ratios, for any angle,
277 special forms of equation, 275.
; 58 graphs of, 62
;
of some related
;

Submultiple angles, 79. angles, 60.


Substitution, integration by, 194. Truncated prism, pyramid, etc., defin-
Successive approximations, 252. ition of, 362.
Summary, formulae for triangle,
of Turning points, 154.
103 of mensuration formulae. 396.
;

Surface, of cone. 390 of cylinder, 388 ;


:
Undetermined coefficients, principle
of spherical frustrum, 393. of, 29.
Surface of revolution, area of, 230.
Variable, dependent, 119; inde-
Tangent, addition theorem for, 76 pendent, 119.
formula for triangle, 102. Vertex of ellipse, 318; of hyperbola,
Tangent, to circle, 298 to curve, 122 ; ; 325 of parabola, 311.
;

to ellipse, 319 to hyperbola, 326


; ; Volume of cone, 389 of cylinder, 388
; ;

to parabola, 311. of of revolution, 183


figures of ;

Tetrahedron, definition of, 3C1 ; vol- frustrum of sphere, 392 of oblique ;

ume of. 385. prism, 384 of prism, 382


;
of ;

Transverse axis, of hyperbola, 325. pyramid, 384 of tetrahedron, 385.


;

Trapezium, area of, 382.


Trapezoidal rule, 205. Zero indices, 22

2040

You might also like